You are on page 1of 375

<A

O
X
>

h
SCHAUM'S OUTLINE OF

1
(i

THEORY AND PROBLEMS


OF

THEORETICAL MECHANICS
with an introduction to

Lagrange's Equations

and Hamiltonian Theory

BY

MURRAY

R.

SPIEGEL, Ph.D.

Professor of Mathematics

Rensselaer Polytechnic Institute

SCHAUM'S OUTLINE SERIES


McGRAW-HILL BOOK COMPANY
New

York, St. Louis, San Francisco, Toronto, Sydney

Copyright

1967 by McGraw-Hill, Inc.

United States of America.

No part

All rights reserved.

of this publication

may

Printed in the

be reproduced,

stored in a retrieval system, or transmitted, in any form or by any means,


electronic, mechanical, photocopying, recording, or otherwise, without the prior

written permission of the publisher.

60232
8 9 10 11 12 13 14 15

SH SH

7 5

Preface
In the 17th century, Sir Isaac Newton formulated his now famous laws of mechanics.
These remarkably simple laws served to describe and predict the motions of observable
objects in the universe, including those of the planets of our solar system.

Early in the 20th century it was discovered that various theoretical conclusions defrom Newton's laws were not in accord with certain conclusions deduced from theories
of electromagnetism and atomic phenomena which were equally well founded experimentally.
These discrepancies led to Einstein's relativistic mechanics which revolutionized the concepts of space and time, and to quantum mechanics. For objects which move with speeds
much less than that of light and which have dimensions large compared with those of atoms
and molecules Newtonian mechanics, also called classical mechanics, is nevertheless quite
satisfactory. For this reason it has maintained its fundamental importance in science and
rived

engineering.

purpose of this book to present an account of Newtonian mechanics and its


The book is designed for use either as a supplement to all current standard
textbooks or as a textbook for a formal course in mechanics. It should also prove useful to
students taking courses in physics, engineering, mathematics, astronomy, celestial mechanics, aerodynamics and in general any field which needs in its formulation the basic
principles of mechanics.
It is the

applications.

Each chapter begins with a clear statement of pertinent definitions, principles and
theorems together with illustrative and other descriptive material. This is followed by
graded sets of solved and supplementary problems. The solved problems serve to illustrate
and amplify the theory, bring into sharp focus those fine points without which the student
continually feels himself on unsafe ground, and provide the repetition of basic principles
so vital to effective learning. Numerous proofs of theorems and derivations of basic results are included in the solved problems.
The large number of supplementary problems
with answers serve as a complete review of the material of each chapter.
Topics covered include the dynamics and statics of a particle, systems of particles and
Vector methods, which lend themselves so readily to concise notation and to
geometric and physical interpretations, are introduced early and used throughout the book.
An account of vectors is provided in the first chapter and may either be studied at the beginning or referred to as the need arises. Added features are the chapters on Lagrange's
equations and Hamiltonian theory which provide other equivalent formulations of
Newtonian mechanics and which are of great practical and theoretical value.
rigid bodies.

Considerably more material has been included here than can be covered in most courses.
This has been done to make the book more flexible, to provide a more useful book of
reference and to stimulate further interest in the topics.
I wish to take this opportunity to thank the staff of the
for their splendid cooperation.

Schaum Publishing Company

M. R. Spiegel
Rensselaer Polytechnic Institute
February, 1967

CONTENTS
Page
Chapter

VECTORS, VELOCITY AND ACCELERATION

Mechanics, kinematics, dynamics and statics. Axiomatic foundations of meMathematical models. Space, time and matter. Scalars and vectors.
Vector algebra. Laws of vector algebra. Unit vectors. Rectangular unit vectors. Components of a vector. Dot or scalar product. Cross or vector product.
Triple products. Derivatives of vectors. Integrals of vectors. Velocity. Acceleration. Relative velocity and acceleration. Tangential and normal acceleration. Circular motion. Notation for time derivatives. Gradient, divergence
and curl. Line integrals. Independence of the path. Free, sliding and bound
chanics.

vectors.

Chapter

NEWTON'S LAWS OF MOTION. WORK, ENERGY


AND MOMENTUM

librium of a particle.

Chapter

33

Newton's laws. Definitions of force and mass. Units of force and mass.
Inertial frames of reference. Absolute motion. Work. Power. Kinetic energy.
Conservative force fields. Potential energy or potential. Conservation of
energy. Impulse. Torque and angular momentum. Conservation of momentum.
Conservation of angular momentum. Non-conservative forces. Statics or equiStability of equilibrium.

MOTION IN A UNIFORM
AND PROJECTILES
Uniform force

FIELD. FALLING BODIES


62

Uniformly accelerated motion. Weight and acceleration


due to gravity. Gravitational system of units. Assumption of a flat earth.
Freely falling bodies. Projectiles. Potential and potential energy in a uniform
force field. Motion in a resisting medium. Isolating the system. Constrained
motion. Friction. Statics in a uniform gravitational field.

Chapter

fields.

THE SIMPLE HARMONIC OSCILLATOR AND


THE SIMPLE PENDULUM

86

The simple harmonic oscillator. Amplitude, period and frequency of simple


harmonic motion. Energy of a simple harmonic oscillator. The damped harmonic oscillator. Over-damped, critically damped and under-damped motion.
Forced vibrations. Resonance. The simple pendulum. The two and three
dimensional harmonic oscillator.

Chapter

CENTRAL FORCES AND PLANETARY MOTION


Central forces. Some important properties of central force fields. Equations
of motion for a particle in a central field. Important equations deduced from
the equations of motion. Potential energy of a particle in a central field. Conservation of energy. Determination of the orbit from the central force. Determination of the central force from the orbit. Conic sections, ellipse, parabola
and hyperbola. Some definitions in astronomy. Kepler's laws of planetary
motion. Newton's universal law of gravitation. Attraction of spheres and
other objects. Motion in an inverse square field.

116

CONTENTS
Page
Chapter

MOVING COORDINATE SYSTEMS

144

Non-inertial coordinate systems. Rotating coordinate systems. Derivative


operators. Velocity in a moving system. Acceleration in a moving system.
Coriolis and centripetal acceleration. Motion of a particle relative to the earth.
Coriolis and centripetal force. Moving coordinate systems in general. The

Foucault pendulum.

Chapter

SYSTEMS OF PARTICLES

165

Discrete and continuous systems. Density. Rigid and elastic bodies. Degrees
of freedom. Center of mass. Center of gravity. Momentum of a system of
Angular
particles. Motion of the center of mass. Conservation of momentum.
system.
on
a
acting
torque
external
Total
particles.
of
system
momentum of a
Relation between angular momentum and total external torque. Conservation
particles. Work. Poof angular momentum. Kinetic energy of a system of
the center of mass.
tential energy. Conservation of energy. Motion relative to
constraints. Virtual disnon-holonomic
and
Holonomic
Constraints.
Impulse.
work. Equiplacements. Statics of a system of particles. Principle of virtual

librium in conservative

Chapter

fields.

Stability of equilibrium. D'Alembert's principle.

APPLICATIONS TO VIBRATING SYSTEMS,


ROCKETS AND COLLISIONS

194

mass. Rockets.
Vibrating systems of particles. Problems involving changing
vibrating string.
The
particles.
of
systems
Continuous
particles.
Collisions of
problems. Fourier series. Odd and even functions. Con-

Boundary-value
vergence of Fourier

Chapter

series.

PLANE MOTION OF RIGID BODIES

224

Instantaneous
Rigid bodies. Translations and rotations. Euler's theorem.
rigid body. Chasle's
of
a
motion
General
freedom.
of
Degrees
axis of rotation.
inertia. Radius of gyratheorem. Plane motion of a rigid body. Moment of
theorem. Perpendicular
axis
Parallel
inertia.
of
moments
tion. Theorems on
Kinetic energy and
axes theorem. Special moments of inertia. Couples.
rigid body about a fixed
of
a
Motion
axis.
fixed
a
about
angular momentum
angular momentum. Principle of conservation of energy.
axis.

Principle of

Work and

power.

Impulse.

Conservation of angular momentum. The comInstantaneous center.

pound pendulum. General plane motion of a rigid body.


Principle of virtual work
Space and body centrodes. Statics of a rigid body.
energy. Stability.
potential
minimum
and D'Alembert's principle. Principle of

Chapter

10

SPACE MOTION OF RIGID BODIES

Pure rotation
General motion of rigid bodies in space. Degrees of freedom.
of a rigid body with one point
of rigid bodies. Velocity and angular velocity
inertia. Products of inertia. Moment
fixed Angular momentum. Moments of
of rotation. Principal axes of
of inertia matrix or tensor. Kinetic energy
about the principal axes. The
energy
kinetic
and
momentum
inertia. Angular
Force free motion. The inmotion.
ellipsoid of inertia. Euler's equations of
Herpolhode. Space
Polhode.
construction.
Poinsot's
plane.
variable line and
earth. The Euler
the
of
Rotation
and body cones. Symmetric rigid bodies.
of Euler angles. Motion
terms
in
energy
kinetic
and
velocity
angles. Angular
of a spinning top. Gyroscopes.

253

CONTENTS
Page
Chapter

//

LAGRANGE'S EQUATIONS

282

General methods of mechanics. Generalized coordinates. Notation. Transformation equations. Classification of mechanical systems. Scleronomic and
rheonomic systems. Holonomic and non-holonomic systems. Conservative and
non-conservative systems.
Kinetic energy.
Generalized velocities.
Generalized forces. Lagrange's equations. Generalized momenta. Lagrange's
equations for non-holonomic systems. Lagrange's equations with impulsive
forces.

Chapter

12

HAMILTONIAN THEORY
Hamiltonian methods.

The Hamiltonian.

311
Hamilton's equations.

The Hamil-

tonian for conservative systems. Ignorable or cyclic coordinates.


Phase space.
Liouville's theorem. The calculus of variations. Hamilton's
principle. Canonical or contact transformations. Condition that a transformation
be canoni-

Generating functions. The Hamilton-Jacobi equation. Solution


of the
Hamilton-Jacobi equation. Case where Hamiltonian is independent of
time.
Phase integrals. Action and angle variables.
cal.

APPENDIX A

UNITS AND DIMENSIONS

339

APPENDIX B

ASTRONOMICAL DATA

342

appendix c

SOLUTIONS OF SPECIAL DIFFERENTIAL EQUATIONS

APPENDIX D

INDEX OF SPECIAL SYMBOLS AND NOTATIONS

356

INDEX

361

....

344

and ACCELERATION

MECHANICS, KINEMATICS, DYNAMICS AND STATICS


Mechanics

is

physical objects.

a branch of physics concerned with motion or change in position of


It is sometimes further subdivided into:

1.

Kinematics, which

2.

Dynamics, which

3.

Statics,

which

is

is

is

concerned with the geometry of the motion,


concerned with the physical causes of the motion,

concerned with conditions under which no motion

is

apparent.

AXIOMATIC FOUNDATIONS OF MECHANICS


An

axiomatic development of mechanics, as for any science, should contain the following

basic ingredients:
1.

This is clearly necessary since ultimately any


Undefined terms or concepts.
must be based on something which remains undefined.

definition
2.

Unproved assertions. These are fundamental statements, usually in mathematical


form, which it is hoped will lead to valid descriptions of phenomena under study.
In general these statements, called axioms or postulates, are based on experimental
observations or abstracted from them. In such case they are often called laws.

These definitions are given by using the undefined

3.

Defined terms or concepts.


terms or concepts.

4.

Proved assertions. These are often


definitions and axioms.

called

An example of the "axiomatic way of thinking"


which point and line are undefined concepts.

theorems and are proved from the

is

provided by Euclidean geometry in

MATHEMATICAL MODELS
A mathematical description

of physical phenomena is often simplified by replacing


actual physical objects by suitable mathematical models. For example in describing the
rotation of the earth about the sun we can for many practical purposes treat the earth

and sun as

points.

SPACE, TIME AND MATTER


From everyday experience, we

all have some idea as to the meaning of each of the


following terms or concepts. However, we would certainly find it difficult to formulate
completely satisfactory definitions. We take them as undefined concepts.

VECTORS, VELOCITY AND ACCELERATION

[CHAP.

closely related to the concepts of point, position, direction and


Measurement in space involves the concepts of length or distance,
with which we assume familiarity. Units of length are feet, meters, miles, etc.
In this book we assume that space is Euclidean, i.e. the space of Euclid's geometry.

1.

Space. This
displacement.

2.

This concept is derived from our experience of having one event taking
Measurement of time is
place after, before or simultaneous with another event.
achieved, for example, by use of clocks. Units of time are seconds, hours, years, etc.

3.

Matter.

is

Time.

Physical objects are composed of "small bits of matter" such as atoms


and molecules. From this we arrive at the concept of a material object called a
particle which can be considered as occupying a point in space and perhaps moving
as time goes by. A measure of the "quantity of matter" associated with a particle
is called its mass.
Units of mass are grams, kilograms, etc. Unless otherwise
stated we shall assume that the mass of a particle does not change with time.

Length, mass and time are often called dimensions from which other physical quantities
are constructed. For a discussion of units and dimensions see Appendix A, Page 339.

SCALARS AND VECTORS


Various quantities of physics, such as length, mass and time, require for their specificanumber (apart from units of measurement which are decided upon in
advance). Such quantities are called scalars and the real number is called the magnitude
of the quantity. A scalar is represented analytically by a letter such as t, m, etc.

tion a single real

Other quantities of physics, such as displacement, require for their specification a


Such quantities are called vectors. A vector is repreGeometrically it is
sented analytically by a bold faced letter such as A in Fig. 1-1.
represented by an arrow PQ where P is called the initial point and Q is called the terminal
point. The magnitude or length of the vector is then denoted by |A| or A.
direction as well as magnitude.

Fig. 1-3

Fig. 1-2

Fig. 1-1

VECTOR ALGEBRA
The operations of addition, subtraction and multiplication familiar in the algebra of
real numbers are with suitable definition capable of extension to an algebra of vectors.
The following definitions are fundamental.
Two vectors A and B are equal if they have the same magnitude and direction
1.
regardless of their initial points. Thus A = B in Fig. 1-2 above.
2.

A vector having
denoted by

3.

-A

direction opposite to that of vector


as in Fig. 1-3 above.

but with the same length

is

The sum or resultant of vectors A and B of Fig. l-4(a) below is a vector C formed
by placing the initial point of B on the terminal point of A and joining the initial

B [see Fig. l-4(b) below]. We write C = A + B.


law for vector addition as indicated
parallelogram
This definition is equivalent to the
in Fig. l-4(c) below.
point of

A to the terminal point

of

CHAP.

VECTORS, VELOCITY

11

AND ACCELERATION

Fig. 1-4

Extensions to sums of more than two vectors are immediate. For example,
shows how to obtain the sum or resultant E of the vectors A, B, C
and D.
Fig. 1-5 below

rrfB+c+D
Fig. 1-5

A B,

is that vector C which


be denned as A + (-B). If
A = B, then A B is defined as the null or zero vector represented by 0. This has
a magnitude of zero but its direction is not defined.

The difference of
when added to B

vectors
gives A.

and B, represented by
Equivalently,

A-B

may

of a vector A by a scalar p is a vector pA or Ap with magnitude


times the magnitude of A and direction the same as or opposite to that of A
according as p is positive or negative. If p = 0, pA = 0, the null vector.

The product

5.

\p\

LAWS OF VECTOR ALGEBRA


B and C are vectors, and
A+B = B+A
A + (B + C) = (A + B) + C
p(qA) = (pq)A = q(pA)
(p + q)A = pA + qA
p{A + B) = pA + pB

If A,
1.

2.
3.
4.
5.

Note that
defined.

in these

p and q are scalars, then

Law for Addition


Associative Law for Addition
Associative Law for Multiplication
Distributive Law
Distributive Law
Commutative

laws only multiplication of a vector by one or more scalars


5 we define products of vectors.

is

On pages 4 and

UNIT VECTORS
Vectors having unit length are called unit vectors. If A
then A/A = a is a unit vector having the same direction as

is

a vector with length

and

A>

0,

A = A a.

RECTANGULAR UNIT VECTORS


The rectangular unit vectors
directions of the positive x,

i,

y and

and k are mutually perpendicular unit vectors having


a rectangular coordinate system

z axes respectively of

VECTORS, VELOCITY AND ACCELERATION

[CHAP.

[see Fig. 1-6]. We use right-handed rectangular coordinate


systems unless otherwise specified. Such systems derive
their name from the fact that a right threaded screw rotated through 90 from Ox to Oy will advance in the positive z direction. In general three vectors A, B and C which
have coincident initial points and are not coplanar are said
to form a right-handed system or dextral system if a right
threaded screw rotated through an angle less than 180 from
A to B will advance in the direction C [see Fig. 1-7 below].

\/o.

Fig. 1-6

(4i,A2,As)

Fig. 1-8

Fig. 1-7

COMPONENTS OF A VECTOR
Any vector A in 3 dimensions

can be represented with initial point at the origin O of


a rectangular coordinate system [see Fig. 1-8 above]. Let (Ai,A 2 ,A 3 ) be the rectangular
coordinates of the terminal point of vector A with initial point at 0. The vectors Aii,
A 2j and A 3k are called the rectangular component vectors, or simply component vectors,
of A in the x, y and z directions respectively. A lf A 2 and A 3 are called the rectangular
components, or simply components, of

The sum or

resultant of Aii,

A 2j

and Ask

A =
The magnitude

of

A =

is

|A|

y and

in the x,
is

z directions respectively.

the vector A, so that

(1)

= v^f+AfTAf

(2)

|r|

y/x 2

write

+ A 2j + A 3k

Aii

In particular, the position vector or radius vector r from


written
.
r = xi + yj + zk

and has magnitude r

we can

+ y2 + z 2

to the point {x,y,z) is

(3)

DOT OR SCALAR PRODUCT


The dot or

scalar product of

two vectors

is defined as the product of the magnitudes of


between them. In symbols,
cos 0,
AB =

AS

Note that

A*B

is

a scalar and not a vector.

and B, denoted by A-B (read A dot B)


A and B and the cosine of the angle

0^0^77

(4)

CHAP.

VECTORS, VELOCITY AND ACCELERATION

11

The following laws are


1.

2.
3.
4.
5.

valid:

Commutative Law for Dot Products


A B = B A
Distributive Law
A-(B + C) = AB + AC
=
where p is a scalar.
(A B)p,
p{A B) = (pA) B = A (pB)
i*i = j*j = k*k = l,
i*j = j*k = k*i =
If A = Aii + Aaj + Aak and B = Bd + B 2j + Bak, then
A B = AiBi + A2B2 + A3B3
A- A = A 2 = A\ + A\ + A\
B-B = B 2 = B\ + B\ + B%

6.

A-B =

If

and

and

are not null vectors, then

and

are perpendicular.

CROSS OR VECTOR PRODUCT


The cross or vector product of A and B is a vector C = A x B (read A cross B). The
magnitude of A x B is denned as the product of the magnitudes of A and B and the sine
of the angle between them. The direction of the vector C = A x B is perpendicular to the
plane of A and B and such that A, B and C form a right-handed system. In symbols,

A X B = AB sin 6 u,
where u

is

a unit vector indicating the direction of

to B, then sin 6

and we

The following laws are


1.

2.
3.
4.
5.

define

AxB =

A x B.

tt

If

(5)

A=B

or

if

is

parallel

0.

valid:

A x B = B x A
(Commutative Law for Cross Products Fails)
Ax(B + C) = AxB + AxC
Distributive Law
p{A x B) = {pA) x B = A x (pB) - (A x B)p,
where p is a scalar.
ixi = jxj = kxk = 0, ixj = k, x k =
kxi = j
If A = Aii + Aaj + Aak and B = Bd + B2j +
then
i,

AxB

At
Bx

6.

|A

7.

If

A2 A3
B 2 B3

= the area of a parallelogram with sides A and B.


A x B = and A and B are not null vectors, then A and B

B|

are parallel.

TRIPLE PRODUCTS
The scalar

triple

product

is

defined as

At
(B x C)

Bx
Ct

where A =

Aii

+ A 2j + A3k,

B =

B\\

+ B2j + Bak,

A 2 As
B2 Bz

(')

C2 Cz
C = Cd + C2j + Cak.

It

represents the

volume of a parallelepiped having A, B, C as edges, or the negative of this volume according


as A, B, C do or do not form a right handed system. We have A (B x C) = B (C x A) =

C-(AXB).
The vector
Since

triple

(A x B) x

product

C = (A

is

defined as

AX(BXC) = (A-C)B-(A-B)C
C)B - (B C) A, it is clear that A x (B x C)

{7)

(A x B) x C.

VECTORS, VELOCITY AND ACCELERATION

[CHAP.

DERIVATIVES OF VECTORS
assumed by a scalar variable u there corresponds a vector A(u), or
The derivative of A(u) is defined as

If to each value

briefly A, then A(u) is called a (vector) function of u.

dU
provided this limit exists.

Similarly

we can

if it exists is

dA

du

~~

A =

If

dAi
du

&A _

dAi.

3tt)i +

(2m2

(4m

5 cos

- 3)i -

du*

5 sin

,q*

du

For example the second derivative of A(u)

&A2.

+ du2

3 sin

'

then

c?A 3 .

du

^Ai.

+ Ai{u)i + A 3 (w)k,

Ai(u)i

define higher derivatives.

given by

du2
Example.

A(u)

If

AW

Au~0

d?As

1/m
"*

then

k,

3 cos

k,

-j-g-

4i

5 cos

3 sin

uk

The usual rules of differentiation familiar in the calculus can be extended to vectors,
although order of factors in products may be important. For example if ${u) is a scalar
function while A{u) and B(u) are vector functions, then

t(*-*)

A(AXB) =
INTEGRALS OF VECTORS
Let A{u) = Ai(u)i + A 2 (u)j + A s (u)k

<">

Axf + fxB

be a vector function of

(IS)

We

u.

define the indefinite

integral of A(u) as

( A(u)du
If there exists

limits

c is

{u)du

A 2 (w)^ + kj A

r^{B(tt)}<fa

definite integral

is in

(U)

(u)du

^-{B(w)}, then

an arbitrary constant vector independent of

u = a and u = /?

C A(u)du
The

a vector function B(u) such that A(w)

J*A(M)dM
where

f A

B(m)

u.

The

(*5)

definite integral

between

such case, as in elementary calculus, given by

f'-fo{B{u))du

B(u)

can also be defined as a limit of a

'

B(/3)

sum analogous

B()

(iff)

to that of elementary

calculus.

VELOCITY
Suppose that a particle moves along a path or curve C [Fig. 1-9 below]. Let the position
vector of point P at time t be r = r(t) while the position vector of point Q at time t + At is

CHAP.

VECTORS, VELOCITY AND ACCELERATION

11

+ Ar =

at

r(t

+ At). Then

the velocity (also called the instantaneous velocity) of the particle

given by

is

dr
-

Ar
hm
..

at

At-*o

At

and

a vector tangent to

is

If

we can

r(t)

x(t)i

at P.

+ y(t)j + z{t)k =

xi

+ yj + zk,

write

dt

dx.

dt

dt

The magnitude of the

where

(17)

AC

At-*o

dy

dz

+ Tt k

18)

velocity is called the speed

|v|

is

given by

V(t y+ (fr

dt

s is the arc length along

and

dr

Fig. 1-9

measured from some

(t

(19)

dt

point to P.

initial

ACCELERATION
If v = dr/dt is the velocity of the particle, we define the acceleration (also called the
instantaneous acceleration) of the particle at P as

a
In terms of r

xi

+ yj + zk
*

and

its

magnitude

dv

dt

Hm

the acceleration

_
~

d^r
dt*

v(t

_
~

+ At)-v(t)
At

At-0

is

d?x.
21i
dt

dV

d?y.

~
+ ^5k
+ 5^J
dt2
dt2

(21)

'

is

V(S)' +

(SWS

(22)

RELATIVE VELOCITY AND ACCELERATION


If

two particles Pi and Pz are moving with respective


and a2, the vectors

velocities vi

and

V2

and

accelera-

tions ai

vpg/Pj

v2

vi

and

ap2 /Pj

a2

ai

(23

are respectively called the relative velocity and relative acceleration of P2 with respect to Pi

TANGENTIAL AND
NORMAL ACCELERATION
Suppose that particle P with position vec= r(t) moves along curve C [Fig. 1-10].
can consider a rectangular coordinate
system moving with the particle and defined
by the unit tangent vector T, the unit principal normal N and the unit binormal B to
curve C where
tor r

We

Fig. 1-10

VECTORS, VELOCITY AND ACCELERATION

T =
from some

s being the arc length

The

R
N
* = K ds'

ds'

TxN

B =

point to

initial

and

[CHAP.

(U)

the radius of curvature of C at P.


and is given by k= 1/R.

reciprocal of the radius of curvature is called the curvature

We can

show

[see

Problem

1.35,

page 20] that the acceleration along


dt

The

and second terms on the right are

first

is

given by
(25)

R*

called the tangential acceleration

and normal

or centripetal acceleration respectively.

CIRCULAR MOTION
Suppose particle P moves
R.

If s is the arc length

and

on a circle C of radius
measured along C from

the corresponding angle subtended


= Rd. Thus the magnitudes
of the tangential velocity and acceleration are given
respectively by
to

9 is

at the center O, then s

dv

and

dt

d?s

dt2

( e >

fi

2
_ K
p d 2 = Ra

Fig. 1-11

(27)

dt

dd/dt and a - d2 9/dt 2 the angular speed and angular acceleration respectively.
normal acceleration as seen from (25) is given by v 2 /R = <a 2 R.

We call
The

ft =

at

NOTATION FOR TIME DERIVATIVES


We shall sometimes find it convenient

symbol to denote
for a second
dots
derivatives with respect to time t, one dot for a first derivative, two
2
2
=
=
etc.
dvldt,
d r/dt , v
derivative, etc. Thus for example r = dr/dt, r
to use dots placed over a

GRADIENT, DIVERGENCE AND CURL


If to each point (x, y, z) of a rectangular coordinate system there corresponds a vector A,
that A = A(x, y, z) is a vector function of x, y, z. We also call A(x, y, z) a vector
say
we
Similarly
we call the (scalar) function <f>(x, y, z) a scalar field.
field.
It is

convenient to consider a vector differential operator called del given by


(28)

Using
1.

this

we

Gradient
This

2.

define the following important quantities.

is

V4>

(i^
1
+
a^

a vector called the gradient of

Divergence

V'A =

Jx

Mi
a

This

is

^
^ + k ^j<^

J
J^7

+
+

<

and

dz

Mi

a scalar called the divergence of

dx

is also

dy

a?/

dy

+ K dz

written grad

(Ad +

</>.

A 2j + AOt)
(SO)

dz

and

(29)

is

also written

div A.

CHAP.

3.

VECTORS, VELOCITY AND ACCELERATION

1]

Curl

A =

^+

5S

dX

dy

dz

Two

<

A i + A2i + A *
'

At

A2

As

(31)

BA

\dy
is

fdAs
This

a vector called the curl of

dz

dA s \.

and

dX

is also

+ fdA 2

[iF-f ]k

written

curl A.

important identities are

divcurlA

= V(V'XA) =

curl grad

= Vx

<

(32)

(V4>)

LINE INTEGRALS
Let r(t) = x(t)i + y(t)j + z(t)k, where r(t) is the position vector of (x,y,z), define a
Let
curve C joining points Pi and Pi corresponding to t = U and t = t 2 respectively.
A = A(x, y, z) = Ad + A 2j + Ask be a vector function of position (vector field). The
integral of the tangential component of A along C from Pi to P%, written as

JA'dr
is

an example of a

Adr

A 2 dy + A 3 dz

Aidx +

(84)

line integral.

a closed curve (which we shall suppose is a simple closed curve, i.e. a curve
which does not intersect itself anywhere) then the integral is often denoted by
If

is

A-dr

<k

Aidx + A 2 dy +

dz

(35)

In general, a line integral has a value which depends on the path.


evaluation see Problems 1.39 and 1.40.

For methods of

INDEPENDENCE OF THE PATH


The

A = v<,

line integral (34) will

or equivalently

xp2

Jp
Pt

be independent of the path joining Pi and P2


In such case its value is given by
0.

A-dr

if

and only

if

VxA =
I

J Pi

d<f>

<j>(P 2 )

4>(Pi)

<f>(x 2

,y 2 ,z 2 )

<f>(x

u yi,Zx)

(36)

assuming that the coordinates of Pi and P2 are (x v y v z t ) and (x 2 y2 z 2 ) respectively while


The integral (35) in this case is zero.
<f>(x,y,z) has continuous partial derivatives.
,

FREE, SLIDING AND BOUND VECTORS


Up to now we have dealt with vectors which are specified by magnitude and direction
only.
Such vectors are called free vectors. Any two free vectors are equal as long as
they have the same magnitude and direction

[see Fig. l-12(a) below].

VECTORS, VELOCITY AND ACCELERATION

10

(a)

Equal free vectors

(b)

Equal sliding vectors

[CHAP.

(c)

Bound vector

Fig. 1-12

Sometimes in practice the particular line of action of a vector is important. In such


case two vectors are equal if and only if they have the same magnitude, direction and line
of action. Such vectors are often called sliding vectors [see Fig. 1-12(6)].
Sometimes

important to specify the point of action of a vector. Such a vector


a bound vector. In this case two vectors will be equal if and

it is

[see Fig. l-12(c)] is called

only

if

they are identical.

Most cases with which we shall deal involve free vectors. Cases where sliding vectors
or bound vectors need to be employed will in general be clear from the context.

Solved Problems

VECTOR ALGEBRA
1.1.

Show

that addition of vectors

commutative,

is

i.e.

A + B = B + A.

See Fig. 1-13

below.

OP + PQ = OQ
OR + RQ = OQ

and

Then

1.2.

A+B = B+

A+B = C
B+A = C

or

or

A.

Show

that the addition of vectors is associative,


Fig. 1-14 above.

OP + PQ = OQ =
Since

we have

A+

(B

+ C)

(A + B)

OP + PR = OR =
OQ + QR = OR =
= A + B) + C.

D,

A + (B + C) =

PQ + QR = PR =

and
D,

i.e.

i.e.
i.e.

(B

(A

4-

B)

+ C.

See

+ C)

A + (B + C) = D
A + B) + C = D
(

Extensions of the results of Problems


number of vectors is immaterial.

1.1

and

1.2

show that the order of addition of any

CHAP.

1.3.

VECTORS, VELOCITY AND ACCELERATION

1]

Given vectors A,

and C

[Fig. l-15(a)] construct

(a)

11

A - B + 2C,

(b)

3C - (2A - B).

(a)

-B

-|(2A-B)

Pig. 1-15

1.4.

Prove that the magnitude

Ad + Aaj + Ask

is

A =

of the vector

A=

+ A\ + A\.

See

y/A\

Fig. 1-16.

By

the Pythagorean theorem,

(OP)*

where

OP

Similarly,

A*

1.5.

(OQ)*

(QP) 2

denotes the magnitude of vector OP,

(OQ)*

(OP) 2

Then

=
=

= A\ + A\ +

(OR)*
(OR)*

A\,

+
+

etc.

(RQ)*.

+ (QP)* or
A = VA* + A* + A*

(RQ)*

i.e.

Determine the vector having the initial point


P(xi,yi,zi) and the terminal point Q(* 2 3/2, z 2)
and find its magnitude. See Fig. 1-17.
,

The

position vector of

is

rt

The

position vector of

is

r2

x x\ +

+ Zjk.
= x 2i + 2J + 2k
2/ x j

2/

r + PQ = r2 or
r = (^2* + Vzi + *2k (M + #ii + *ik
PQ =
= (2 - *i)i + (Va - 2/i)J + (2 - i)k
x

1*2

Magnitude of

PQ = PQ
= V(*2 - *l) 2 +

Note that this

is

(^2

~ Vl) 2 +

the distance between points

(*2

~ *l) 2

and Q.

Fig. 1-17

VECTORS, VELOCITY AND ACCELERATION

12

1.6.

Find

(a) graphically and


displacements:

sum

analytically the

(b)

[CHAP.

or resultant of the following

A, 10 ft northwest; B, 20 ft 30 north of east; C, 35 ft due south.

See Fig. 1-18.

Graphically.

At the terminal point of


place the initial point
of B. At the terminal point of B place the initial
point of C.

D is formed by joining the initial


the terminal point of C, i.e. D =

The resultant
point of

A+B+

C.

to

The resultant

4.1 units

measured to have magnitude of


and direction 60 south of east.

is

20.5 ft

Analytically.

From

E and N

Fig. 1-18, if
directions,

and

A = - 10

are unit vectors in the


Unit

cos 45

= 20 cos 30
C = 35j

Then the resultant

we have

10 sin 45

20 sin 30

5 ft

is

D = A + B + C =
=
Thus the magnitude of

is

(-10

cos 45

(-5V2 +

V(10.25)

10VS

20cos30)i
)i

+ (17.93) 2 =

tan-i 17.93/10.25

(5 V^ +

(10 sin 45

10

35)j

20 sin 30

10.25i

20.65 ft and the direction

tan" 1 1.749

35)j

17.93J

is

6045' south of east

Note that although the graphical and analytical results agree fairly

well, the analytical result is

more accurate.

of course

THE DOT OR SCALAR PRODUCT


Prove that the projection of A on B is equal to A
1.7.
where b

is

b,

a unit vector in the direction of B.

Through the initial and terminal points of A pass


respectively as in the
planes perpendicular to B at G and
adjacent Fig. 1-19; then

Projection of

1.8.

Prove

A on

B = GH = EF = A

A(B + C) =

cos 6

= A

G
Fig. 1-19

AB + AC.

Let a be a unit vector in the direction of A; then

[see

Fig. 1-20]

Projection of (B

+ C)

(B

on

A =

+ C)a =

B on A
+ projection of C on A
B*a + C*a
projection of

Multiplying by A,

BAa + OAa
A = B A + C A

(B+C)'Aa =
and

(B

+ C)

Then by the commutative law for dot products,

(B

+ C) =

and the distributive law

A B + A C

is valid.

Fig. 1-20

CHAP.

1.9.

= |i||i|cosO = (1)(1)(1) = 1
= |i||k|cos90 = (1)(1)(0) =
k-j = |k||j|cos90 = (1)(1)(0) =
= -3
(2i - 8j + k) = 2j i - 3j j + j k = 0-3 +
j
+ 2i k - 3j
(2i - j) (3i + k) = 2i (3i + k) - j (3i + k) = 6i
= 6 + 0-0-0 =

(a)

ii

(6)

ik

(d)
(e)

If

A=

Aii

AsBa.

+ Aaj+Agk and B = Bd + B 2 2 + B^k,

A-B =

prove that

Ai#i

+ A 2JB 2 +

A =

Aii

+ A 2j + A 3k,
cos 0

A = \/A-A =
A = VA A.

show that

= A2

Then

A\.

(Aji + A 2j + A 3k) (A i + A 2j + A 3k)


= (AJiAJ + (A 2)(A 2 + (A S )(A S = A\ + A\ + Al
taking B = A.

1.10,

A =

+ Al +

y/A\

Then

AA = (A)(A)
AA =
Also,
by Problem

If

(Aii + Aa + AM-iBj + Bzi + Bak)


= A \ (BJ + Baj + B 3k) + A 2j ( i + B 2j + B 3k) + A 3k (BJ + B 2j + B 3k)
= A B i i + A^i j + A B Z \ k + A 2Btf i + A 2 2 j + A 2 3j k
+ A^k i + A 3 2k j + A 3 3k k
= A B + A 2B % + A 3fi 3
k = 1 and all other dot products are zero.
j = k

since

A-B =

1.11.

13

Evaluate each of the following.

(e)

1.10.

VECTORS, VELOCITY AND ACCELERATION

1]

y/A*

A =

-^A\

+ A 2 + A\

the magnitude of A.

is

Sometimes

A A

is

written

A2

5(4,6,0)
1.12.

Find the acute angle between the diagonals of a


quadrilateral having vertices at (0, 0, 0), (3, 2, 0),
(4,6,0), (1,3,0) [Fig. 1-21].

We have
from which

OA =

CA =

3i

+ 2j, OB =

OA-OC =

2i

OB CA =

Then

4i

+ 6j, OC = i + 3j

[OB| |CA| cos $

i.e.

(4i

+ 6j)

from which

(2i

- j) =

cos 6

+ (6)2 V(2) 2 + (-1) 2 cos e


= 2/(v1S2 Vs = -1240 and = 8253'.
V(4) 2

(0,0,0)
Fig. 1-21

THE CROSS OR VECTOR PRODUCT


1.13. Prove AxB = -BxA.

BXA = D
(&)

Fig. 1-22

A X B = C has magnitude
system [Fig. l-22(a) above].

AB

sin $

and direction such that A,

and C form a right-handed

VECTORS, VELOCITY AND ACCELERATION

14

B X A = D has magnitude
system [Pig. 1-22(6) above].
Then

BA

and

D has the same magnitude as C but is opposite in direction, i.e. C = D

The commutative law for cross products

1.14.

and direction such that B,

sin e

[CHAP.

is

form a right-handed
or

A X B = -B X

A.

not valid.

Prove that

Ax(B + C) = AXB + AXC


the case where A is perpendicular

for
to

B and

also to C.

Since A is perpendicular to B, A X B is a
vector perpendicular to the plane of A and B
and having magnitude AB sin 90 =
or
magnitude of AB. This is equivalent to multiplying vector B by A and rotating the
resultant vector through 90 to the position

AB

shown

in Fig. 1-23.

A X C is the vector obtained by


multiplying C by A and rotating the resultant
vector through 90 to the position shown.
In like manner, A X (B + C) is the vector
obtained by multiplying B + C by A and rotating the resultant vector through 90 to the
position shown.
Similarly,

Since A X (B + C) is the diagonal of the


parallelogram with A X B and A X C as sides,

we have

1.15.

AX(B-f-C)

Prove that

AXB + AXC.

Fig. 1-23

A x (B + C) = AxB + AxC

in the general case

non-coplanar.

B and C

where A,

are

See Fig. 1-24.

Resolve B into two component vectors, one


perpendicular to A and the other parallel to A,
and denote them by B^ and B respectively.
(

Then

B = B +

between

If 6 is the angle

B j_ = B

B.

and B, then

Thus the magnitude of A X B j_ is


AB sin 9, the same as the magnitude of A X B.
Also, the direction of A X B is the same as the
direction of A X B. Hence A X B =AXB.
sin

e.

Similarly
vectors

if

resolved into two component

is

and C^,

parallel

respectively to A, then

Also, since

and perpendicular

A X C = A X

Fig. 1-24

C.

B + C = B + B n + C + C M = (Bj_ + C +
AX(B + C 1 = AX(B + C)
)

(B,
(

+ CM

it

follows that

Now

A and so by Problem
AX(B 1 + C 1 = AXB i +AXC 1
A X (B + C) = AXB + AXC

Bjl and Cj_ are vectors perpendicular to

1.14,

Then

and the distributive law holds. Multiplying by 1, using Problem


B X A + C X A. Note that the order of factors in cross products
algebra apply only

if

proper order

is

1.13, this
is

+ C) X A =
The usual laws of

becomes (B

important.

maintained.
i

1.16.

If

A = Ad + A + Ask
2j

and

B=

Bii

+ B 2j + Bak,

prove that

AxB

Ai

A
B

As

Bs

CHAP.

VECTORS, VELOCITY AND ACCELERATION

1]

AXB

=
=
=

Aii X (S x i

(A 2B 3

15

+ Atf + AMxiBii + Btf + BJi.)


+ Ba + B sk) + Aa X (BJ + Brf + 3k) + A 3k X (BJ + # 2j + B 3k)
A^i X i + A xB2i X j + A XB Z\ X k + AgBJ X i + A 252j X J + ^2#3J X k
+ A 3Bik X i + A 3B 2k X j + A 3B 3k X k
(Aii

1.17.

If

A = 3i-j + 2k

and

- A 3B 2)i +

B=

AXB

-1

-1

-1

- A^k =

At

A 2 ^3

B,

B2

*3

-5i

-1

7j

-1

+ k

-1

Ilk

Prove that the area of a parallelogram with


sides A and B is |AxB|.
Area of parallelogram

= h |B|
= A| sin 6
= |AXB|
|

Note that the area of the triangle with

B
1.19.

=
1.18.

(A t B 2

+ 3j-k, findAxB.

2i

- A B Z )\ +

(A sB t

is

|A X

|B|

sides

A and

B
Fig. 1-25

B|

Find the area of the triangle with vertices at P(2,3,5), Q(4,2,-l),


PQ = (4_ 2 )i + (2-3)j + (-l-5)k = 2i-j-6k
PR = (3 - 2)1 + (6 - 3)j + (4 - 5)k = i + 3j - k
Area of triangle

|PQXPR|

= il

-1

k
-6

+ 3j-k)|

(i

= |19i-4j + 7k|

3-1

= |(2i- j-6k) X

72(3,6,4).

iV(19)

+ (-4)2 + (7)2 =

Jv'426

TRIPLE PRODUCTS
1.20.

Show

that

(B x C)

is in

absolute value equal

to the volume of a parallelepiped with sides


A, B and C.
Let n be a unit normal to parallelogram /, having
the direction of B X C, and let h be the height of the
terminal point of
above the parallelogram /.

Volume

If A,

1.21.

(a) If

of parallelepiped

= (height fc)(area of parallelogram J)


= (A.n)(|BXC|)
= A{|BXC|n} = A'(BXC)

and C do not form a right-handed system,

A =

Aii

Fig. 1-26

A n <

and the volume

+ A 2j+Ask, B = Bd + B 2j + Bsk, C =
Bi

A 2 Az
B2 B3

C\

C2

Ai

A-(BxC)

Ci

Cii

= A
|

(B

+ C2j + Ctk show

C)

that

VECTORS, VELOCITY AND ACCELERATION

16

(6)

Give a geometric significance of the case where

(a)

A'(BXC) = A* B t

(A i i

= A

(B x C)

0.

B2 B 3
C2

C3

+ A 2j + A 3k)

[(B 2 C3

- 3 C2)i +

-B3C2)+A 2 (B C -B

(B 2 C3

- B C3 )\ +

{B 3 C X

(B X C2 -

C3) + A 3 {B X C2 -

A
B

Ct =
)

B 2C t )k]
x
x

Cx
(b)

By Problem

1.22.

1.20 if

and conversely

A, B,

if

(B

X C) =
then A,
are coplanar then

B and C

(B

and

1.20

are coplanar,

volume of parallelepiped

1.21,

C)

A 2 A3
B2 B 3
C2

c3

are in the same plane,

i.e.

0.

A = 3i

Find the volume of a parallelepiped with sides


By Problems

Cj

[CHAP.

j,

B = j + 2k, C = i + 5j + 4k.

3-10

|A(BXC)|

1-201

12
15

1.23.

If

A = +

(a)

AXB =

B = 2i-3j + k, C = 4j-3k,

j,

2-3
i

(6)

= - j - 5k.

23i

+ 3j + 4k.

4-3

-5

-1

2-3 1
4-3

BXC =

Then (A X B) X C

110

20.

(a)(AxB)xC, (b)Ax(BxC).

find

5i

+ 6j + 8k.

(A X B) X

It follows that, in general,

AX

Then

C ^ AX

(B

(B

C)

= 8i-8j + k.
8

C).

DERIVATIVES AND INTEGRALS OF VECTORS


1.24.

If r

(a)

(i

~(*s +

At

= 0,

(c)

W = Jtilt)

(a),

From

(c),

Prove that
tions of u.

\dr/dt\

t =t 0,

2i

V(2) 2

(b)

(3*2

dt2

+ 2)i +

6e~2tj

at

= 0.

10 cos ht k

6e

~ 2t

2\/35

10cos5k}

at

= 0.

= 6<i-12e-2tj-50sin5tk

= 12j.

=12

-f-(A-B)

sin 5*)k

+ (6) 2 + (10)2 = ^140 =

d?r/dtf

^,

(a)

+ 6j + 10k.

= ^<( 3 *2 + 2 i +

Idtr/dP]

du

find

+ ^(-3e- 2 *)j + ^(2

dr/dt

From

At

1.25.

2t)i

(6)

(d)

d2 r

+ 2i)i-3<?- 2tj + 2sin5*k,

at

= 0.

A-^
+ ^-B,
du
tint,

where

and

are differentiate func-

CHAP.

VECTORS, VELOCITY AND ACCELERATION

1]

L ,
Method

d ,. ,
-j-(AB)
du v

1.

(A

..

lim

+ AA)

Am

am-*o

+ AB) - A'B

(B

A'AB + AA'B + AA'AB

lim

AM

Au-0

= A dB + dA B
S 1S'

\
AA
AB
AA
/
= ?o{ A '^ + -^- B+ -^' AB )

Method

A = A i + A 2 + A 3k, B =

Let

2.

< A

"

^A

>

A
.

^+A

$A =
T-(*A)
02

32

If

Evaluate

- xzk,

Sx 2yi + yz 2 \

3x*y 2 zi

aV*^ -*) =

OZ

+ A

-2, z

Sx*y 2 i

A(u)

if

(dA x
dA*
.dA 2
\
+ {~dV Bl + ~dV B2 + ~du~ Bs
)
,

d2

12j

at the point

(1,

-2, -1).

x 3 2/s 2k

3x 2y 2 z 2j

- l)i +

(3w2

^-^ (<f>A)

find

6as 4 ]/i

-1, this becomes -12i

A(w)dw

*r;

x2 y2z3 j

3
3
2 2 2
= ^(3a^i/2 i + Sx y z j - 2x wzk) =

1,

df? 3 \

du

+ yz 2\ - xzk) =

^-(SajVrf +

(*A)

A=

z 2 2/s and

{x 2yz){Sx 2 yi

_,

dydz

1.27.

If <(#, y, z)

<Zi? 2

*~dV

dA

dt*

1.26.

Then

aa + aw

*-dir

dB

+ B^ + 3k.

Bji

17

6; 2i/3 2 j

2k.

(2i*

2x3yzk

- 3) j +

2a; 3 *k

{Qu2

- 4tt)k.

The given integral equals


I

{(3m2

- 3)j + (6m2 - 4M)k} du


2
3
(m3 - M)i + (m2 - 3m) j + (2m - 2M )k

- l)i +
=

(2m

u=l

- 3)j +

- Di +

(1

A particle moves along a curve whose parametric equations are

=
=

{(8
6i

- 2)i +
+ 8k

(4

- 6)j +

(16

- 8)k> -

{(1

(2

- 2)k}

VELOCITY AND ACCELERATION


1.28.

5 cos St where

t is

(6)
(a)

The

Then the

and the acceleration


a
t

= 0,

xi

yj

zk

3e~ 2t i

4 sin St

velocity is

At

= 0.

position vector r of the particle is

(6)

3e~ 2t y

the time.

Find its velocity and acceleration at any time.


Find the magnitudes of the velocity and acceleration at

(a)

dr/dt

dt/dt

= -6e- 2ti +

12 cos Zt

5 cos Bt

15 sin St

is

dv/dt

-6i

d2v/dt2

+ 12j

and

magnitude of velocity at

a
t

12e- 2t i

d2r/dt2

magnitude of acceleration at

is
*

36 sin St

12i-45k.

V(-6) 2
is

V(12)

(12) 2
2

45 cos 3*

Then

= 6^5

(-45) 2

3 V241

4 sin 3t,

VECTORS, VELOCITY AND ACCELERATION

18

1.29.

particle travels so that its acceleration is given

2e -ti

5 cos

at

by

- 3 sin t k
t =
and is moving

with a velocity
the displacement of the particle

(b)

any time t>0.


a
8

()

Integrating,

Since

dPr
-no

= 4i-3j + 2k
4i -

3j

Thus

5 cos

t j

3 sin

cos

k) dt

3 sin

ct

ct

6i

-k

3j

(1)

and integrating, we have


[(6

(6t

- 2e-*)i +

+ 2~*)i =

(6t

+ 2~*-l)i +

2i

3)j

(3 cos t

+ 8)j + (3
=
t
0, we have

(5 cos *

3j

2k

(5 sin *

located at (1,-3,2) at

is
i

(2e"'i

* j

+ 5 sin t j + 3 cos t k + 6i - 3j - k
2e-*)i + (5 sin t - 3)j + (3 cos * - l)k
(6

=
Since the particle

2e-'i + 5cos
2e_1
5 cos

-2-*i

(I)

~dt

=
=

Replacing v by dr/dt in

-jT

= 2e~H + 5 sin t j + 3
= 0, we have
+ 2k = -2i + 3k + c t
or

at

dv

Then

(6)

t j

If the particle is located at (1,-3,2) at time


given by 4i - 3j + 2k, find (a) the velocity and

[CHAP.

5j

(2

or

c2

5 cos

- 3)j +

l)k) dt

sin

c2

t)k + c 2

i
3 j + 2k

-i

2j

(3 sin

at

= 0,

so that

2k

-t+

2)k

{2)

RELATIVE VELOCITY AND ACCELERATION


1.30. An airplane moves in a northwesterly
direction at 125 mi/hr relative to the

ground, due to the fact that there is a


westerly wind [i.e. from the west] of
50 mi/hr relative to the ground. Determine (a) graphically and (b) analytically how fast and in what direction
the plane would have traveled if there
were no wind.
(a)

Graphically.

Let

=
=

Va

wind velocity
velocity of plane

with wind

V(,

velocity of plane

without wind.

Then

Vb
(6)

[see Pig. 1-27]

has magnitude

6.5

Fig. 1-27

Va = Vb + W or v 6 = va
units = 163 mi/hr and direction

-w

= V +

(-W).

33 north of west.

Analytically.

Letting

and

E and N

be unit vectors in directions

respectively,

that

= - 125

V
Then

= Va -

(- 125 cos 45

Thus the magnitude of


tan-i 88.39/138.39

cos 45

Vb

tan~i .6387

is

125 sin 45

50)i

+
2

125 sin 45

+ (88.39) 2 =

V(-138.39)
32 34' north of west.

and
j

we

W =

from

Fig. 1-27

50i

-138.391

164.2 mi/hr

see

88.39J.

and the direction

is

CHAP.

1.31.

VECTORS, VELOCITY AND ACCELERATION

1]

19

= 2ti - t j + (St - 4)k and


ri
given by
and (b) the relative
velocity
relative
the
Find (a)
instant where t = 2.
the
the
first
at
to
acceleration of the second particle with respect

Two particles have position


r 2 = (U 2 - 12* + 4)i + Fj - 3tk.
(a)

The

t-2

velocities of the particles at

Vl

rt

are respectively

= 2i-2tj +

(6t-4)k
t

v2

f2

- 12)i +

(lOt

vectors

St2}

2i

4j

8k

=2

3k
t

8i

12j

3k

=2

Relative velocity of particle 2 with respect to particle 1

=
(6)

y2

- Vl =

(8i

+ 12j - 3k) -

(2i

The. accelerations of the particles at


ai

a2

*i

v2

=2

vi

- 4j + 8k) =

r2

Ilk

-2j

10i

6i

16j

are respectively

~2j

lOi

6k
t

=2

=2

6tj

6k

12j

Relative acceleration of particle 2 with respect to particle 1

a2

a!

+ 12j) -

(lOi

(-2j

+ 6k) =

lOi

14j

6k

TANGENTIAL AND NORMAL ACCELERATION


1.32.

Given a space curve

with position vector

3cos-2ti'+ 3 sin 2t

(a)

Find a unit tangent vector

(b)

If r is the position vector of

case that v
(a)

to the curve.

a particle moving on

tangent vector to

The magnitude of

T =

= -6

ds/dt
to

is

8k

6 cos 2t

dx/dt

(a)

dr/dt

sin 2t)*

(6 cos 2*)*

(8)*

-6

dt
ds

sin 2t

+ f

sin 2*

cos 2t

4-

6 cos 2t
10

is

10

8k

since

= - 6 sin 2t i +
= (10)(- f sin 2*

6 cos 2t
i

+ f

cos2t

8k
j

$k)

= vT

in this case the speed of the particle along the curve is constant.

a unit tangent vector to a space curve C, show that dT/ds

Since

verify in this

is

dx/dt
ds/dt

dr/dt

This follows at once from

sin 2*

= V(- 6

= -1

If

t,

this vector is

Then a unit tangent vector

1.33.

at time

is

\dr/dt\

Note that

= vT.
dr/dt

(6)

+ (8*-4)k

a unit vector, we have

T*T =

T .f + *. T =
d8
da
which states that dT/ds

is

normal,

i.e.

2T

1.

M
d8

Then

is

normal to T.

differentiating with respect to

perpendicular, to T.

or

T.=
da

8,

we

obtain

VECTORS, VELOCITY AND ACCELERATION

20

If

is

a unit vector in the direction of dT/ds,

while

1.34.

call

R=

N
1/k

we have

the unit principal normal to C. The scalar


is called the radius of curvature.

From Problem

ds

ds/dt

(c)

From

(a), (6)

= R =

and Problem

1/K

sin 2t

to

^ sin It

1.33,

1 dT
=
N = KdF

Show

unit principal normal

25/3

n dT

.,

1.35.

called the curvature,

= V(-^cos 2 *) 2 + (-^sin2)2 =

is

ds

Radius of curvature

(c)

is

10

= Jj cos 2t

(b)

\dT/ds\

T = -f sin It i + cos 2t j + |k. Then


dT _ dT/dt _ (-6/5) cos 2t i - (6/5)

1.32,

Thus the curvature

Find the (a) curvature, (b) radius of curvature and


any point of the space curve of Problem 1.32.
(a)

= kS

dT/ds

and we

[CHAP.

^-rfT

x
-cos2*i-sm2j
.

that the acceleration a of a particle which travels along a space curve with

velocity

is

given by

= 3t T -+ff N

where T is the unit tangent vector to the space curve,


and R is the radius of curvature.
Velocity v

is its

unit principal normal

magnitude of v multiplied by unit tangent vector T, or


v

Differentiating,

dt

ti.
Then

dt

-^ =

But

*
dt

?I

ds_

ds

dt

^V m
T

= vT
=

{vT)

dt

dt

ds
dt

KXN

/vfi\

^T + ^

+ *(-;

=
=

tjN

KVS*

dv _
d*

T +
,

i;2

bn
T

This shows that the component of the acceleration is dv/dt in a direction tangent to the path and
v 2 /R in the direction of the principal normal to the path. The latter acceleration is often called
the centripetal acceleration or briefly normal acceleration.

CIRCULAR MOTION
1.36.

its position vector is given by r = cos o>t i + sin


j where
that (a) the velocity v of the particle is perpendicular to r,
(b) the acceleration a is directed toward the origin and has magnitude proportional
to the distance from the origin, (c) r x v = a constant vector.
o>

particle

is

moves so that

a constant.

dr

di

Show

~ ~ a sm ut
r

and

* "*"

" cos w *

=
=

Then

*'

[cos at

sin at

j]

(cos at)(a sin at)

and v are perpendicular.

[ sin at

w cos at

(sin at)(a cos at)

j]

CHAP.

VECTORS, VELOCITY AND ACCELERATION

1]

(b)7

^=^=
at2

-a2

at

cos at

a2 sin t

= -w2

[cos

<ot i

sin at

21

= -2r

j]

is opposite to the direction of r, i.e. it is directed toward the origin.


proportional to |r| which is the distance from the origin.

Then the acceleration


Its
(c)

magnitude

Xv

is

[cos at

sin at

X [ a

j]

a cos at

j]

cos at

sin at

sin at

(cos2 at

sin2 at)k

a constant vector.

k,

a COS at

sin at

Physically, the motion is that of a particle moving on the circumference of a circle with
constant angular speed a. The acceleration, directed toward the center of the circle, is the

centripetal acceleration.

GRADIENT, DIVERGENCE AND CURL


= x 2yz* and A = xzi-y2j + 2x2yk,
1.37. If
(e)

(<j>A),

'

curl

(6)

VA

(c)

VXA

dy

-^(x2yz3)i

}*

dz

'

dx

-(x*yz3)j

dy*

(^

(c)

x A,

3
2<ci/z i

****j

3x*y**k

X( ^i " 2/2J + 2x 2yk)

k
)

3/fla;

d/dy

d/dz

xz

y2

2x^y

(x

4*2/) j

(*A)

= V

2x 2 i

div (*A)

^ +^

curl (*A)

VX

(*A)

(x^yzH

A =

(a)

If

(b)

Find a

(a)

VXA

VX

1.38.

V'A,

dz

|^(asy*)k

- x*yHH +
+

(e)

(&)

= (l + j + k)-<^-lrt + **>

=
(d)

(a) V</>,

(<f>A).

\3

find

(x*yz*i

d/dy

d/dz

xSyz*

-x2y*z*

2ar%28

2 3
f-(2a*2/ z )
oz

*WJ + 2*

4 2 z3k)
i/

d/dx

(4x*yzs

2*4i/2z3k)

3 2i/3 2 )i

(43j/2 3

8x3 i/ 2z3 )j

(2xys z*

+ z?)i + (x 2 + 2y)j+{Sxz 2 -2)k, show


such that A = V/>scalar function
(2xy

<j>

d/dx

d/dy

d/dz

2xy

a2

2y

3s 2

=
2

that

o3 4 )k

VXA

0.

(d)

div

VECTORS, VELOCITY AND ACCELERATION

22

(b)

Method

A =

If

1.

(1)

= ^i +
ox

2xy

f
da;

we

Integrating,

V<&

f2j
dy

*3

^k
dz

x2

we must have

then

(*)

[CHAP.

2y

'& =

(3)

fly

3xz2

flz

find

U)

4>

+ F^x)
(5)
* = x2y +
= xz*-2z + F3 (x,y)
(6)

sfiy

xtfl

y2

+ F2 (x,z)

<f

Comparing these we must have F^y^)


= x 2 y + xzs + y 2 2z.
and so

y2

2z, F2 (x,z) =

xz*

2z, Fz (x,y) =

x*y

+ y*

<f>

Method

We

2.

have

if

A=

A-dr

an exact

differential.

<p

x 2y

^i + ^j + ^k^.^i + ^j + ^k)

H dx + % dy

fz

dz

d*

this case,

= A'dr =
=
=
-

d<f>

Then

For

v>,

+ z*)dx + (x 2 + 2y)dy + (Sxz2 -2)dz


[(2xy + zs dx + x 2 dy + Sxz2 dz] + 2y dy d(x 2y + xz*) + d(y2 + d(-2z)
d(x 2y + xz* + y2 - 2z)
(2xy

2 dz

+ xz s + y 2 2z.

Note that an arbitrary constant can

also be

added to

<f>.

LINE INTEGRALS AND INDEPENDENCE OF THE PATH


1.39.

If

A =

(Sx 2

- 6yz)i +

(2y

+ 3xz)j +

(1

- 4xyz 2 )k,

(1, 1, 1)

= t, y = t2

(a)

(b)

the straight lines

(c)

the straight line joining (0,0,0) and (1,1,1).

f A

(a)

If

J c A'dr from

evaluate

along the following paths C:

t,

z-i^.

dr

= t2

(0,0,0) to

(0, 0, 0)

{(Sx2

(Sx 2

A-dr
Jfo

from

*,

points

to

- 6yz)i +

(2y

-6yz)dx +
and

(0, 0, 0)

+ 3xz)j +

(2y

(1, 1, 1)

- 6(*2)(*3)} dt +

Jft=0

{3*2

f
J t=o

(3*2-6*5 )d*

then to

(0, 0, 1),

(4t?

(1

and then

(0, 1, 1),

- 4xyz 2 )k} '(dxi +

dyj

to

(1, 1, 1).

dz k)

+ Sxz)dy + (l-4xyz2 )dz


correspond to
{2*2

and

+ 3(*)(*3)} d(t2 +
- 12* 11

(St 2

=1

{1

+ 6t*)dt +

dt

respectively.

Then

- 4(*)(*2)(*3)2} d(*)
=

Another method.

A = (3*2 - 6*5)i +
+ 2<j + 3<2k) dt. Then

Along C,
dx

(i

J
(6)

A'dr

(3*2-

Along the straight line from (0,


to 1. Then the integral over

{3(0)

-6(0)(z)}0

(2*2

0, 0)

+ 3*4)j +

6*5) dt

to

(0, 0, 1),

(i

_ 49) k

(4*8

an d

+ 6*5) dt +

0,

= 0,

dx

(St2

+ yj + k =

- 12*") dt

= 0,

dy

*i

+ *2j + *3fc,
2

while z varies from

this part of the path is

{2(0)

+ 3(0)()}0 +

{1

- 4(0)(0)(z2)} dz

dz

CHAP.

VECTORS, VELOCITY AND ACCELERATION

1]

Along the straight line from (0, 0, 1) to (0, 1, 1), x = 0,


from
to 1. Then the integral over this part of the path

= 1,

23

= 0,dz =

dx

r1

Jfy =o

{3(0)

- 6(y){l)}0 +

{2i/ + 3(0)(1)} dy +

Along the straight line from (0,


from to 1. Then the integral over

{3*2

- 6(1)(1)} dx +

{2(1)

1, 1)

{1

- 4(0)(i/)(l) 2 }0

= l,z
path is

to (1, 1, 1),

= 1,

= 0,dz =

dy

- 4*(1)(1) 2 >0 =

and (1,1,1) we have x

= t,

{1

2ydy

^y=o

this part of the

+ 3z(l)}0 +

while y varies

is

while x varies

(3* 2

- 6) dx = -5

= t,

x=0

Adding,

dr

+ 1-5 =

-3.

Jr.

Along the straight


dx = dy = dz = dt,

(c)

line joining (0,0,0)

( A'dr

(3t2

(2

(3x 2

-6yz)dx +

(2y

+ Sxz) dy +

(2t

+ l-4t*)dt

-6t*)dt

(1

+ St2 )dt +

- 4xyz2

= t. Then

since

dz

(l-4t*)d

6/5

Note that in this case the value of the integral depends on the particular path.

1.40.

A =

If

+ z?)i +

(2xy

+ 2y)j +

(3zz 2

-2)k show that

J A -dr

(a)

joining the points (1,-1,1) and (2,1,2) and

of the path

(x 2

By Problem

VXA =

1.38,

independent of the path aijd

its

or
value

dr

d<p

d(x 2y

A'dr

independent

(b) find its value.

+ xz* + y 2 - 2z). Then

the integral

is

is
/(2.1.2)

(2,1,2)

is

d(x*y

+ xz3 + y 2 - 2c)

*^(i,-i.D

ii.-i.i)

(2.1,2)

x2 y

xz*

+ |/2-22

18

(1,-1.1)

MISCELLANEOUS PROBLEMS
1.41.

Prove that

if

Suppose x
to the

1.42.

same

a and b are non-collinear, then xa + yb

= 0.

Then

a?a

line (collinear),

+ yb =

implies xa

Let

bi-

ABCD be the given parallelogram with diagonals


P

as shown in Fig. 1-28.

BD + a = b, BD = b - a. Then BP = x(b - a).


Since AC = a + b, AP = y(a + b).
But AB = AP + PB = AP - BP,
a = i/(a + b) x(b a) = (x + y)a + (y x)b.
Since

i.e.

Since a and b are non-collinear


is

we have by Problem
x = y = % and P

x + y = 1 and y x = 0, i.e.
the midpoint of both diagonals.

1.41,

implies

-y=

0.

= yb or a = (y/x)b, i.e. a and b must be parallel


Thus x = 0; then 2/b = 0, from which y = 0.

contrary to hypothesis.

Prove that the diagonals of a parallelogram


sect each other.
intersecting at

VECTORS, VELOCITY AND ACCELERATION

24

1.43.

[CHAP.

Prove that for any vector A,

A =
A =

(a)

(b)

(A-i)i

A(cos a

cos

where a, /?, y are the angles which


makes with
cos y are called the direction cosines of A.
(a)

We

A = A + A 2 + A 3k.

have

ti

+ (A-k)k
/? j + cos y k)

(A-j)j

i, j,

and cos a, cos (3,

respectively

Then

Ai =
Aj =

= A
+ 2 j + A 3k)
+ Aaj + Agk)'! = A 2
A'k = (Aii + AaJ + AgkJ-k = A 3

A =

Thus

(A x i

(A

=
Ak

(b)

Prove that V<

is

(A

j)j

cos a

|i|

|A||j|cos/?
|A| |k|

cos y

(A k)k

= A cos a
= Acos/3
= A cos y

(a),

A =
1.44.

i)i

|A|

Aj
Then from part

(Aji

(A

i)i

(A

j)j

(A k)k

A(cosa

</>(#, 2/,

2)

a vector perpendicular to the surface

cos/3

cosy k)

c,

where

c is

constant.
Let r

Then

dv

*
i.e.

1.45.

V#

dr

vector to any point P(x, y,

so that

Find a unit normal


By Problem

1.44,

is

k
i
ff + lf i+ lf

or

2# 2 + 4yz - 5z 2

to the surface

Then a unit normal

4ai

4zj

to the surface at

1.46.

(&i +

<

"' i

But

+ &k)

(Ay

to the surface at

- 10z)k =
12i

is

= -10

at the point P(3,-l,2).

is

V(12)

Another unit normal

on the surface.

perpendicular to dr and therefore to the surface

a vector normal to the surface

+ Ayz - 5z2

V(2ar2

V^

z)

in the plane tangent to the surface at P.

^ dx + ^ dy+ ^ dz

+ j/j + zk be the position


dx i + dy j + dz k lies

xi

3i

12i

8j

+ 8j - 24k
+ (8) 2 + (-24)2

2j

24k
3i

at (3,-1,2)

2j

6k

6k

is

ladder AB of length a rests against a vertical wall OA [Fig. 1-29]. The foot B
{a) Show that the midpoint
of the ladder is pulled away with constant speed vo.
with center at O. (b) Find
radius
a/2
of the ladder describes the arc of a circle of
ladder
at
the
instant where B is distant
of
the
the velocity and speed of the midpoint
b < a from the wall.

(a)

Let r be the position vector of midpoint


If angle OBA = o, we have

M of AB.

OA = a sin e j
OB = a cos e

=
=
a
cos i a sin
OB OA
AB
i,

Then
r

= OA + AM = OA + AB
= a sin e j + \(a> cos e a sin e j)
= ^a(cos 9 + sin e j)
= ^a, which is a circle of radius
i

Thus

|r|

with center at 0.

a/2
Fig. 1-29

CHAP.

(6)

AND ACCELERATION

VECTORS, VELOCITY

1]

The

-jr

where

The

is

-jrtya(co& 8

velocity of the foot

is

-r-(OB)
at

the instant where

of the ladder

ei

cos 8 8j)

(i)

distant 6

62

the required velocity of

(1)

a sin

or

8i

from the wall we have from

Va -

am 8 =

is

= a&m8

37 (a cos 0i)
at

Thus from

= ^a( sin e

sin e j)}

d8/dt.

vA =
At

velocity of the midpoint

25

dr

-v

()

(2),

-n

y a2 _

a sin*

at this instant
1

= -v

8 8

b2

is

y/cP-b*
and

1.47.

Let

speed

its

av/2 Va2

is

o2

represent the polar coordinates describing the position of a particle. If


a unit vector in the direction of the position vector r and $i is a unit vector
perpendicular to r and in the direction of increasing 6 [see Fig. 1-30], show that
(r, 0)

ri is

(a) ri
i

(6)

(a)

cos 9
cos

i 4-

ri

sin 6

j,

0i

sin

0i,

If r is the position vector of the particle at

= sin 6 i +

sin

ri

cos

cos 6 $i

any

time t, then dr/dr is a vector tangent to the


curve 8 = constant, i.e. a vector in the direction of r (increasing

direction

is

r).

unit vector in this

thus given by

5r /Idrl

dr/

(1)

\dr\

Since

xi

yj

rcosfli

cos*

\dr\

rsintfj

we have

as seen from Fig. 1-30,


3r

dr

sin

j,

\dr\

so that

cos 8

sin 8

Fig. 1-30

(3)

Similarly, Br/d8 is a vector tangent to the curve r


direction is thus given

Now from
so that

(-4)

(2),

dr

do

...
= r sin
8i +

constant.

unit vector in this

rcosff

3r

j,

\de

yields
8X

(b)

by

= sin 8

cos 8

These results follow by solving the simultaneous equations

(5)

(3)

and

(5)

for

and

j.

VECTORS, VELOCITY AND ACCELERATION

26

1.48.

(a)r a

Prove that
Prom

(a)

(S)

(b)di

00i

= -0r u

we have

of Problem 1.47
Tl

dr dt

dt

=
From

(b)

(5)

1.49.

_
~

(0)(f )

_
~
=

d*i

dt

Prove that in polar coordinates

d*idr

(b)

the acceleration

is

We

have

by Problem
(6)

From

part

cos e

})($)

e0 t

^ide_

de dt

(0)(r)

(a)

( cos

the velocity

is

= er x

sin e j)(e)

given by

r$6i

rri 4-

r - r6 2 )ri

(r 6

so that

rr 1

given by
a

(a)

(- sin

dr dt

v =
and

de dt

we have

of Problem 1.47
01

[CHAP. 1

dr

Tt

dr

= Tt r +

^r i

-dT

+ 2r9) 9

...
^+

rr

rr i

+ *"i

1.48(a).

(a)

and Problem 1.48 we have


d .*
= dv

Tt =

(rr

de

l)

= rr t + rr x + r0x + rtf*! + rebi


= Vr! + f(5#i) + ffoi + r'e'9 + (re^eri)
= (r r$ 2 )r + (r6>'+2r)0
1

Supplementary Problems
VECTOR ALGEBRA
1.50.

Given any two vectors

1.51.

Given vectors A,

1.52.

If
is

1.53.

(a)

and B,

and C, construct the vectors

Determine the vector having

2A - 3B + C,

(6)

C - A + JB.

to the side

AB.

point (2,-1,3) and terminal point (3,2,-4).

initial
(a).

triangle has vertices at the points

median

1.55.

(a)

4A+3(B-A) = A + 3B.

A and B are any two non-zero vectors which do not have the same direction, prove that
a vector lying in the plane determined by A and B.

distance between the two points in

1.54.

illustrate geometrically the equality

Ans.

(a)

+ 3j 7k,

(6)

Find the

\/59

A(2,l,-1), (-1,3,2), C(l,-2,1).

Find the length of the

Ans. \/66

travels 25 miles northeast, 15 miles due east and 10 miles due south.
appropriate scale determine (a) graphically and (6) analytically how far and in
Ans. 33.6 miles, 13.2 north of east.
he is from his starting position.

A man

(6)

pA + qB

By

using an

what

direction

CHAP.

1.56.

VECTORS, VELOCITY AND ACCELERATION

1]

27

A =

Find a unit vector in the direction of the resultant of vectors


Ans. (61-21 +
C = 3i-2j + 4k.

2i

j + k, B =

+ j + 2k,

7^/^

THE DOT OR SCALAR PRODUCT


1.57.
Evaluate |(A + B) (A-B)|

if

1.58.

1.59.

Find a so that
If

A =

2i

2i

3j + 5k

+ j + k, B =

1.61.

- 2j + 2k

3i

- 4j + 2k,

Ans. 24

Ans. a

Take the

ABC, i.e. c 2 = a2 + b 2 2ab


A,B,C where C = A B. Then use

sides as

in the

cos C.

OC

(A

B)

|(2A

find

+ B) X

Find a unit vector perpendicular to the plane of the vectors

(A-2B)|.

(A

B).]

A=

3i

Ans. 2byfl

2j + 4k and B = i + j 2k.

+ k)/V5

1.65.

Find the area of the triangle with vertices

1.66.

Find the shortest distance from the point (3,2,1) to the plane determined by
Ans. 2
(-1,0,2).

t>

/.ft

A+C

Prove that the diagonals of a rhombus are perpendicular to each other.

Ans. (2j

1.67.

4/3

find the projection of

Prove the law of cosines for triangle

THE CROSS OR VECTOR PRODUCT


1.63.
If A = 2i-j + k and B = i + 2j-3k,
1.64.

+ j-2k.

are perpendicular.

C =

and

3i

to side

[Hint.
1.62.

+ aj 2k

3i

B =

and

has vertices at A(2,3,l), B(-l, 1,2), C(l,-2,3).


Find the acute angle which the
AC makes with side BC.
Ans. cos -1 V91/14

triangle

median

and

+ 5k

2i-3j

Ans. 17/3

direction of B.
1.60.

A =

j.
*
x
i
a ns,
Prove 0.1.7
the law of sines for triangle ABC,

Consider the sides to be A, B,

[Hint.

sides with

and

Ans. \yf%

(2,-3,1), (1,-1,2), (-1,2,3).

sin

i.e.

A =

sin B

C where A + B + C =

sin

(1,1,0), (3,-1,1),

and take the cross product of both

respectively.]

TRIPLE PRODUCTS
1.68.

If
(c)

1.69.

1.70.

A = 2i + j - 3k, B = i - 2j + k
A X (B X C), (d) (A X B) X C.
A

Prove that

(B

C)

(A X B)

and

C = -i + j - 4k,

Ans.

C,

i.e.

(a)

20,

(6)

find

20,

(c)

(a)

8i

(B

C),

- 19j - k,

(b)

{d)

25i

(A X

B),

- 15j - 10k

the dot and the cross can be interchanged.

Find the volume of a parallelepiped whose edges are given by


Ans. 31

A =

+ 3j k, B =

2i

2j + 2k,

C = 3i-j-2k.
1.71.

Find the volume of the tetrahedron with vertices at

(2, 1, 1),

(1,

1, 2),

(0, 1,

-1), (1 -2,

1).

Ans. 4/3
1.72.

Prove that

(a)
(b)

1.73.

(a)
(r

A-(BXC) = B(CXA) =a C
A X (B X C) = B(A C) - C(A

(A X B),

B).

P 1 ,P 2 ,P3 respectively. Prove that the equation


where r = xi + yj + zk, represents an equation for the plane
(2,-1,-2),
(&) Find an equation for the plane passing through
Ans. (b) 2x + y-3z = 9

Let r v r2 r3 be position vectors to three points


,

rj)

[(r

r2 X (r r3 =
by P u P2 and P3
)

0,

)]

determined
(-1,2,-3), (4,1,0).

DERIVATIVES AND INTEGRALS OF VECTORS


1.74.

1.75.

Let

A =

Ans.

(a)

If r

3fi

(t 2

3i-3j-k,

a cos ut

scalar, prove that

+ t)j +
(6)

sin at,
(a) r

- 2t2 )k.
+ 2k

(t*

-2j

Find

(a)

dA/dt

and

(6)

cPA/dt2

at

= 1.

where a and b are any constant non-collinear vectors and


dr/dt = <o(a X b), (6) d2r/dt2 + <o2r = 0.

is

a constant

VECTORS, VELOCITY AND ACCELERATION

28

A =

1.76.

If

1.77.

Prove that

1.78.

1.79.

sin

B =

and

cos

t i

sin

t j

+ -r-XB
t-(AXB) = AX-rdu
du
'

du

= 4(tt - l)i - (2u + 3)j +


- 8j + 38k, (b) -28

A(m)

If

Ans.

t\

Gu 2k,

k,

find

where

evaluate

(a)

(a) 6i

[CHAP.

^(AB).

and

J2

Ans.

sin

are differentiate functions of u.

A(u) du,

Find the vector B(u) such that d2B/du 2 = Gul - 48w2j + 12k where
Ans. (us + u + 2)i + (5u - 4u4 )j + (Gu2 - 3)k
+ 5j for u = 0.

(b)

B =

(ui

J*
2i

- 2k)

and

dB/du

- 3k

A(u) du.

Xd?A
AX

1.80.

1.81.

1.82.

-j-^

R =

If

x 2 yi

dA
= A X -rr
+

dt

32

- 2y 2 zj + xy 2 z>k,

A = x*i - y] + xzk
Ans. 4i + 8j
If

find

B =

and

aa; 2

where

fl

is

a constant vector.

Ans. IGyfh

at the point (2,1,-2).

3j/ 2

yi

+ xj-xyzk,

find

3
^y(AXB)

at the point (1,-1,2).

VELOCITY AND ACCELERATION


1.83.

particle

moves

(a) velocity,

at time

tion

1.84.

the

= 2.

Ans.

(c)

(a)

Find the
r = (t2 + *)i + (8* - 2)j + (2t - 4t2 )k.
space curve
speed or magnitude of velocity and (d) magnitude of accelera5i

+ 3j + 8k,

(6)

21

16k,

(c)

ly[2,

(d)

2^

moves along the space curve defined by x = e-* cost, y e~ t


the magnitude of the (a) velocity and (6) acceleration at any time t.

particle

Ans.

1 .85.

along

acceleration,

(6)

(a)

V3~e _t

(6)

sin

t,

e~ t

Find

VEe-*

2
position vector of a particle is given at any time t by r = a cos at i + 6 sin ut j + ct k.
of
the
magnitude
the
time
with
increases
particle
the
of
speed
the
(a)
acceleration is always constant. (&) Describe the motion of the particle geometrically.

The

Show that although

RELATIVE VELOCITY AND ACCELERATION


1.86.

2
two particles are given respectively by r x = ti \ + (2t + 3)k and
of the
acceleration
relative
the
and
velocity
relative
(&)
the
Find
(a)
r2
Ans. (a) -5i + 6j-5k, (6) -6i + 2j-6k
second particle with respect to the first at t = l.

The

1.87.

position vectors of

(2t - 3 2 )i + 4tj - %.

An automobile driver traveling northeast at 26 mi/hr notices that the wind appears to be coming
from the northwest. When he drives southeast at 30 mi/hr the wind appears to be coming from
60 south of west. Find the velocity of the wind relative to the ground.
Ans. 52 mi/hr in a direction from 30 south of west

1.88.

the other
a boat on one side of a river wishes to reach a point directly opposite him on
Assuming that the width of the river is D and that the speeds of the boat and
at an angle
current are V and v < V respectively, show that (a) he should start his boat upstream
2 - v2
D/y/V
is
the
river
1
cross
to
time
the
and
shore
(6)
the
of sin- (v/V) with

A man

in

side of the river.

TANGENTIAL AND NORMAL ACCELERATION


1.89.

1.90.

given
Show that the tangential and normal acceleration of a particle moving on a space curve are
initial point
by d2 s/dt2 and K (ds/dt) 2 where s is the arc length of the curve measured from some
and k is the curvature.
unit tangent T, (6) principal normal N,
2
k to the space curve x = t, y = t /2, z = t.

Find the

Ans.

(a)

(a)

(i+tj

+ k)/v^T2,

(&)

(-ti

(c)

+ 2j - tk)/VW+4,

radius of curvature

(c)

(t 2

+ 2f' 2lyf2,

R
(d)

and

(d)

V2/(*

curvature

+ 2) 3

'2

CHAP.

1.91.

VECTORS, VELOCITY AND ACCELERATION

1]

moves in such a way that its position vector at any time t is r = i + |^ 2j + ik.
the velocity, (6) the speed, (c) the acceleration, (d) the magnitude of the acceleration,
the magnitude of the tangential acceleration, (/) the magnitude of the normal acceleration.

particle

Find
(e)

(a)

Ans.
1.92.

29

(a)

*j

+ k,

(6)

y/W+2,

(c)

(d)

j,

Find the (a) tangential acceleration


the ellipse
r = a cos at i + b sin at

and

t/V& +

(e)

(6)

normal acceleration of a particle which moves on

(a)

_ fr2) sm at cos at
Va sin 2 at + b 2 cos 2 u
,

2,

(/)

j.

M 2( a2

An8.

V2/V^T2

1,

a2 ab
(6)

sin 2 <ot

6 2 cos2 at

CIRCULAR MOTION
1.93.

particle

moves

angular speed,
Ans.
1.94.

in a circle of radius 20 cm.


its

(6)

(a) 2 radians/sec,

angular acceleration,
radians/sec 2

(6)

its

(a)

its

normal acceleration.

80 cm/sec 2

(c)

moving on a circle of radius


from rest, show that after time
covered is 8 = ^Rat2
particle

starts

If its tangential speed is 40 cm/sec, find

(c)

has a constant angular acceleration


(a) its angular velocity is a at,

If the particle
the arc length

a.

(b)

1.95.

particle

moves on a

circle of radius

1.96.

Ans.

at the point (1,-1,1).


If

<p

xy

+ yz + zx

at the point

1.99.

a).

If the particle in Problem 1.95 is travelling at 3600 revolutions per minute in a circle of radius
100 cm and develops a constant deceleration of 5 radians/sec2 , (a) how long will it be before it
comes to rest and (6) what distance will it have travelled?
Ans. (a) 75.4 sec, (6) 1.42 X 10e cm

GRADIENT, DIVERGENCE AND CURL


1.97.
If A = xzi+(2x 2 -y)j-yz 2k and

1.98.

At time t = it starts
Show that (a) it comes to

with constant angular speed

down

so that its angular acceleration is a (or deceleration


rest after a time u /a and (6) has travelled a distance Raf,/2a.

to slow

Show

1.101.

Prove that

1.102.

If

that

A =

(r2 r)

x 2y\

Ans.

= V A+ V

VX
(a)

and

2).

U,V,A,B have

Prove that if
(b) V (A + B)

1.100.

-1,

(3,

(a)

div curl

B,

+ yH\ + z 2 xk,

(a) 25,

(b)

56i

(c)

V^,

(a)

-i

and

xi

grad

A = (2x 2 -yz)i + (y 2 -2xz)j + x 2z*k


A =
and curl grad $ 0.

<j>

and

<p

and

VXA

(c)

(c)

(V#) X

- 30j + 47k

+ yj + zk and

curl

(6)

VA

(6)

+ j + 4k

(a)AV^, (6)^VA and

find

(c)

2,

find
2,

continuous partial derivatives, then


(c) V X (A + B) = V X A + V X B.

where

A=

= $x 2y + yW,
+ j + 3k, (6)

<f>

-6i

(o)

V(17

+ V) = V17+VV,

|r|.

under suitable conditions on

x 2y

- Sxz2 + 2xyz,

show

and

<p.

directly

that

div curl

A =

1.103.

If

1.104.

(a)

1.105.

Prove:

3z 2 i

- yzj + (x + 2z)k,
VX

Prove that
Problem 1.103.
(a)

VX

(V X A)

(UA)

= -V 2A+ V(V
X

(VC7)

Ans. -6xi

curl curl A.

find

A),

A+ U{V X A).

(b)

+ (6*-l)k

Verify the result in

(a) if

is

as given in

V (AX B) = B (V X A) - A' (V X B).

(6)

LINE INTEGRALS AND INDEPENDENCE OF THE PATH


1.106.

If

F =

(3aj

- 2y)i + (y + 2z)j - x 2k,

consisting of:
straight lines
z

= y2

the curve x

= t,

evaluate

t2 ,

= 3

>

dr from

(0, 0, 0)

to (1, 1, 1),

where

is

a path

(c) the
(6) a straight line joining these points;
from (0,0,0) to (0,1,0), then to (0,1,1) and then to (1,1,1); (d) the curve x = z2
Ans. (a) 23/15, (6) 5/3, (c) 0, (d) 13/30

(a)

VECTORS, VELOCITY AND ACCELERATION

30

1.107.

Evaluate
to (2,1,3),

by x 2
1.108.

Find
x

1.109.

(a)

(b)

= 4y,

t,

A =

If

the space curve x = 2t2 y


= 8z from x = to x = 2.

F =

where

i/

3x 2 i -K (2xz
,

3x*

Fdr

4>

2 cos

A-dr where A =

3 sin

{Axy

t,

4t2

Ans.

3#)i + (y 2x)i

(a;

t,

y) j + zk

from

to

along

16,

and

= 2ir.

to

14.2,

prove that

A'dr

is

independent of the

is

the curve from the

C joining two given points. (6) Evaluate the integral in (a)


points (1,-1,1) to (2,-2,-1).
Ans. (b) -19
Determine whether
x 2zj + x 2yk,
determine

<j>

Ans.

1.111.

(a)

2xzi

(b)

Jc
+

dr

(x 2

y)j + (2z x 2 )k.

A=

such that

(J)

where

dr

Let l^m^n! and


such that cos

is

Z2

C joining any

In the case where

<f>

E=

x 2 yz

+ c;

(b)

IJ2

find

+m

m2 + n %2
1

Show that

1.115.

Prove that

1.116.

If A,

not

xtA

xx

1.117.

(A B) 2

= A2 B2

B and C are non-coplanar vectors [vectors which do


+ y{B + ]C = x 2 A + y 2B + z 2 C, prove that necessarily

1.119.

Find a unit normal to the surface x 2y

is parallel to

the third side

in the

lie

all

x2

y\

y%,

zx

same

z2

and

plane]

is

a right angle.

- 2xz + 2y 2 z^ =

at the point

10

(2, 1,

-1).

+ 4}-6k)/^

Ans. (Zi
1.120.

m ^
^
Prove that

1.121.

If A(w) is

1.122.

Prove

1.123.

If

1.124.

between them

the angle

Let ABCD be any quadrilateral and points P, Q, R and S the midpoints of successive sides. Prove
that (a) PQRS is a parallelogram, (6) the perimeter of PQRS is equal to the sum of the lengths
of the diagonals of ABCD.

Prove that an angle inscribed in a semicircle

<

1.118.

2xyzi

Prove that the line joining the midpoints of two sides of a triangle
and has half its length.

A=

and B.

1.114.

(A X B) 2

(a)

independent of the path,

is

it

if

Ans.

rr.

>2 w2 be direction cosines of two vectors.

two points

dependent on path

MISCELLANEOUS PROBLEMS
1.112.
If A X B = 8i - 14j + k and A + B = 5i + 3j + 2k,
Ans. A = 2i + j-2k, B = 3i + 2j + 4k
1.113.

if

V#.

Independent of path,

Evaluate

independent of the path

is

the curve defined

(c)

16

6tt

curve

1.110.

= l,

(c)

the closed curve in the xy plane,

is

Ans.

- Zx2z 2 )i + (4y + 2ar 2 )j + (1 - 2xs z)k,

(6)

the straight line from (0,0,0)

(a)

t from

(a)

[CHAP.

dA
dA =
A -^
A. -^
.

a differentiate function of u and

V-(*A) = (V*)

AXB = AX

does

C,

A + <f>(V
B=C

|A(te)|

1,

prove that dA/du

is

perpendicular to A.

A).

necessarily?

Explain.

man on this ship observes that another ship


ship is traveling northeast at 15 miles per hour.
located 5 miles west seems to be traveling south at 5 miles per hour, (a) What is the actual
velocity of this ship? (6) At what distance will the two ships be closest together?

(A X B)

1.125.

Prove that

1.126.

Solve the equation

Ans. r

(v t

(C

X D) +

d2 r/dt2

^gt 2 )k

(B

C)

(A X D)

= -gk where

is

(C

X A)

(B

X D) =

a constant, given that r

0.

= 0,

dr/dt

vjs.

at

= 0.

CHAP.

1]

1.127.

If

1.128.

1.129.

VECTORS, VELOCITY AND ACCELERATION

<t>

+ 2 2 )- 1/2

2
2/

V 2^ = V

show that

(Vtf>)

at all points except (0,0,0).

The muzzle velocity of a gun is 60 mi/hr. How long does it take for a bullet to travel through the
Ans. .05 sec
gun barrel which is 2.2 ft long, assuming that the bullet is uniformly accelerated?
25 foot ladder AB rests against a vertical wall OA as in Fig. 1-29, page 24. If the foot of the
ladder B is pulled away from the wall at 12 ft/sec, find (a) the velocity and (6) the acceleration
of the top of the ladder A at the instant where B is 15 ft from the wall.

Ans.
1.130.

( 2

31

downward,

9 ft/sec

(a)

Prove that (a)|A

+ B|^

|A|

downward

11.25 ft/sec 2

(6)

|B|,

(6)

|A+B + C| ^

|A|

|B|

Give a possible geometric

|C|.

interpretation.
1.131.

from rest with uniform acceleration. After 10 seconds it has a speed of 20 mi/hr.
far has it traveled from its starting point after 15 seconds and (6) what will be its speed
Ans. (a) 330 ft, (6) 30 mi/hr
in mi/hr?
train starts

How

(a)

1.132.

Prove that the magnitude of the acceleration of a particle moving on a space curve
y/(dvldt)*

where v
1.133.

If

is

the tangential speed and

is

the radius of curvature.

is

the unit tangent vector to a curve

C and A

JA'dr
c
where
1.134.

If

s is the

A =

(2x

a vector

is

J\c

field,

prove that

A'Tds

arc length parameter.

- y + 4)i + (5y + Sx - 6)j,

evaluate

A'dr

4>

Ans. 12

(0,0,0), (3,0,0), (3,2,0).


1.135.

+ vVR

is

around a triangle with vertices at

automobile driver starts at point A of a highway and stops at point B after traveling the
in time T. During the course of the trip he travels at a maximum speed V. Assuming
distance
that the acceleration is constant both at the beginning and end of the trip, show that the time
during which he travels at the maximum speed is given by 2D/V T.

An

1.136.

Prove that the medians of a triangle (a) can form a triangle,


length of each median in the ratio two to one.

1.137.

If a particle

curvature of

1.139.

(a)

1.140.

Prove that the equation


Show that one solution

X = B

Find

all

Ans.

X =

a point which divides the

^-r
|vXa|

Prove that the area of a triangle formed by vectors A,

(c)

in

has velocity v and acceleration a along a space curve, prove that the radius of
path is given numerically by

1.138.

Ans.

meet

its

R =

(6)

(b)

vectors

x A/A 2

AXX

+ XA

such that

pA/A 2 +

VXA

= B

can be solved for

X = B X A/A 2

is

where X

B and Cis |AXB +

is

A X = p.
V is an

(c)

any

Can you

if

and only

if

BXC + CXA|.

AB =

and

A^0.

find the general solution?

scalar.

where

arbitrary vector.

inside a triangle three lines are constructed parallel respectively to each of


the three sides of the triangle and terminating in the other two sides. Prove that the sum of the
ratios of the lengths of these lines to the corresponding sides is 2.

1.141.

Through any point

1.142.

If T,

and

B = TXN

a space curve r =

r(u),

are the unit tangent vector, unit principal normal and unit binormal to

assumed differentiate, prove that

dT
Is-

= kN

dB
'

*t
~ tN

<
'

Is-

-i'
- tB ~ kT

These are called the Frenet-Serret formulas. In these formulas k is called the curvature, r is the
and their reciprocals R = 1/k, o = Vt are called the radius of curvature and radius of

torsion

torsion.

VECTORS, VELOCITY AND ACCELERATION

32

1.143.

[CHAP.

In Fig. 1-31, AB is a piston rod of length I. If A moves along horizontal line CD while B moves
with constant angular speed w around the circle of radius a with center at O, find (a) the velocity
and (6) the acceleration of A.

P
P

l-S
l

Fig. 1-32

Fig. 1-31

1.144.

boat leaves point P [see Fig. 1-32] on one side of a river bank and travels with constant velocity
in a direction toward point Q on the other side of the river directly opposite P and distance D
is the angle between r and PQ,
from it. If r is the instantaneous distance from Q to the boat,
and the river travels with speed v, prove that the path of the boat is given by

_
~
1.145.

If v

=V

L146.

(a)

Prove that in cylindrical coordinates

in

Problem

1.144, prove that the

path

<t>

tan

[see Fig. 1-33] the position vector is


i

p sin

(6)

Express the velocity in cylindrical coordinates,

(c)

Express the acceleration in cylindrical coordinates.

Ans.

(6)

(c)

= ppx 4- p$$\ + k
= (p- P* 2 )pi + (p'4+

2p)tfi

e) v/v

an arc of a parabola.

is

(p,<j>,z)

p cos

sec 6

(sec e

<f>

zk

+ *k

Illlplllilplllliplim

1.147.

(a)

p'v.

Cylindrical coordinates

Spherical coordinates

Fig. 1-33

Fig. 1-34

Prove that in spherical coordinates


r

(r, $,

r sin e cos

<j>)

<f>

r sin 9 sin #

Express the velocity in spherical coordinates,

(c)

Express the acceleration in spherical coordinates.


(6)

(c)

=
=

rt x
(V

+ rb x + rj> sin $ f t
- rff 2 r 2 sin 2 tf)r x + (2re + rV r^ 2
sin 0)#
+ (2re$ + 2r# sin 6 +
r'<p

1.148.

(6)

Ana.

the position vector

[see Fig. 1-34]

sin

r cos

is

cos *)!

Show that if a particle moves in the xy plane the results of Problems 1.146 and 1.147 reduce to
those of Problem 1.49.

WORK, ENERGY and MOMENTUM

NEWTON'S LAWS
The following three laws of motion given by

Sir Isaac

Newton are considered the

axioms of mechanics:
1.

Every
(i.e.

2.

If

particle persists in a state of rest or of uniform motion in a straight line


with constant velocity) unless acted upon by a force.

is

the (external) force acting on a particle of mass


velocity v, then

which as a consequence

is

moving with

F = 5<""> =
where p = mv

is called

the

momentum.

If

is

%
independent of time

this

becomes

dx

F = m-jT = ma
where a
3.

is

(#)

the acceleration of the particle.

on particle 2 with a force Fi 2 in a direction along the line joining


the particles, while particle 2 acts on particle 1 with a force F2 i, then F2 i = 12
In
other words, to every action there is an equal and opposite reaction.

If particle 1 acts

DEFINITIONS OF FORCE AND MASS


The concepts

of force and mass used in the above axioms are as yet undefined, although
idea of mass as a measure of the "quantity of matter in an object"
and force as a measure of the "push or pull on an object".
can however use the above
intuitively

we have some

We

axioms to develop definitions

[see

Problem

2.28,

page

49].

UNITS OF FORCE AND MASS


Standard units of mass are the gram (gm) in the cgs (centimeter-gram-second) system,
kilogram (kg) in the mks (meter-kilogram-second) system and pound (lb) in the fps (footpound-second) system. Standard units of force in these systems are the dyne, newton (nt)
and poundal (pdl) respectively. A dyne is that force which will give a 1 gm mass an acceleration of 1 cm/sec2
A newton is that force which will give a 1 kg mass an acceleration of
A poundal is that force which will give a 1 lb mass an acceleration of 1 ft/sec2
1 m/sec2
For relationships among these units see Appendix A, page 341.
.

INERTIAL FRAMES OF REFERENCE. ABSOLUTE MOTION


all

It must be emphasized that Newton's laws are postulated under the assumption that
measurements or observations are taken with respect to a coordinate system or frame

33

NEWTON'S LAWS OF MOTION. WORK, ENERGY AND MOMENTUM

34

[CHAP. 2

of reference which is fixed in space, i.e. is absolutely at rest. This is the so-called assumpIt is quite clear, however, that a particle can be
tion that space or motion is absolute.
at rest or in uniform motion in a straight line with respect to one frame of reference and
be traveling in a curve and accelerating with respect to another frame of reference.

We

Newton's laws hold in one frame of reference they also hold in


any other frame of reference which is moving at constant velocity relative to it [see
Problem 2.3]. All such frames of reference are called inertial frames of reference or
Newtonian frames of reference. To all observers in such inertial systems the force
acting on a particle will be the same, i.e. it will be invariant. This is sometimes called
can show that

if

the classical principle of relativity.

The earth is not exactly an inertial system, but for many practical purposes can be
considered as one so long as motion takes place with speeds which are not too large. For
For speeds comparable with the
non-inertial systems we use the methods of Chapter 6.
speed of light (186,000 mi/sec), Newton's laws of mechanics must be replaced by Einstein's
laws of relativity or relativistic mechanics.

WORK
If a force F acting on a particle gives it a
displacement dr, then the work done by the
force on the particle is defined as

dW =
since only the

of dr

F-dr

component of F

is effective in

(3)

in the direction

producing the motion.

The total work done by a force field (vector


F in moving the particle from point Pi

field)

P2 along the curve C of Fig. 2-1 is


given by the line integral [see Chap. 1, page 9].

to point

W
where

ri

and

F-dr

Fig. 2-1

F-dr

*yPi

and

r 2 are the position vectors of Pi

P2

dr

(*)

"n

respectively.

POWER
The time

rate of doing

work on a

particle is often called the instantaneous power, or

Using the symbols

briefly the power, applied to the particle.

power respectively we have

is

and

<P

for

work and

r
If

dt

the force acting on a particle and v


<P

is

the velocity of the particle, then

Fv

w
we have
(6)

KINETIC ENERGY
Suppose that the above particle has constant mass and that at times U and U it is
located at Pi and P 2 [Fig. 2-1] and moving with velocities vi = dn/dt and v2 = drjdt
respectively. Then we can prove the following [see Problem 2.8].

CHAP.

NEWTON'S LAWS OF MOTION WORK, ENERGY AND MOMENTUM

2]

Theorem 2J.

The

total

work done

in

moving the

particle along

35

C from

Pi to

P2

is

given by

W
we

If

call

C Fdr =

the kinetic energy of the particle, then

or, in

where

Theorem

(8)

2.1 is equivalent to the statement

Work done from Pi to P2 along C


= Kinetic energy at P2

symbols,
Ti

^mv 2v T2

(7)

T = \mv^

the quantity

Total

^m(v\-v\)

Kinetic energy at Pi

= T 2 -Ti

(10)

\mv\.
^

CONSERVATIVE FORCE FIELDS


Suppose there exists a scalar function
following [see Problem 2.15].

Theorem

2.2.

The

total

work done

in

F = - vV. Then we

such that

moving the

particle along

can prove the

C from

Pi to

P2

is

F-dr

V(Pi)

- V(P2

(11)

is independent of the path C joining points Pi and P2


If the
force field in moving a particle from one point to another point is
independent of the path joining the points, then the force field is said to be conservative.

In such case the work done

work done by a

The following theorems are

Theorem 23.

valid.

A force field F is conservative


V such that F=-vF

differentiable scalar field

VXF =
if

curl

if

and only

if

there exists a continuously

or, equivalently, if

F =

and only

identically

(12)

Theorem 2.4. A continuously differentiable force field F is conservative


for any closed non-intersecting curve C (simple closed curve)

i F-dr =
i.e.

the total

work done

in

moving a

particle

if

if

and only

(13)

around any closed path

is zero.

POTENTIAL ENERGY OR POTENTIAL


The scalar V such that F=-yF is called

the potential energy, also called the scalar


potential or briefly the potential, of the particle in the conservative force field F. In such
case equation (11) of Theorem 2.2 can be written

Total

or, in

where

Work done from Pi to P2 along C


= Potential energy at Pi -

symbols,

Vi

V(Pi),

V2 = V(P2

).

Vi

- V2

Potential energy at

P2
(15)

NEWTON'S LAWS OF MOTIQN. WORK, ENERGY AND MOMENTUM

36

should be noted that the potential


can express the potential as

It

We

V
V=

where we suppose that

when

defined within an arbitrary additive constant.

is

- f F-dr

i=

[CHAP.

(16)

CONSERVATION OF ENERGY
For a conservative force

we have from

field

T 2 -Ti = Vi- Va

The quantity

E = T + V,

total

energy at

P2

which

the

sum

we

s^ee

is

V1

= T 2 + V2

(17)

V2

(18)

the total energy [i.e. the sum of kinetic


In symbols, T + V = constant = E.

Theorem 2.5. In a conservative fofce


energy and potential energy] is a constant.
This theorem

(15),

of the kinetic energy and potential energy, is


that
the total energy at Pi is the same as the
(18)
can state our results in the following
is

From

We

T1

or

\mv\ + Vi \mv\ +

which can also be written

called the total energy.

equations (10) and

field

often called the principle of conservation of energy.

IMPULSE
Suppose that

in Fig. 2-1 the particle is located at Pi

has velocities vi and v2 respectively.

Thb time

r
is called

the impulse of the force F.

Theorem

2.6.

The impulse

is

and P2 at times

integral of the force

and U where

Ydt

(19)

The following theorem can be proved


equal to the change in

it

given by

momentum;

[see

or, in

Problem

2.18].

symbols,

Xt F dt = m\2 mvj
The theorem

is

true even

when

the mass

is

(20)

Pi

variable and the force

is

non-conservative.

TORQUE AND ANGULAR MOMENTUM


If

a particle with position vector r moves in a

force field

[Fig. 2-2],

we

define

A = rxF
as the torque or

moment

of the force

(21)

F about

0.

The magnitude of A is a measure of the j'turning


effect" produced on the particle by the fcjrce. We
can prove the following [see Problem 2.5J50]

Theorem

2.7.

rxF =

-3r{m(rXv)}

(22)

Fig. 2-2

CHAP.

NEWTON'S LAWS OF MOTION. WORK, ENERGY AND MOMENTUM

2]

The quantity
12

37

= m(rxv) = rxp

(23)

In words the theorem


is called the angular momentum or moment of momentum about 0.
states that the torque acting on a particle equals the time rate of change in its angular

momentum,

i.e.,

- =
This theorem

is

true even

if

the

mass

is

variable or the force non-conservative.

CONSERVATION OF MOMENTUM
If

we

let

F=

in

Newton's second law, we find


-jT-(rav)

or

mv =

constant

(25)

This leads to the following

Theorem

If the net external force acting

2.8.

on a particle

is

zero, its

momentum

remain unchanged.

will

This theorem
of constant

mass

is

often called the principle of conservation of


equivalent to Newton's first law.

momentum. For the

case

it is

CONSERVATION OF ANGULAR MOMENTUM


If

we

let

A=

in (2U) f

we

find

-jr{m(rXv)}

m(rXv) = constant

or

(26)

This leads to the following

Theorem

momentum

If the net external torque acting

2J9.

will

This theorem

is

is

is

zero, the

often called the principle of conservation of angular

NON-CONSERVATIVE FORCES
If there is no scalar function V
then

on a particle

angular

remain unchanged.

such that

F = - yV

called a non-conservative force field.

for all types of force fields, conservative or not.


for conservative force fields.

The

[or,

results

However,

momentum.

equivalently,
(7), (20)

(11)

and

and

if

(2U)

VxF *

0],

above hold

(17) or (18) hold only

STATICS OR EQUILIBRIUM OF A PARTICLE


An important special case of motion of a particle

occurs when the particle is, or appears


to be, at rest or in equilibrium with respect to an inertial coordinate system or frame of
reference.
necessary and sufficient condition for this is, from Newton's second law, that

F =
i.e.

the net (external) force acting on the particle be zero.

(27)

NEWTON'S LAWS OF MOTION. WORK, ENERGY AND MOMENTUM

38

[CHAP.

If the force field is conservative with potential V, then a necessary and sufficient
condition for a particle to be in equilibrium at a point is that

F =

dV = dV = V
=

'

at the point.

Bx

dy

dz

STABILITY OF EQUILIBRIUM
a particle which is displaced slightly from an equilibrium point P tends to return
we call P a point of stability or stable point and the equilibrium is said to be
stable.
Otherwise we say that the point is one of instability and the equilibrium is
unstable. The following theorem is fundamental.
If

to P, then

Theorem
of stability

A necessary and sufficient condition that an equilibrium point be one


that the potential V at the point be a minimum.

2.10.

is

Solved Problems

NEWTON'S LAWS
2.1.

Due

given as a function of time

is

Find
field

2.2.

(a)

at

the velocity,

any time

(6)

Momentum = p = mv =

(c)

Acceleration

(d)

Force

= F = j^ =

mass

particle of

+ l)i +

(6* 2

Velocity

+ t)i +

(St4

-t 2 + 8) j -

momentum,

the

(b)

dr
=
^

(2ts

12t 2k

the acceleration

(c)

and

(d)

the force

(a)

mass 5 units moves along a space curve whose


t by

to a force field, a particle of

position vector

5v

(30*2

= -^ -

m^

moves

(12*3

60ti

in the

+ 5)j +

12ti

- 2*)j -

(36*2

(180t2

24tk

60 f3

_ io*)j - 120*

_ 2 )j -

- 10) j -

24k

120k

xy plane so that
a cos (d

b sin

its
<*>t

position vector

is

and > are positive constants and a> b. (a) Show that the particle moves
an ellipse, (b) Show that the force acting on the particle is always directed
toward the origin.

where

a, b

in

(a)

The

position vector
r

xi

y]

is

a cos at

b sin at

x = a cos at, y = b sin at which are


the parametric equations of an ellipse having
semi-major and semi-minor axes of lengths a

and so

and

b respectively [see Fig. 2-3].

Since
(*/o) a

the ellipse

(y/b) 2

is

cos2 at

also given

by

as

sin 2 at

2 /a 2

+ y2 /b 2 =

1.

Fig. 2-3

CHAP.

NEWTON'S LAWS OF MOTION. WORK, ENERGY AND MOMENTUM

2]

Assuming the

(6)

particle has constant

mass m, the force acting on


d2r

dv

= m Tt = m
= m[ 2 a cos wt
mw 2 [a cos ut i +
i

which shows that the force

2.3.

d2
-r- [(a

w 2 6 sin ut
b sin ut

cos ut)i

39

it is

(b sin t)j]

j]

= mw 2 r

j]

always directed toward the origin.

is

observers O and 0', fixed relative to


two coordinate systems Oxyz and O'x'y'z'

Two

respectively, observe the motion of a particle

P in

space [see Fig.

Show

2-4].

that

to both observers the particle appears to

have the same force acting on it if and


only if the coordinate systems are moving
at constant velocity relative to each other.
Let the position vectors of the particle in the
O'x'y'z' coordinate systems be r and r'
respectively and let the position vector of 0'
with respect to O be R = r r\

Oxyz and

Relative to observers
respectively

by

F =
The

difference in observed forces

and only

this will be zero if

r
m dw'

according to Newton's laws are given

" r)

'

pr

F =

<

is

F ~ F
and

Fig. 2-4

and O' the forces acting on

= m

(r

m l#

if

d2 R

dR =

dt2

dt

constant

the coordinate systems are moving at constant velocity relative to each other.
systems are called inertial coordinate systems.
i.e.

The

2.4.

result

particle of

is

sometimes called the classical principle of

mass 2 moves

F
Assuming that
vo
(a)

at

= 6i + 15j - 8k,
By Newton's

24t 2 i

relativity.

depending on time

in a force field

(36-16)j

12tk

second law,

2dv/dt

24t2 i

(36t-16)j

dv/dt

12tH

(18t-8)j

given by

Integrating with respect to

the particle is located at r = 3i - j + 4k and has velocity


find (a) the velocity and (b) the position at any time t.

or

Since

Such coordinate

6i

+ 15j - 8k
v =

and calling

(4t3

6tk

(9 2

-8t)j

3t2k

have

cx

6i

we have

ct

+ 15j - 8k and
+ (9t2 -Bt+ 15)j - (3*2 + 8)k

= 0, we

+ 6)i

12tk

c x the constant of integration,

= UH +
at

so

NEWTON'S LAWS OF MOTION. WORK, ENERGY AND MOMENTUM

40

Since

(6)

we have by part

dr/dt,

dr

Integrating with respect to

Since r

3i

constant force

in time

(t*

+ 6 )i +

and calling

at

+ 6)i +

- 42 + I5t)j -

have c2

+ 6t + 3)i +

(3t 2

+8k
)

c 2 the constant of integration,

(3*3

3i

(tf*

+ 8t)k +

j + 4k

c2

and so

- 4*2 + 15t - l)j +

acting on a particle of mass

(4

- & - St)k

changes the velocity from

vi to V2

F = m(v2 vi)/t.

Prove that

(b)

Does the result in

(a)

By Newton's

hold

(a)

if

second law,

the force
,

Then

F and

if

= 0,

v=v

= r,

so that

= v2

c x = Vj

Explain.

F
=

dv

or

we have on

are constants

variable?

is

= _
F

dv
m-jdt

At

_ 8t + 16) j _

t.

(a)

At

(9*2

(3t3

= 0, we

(a)

(4^3

(t4

j + 4k

2.5.

[CHAP.

-77

/<v
(1)
y '

dt

integrating,

(F/m)t

c2

(F/m)t

vj

v2

(F/m)r

vt

i.e.

so that

(2)

F = m(v2 vi)/t

i.e.

(S)

Another method.

Write

(1)

as

mdv =

Fdt.

Then

md\

since

v=v!

Ydt

at

and

m(v 2 v^

or

= v2

at

r, we

have

Ft

vi

which yields the required


(b)

2.6.

result.

No, the result does not hold in general if F


obtain the result of integration achieved in

is

not a constant, since in such case

we would not

(a).

Find the constant force in the (a) cgs system and (b) mks system needed to
accelerate a mass of 10,000 gm moving along a straight line from a speed of
54 km/hr to 108 km/hr in 5 minutes.
Assume the motion to be in the direction of the positive x axis. Then if v x and v 2 are the
we have from the given data V! = 54i km/hr, v2 = 108i km/hr, m = 10,000 gm,

velocities,
t

(a)

5 min.
In the cgs system

m =

10 4 gm,

Then

km/hr =

vx

= ma = m (

54i

/v 2

Thus the magnitude


(6)

In the

0.5

10 5 i

gm

(104

cm/sec2

3.0

10 4 i

103i cm/sec,

/1.5

X 103 i cm/sec \

gm)

v2

lQ3i cm/sec,

vt \

1.5

102 sec

dynes

of the force is 50,000 dynes in the direction of the positive

axis.

mks system

m =

10 kg,

vj

54i

km/hr

15i m/sec,

v2

30i m/sec,

300 sec

300 sec

CHAP.

NEWTON'S LAWS OF MOTION. WORK, ENERGY AND MOMENTUM

2]

F = wa

Then

kg

0.5i

v2

41

15i m/sec

Vl

(10 kg)

300 sec

m/sec 2

0.5i

newtons

This result could also have been


Thus the magnitude is 0.5 newtons in the positive x direction.
newtons.
or
10*
dynes
=
1 dyne = 10
newton
obtained from part (a) on noting that 1
omitted, it being understood that the
In this simple problem the unit vector i is sometimes
it is good practice to work this
However,
*
axis.
positive
force F will have the direction of the
to emphasize the vector character of force,
and similar problems with the unit vector present so as
velocities may change their directions.
where
cases
in
This is especially important
velocity, etc
See, for example, Problem 2.46, page 56.

2.7.

at a speed of 60 mi/hr
constant force is needed to bring a 2000 lb mass moving
to rest in 4 seconds?

What

which we choose as the positive


assume that the motion takes place in a straight line
units, we have
of
system
absolute
direction of the x axis. Then using the English
m = 2000 1b, v = 60i mi/hr = 88i ft/sec, v2 = Oi ft/sec, t = 4 sec

We

shall

= wa = m

Then

Thus the force has magnitude


opposite to the motion.

-4.4

This

v2

'-88i ft/sec \
(2000 lb)

4 sec

104 i ft lb/sec2

-4.4 X 104 i poundals

X 10* poundals in the negative x direction,


of course to be expected.

4.4

is

i.e.

in

WORK, POWER, AND KINETIC ENERGY


2.8.

A particle

of constant

Assuming that
the work done

U and U

at times
is

under the influence of a force field F


that
the velocity is vi and v2 respectively, prove

m moves

mass

in space

the change in kinetic energy,

i.e.,

tt

X
Work done

- X. F*\dt
dv

2.9.

im f

d(v

v)

mv 2

%mv\ -

-Jmv

Find the work done in moving an object along a


vector

F=

2i

r = 3i + 2j-5k
- j - k. Refer to

if the applied force


Fig. 2-5.

is

Work done = (magnitude

of force in direction
of motion) (distance moved)

(Fcostf)(r)

Ft

= (2i-j-k)-(3i + 2j-5k)
= 6-2 +

a direction

r
Fig. 2-5

NEWTON'S LAWS OF MOTION. WORK, ENERGY AND MOMENTUM

42

2.10.

[CHAP.

Referring to Problem

2.2, (a) find the kinetic energy of the particle at points A


the work done by the force field in moving the particle from A to B,
(c) illustrate the result of Problem 2.8 in this case and (d) show that the
total work
done by the field in moving the particle once around the ellipse is zero.

and B,

(a)

(b) find

Velocity

ua sin ut i + ub cos at j.
= |w 2 = ^m(u 2 a 2 sin 2 ut + u 2 b 2 cos 2 ut).

Kinetic energy

(6)

Method

dr/dt

==

[where cos ut

Kinetic energy

[where cos ut

From

1.

A
at B

Kinetic energy at

part

(6)

Work done

1,

sin ut

ut

0, sin

=
=

1]

dr

d(r

r)

We

Work

done

JA

Xir/2u

dr

^mu 2r2

^mw 2 (a2 -

and

\mu 2 a2 -

can assume that at

raw2

(mu 2 r) dr

=
2.

4mw 2 6 2
^mu 2 a2

of Problem 2.2,

Method

=
=

0]

|mo> 2 6 2

and B,

62)

tt/2u respectively.

Then:

dr

[ mu 2 (a cos ut

b sin ut

j)]

[ua sin ut

ub cos ut

j]

dt

s>ir/2(>)

mw 3 (a2

6 2 ) sin wt cos ut dt

%mu 2 (a2 -

ir/2w

b 2 ) sin 2 ut

%mu 2 (a2 -

\mu 2 a2 - \mu 2 b 2

b2 )

o
(c)

From

parts

(a)

and

(6),

Work done

(d)

Using Method 2 of part


around the ellipse,

(6)

|ww 2 (a2 -

kinetic energy at

we

have, since

62)

goes from

J-..S7JVCU

muz (a2 b 2

kinetic energy at
to

2v/u

for a complete circuit

sin ut cos wt dt

mw 2 (a2 -

=
Method

2.11.

27T/W

6 2 ) sin 2 ut

can also be used to show the same result.

Prove that if F is the force acting on a particle and v is the (instantaneous) velocity
of the particle, then the (instantaneous) power applied to the particle is given by
cp

By

definition the

work done by a force F

- f-v
in giving a particle a displacement dr is

dW =
Then the (instantaneous) power

is

given by

* =
as required.

F*dr

'! = -'

CHAP.

2.12.

NEWTON'S LAWS OF MOTION. WORK, ENERGY AND MOMENTUM

2]

Find the (instantaneous) power applied to the particle

Problem

in

2.1

43

by the force

field.

By Problem

2.1,

the velocity and force are given respectively by

+ l)i + (12*3 _ 2t)j - 24<k


60ti + (180* 2 - 10)j - 120k

F =

(6* 2

Then the power

[by
q>

2.13.

Problem

= F

given by

2.11] is

=
=

2160t 5

Find the work done by the force


(a)

Then by Problem

vx

In the cgs system:

|v x

done

(10 4 gm)(9.0

3.38

part

v2

Problem

(b)

2.6,
|v 2

3.0

2.7.

103 cm/sec,

m =

energy

in kinetic

106

V*L

lOio

sec2

3.

_^

106)

38xlo io/sn_|n
z
\

3.38

sec

/)

(cm)

10 10 ergs

similarly:

(10 kg)(900

- 225)

/ kg
\
10 3 -j^g- ) (m)

3.38

(2000 lb)(88 2

7.74

(a),

Work done

cm2

2.25

X 10 10 dyne cm

3.38

=
in

(120)(24t)

2960

cm/sec,

change

mks system we have

Work done

As

1.5

10 3

- 10)(12*3 - 2t) +

Problem

(a)

(6)

in

(180t2

120*3

2.8,

Work

In the

+ 1) +

(60t)(6t 2

106(ft)

3 38
-

x 10 newton meters

-i^

(^f)

7.74

10 ft pdl

CONSERVATIVE FORCE FIELDS, POTENTIAL ENERGY, AND


CONSERVATION OF ENERGY
2.14.

Show

that the force

field

F =
is

a conservative force

Method

1.

The force

field

V XF

defined
2 8

(y z

by

- 6xz 2)i +

2xyz 3 j

is

conservative

if

and only

d/dy

d/dz

2xyzs

q xzz

\j-{3xy 2z2 -6x 2 z)

3xy2z2

- 6x 2 z)k

F = VXF =

6x 2 z

-j^(2xyzS)

\^-(y2 z^-6xz2)

+ k
conservative.

curl

if

d/3x

field is

{Sxy 2 z 2

field.

y 2 zz

Then the force

- ^(SxyW - x2 z) 1
9a?

["^(fcsy* 8 )

- j- (y2z* - 6xz 2 )~\

0.

Now

10 4 gm.

NEWTON'S LAWS OF MOTION. WORK, ENERGY AND MOMENTUM

44

Method

z)

field

such that

F is conservative
F = -grad V =

and only
Then

if

=
if

is

Integrate the

we must

conservative

dV/dx

2 3
(y z

6xz 2

dz

- 6xz 2 )i +

2xyz 3 j

- 6x 2 z)k

(3xy 2 z 2

V such that
= -2xyz 3
BV/dz =

be able to find

y 2z3

dV/dy

V =
is

6x 2 z

equation with respect to x keeping y and z constant.

first

where g t (y, z)

dy

there exists a scalar function or potential

if

-W
dx

Hence

2.

The force
V(x,y,

[CHAP.

a function of y and

3x 2 z 2

xy 2 z 3

3xy 2 z 2

(1)

Then

g x {y, z)

(2)

z.

Similarly integrating the second equation with respect to y (keeping x and z constant) and the
third equation with respect to z (keeping x and y constant), we have

V = -xy 2z 3 +

V =
Equations

(2), (3)

and

where

c is

g 2 (x, z)

c,

any arbitrary constant, and

we

if

g 2 (x,

z)

(3)

g 3 (x, y)

(4)

choose

3x 2 z 2

2xyz 3 dy

g 3 (x, y)

c,

(5)

follows that

it

V =
is

xy 2 z 3

common V

(4) yield a

ffi(V, *)

Zx 2 z 2

3x 2 z 2

xy 2 z3

(2/2*3

Qxz 2 )dx

the required potential.

Method

3.

F-dr

(.x

.'o

(3xy 2 z 2

6x 2 z)dz

,y ,z Q )

s*(x,y,z)

-I

where

2.15.

x y2 z3

3x 2 z 2 )

3x 2 z 2

xy 2 z3

3x 2 z^

Prove Theorem 2.2, page 35: If the force acting on a particle is given by F = vV",
then the total work done in moving the particle along a curve C from Pi to P% is

W
We

F-rfr

V(Pi)

- V(P

2)

have

2.16.

d(xy 2 z 3
(x ,y ,z

J Pi

F-dr

J Pi

-VV*dr =

1
I

J Pi

-dV

Find the work done by the force field F of Problem 2.14


the point A(-2,l,3) to 5(1,-2,-1).

Work done

JA

F'dr

f
JA

-VV

-dV

in

moving a

particle

(1,-2,-1)

-V(x,y,z)
(-2,1,3)

(-2,1,3)

- V(P 2

dr

(1,-2,-1)

V(P X )

3x2 z2 +

Pi

xy 2 z3

(1,-2,-1)
c
(-2,1,3)

155

from

CHAP.

2.17.

NEWTON'S LAWS OF MOTION. WORK, ENERGY AND MOMENTUM

2]

(a)

Show

(6)

Find the potential energy at points

(c)

(d)

(a)

that the force

of Problem 2.2

field

conservative.

is

and

of Fig. 2-3.

From Problem

show that

F = ma 2r = -ma2 {xi + yj).

2(6),

V XF

d/dx

d/dy

d/dz

ma2 x

mu2y

+ k \h ~ mu2y) ~
{

Hence the
Since the

and compare

constant,

i.e.

demonstrate

~ ma2x)
\

conservative.

field is

field is

it is

to

Then

(6)

Find the work done by the force in moving the particle from
with Problem 2.10(6).
Find the total energy of the particle and
the principle of conservation of energy.

45

conservative there exists a potential

such that

WdV/dx

Then

= ma 2 x,

= mw 2y,

dV/dy

dV

dV/dz

dV u

from which, omitting the constant, we have

V = \ma 2 x 2 + %ma2 y 2 = $ma 2 (x 2 + y2 = \mJh2


)

which

(c)

is

the required potential.

A of Fig. 2-3
point B of Fig. 2-3

Potential at point
Potential at

= a] = $ma2 a2
= mu2 6 2
[where r = b]
[where r

Then

Work done from A

to

B =
=

Potential at

\ma

2 a2

B
= $ma2 (a2 - b 2

Potential at

m<o2 6 2

agreeing with Problem 2.10(6).

(d)

By Problems

2.10(a)

and part

(6),

Kinetic energy at any point

Potential energy at any point

= T = %mv 2 = |mf2
= |ra(<o 2 a 2 sin 2 at +

a2 b 2 cos 2

= V = ^m2r2
= %ma 2 (a2

b 2 sin2 at)

1,

Thus at any point we have on adding and using

T+V
which

is

sin2 at

= $ma2 (a2 +

cos 2 at

cos2 at

at)

b2)

a constant.

AND

IMPULSE, TORQUE, ANGULAR MOMENTUM,

CONSERVATION OF MOMENTUM
2.18.

Prove Theorem

2.6,

page

36:

The impulse

of a force

is

equal to the change in

momentum.
By

definition of impulse [see (19),


2

C* dt = J

page

36]

and Newton's second law, we have


t2

-^(m v) dt

= J

d (mv)
^

= mv 2 mxi

NEWTON'S LAWS OF MOTION. WORK, ENERGY AND MOMENTUM

46

2.19.

mass of 5000 kg moves on a straight

in 2 minutes.

Method

What

is

line from a speed of 540


the impulse developed in this time?

km/hr

that the mass travels in the direction of the positive x axis.

_
=

V,

Then from Problem

_.

_
=

km

n
5401-:;
.

hr

m =

X 1000

540i

In the

1Kvin2
1.5 X 10 2 1
.

km/hr

m =

X 1000

720i

hr

.
nn
2i
2.0Xl0

2.18,

= (5000 kg)(0.5 X 10 2 i m/sec)


= m(v2
= 2.5 X 10 5 i kg m/sec = 2.5 X 10 5 newton
kg m/sec 2 or 1 newton sec = 1 kg m/sec.
x)

newton

Thus the impulse has magnitude


Method 2.
Using the cgs system,
2.0 X 10 4 i cm/sec.
Then

dyne

gm

vx

2.5

540i

or

1.5

(5000

km/hr

cm/sec 2

sec

10 5 newton sec in the positive x direction.

= w(v 2 - v t =
= 2.50 X 10 10 igm

Impulse

system,

sec

3600 sec

Impulse

mks

sec

3600 sec

km
= .
720i=^ =

v2

since

to 720

1.

Assume

since

[CHAP.

X 10 4 i cm/sec
10 3 gm)(0.5

cm/sec

dyne sec

2.50

and

v2

720i

km/hr

10 4 i cm/sec)

10 10 i dyne sec

gm cm/sec

Note that in finding the impulse we did not have to use the time 2 minutes as given in the
statement of the problem.

2.20.

Prove Theorem 2.7, page 36:


a coordinate system is equal

The moment

of force or torque about the origin

rXF =

is

rX-j- (mv)

The angular momentum or moment of momentum about O

B =

Now we

42L

have

which gives the required

2.21.

m(r X

= ^(rXmv)

v X (mv)

v)

J^

X (mv) +

(a)

Torque

at

+ 6* + 3)i +

(3*3

- 4*2 + 15* - 1) +
j

t4

6*

24t2

rXF =

momentum

about the origin for the

rXF

A =
4

result.

(a)

[(*

rx|(mv)

X -(mv)

is

X (mv)

the torque and (6) the angular


particle of Problem 2.4 at any time t.

Determine

(4

- *3 - 8*)k] X
k

of

change of angular momentum.

to the time rate of

The moment of force or torque about the origin O

3*3

4*2

36*

+ 15* - 16

t3

8*

-12*

(32*3 + 108*2 _ 260* + 64)i - (12* 5 + 192* 3 - 168*2 - 36*)


- (36* 5 - 80*4 + 360*3 _ 240*2 - 12* + 48)k

[24* 2 i

(36*

- 16)j -

12*k]

CHAP.

NEWTON'S LAWS OF MOTION. WORK, ENERGY AND MOMENTUM

2]

(b)

Angular momentum

SI

X (mv)

m(r X

47

v)

2[(t* + 6* + 3)i + (3*3 - 4*2 + 15* - l)j + (4 - *3 - 8*)k]


X [(4*3 + 6)i + (9*2 - 8* + 15)j - (3*2 + 8)k]

*4

6*

3*3

4*2

15*

*3

- 8*
-8

9*2 - 8* + 15
+6
= (8*4 + 36*3 _ 130*2 + 64t _ I04)i - (2* + 48** - 56*3 - 18*2 - 96)j
- (6* - 16*5 + 90*4 _ 80*3 _ 6*2 + 48* - 102)k

-3*2

4*3

Note that the torque is the derivative with respect to


the theorem of Problem 2.20.

2.22.

of the angular

momentum,

illustrating

moves in a force field given by F = r2 r where r is the position vector of


the particle. Prove that the angular momentum of the particle is conserved.

particle

The torque acting on the

particle is

A = rXF = rX
Then by Theorem

2.9,

page

37, the

angular

(r 2 r)

r2 (r X

momentum

is

r)

constant,

i.e.

the angular

momentum

is

conserved.

NON-CONSERVATIVE FORCES
2.23.

Show

that the force

We

field

have

VXF =

given by
i

d/dx

d/dy

BlBz

since

V XF#

0,

the

-xz*\

is

non-conservative.

(x 2 y

+ yz 2 )} -

x2zk

xyz2

x 2yz

Then

F = x 2 yzi - xyz^k

field is

non-conservative.

STATICS OF A PARTICLE
2.24.

particle

is

acted upon by the forces Fi,

F2 F3 F4 F8 and F6 shown
,

Represent geometrically the force needed to prevent

Fig. 2-6

Fig. 2-7

The resultant R of the forces


indicated in Fig. 2-7.
have R

We

from moving

is

which

in Fig. 2-6.

from moving.

is

F v F2 F3 F4 F5 and F6 can be found by vector addition as


= F 1 + F2 + F3 -|-F4 + F5 + F6 The force needed to prevent
,

a vector equal in magnitude to

sometimes called the equilibrant.

but opposite in direction and

NEWTON'S LAWS OF MOTION. WORK, ENERGY AND MOMENTUM

48

2.25.

[CHAP.

upon by the forces Fi = 5i - lOj + 15k, F2 = lOi + 25j - 20k and


Find the force needed to keep the particle in equilibrium.

particle is acted

F3 = 15i-20j + 10k.

The resultant of the forces

is

R = F + F2 + F3 =
=

- lOj + 15k) +
30i - 5j + 5k

Then the force needed

2.26.

to

(lOi

(5i

+ 25j - 20k) +

keep the particle in equilibrium

(151

- 20j + 10k)

R = 30i + 5j 5k.

is

The coplanar

forces as indicated in Fig. 2-8 act on a particle P. Find the resultant


of these forces (a) analytically and (b) graphically. What force is needed to keep
the particle in equilibrium?
Unit

20 lb

''/

V
30

/\
3/

.#////30

iV

Fig. 2-8
(a)

From

Analytically.

Fx =

Fig. 2-9

Fig. 2-8

we

160(cos45

R is
R = F + F2 + F3
= (160 cos 45 - 100
= -33.46i + 59.21J

have,

sin 45 j),
F2 = 100(- cos 30
F3 = 120(- cos 60 i - sin 60 j)
i

sin 30

j),

Then the resultant


t

Writing

R = R

counterclockwise,

cos 30

cos a i + R sin a
see that

120 cos 60)i

where a

is

(160 sin 45

100 sin 30

120 sin 60)j

the angle with the positive x axis measured

we

R
Thus the magnitude of
the positive x axis

is

is

cos a

R =

V(-33.46) 2

given by tan a

-33.46,

sin a

59.21

+ (59.21) 2 = 68.0
= -1.770

59.21/(-33.46)

lb,

or

and the direction a with


a

119

28'.

Choosing a unit of 20 lb as shown in Fig. 2-9, we find that the resultant has
magnitude of about 68 lb and direction making an angle of about 61 with the negative
x axis [using a protractor] so that the angle with the positive x axis is about 119.

Graphically.

(6)

force

R,

i.e.

opposite in direction to

but with equal magnitude,

is

needed to keep

in equilibrium.

STABILITY OF EQUILIBRIUM
2.27.

particle

moves along the x axis

in a force field having potential

(a)

Determine the points of equilibrium and

(a)

Equilibrium points occur where

VV =

dV/dx
Thus there

is

(5)

kx

only one equilibrium point, at x

or

= 0.

fax

investigate the stability.

or in this case

V=

2
,

> 0.

CHAP.

NEWTON'S LAWS OF MOTION. WORK, ENERGY AND MOMENTUM

2]

Method

(6)

49

1.

d2 V/dx 2 = k > 0, it follows that at x = 0, V is a minimum. Thus by Theorem 2.10,


This is also seen from Problem 2.36 where it is shown
is a point of stability.
that the particle oscillates about x = 0.
Since

page

38,

V(x)

Method

2.

^-i = kxi.
We have F = -VV = dV.
ax

when x >

the left,
goes a force to the right.
point of stability.

Method

Then

the particle undergoes a force to


the particle underand when x <

Thus x

is

3.

The fact that x =


be seen from a graph

is a minimum point can


of V(x) vs x [Fig. 2-10].

Fig. 2-10

MISCELLANEOUS PROBLEMS
2.28.

Show how Newton's laws can be used

to develop definitions of force

and mass.

consider some given particle P, assuming for the present that its mass raP is not
simply some constant scalar quantity associated with P. Axiom 1 states that if P
moves with constant velocity (which may be zero) then the force acting on it is zero. Axiom 2
P aP where
states that if the velocity is not constant then there is a force acting on P given by
a P is the acceleration of P. Thus force is defined by axioms 1 and 2 [although axiom 1 is unnecessary
since it can in fact be deduced from axiom 2 by letting F = 0]. It should be noted that force is a
vector and thus has all the properties of vectors, in particular the parallelogram law for vector

Let us

defined but

first
is

addition.

mass mP of particle P, let us now allow it to interact with some particular


which we shall consider to be a standard particle and which we take to have unit mass.
If aP and as are the accelerations of particle P and the standard particle respectively, it follows
from axioms 2 and 3 that mP aP = s Thus the mass mP can be defined as s /aP

To

define the

particle

2.29.

Find the work done in moving a particle once around a circle


the circle has center at the origin and radius 3 and if the force

F
In the plane z

work done

(2x-y + z)i +

= 0, F =

- y)\ +

(2x

(x

(x

+ y-z 2 )j +

(Sx

- 2y)k

and

+ y)j +

(3x

xy plane,

in the

field is

- 2y + 4z)k
rfr

dx

dy

so that the

is

f F'dr
Jc

[(2x-y)i

(2x

Jc

Jc

+ y)j +

(x

y) dx +

(x

(Sx-2y)k]'[dxi

dyj]

+ y) dy

Choose the parametric equations of the circle as x = 3 cos t,


Then the
to 2v [see Fig. 2-11].
3 sin t where t varies from
line integral equals

,2tt
I

[2(3 cos

t)

X27T

3 sin

(9-9

t]

sin

[-3 sin

cos

t)

t]

dt

dt

[3

9t

cos

3 sin

sin 2

t]

[3

cos

t]

dt

lSir

In traversing C we have chosen the counterclockwise direction indicated in Fig. 2-11. We call this the positive direction, or say that C
has been traversed in the positive sense. If C were traversed in the

clockwise (negative) direction the value of the integral would be 18tt.

if

given by

=
=

xi

3 cos

yj
t i

Fig.2-ll

3 sin

t j

NEWTON'S LAWS OF MOTION. WORK, ENERGY AND MOMENTUM

50

2.30.

(a)

F=

-vV

where V is single-valued and has continuous partial derivatives,


show that the work done in moving a particle from one point Pi s (xi, yi, zi) in
this field to another point P2 = (x 2 2/2, z 2 is independent of the path joining the
two points.
If

(b)

Conversely,

J F'dr

if

show that there


(a)

J *x

-J Pi
J Pl

-f

Then the
This

Fdr

d%

F = F \ + F2j + F 3k.

dz

two

points,

V(x,y,z)

if

By

which we take as

is

hypothesis,

dx

V(x, y,z)

dt

work done along

from

(x it

yv

'>

<.x

y, z)

z x ) to (x,

Thus

+ F 2 (x, y, z) dy + F 3 (x, y, z) dz]

dx

yx

+ F2 dy + F s dz)

v y lt zi>

[F x (x,

Let us choose as a particular path the


and call V(x, y, z) the

z x ) to (x, y, z x ) to (x, y, z)

Then

this particular path.

XX F

pV
x

joining any

Then

(F x dx

a path joining (x x ,y x ,z x ) and (x,y,z).

straight line segments

p(.x,y,z)

F
(.x i ,y 1 ,z 1

independent of the path

is

(x,y,z)

is

V(x 2 ,y 2 ,z 2)

P x and P2 and not on the path joining them.


single-valued at all points P x and P 2

independent of the path joining (x x y x z x ) and (x,y,z).

V(x x ,y x ,z x )

and (x,y,z) respectively.

(x x ,y x , z x )

where

dzk)

integral depends only on points

Let

is

-^kj-idxi + dyj +

= V(P )-V(P2

2<iV

F=-vV.

such that

VVdr

J P!

ty

true of course only

is

joining any two points,

pP2

aP +

(to

independent of the path

is

exists a function

ft

Work done

"

(6)

[CHAP. 2

(x,y x ,z x )dx

J *i

"i

pz

F2 (x,y,z

)dy

J ^1

F3 (x,y,z)dz

It follows that

= -F3 {x,y,z)

dV =
7dy

-F2(x,y,z x )

C dF (x,y,z)dz
3

^ Xl

-F2 {x,y,z

dy

x)

C z 9F2

-(x,y,z)dz

*s
Zl

dz

= -F2 (x,y,z

x)

- F2 (x,y,z)

x)

- F2 (x,y,z) + F2 (x,y,z

*i

V_

dx

-F 2 (x,y,z

= -f

C v SF2

(x,

x,

x)

-j

-^-(*> v>

*i)

dy

dF x

J'*
j/j

-F

(x,y x ,z x )

- F

(x,y x ,z x )

- F

-r-{x,y,z x )dy

f 8F
-fate*

(x,y,z x )

-F

(x,y,z x )

C* dF x
I

^ zx

y> *)

dz

(x,y,z) dz

Z
\z

- F

(x,y,z)\

v\

-j

<>y

y
x

-F 2 (x,y,z)

x)

\*i

F(x,y x ,z x )

- F

(x,y,z)

F(x,y,z x )

-F

(x,y,z)

CHAP.

NEWTON'S LAWS OF MOTION. WORK, ENERGY AND MOMENTUM

2]

Thus a necessary and

2.31.

Show

(a)

-fi-fi-fk
3"
dy'

F = r.l+J-.J+F.k =

Then

sufficient condition that

field

-W

dz

F be conservative is

51

that curl

F= V XF=

F =

that

the potential,

(2xy + z*)i + x 2j + Sxz 2k is a conservative force field,


Find the work done in moving an object in this field from

(c)

Find

(b)
(1,

0.

-2,

1)

to (3,1,4).

(a)

necessary and sufficient condition that a force will be conservative

VXF =

Now

d/dx

d/dy

d/dz

x2

Zxz 2

2xy

Methods 2 or

(6)

As

(c)

Work done = (x 2y +

Problem

in

2.14,

z*

we

3,

(3,1,4)

xz 3 )

V =

find

Thus F

0.

~(x 2 y

is

is

that curl

F =

V X F 0.

a conservative force

field.

+ xz3 ).

-202.

(1,-2,1)

F'dr
Pi

P AP 2 BP
1

F*dr

independent of the path joining any two points Pi and

i F

in a given region, then

Let

is

[see Fig. 2-12]

F-dr- J

F dr +

Then

if

<i>

F*dr =

F'dr

J
P^PiBP!

F-dr =
Fdr

PiAP2

2.33.

(a)

is

Fig. 2-12

F'dr +

then

0,

F'dr

P2 BPt

PiAP2

so that,,

L^
S^

F-dr

\p
\

^^^^^^

dr

PiBPa

since the integral from P x to P2 along a path through


the same as that along a path through B, by hypothesis.

Conversely

^^^

P2BP1

PiAP2

PiAP2

and conversely.

f or all closed paths in the region

be a closed curve.

dr

PiAPtBP!

dt

P2

Fdr -

J
P1AP2

dr

P 1 BP2

F
Fdr

PiBP2

Show that a necessary and sufficient condition that Fi dx + F2 dy + Fs dz be an


=
where F = Fii + F2 j+F3 k.
exact differential is that

VxF

(b)

Show

2
(y z? cos

that

differential of
(a)

Suppose

since x, y

and

dx

- 4x s z) dx +

a function

<f>

and

+ F2 dy + F3 dz =

z are

d<f>

2zs y sin x dy

find

independent variables,

(Sy 2z2 sin x

x 4) dz

is

an exact

<f>.

-^-dx

-^-dy

+ -^-dz,

an exact

differential.

Then

NEWTON'S LAWS OF MOTION. WORK, ENERGY AND MOMENTUM

52

30
dx

and so

Conversely

2.34.

d<f>

VXF =

if

F =

0,

an exact

d<p,

dx

dy

then

F = V> and

30

dy

dz

F = F x i + F2j + F sk = |*i + |* j + |^k = V*.

F2 dy + F3 dz =
(b)

jt,

[CHAP.

VXF

Thus

dz

VXV^^

F'dr = V#*dr =

so

i.e.

d<f>,

0.

Fj

da;

differential.

2 s
2 2
Axz z) i
2z3j/ sin x j
x*)
(y z cos a;
(32/ sin x
zero, so that by part (a) the required result follows.

VXF

and

is

computed to be

the kinetic energy of the particle at t - 1 and


field in moving the particle from the point where
t = 1 to the point where t = 2,
(c) the momentum of the particle at t = 1 and t = 2
and (d) the impulse in moving the particle from t = 1 to = 2.

Referring to Problem 2.4 find


= 2,
(6) the work done by the

(a)

From part

of Problem 2.4,

(a)

Then the

velocities at

=1

and the kinetic energies at

(6)

Work done

mv ?

6)i

=2

lOi

=1

(9i 2

- 8t + 15)j -

(St2

+ 8)k

are

16j

and

(2)[(10)2

=2

v2

Ilk,

38i

35j

20k

are

+ (16)2 + (-11)2] =

T2 = mv 2 =

477,

3069

dr

[24* 2 i

-s.

(4*3

and

vx

(a)

(36t

- 16)j - 12tk]

[(4

+ 6)i +

(9* 2

- 8t + 15)j -

(3t 2

+ 8)k]dt

-X

[(24t 2 K4*3

Note that by part

3069-477 =

2592,

+ 6)

- 16)(9t 2 -8t + 15) +

4- (36

(12t)(3 2

+ g)]dt =

same as the difference or change


Theorem 2.1, page 35, that Work done

this is the

(a)

illustrating

2592

in kinetic energies
change in kinetic

energy.
(c)

By part

(a)

the

momentum
p

= mv =

Then the momenta at

(d)

Impulse

J t=i

at any time

2v

(8 3

is

+ 12)i +

= 1 and t = 2 are
- 22k,
Pi = 20i + 32j

- 16* + 30)j -

(18t2

p2

76i

70j

(6t 2

16)k

40k

F dt

.2

=/,:
't=i

[24t*i

Note that by part


P2

page

2.35.

(36t

(6)

40k )
7 J
Pi = ( 76i
Impulse
36, that

- 16) j -

this is the

20i

12tk]dt

56i

same as the

+ 32J ~~ 22k =
>

56i

38 j

18k

difference or change in

38 '

18k

'

momentum,
Theorem

illu strating

i.e.

2.6,

change in momentum.

moves along the x axis under the influence of a conservative


force field having potential V(x). If the particle is located at positions x x and x 2 at
respective times U and U, prove that if E is the total energy,
H
dx

particle of

mass

*2

"

*
*

"

[ C

y/E

V(x)

CHAP.

NEWTON'S LAWS OF MOTION. WORK, ENERGY AND MOMENTUM

2]

By

53

the conservation of energy,

+
+

Kinetic energy

$m{dx/dt) 2

from which we obtain on considering the

(2/m){E

- V(x)}

(1)

positive square root,

dt

V{x)

{dx/dt) 2

Then

= E
= E

Potential energy

yfm/2(dx/\/E-V{x))

Hence by integration,

~ **-** -

dt

\f f

2.36.

(a) If

= a,

(a)

From

(i)

E-

find

= acosy^Jmt and

sin _1 (*/a)

2.37.

= y/ic/m t +

= sjic/m t +

Since

).

dx/y/a2

cx

and

from

starts

rest at

a sin

dx/dt

where

a,

we

- x 2 = y/Umdt
=a

Since x

or

jt/2

- fax 2

or

fax

describe the motion.

(b)

(2/m)(E

-x2

(K/m)(a2

sin -1 (x/a)

Integration yields

(dx/dt) 2

of Problem 2.35,
2
faa so that
(dx/dt) 2

(6)

V=

the particle of Problem 2.35 has potential

prove that x

Ve^vW)

Xi

ti

(jt/2

at

= 0,

y/ic/m

cl

t)

v/2.

Then

a co^yfic/mt

particle oscillates back and forth along the x axis from x = a to x a. The time for
one complete vibration or oscillation from x a back to x = a again is called the period of the
oscillation and is given by P = 2v VWk.

The

particle of mass 3 units moves in the xy plane under the influence of a force field
having potential V = l2x{Zy - 4x). The particle starts at time t = from rest at
the point with position vector lOi - lOj. (a) Set up the differential equations and
conditions describing the motion.
(c) Find the
(6) Solve the equations in (a),
position at any time, (d) Find the velocity at any time.
(a)

Since

V =

12x(3y

- 4*) =

- 48a 2

S6xy

= ~ Vy =

the force

-f^-^-f

1'

field

is

+ 96*)i-36*j

(-362/

Then by Newton's second law,


3

or in component form, using

where

d2*
dt

;/72
2

xi

(~36y

= -12y +

32a,

-10, y

10,

From

the second equation of

(i),

36;rj

+ yj,

dtx/dt2

0,

using the fact that the particle starts at r

(6)

+ 96x)i -

=
=

lOi

lOj

^d2yfdt2

= -12*

cPy/dt2

at

(1)

(2)

with velocity v

0.

Substitution into the first equation of

(1)

yields

d*y/dt*
If a is constant then
4

32a2

144

= e*
= 0,

is

32 d2y/dt2

a solution of
i.e.

(a 2

($)

4)(a 2

144y

(3)

provided that

- 36) =

or

2i, a

= 6

NEWTON'S LAWS OF MOTION. WORK, ENERGY AND MOMENTUM

54

Thus solutions are e 2it e~ 2it


and the general solution is
,

Thus from

a;

= ~^cPy/dt

Using the conditions

(2) in (4)

<?!

ci

a?

(d)

The

velocity at

= 6,

cx

+
=

3c4
c4

\c2 sin 2*

we

(5),

xi

any time

xi

c 3 e 6t

3c 3 e

terms of real functions]

c 4 e~ 6t

(4)

3c4 e~ 6t

(5)

obtain
10,

c2

18c 3

2c2

6c 3

= 10,

c4

c2

+ 18c4 =
6c4 =

= 2
= -2

0,

so that

cos 2t

6e 6t

+ 6e-

is

yj

[in

(.4),

*,

The position at any time


r

c3

c 2 sin2*

= 0, c 3 = 2,
= -6 cos2t- 2e6t - 2e~ 6

Solving simultaneously,

(c)

3c 3

using

Cos 2t

and

cos2t, sin2t, e 6t , e -6t

or

cos2t

find,

a?

ct

we

e~ 6t

e 6t ,

[CHAP. 2

(-6 cos2t

2e 6t

2e~ flt )i

(-2 cos 2*

6e 6t

6e~ 6t )j

is

yj

(12 sin 2*

12e'

12e" 6t )i

(4 sin 2*

36e 6 '

36e~ 6t)j

In terms of the hyperbolic functions


sinh at

(*-

-*)

cosh at

^(e at

-\-

e~ at
)

we can also write


v

2.38.

(-6 cos 2t

(12 sin 2t

Prove that in polar coordinates

4 cosh 6t)i

24 sinh

VV =

where G and H are to be determined. Since


y = r sine and Problem 1.47(6), page 25,

(cos *

dr

r sin *)(cos $ r x

or

Using

(i)

and

(2) this

(Gr x

(4 sin It

12 cosh 6t)j

72 sinh

6)j

Gri

+ H*!

dr

sin

9^

dr

dr r x

dx i

(i)

we have on using

dy j

+ (sin dr +
+ r dff #
tf

r cos

d*)(sin e r t

cos

a?

(1)

rd69i)

G ~

relativity, the

the speed,

= jp dr +

Gdr + Hrde

'gf

de

H ~ rle
i

W ~
is

Ifr'

^ a7

mass

VI - vVc 2

#i

of a particle

is

given by

mp

VI

r cos

(2)

VV = ^T r +

becomes

According to the theory of

where v

# x)

becomes

+ H9 1 )-(drr +

so that

2.39.

VV'dr = dV = -^dr + -^de

Now

Then

(-2 cos 2t

(r, 0),

Let

dr

6t)i

-)82

the rest mass, c the speed of light and

= v/c.

CHAP.

NEWTON'S LAWS OF MOTION. WORK, ENERGY AND MOMENTUM

2]

Show

(a)

work

that the time rate of doing

moc 2

Deduce from

(b)

(a)

^!-/? )-"
2

much

If

(a)

By Newton's

is

less

than

if

second law,

mv

-jT(mv)
dt

dt \y/

the work done,

is

dW

-l}

{(l-/3 2 )- 1/2

show that T = %mv 2 approximately.

c,

F
Then

is

mc

T = (m-mo)c 2 =
(c)

given by

is

that the kinetic energy

55

"W

d /

d f

d /

as proved by direct differentiation.

Work done = change

Since

(6)

Time rate of
_*
or u
by part

doing work =

dW

(a),

dT

/?

< 1 we

(^f^p)

+
T=

definition,

cx

when v =

= z^^ m

c2

or

/?

= 0,

so that

Cj

(t*

(m mo)c2

have by the binomial theorem,


1

Then

d
mc2*dt

WoC2

T =

To determine Cj note that, by


Hence we have, as required,

For

time rate of change in kinetic energy

= Hi =

~dT

Integrating,

(c)

we have

in kinetic energy,

=
=

(I-/?*)-"*

m c2 r 1 +

i v2
-^

+ 1^2 + 1^1^ + 1^111^+...

c2

-^mv 2

approximately

Supplementary Problems

$
NEWTON'S LAWS
2.40.

particle of

Find
Ans.

2.41.

(a)
(a)

the

mass 2 units moves along the space curve defined by

momentum and

10i-10j

particle

moving

+ 8k,
in

(6)

(6) 4i

a force

2.42.

Under the

Ans.

3e~*i +

field

2 sin

F has

=
j

is

the

its

3e _t i

(4t2

t3 )i

5tj

= 1.

3 cos

momentum

given at any time

2 cos

t j

3 sin

by

influence of a force field a particle of


r

If

at

it

+ 24k

p
Find F.

the force acting on

mass

a cos at

momentum, prove that (o)rXp = mabak,

m moves
b sin ut
(6)

along the ellipse

= m(& 2 a2

sin 2at.

2)k.

NEWTON'S LAWS OF MOTION. WORK, ENERGY AND MOMENTUM

56

2.43.

If

is

means
2.44.

2.46.

2.44 if the force

gm

is

20 newtons and the mass

+ 16j - 16k

(a)

8i

(6)

24 newtons

An

elevator

(a)

Explain

all.

(b)

newtons
24

or

(a) 4i

(8i

+ 16j - 16k) X

floor of a tall building to the ground floor without stopping.


blindfolded person in the elevator may believe that the elevator is not moving
the person tell when the motion begins or stops? Explain.

Can

mass moves

and

a force

in

given in terms of time

field

(6-8)i

60t3j

velocity of the particle at

- 88j + 77k,

(6) i

particle of

cos at)

by

(20t3

and v

+ 4j.

5i

Find the

= 2.

mass

is

given in terms of time

a cos ut

If the particle is initially at rest at the origin, find its

ma 2

+ 362)k
= 2i 3k

- 236j + 176k

(a) - ^-(1

and velocity are given respectively by

(b)

The force acting on a

Am.

10 5 dynes

10 5 dynes

moves from the top

Its initial position


(a) position

or

+ 3k

why a

particle of unit

Ans.

for

Ans. 1200 m/sec

10 kg.

is

(a) Find the constant force needed to accelerate a mass of 40 kg from the velocity 4i 5j
m/sec to 8i + 3j 5k m/sec in 20 seconds. (6) What is the magnitude of the force in (a)?

2.49.

Explain what this

0.

X 104 cm/sec

Work Problem

at

2.48.

XF =

Ans.

2.47.

physically.

force of 100 dynes in the direction of the positive x axis acts on a particle of mass 2
10 minutes. What velocity does the particle acquire assuming that it starts from rest?
Arts.

2.45.

the force acting on the particle of Problem 2.42, prove that

[CHAP. 2

^(at

ma 2

sin at)

j,

(6)

b sin at
(a)

position

ma

by

sin at

and
H

(6) velocity

ma

(1

at any later time.

cos at)

WORK, POWER AND KINETIC ENERGY


A particle is moved by a force F = 20i 30j + 15k along a straight line from point A to
2.50.
with position vectors 2i + 7j 3k and 5i 3 j 6k respectively. Find the work done.

point

Ans. 315
2.51.

2.52.

2.53.

Find the kinetic energy of a particle of mass 20 moving with velocity

3i

5j + 4k.

Ans. 500

Due to a force field F, a particle of mass 4 moves along the space curve r = (3t2 2t)\ + t3 j t4k.
Find the work done by the field in moving the particle from the point where t = 1 to the point
Ans. 2454
where t = 2.
At one particular instant of time a particle of mass 10 is traveling along a space curve with velocity
given by 4i + 16k. At a later instant of time its velocity is 8i 20j. Find the work done on the
Ans. 192
particle between the two instants of time.

2.54.

Verify Theorem

2.55.

2.1,

page 35 for the particle of Problem

2.52.

a(sin at
moves under the influence of the force field given by F
particle of mass
If the particle is initially at rest at the origin, prove that the work done on the particle
is

given by

(a 2 /m<o 2 )(l

Prove that the instantaneous power applied to the particle in Problem 2.55

2.57.

cos at

j).

to time

is (a2 /mw) sin ut.

moves with velocity 5i-3j + 6k under the influence of a constant force


Ans. 160
is the instantaneous power applied to the particle?

10j + 15k. What

cos at).

2.56.

particle

up

F =

20i

CHAP.

NEWTON'S LAWS OF MOTION. WORK, ENERGY AND MOMENTUM

2]

CONSERVATIVE FORCE FIELDS, POTENTIAL ENERGY AND


CONSERVATION OF ENERGY
2.58.
(a) Prove that the force field F = (y 2 2xyz s )i + (3 + 2xy x 2 z 3 )j +
(6)

Ans.
2.59.

2.60.

Find the potential


(b)

xy 2

associated with the force field in

Find constants

a, b, c

2.61.

conservative.

is

+ 2y + az)i +

-3y- z)j +

(bx

1, 2)

Find the

to (1, 3, 2).

by

so that the force field defined


(x

(2,

+ cy + 2z)k

(4x

conservative.

What

(6)

Ans.

Problem 2.58 from the point

field of

F =
is

- 3x 2 yz2 )k

(a).

x 2 yz3 + 3y + z4

A particle moves in the force


work done.
Ans. 55
(a)

(6z 3

57

the potential associated with the force field in (a)?

is

(a)

= 4,

= 2,

= -1

(6)

V = -\x 2 + f y 2 -

z2

2xy

4xz

yz

Find the work done in moving a particle from the point (1,-1,2) to (2,3,-1) in a force
V = x 3 y 3 + 2xy y 2 + 4x.
Ans. 15

field

with

potential

2.62.

Determine whether the force

F =

field

(x 2 y

z3 )i +

(3xyz

+ xz2 )j +

(2x 2 yz

+ j/z4 )k

conservative.

is

Ans. Not conservative


2.63.

field
F = 3x 2 i + (2xz y)j + zk along
the straight line from (0, 0, 0) to (2, 1, 3), (6) the space curve x = 2t2 y = t, z = 4t2 t from t =
to t = l. Is the work independent of the path? Explain.
Ans. (a) 16, (6) 14.2

Find the work done in moving a particle in the force

(a)

2.64.

2.65.

(a)

Evaluate

to

(x
t

3y)i +

= 2v.

(b)

(y

2#)j

and

Give a physical interpretation to the result in

F = kt^t

(a)

Show

Write the potential energy of a particle moving in the force

(c)

If a particle at

that the force field

is

(a).

conservative.

m moves with velocity v = dr/dt in

mass

the closed curve in the xy plane

is

the positive (counterclockwise) direction.

(6)

total

2.66.

F dt where F =

d>

Jc
x 2 cos t, y = 3 sin t from t =
Ans. (a) 6jt if C is traversed in

energy then ^m(dr/dt) z

+ ^kt5 = E. What

field

of

(a).

if E is the constant
important physical principle does this illustrate?

this field,

show that

A particle of mass 4 moves in the force field defined by F = 200r/r3. (a) Show that the field is
conservative and find the potential energy. (6) If a particle starts at r = 1 with speed 20, what will
be

speed at r

its

IMPULSE, TORQUE

= 2?

Ans.

(a)

V-

200/r,

(6)

15^

AND ANGULAR MOMENTUM.

CONSERVATION OF MOMENTUM
2.67.

(6) If

Ans.
2.68.

2.69.

t is

the time,

12j

particle of

the torque
particle

=1

19k,

mass
and

moves

in a force
Find the change

(a)

the velocity at
(a) i

(a)

mass moves

particle of unit

where

m
(b)

in

is 4i

(6) 5i

field

in

given by

F =

(3t2

moves along a space curve defined by

a force

field

momentum about

F=

given by

(12t

- 6)j + (6* - 12^


t 1
to t = 2.

of the particle from time


is the velocity at * = 2?

5j + 10k, what
7j - 9k

the angular

- te)i +

momentum

<f>(r)

the origin.

r.

a cos at i + 6 sin at j. Find


Ans. (a) 0, (6) 2mabak

Prove that the angular momentum of the

particle about the origin is constant.


2.70

Find

the torque and

(a)

particle of

Ans.
2.71.

(a)

Problem
(36i

2.67,

128j

(6)

the angular

+ 60k),

(6)

momentum about

= it is
- 44i + 52j + 16k

assuming that at

Find the impulse developed by a force given by


Ans.

8i

12j

24tk

the origin at the time


located at the origin.

2)j +

F =

4i

(6t 2

12k from

=2

for the

to

= 2.

NEWTON'S LAWS OF MOTION. WORK, ENERGY AND MOMENTUM

58

2.72.

What
5i

is

3j +

[CHAP.

the magnitude of the impulse developed by a mass of 200 gm which changes its velocity from
Ans. 1.8 X 10 5 dyne sec or 1.8 newton sec
7k m/sec to 2i + 3j + k m/sec?

STATICS OF A PARTICLE
2.73.

acted upon by the forces F x = 2i + aj


Find the values of the constants a,
Ans. a 1, 6 = 11, c = 4

particle is

equilibrium.

2.74.

Find

graphically and

(a)

+ cj +

6k,

F3 =

6i

- 5j + 7k,

Ans.

(6) 19.5 dynes in a direction


8522' with the negative x axis

making an angle

The potential of a particle moving in the xy plane is


(a) Prove
given by V = 2x 2 5xy + Sy 2 + 6x-7y.
that there will be one and only one point at which a
particle will remain in equilibrium and (6) find the
Ans.

coordinates of this point.

2.76.

5i

order that the particle will be in

in

b, c

analytically the resultof Fig. 2-13 where

(6)

ant force acting on the mass


all forces are in a plane.

2.75.

- 3k, F2 =

ci 6j + ak.

F4 =

(1, 2)

(6)

Prove that a particle which moves in a force field


of potential

V -

x2

Ay2

can remain

z2

Axy

4yz

2xz

4a;

Sy

many

in equilibrium at infinitely

and locate these

4z

points

points.
Fig. 2-13

2y-Vz 2

Ans. All points on the plane x

STABILITY OF EQUILIBRIUM
2.77.

particle

Ans. x

is

2.77 if

x-\,2
x
is

(a)
(6)

Work Problem

(a)

V=

a;

2.80.

If

2.77 if

V=

0,

moves

is

sin

(c)

Ans.

2.83.

(a)

cos x

field.

(tr/2,

V =

(6)

V=

-1

x4

is

a point of unstable equilibrium.

= f+n

z s )i

are points of stable equilibrium, while x

V=

(2y sin x

x2

- 4)j +

+ y 2 + z 2 - Sx + 16y - 4z.

(3a;z 2

2)k

in

Find the potential corresponding


moving a particle in this field from

4y

-1,2).

y 2 sin x

xz 3

2z

c,

(6)

15

4n-

particle P is acted upon by 3 coplanar forces as indicated in


Fig. 2-14. Find the force needed to prevent P from moving.
Ans. 323 lb in a direction opposite to 150 lb force

(a)

Prove that

ing potential.

F=

r^x

Ans.

is

conservative and

(6)

V = -^r4 +

(6) find

the correspond-

+n

Find the points

(6)

Find the work done

(0,1,-1) to

x).

2^-a;.

in

a conservative force

to F.

x 2 (6

a point of stable equilibrium

a force field with potential


Ans. (4, 8, 2)
of stable equilibrium.
particle

V=

a point of unstable equilibrium

is

are points of stable equilibrium; x

MISCELLANEOUS PROBLEMS
2
(a) Prove that F = {y
2.81.

2.82.

=4

- 8a; 3 - 6a; 2 + 24a;,

then x
1, 2, 3,
are points of unstable equilibrium.

Ans.

having potential

investigate their stability.

(6)

a point of stable equilibrium; x

Work Problem
Ans.

2.79.

in a force field

Find the points at equilibrium and

(a)

2.78.

moves on the x axis

100 lb

Fig. 2-14

CHAP.

2.84.

2.85.

NEWTON'S LAWS OF MOTION. WORK, ENERGY AND MOMENTUM

2]

59

Explain the following paradox: According to Newton's third law a trailer pulls back on an automobile to which it is attached with as much force as the auto pulls forward on the trailer. Therefore
the auto cannot move.

Find the potential of a particle placed in a force


Treat all cases.

field

F = Kr~ nr where

given by

and n are

constants.

2.86.

2.87.

A waterfall 500 ft high has 440,000 ft3 of water flowing over it per second. Assuming that the
density of water is 62.5 lb/ft3 and that 1 horsepower is 550 ft lb/sec, find the horsepower of the
Ans. 25 X 10 6 hp
waterfall.
The power applied to a particle by a force field is given as a function of time t by <P(t) = 3*2 At +
Find the work done in moving the particle from the point where t = 2 to the point where t = 4.

2.

Ans. 36

2.88.

Can the torque on a

2.89.

Can

2.90.

Ans.

2.91.

the force on a particle be zero without the angular

Under the
r

particle be zero without the force being zero?

influence

of a force field

Explain.

momentum

being zero?

a particle of mass 2 moves

Explain.

along the space curve


the point where

5)k. Find (a) the work done in moving the particle from
to the point where t = l, (6) the power applied to the particle at any time.
(a) 756
(6) 72*(48i 4 + St + 1)
6t4 i

3* 2j

(4t 3

force field moves a particle of mass


along the space curve r = acosi+ ftsinwi j. (a) What
is required?
Ans. (a) m(a 2 6 2 )3 sinwt coswi
(6) Discuss physically the case a b.

power

2.92.

The angular momentum of a

particle is given as a function of time

12

Find the torque at the time

2.93.

2.94.

= l.

(2t

Ans. 12i

l)j

constant force of 100 newtons


What is the speed achieved?

Ans.

(a)

600 m/sec,

particle of

given by

mass

(6)

36,000

is
(6)

(12i3

by

- 8*2 )k

2j + 20k

Find the constant force needed to give an object of


starting from rest.
Ans. 1760 poundals

(a)

2.95.

6tH

mass 36,000

lb

a speed of 10 mi/hr in 5 minutes

applied for 2 minutes to a 20 kg mass which


What is the distance traveled?

is initially

at rest,

m moves on the x axis under the influence of a force of attraction toward origin O

F = (k/x 2 )L

If the particle starts

from

rest at x

= a,

prove that

it will

arrive at

in

time given by ^iray'ma/2*.

2.96.

2.97.

2.98.

Work Problem

2.95 if

F = -( K /x*)i.

mass 2 units moves in the force field F = t2 i Btj + (t + 2)k where t is the time.
far does the particle move from * =
to t = 3 if it is initially at rest at the origin?
(6) Find the kinetic energy at times t = 1 and t = 3. (c) What is the work done on the particle by
the field from t = 1 to t = 3? (d) What is the power applied to the particle at * = 1? (e) What is the
impulse supplied to the particle at t = 1 ?
particle of

(a)

How

At

= 0a

particle of unit

mass

is

at rest at the origin. If

it is

acted upon by a force

change in momentum of the particle in going from time


after a long time has elapsed.
Ans. (a) 25e _2 (3 5e -2 )i, (6) 25

find (a) the

=1

to

= 2,

F = 100te _2t i,

(6)

the velocity

NEWTON'S LAWS OF MOTION. WORK, ENERGY AND MOMENTUM

60

2.99.

63

+ 12y 3 + S6xy 48s2

from

its

equilibrium position.

= 0, y =

particle of unit

6x(x 2).

V=
is

Near x

(a)

in the xy plane under the influence of a force field having potential


Investigate the motion of the particle if it is displaced slightly

moves

3 units

V =

[Hint.

2.100.

mass

particle of

[CHAP.

the potential

is

very nearly 36xy

- 48x 2

since

6a; 3

and 12ys are

negligible.]

field having potential


a position of stable equilibrium. (6) Prove that if the mass
position of equilibrium it will oscillate about it with period equal to

mass moves on the x axis under the influence of a force

Show that x =

displaced slightly

from

its

1 is

4rv 3.
[Hint.

2.101.

Let x

+u

and neglect terms

in

u of degree higher than

one.]

mass m moves in a force field F = kx\. (a) How much work is done in moving the
from x = x x to x = x 2 ' (*) If a unit particle starts at x = x lt with speed v lt what is its speed

particle of

particle

= x2 1

on reaching x

2.102.

Ans.

%k{x\

(a)

- x\),

(6)

V v2 +

(ic/m)(cc

a;

2
)

the xy plane under the influence of a force field having potential


from rest at the point (2, 1). (a) Set up the differential
at time t =
(c) Find the
motion.
the
(6) Find the position at any time t.
equations and conditions describing
velocity at any time t.

A particle, of mass 2 moves in


y = z 2 + y The particle starts
2.

V - Bxy.

2.103.

Work Problem

2.104.

Does Theorem 2.7, page


Prove your answer.

2.105.

(a)

2.102

if

36, hold relative to

a non-inertial frame of reference or coordinate system?

field having potential


if a particle moves in the xy plane under the influence of a force
(6) Discuss the relationship
is a point of stable equilibrium.
12x(Sy - 4x), then x = 0, y =
of the result in (a) to Problem 2.37, page 53.

Prove that

V =

2.106.

(a)

Prove that a
V(x, y)

is

(i)

(6)

Use

y =
2.107.

2.108.

(a) to
jj.3

sufficient condition for the point (a,b) to be

that at

V =

to

Z=0,
W=

(ii)

"'

^ (SrYSUS^
*
VteVV*
8

"

any time.

particle

\s

point of the function

and^>0

>

having potential
investigate the points of stability of a particle moving in a force field
stability
of
point
z - 33. _ \2y.
is
a
point
The
(1,
(b)
2)
Ans.
y

Suppose that a particle of unit mass moves in the force


at

minimum

(a, b)

moves once around the

field

+ sin e
F = (xi-yj)/(x 2 + y 2

circle

a(cos B

of Problem 2.106.

j)

in

a force

Find

its

speed

field

(6) Is the force field conservative?


(a) Find the work done.
35? Explain.
page
contradict Theorem 2.4,

2.109.

(c)

Do your answers

to (a)

and

(6)

the assumption that space and


sometimes stated that classical or Newtonian mechanics makes
statement.
this
by
meant
is
time are both absolute. Discuss what
It is

ft2
2.110.

The quantity

Fav =

Fdt

J
1

I
2

Does the result

(3)

is

called the average force acting

on a particle from time

of Problem 2.5, page 40, hold if

is

replaced by

F av ?

Explain.

tx

to

t2 .

CHAP.

2.111.

2]

NEWTON'S LAWS OF MOTION. WORK, ENERGY AND MOMENTUM


particle of

mass 2

gm

moves

2.112.

(a)

1),

what

is its

Find positions of stable equilibrium of a particle moving

V=

2.113.

2,

Sxyi + (4x 2 8z)j


speed at (1, 1, 1)?

F =

in the force field

a speed of 4 cm/sec at the point (1,

in

Syk. dynes.

a force

field

of

potential

18r2 e-2r.

= ,

find the speed

when

reaches the equilibrium position,

(o)

If the particle is released at r

(c)

Find the period for small oscillations about the equilibrium position.

it

of a particle moving with speed


According to Einstein's special theory of relativity the mass
m = m /Vl v2 /c2 where c is the speed of light [186,000
is given by
mi/sec] and m is the rest mass. What is the percent increase in rest mass of (a) an airplane moving
at 700 mi/hr, (6) a planet moving at 25,000 mi/hr, (c) an electron moving at half the speed of light?
What conclusions do you draw from these results?

Prove that in cylindrical coordinates,

dV
ldV +
VF = ^-e
e + i ^-e*
"
dp p
30 **

dV

^-e,

dz

where ep

2.115.

If it has

Ans. 6 cm/sec

v relative to an observer

2.114.

61

e^,

Prove that

ez are unit vectors in the direction of increasing

<f>

and

z respectively.

in spherical coordinates,

Vy =
T7T/

where e r e e e^ are unit vectors


,

p,

^
dV

+
.

dV

3V

_l
e
Jt e + ^In7"s7 *

in the direction of increasing r, e,

<f>

respectively.

UNIFORM FORCE FIELDS


A force field which has constant
tude and direction

is called

urn

magni-

a uniform or con-

-Fk

stant force field. If the direction of this field


is taken as the negative z direction as indi-

cated in Fig. 3-1 and the magnitude is the


constant F > 0, then the force field is given by

F = -F k

Fig. 3-1

{1)

UNIFORMLY ACCELERATED MOTION

moves in a uniform force field, then its acceleration


a particle of constant mass
is uniform or constant. The motion is then described as uniformly accelerated motion.
moving in the uniform force
Using F = ma in (1), the acceleration of a particle of mass
field (1) is given by

If

-^k
m

(*)
v

WEIGHT AND ACCELERATION DUE TO GRAVITY


It is found experimentally that near the earth's
surface objects fall with a vertical acceleration
which is constant provided that air resistance is
negligible. This acceleration is denoted by g and
is called the acceleration due to gravity or the

gravitational acceleration.

The approximate mag-

nitude of g is 980 cm/sec 9.80 m/sec2 or 32 ft/sec 2


according as the cgs, mks or fps system of units
is used. This value varies at different parts of the
earth's surface, increasing slightly as one goes
from the equator to the poles.
2

Assuming the surface of the earth is represented by the xy plane of Fig. 3-2, the force acting
is given by
on a particle of mass

= mgk

(3)

This force, which is called the weight of the par mg.


has magnitude

ticle,

62

Fig. 3-2

CHAP.

3]

MOTION IN A UNIFORM FIELD. FALLING BODIES AND PROJECTILES

63

GRAVITATIONAL SYSTEM OF UNITS

= mg, it follows that = Wig. This fact has led many scientists and engideal to a large extent with mechanics on the earth's surface, to rewrite the
equations of motion with the fundamental mass quantity
replaced by the weight quantity
Because

neers,

who

W.

Thus, for example, Newton's second law

F =

is

rewritten as

fa

In this equation
= Wig is constant. One system of
and g can both vary while
is
the
gravitational
or
English
engineering system where the unit of F or
(4)
is the pound weight (lb wt) while length is in feet and time is in seconds.
In this case
the unit of
is the slug and the system is often called the foot-slug -second (fss) system.
Other systems are also possible. For example, we can take F or
in kilograms weight
(kg wt) with length in meters and time in seconds.
units used in

ASSUMPTION OF A FLAT EARTH

is

Equation (3) indicates that the force acting on mass


has constant magnitude mg and
at each point directed perpendicular to the earth's surface represented by the xy plane.

In reality this assumption, called the assumption of the flat earth, is not correct first because
the earth is not flat and second because the force acting on mass m actually varies with the
distance from the center of the earth, as shown in Chapter 5.
In practice the assumption of a flat earth is quite accurate for describing motions of
objects at or near the earth's surface and will be used throughout this chapter.
However,
for describing the motion of objects far from the earth's surface the methods of
Chapter 5

must be employed.

FREELY FALLING BODIES


If an object moves so that the only force acting upon it is its
weight, or force due to
gravity, then the object is often called a freely falling body. If r is the
position vector and
is the mass of the body, then using Newton's second law
the differential equation of
motion is seen from equation (3) to be

m ddT>r

= ~ mgk
,

or

d*r

~
d = 9k

Since this equation does not involve the mass m, the motion of a freely falling
body
independent of its mass.

PROJECTILES
An object fired from a gun or dropped from

(*)

is

a moving airplane is often called a projectile.


a projectile can be considered as a freely falling body so that
its motion can be found from equation
(5) together with appropriate initial conditions. If air
resistance is negligible the path of a projectile is an arc of a
parabola (or a straight line
which can be considered a degenerate parabola). See Problem 3.6.
If air resistance is negligible,

MOTION IN A UNIFORM FIELD. FALLING BODIES AND PROJECTILES

64

[CHAP.

POTENTIAL AND POTENTIAL ENERGY


IN A UNIFORM FORCE FIELD
potential of the uniform force

The
field, is

where

given by

is

field,

or potential energy of a particle in this force


,

V =

Fo(z-zo)

an arbitrary constant such that when

(6)

= z V = 0. We
,

call z

=z

the reference

level.

In particular for a constant gravitational


particle

field,

F = mg

and the potential energy of the

is

V = mg(z-z

v)

This leads to
potential energy of a particle in a constant gravitational field is
found by multiplying the magnitude of its weight by the height above some prescribed
reference level. Note that the potential energy is the work done by the weight in moving

Theorem

3.1.

The

through the distance

zo.

MOTION IN A RESISTING MEDIUM


well.
In practice an object is acted upon not only by its weight but by other forces as
Such
object.
an
of
motion
the
oppose
to
tend
which
those
are
An important class of forces
air or water, are
forces, which generally arise because of motion in some medium such as
medium is said
corresponding
the
and
forces
dissipative
or
damping
often called resisting,

to be a resisting,

damping or

dissipative

medium.

found experimentally that for low speeds the resisting force is in magnitude proporother power]
tional to the speed. In other cases it may be proportional to the square [or some
m in an
mass
of
particle
of
a
motion
the
then
R,
is
force
of the speed. If the resisting
It is

otherwise uniform (gravitational) force

field is

m-^
dt
2

If

R=

given by

mgk R

(#)

this reduces to (5).

ISOLATING THE SYSTEM


particles, as we shall
In dealing with the dynamics or statics of a particle [or a system of
which act on the
see later] it is extremely important to take into account all those forces
the system.
isolating
particle [or on the system of particles]. This process is often called

CONSTRAINED MOTION
surface as, for
In some cases a particle P must move along some specified curve or
bowl of
hemispherical
of
a
surface
inner
3-3
the
or
example, the inclined plane of Fig.
is called a
move
must
particle
the
which
on
surface
or
Such a curve
Fig. 3-4 below.
constraint and the resulting motion is called constrained motion.
third law
Just as the particle exerts a force on the constraint, there will by Newton's
described
often
is
force
reaction
This
particle.
the
on
constraint
be a reaction force of the
motion
of
direction
the
parallel
to
and
to
normal
f,
and
N
by giving its components
and is
friction
to
due
force
the
f
is
practice,
in
arise
which
respectively. In most cases
motion.
the
taken in a direction opposing

CHAP.

3]

MOTION IN A UNIFORM FIELD. FALLING BODIES AND PROJECTILES

65

n\

>&^

s^

s^\ a
Fig. 3-4

Fig. 3-3

Problems involving constrained motion can be solved by using Newton's second law
and then solving these equations subject to

to arrive at differential equations for the motion


initial conditions.

FRICTION
In the constrained motion of particles, one of the
most important forces resisting motion is that due to
friction. Referring to Fig. 3-5, let N be the magnitude
of the normal component of the reaction of the constraint on the particle m. Then it is found experimentally that the magnitude of the force f due to
friction is given by
/

where

/*

is

/JV

(P)

Fig. 3-5

called the coefficient of friction. The direction of f is always opposite to the


The coefficient of friction, which depends on the material of both

direction of motion.

the particle and constraint,

is

taken as a constant in practice.

STATICS IN A UNIFORM GRAVITATIONAL FIELD


As
forces

indicated in Chapter 2, a particle is in equilibrium under the influence of a system of


and only if the net force acting on it is F = 0.

if

Solved Problems

UNIFORM FORCE FIELDS AND UNIFORMLY ACCELERATED MOTION


3.1.
A particle of mass m moves along a
Fi
straight line under the influence of a constant force of magnitude F. If its initial

speed

is

velocity

find

and

after time

t.

(c)

the speed, (b) the


the distance traveled

(a)

Fig. 3-6

MOTION IN A UNIFORM FIELD. FALLING BODIES AND PROJECTILES

66

(a)

the straight line along which the particle P moves is the x axis, as shown in
Fig. 3-6 above. Suppose that at time t the particle is at a distance x from origin 0. If i is a
unit vector in the direction OP and v is the speed at time t, then the velocity is vi. By Newton's

we have

4r(mvi)

Fi

= F
m^j
dt

or

dt

= dt

dv

Thus

where
t

a constant of integration.
and
so that from (2), c x = v
is

c-^

From

the velocity at time

(5)

= t+C!

To

find c x

Since v

vi,

dse/d*

and F

Fi.

at

0,

we

{2)

note the initial condition that

or

ti

* +
c 2 to

+ t

at

(S)

F
H

find

c2

*=(, + )*

be the constant of integration,

*+

0.

Thus

(k)."

*+

3.2.

we

or

*
a;

dt

(8),

Then on integrating, assuming

Since

If

we have from

dv

(1)

t is

vi

where v

or

i.e.

(c)

Assume that

second law

(6)

[CHAP.

we have

+ *

()*

Referring to Pr oblem 3.1, sho w that the speed of the particle at any position x
vl + (2F/m)x.
given by v =

is

Method

1.

From
we

find

Method

2.

From

(S)

we have

of Problem 3.1,

- v).

(m/2F)(v 2

{1)

of Problem 3.1,

_ F
~ m'

'

i.

Method 3.
Change

when g =

in kinetic

= Work
or

$mv* - %mv%

0,

we

find

energy from

done in moving

F(x

- 0). Then

c3

g/2

to

particle

vdv = ?-dx

e.

_ F

v2

_ F

dx/dt,

Integrating,

dv dx
dx dt

*1 = ,
dx
m'

Since v

Substituting into
)/F.
obtain the required result.

we have
dt

we

Solving for v

dv
or since v

= m(v - v

J*

+
,

c3

and hence v

any time

from *

V{j

(2F/m)x.

= V^ + (2F/m)x.

to

any position x

(4)

and simplifying,

CHAP.

3]

MOTION IN A UNIFORM FIELD. FALLING BODIES AND PROJECTILES

67

LINEAR MOTION OF FREELY


FALLING BODIES
3.3.

object of mass m is thrown vertically upward from the earth's surface with speed vo.

An

F=

Find (a) the position at any time, (b) the time


taken to reach the highest point and (c) the
maximum height reached.
(a)

Let the position vector of


at any time t be
r = xi + yj + zk.
Assume that the object starts
when t = 0. Since the force acting on
at r =
the object is mgk, we have by Newton's law,

d2 r

dv
m-r-

dt2

where v

the velocity at time

is

= mgk
.

Integrating

t.

= gtk +

V(jk

-^ =

or

Then

since

yields

(4)

when

r
t

c2

0,

(6)

The highest point

(c)

At time

or, equivalently,

= VfJg

is

maximum

the

0,

reached when v

(2)

is v<jk>

we have from

et

(2),

Vok so that

gt)k

(v

(S)

(*)

%gt2 )k +

Thus the

0.

ct

-gt)k

(v

(v t

(J)

-fl*

once yields

the initial velocity]

[i.e.

Integrating

(1)

dt

= -gtk +

v
Since the velocity at

Fig. 3-7

dv

c2

(5)

position vector is

(v Q t-$gt2)k

0,

gt)k =

(v

height reached

is,

(6)

^gt2

(7)

0, i.e.

at time

from

(7),

vjg.

= i%/2g.

Another method.
If we assume, as is physically evident, that the object must always be on the z axis, we may
avoid vectors by writing Newton's law equivalently as [see equation (1) above and place r = zk]

= -g
= 0, we find
z = v t - %gt 2
cPz/dt2

from which, using

as above.

3.4.

= 0,

The answers

dz/dt

to (6)

=v

and

at

(c)

are then obtained as before.

Find the speed of the particle of Problem 3.3 in terms of


Method

1.

From Problem

3.3,

equations

v
Solving for

in the first equation

/v

=
Method

2.

From

Vo

dt

3.

(7),

gt,

(1)

distance

from origin O.

we have

\gt2

and substituting into the second equation, we

-v\

= g,

Then on integrating, vV2


Method

and

/v

2 _ 2
-v\ 2 = v^-v
-*-

\-T-)-tg (-T-)

equation

dv

(3)

its

of Problem 3.3

have, since v

dv dz

dz

i.e.

= gz + c 3

we

dt

Since

_
=
~ g9

=v

at

= vk
__.

..

or
z

= 0,

and v

c3

2gz

dz/dt,

dv

Tz =

= v^/2

find

~g

and thus v2

See Problem 3.9 for a method using the principle of conservation of energy.

v2

2gz.

MOTION IN A UNIFORM FIELD. FALLING BODIES AND PROJECTILES

68

[CHAP.

MOTION OF PROJECTILES
3.5.
A projectile is launched with initial speed
v at an angle a with the horizontal. Find
(a) the position vector at any time, (b) the
time to reach the highest point, (c) the
maximum height reached, (d) the time of
flight
(a)

back to earth and

the range.

(e)

Let r be the position vector of the projecThen


tile and v the velocity at any time t.
by Newton's law

d2 r

i.e.,

dv

dt2

v
v

at

we

0,

Replacing v by dr/dt in

(5)

It follows that the projectile

(b)

At

(2)

cx

the yz plane so that the initial velocity

is in
j

(v

sin a

cos a

sin a

is

(4)

gt)k

(5)

(S),

v cos a

and integrating, we obtain


r

equivalents,

from

find

or,

= -gtk +

the initial velocity of the projectile

Assume

-flrk

Integration yields

Since

Fig. 3-8

(1)

=
0,

cos a)tj

(v

(v

cos a)*,

{(v Q sin
2

a)t- %gt2 }k

- \g&

sin a)*

(v

(6)

(7)

remains in the yz plane.

the highest point of the path the component of velocity v in the


Vq sin a
t =
and
gt =
v Q sin a

direction is zero.

Thus
(8)

is

(c)

the required time.

Using the value of

obtained in

we

(&),

find

from

Maximum
(d)

The time of

flight

back to earth
{v

or since

height reached

9
the time

y&
t

(e)

The range

is

is

twice the time in

Show

(v

cos a)

is

{v

sin a)(y/v

cos a)

2v sin a \

- y{yh

a parabola in the yz plane.

aj^sin 2 a

sina\ 2

2g

(9)

when

- gt] =
<i0)

(6).

_
-

the second equation of (7) in Problem 3.5,


into the third equation of (7) in Problem 3.5, we find

i.e.

t>

2v sin a

cos a) 2

i.e.,

vl sin 2a

2v sin a cos a

that the path of the projectile in Problem 3.5

0,

t[(v Q sin a)

From

which

the value of y at the time given by (10),

Range

3.6.

when

t * 0,

Note that this

-y

sin a)

(v

is

sin a)t

that

(7)

sin a

'v

is

a parabola.

we have
or

Substituting this

y/(v cosa).

y tan

(f//2 V )2/

sec2 a

CHAP.

3.7.

MOTION IN A UNIFORM FIELD. FALLING BODIES AND PROJECTILES

3]

Prove that the range of the projectile of Problem 3.5


ing angle a

maximum when

the launch-

45.

By Problem
or a

is

69

3.5(e)

the range

This

is (v sin 2a)/ g.

is

maximum when

sin 2a

1,

i.e.

2a

90

45.

POTENTIAL AND POTENTIAL ENERGY


IN A UNIFORM FORCE FIELD
3.8.

Prove that a uniform force field is conservative, (b) find the potential correspondin a
ing to this field and (c) deduce the potential energy of a particle of mass
uniform gravitational force field.

(a)

(a)

F = F

If the force field is as indicated in Fig. 3-1, then

V XF

d/dx

d/dy

d/dz

k.

We

have

-F
Thus the force

(b)

-F

F =

(c)

3.9.

dV.

dV. dV j -

= -VV

V = FQ z +

conservative.

field is

If

c.

V=

at

dy

dz

Then

k.

then c

dx

dV

dx

= F

=0,

0,

dy

V=F

and so

dV =
=
F
dV

(z

For a uniform gravitational force field, F = -mgls. [see Fig. 3-2, page
to F = mg. Then by part (b) the potential or potential energy is V

Work Problem
According

P.E. at z

v2

Then

v*

K.E. at z

^mvl

we have

P.E. at

mgz

K.E. at z

|mi) 2

- 2gz.

MOTION IN A RESISTING MEDIUM


3.10.

At time

parachutist [Fig. 3-9] having

a.

mg is located at z = and
traveling vertically downward with speed vo.
If the force or air resistance acting on the

weight of magnitude
is

parachute

is

proportional to the instantaneous

speed, find the (a) speed, (b) distance traveled

and
(a)

(c)

acceleration at

Assume
of
If

any time

> 0.

the parachutist (considered as a particle

mass m) is located at distance z from origin O.


k is a unit vector in the vertically downward

direction, then the weight is mgls. while the force


of air resistance is (3vk so that the net force
is

(mg

fiv)k.

Thus by Newton's law,


ra-r-k
at

= (mg

/3v)k

(1)

and corresponds
mg(z z Q).

62]

3.4 using the principle of conservation of energy.

to the principle of conservation of energy,

from which

dz

Fig. 3-9

MOTION

70

IN

A UNIFORM FIELD. FALLING BODIES AND PROJECTILES


dv

wi-jt
dt

i.e.

Integrating,

7f

Since

at

ln

Bv

mdv =
rmg Bv

or

mg ~ Bv ~
^

dt

ci

(*)

-jln

Then from

).

(mg-/3v)

(2),

jm

Thus

(b)

X-7*

From

(5),

= mg/B +

dz/dt

at

c2

0,

e " t/m

(c)

From

(5),

the acceleration

3.11.

Show that
Method

is

~ &t/m

{s)

integration,

mg/B)

C2

and thus

^+7 (*-?) a

-f-

(m/B)(v

f) e

Then by

^_^

v ~ 2

T+

m/
mg\ _ 3t/m
Mlm +
-jf^v --^je

= mpt

or

mg/B)e-W m

(v

Since

AM

= In (ra^r Bv

cx

0,

= jln(mg-pv

wgr

[CHAP.

-""-)

04)

given by

-(-f>-

e" m

('-S)'-"-

<*>

the parachutist of Problem 3.10 approaches a limiting speed given by mglp.

1.

equation (5) of Problem 3.10, v = mg/B + (v mg/B)e-W m


Then as t increases,
v approaches mg/B so that after a short time the parachutist is traveling with speed which is

From

practically constant.

Method

2.

approach a limiting speed, the limiting acceleration must be zero.


we have mg Bv hm =
or v lim = mg//3.

If the parachutist is to

Thus from equation

3.12.

(1)

of Problem 3.10

A particle of mass m is traveling along the x axis such that at t

it is

located at x

and has speed vo. The particle is acted upon by a force which opposes the motion
and has magnitude proportional to the square of the instantaneous speed. Find the
(a) speed, (b) position and (c) acceleration of the particle at any time t > 0.
(a)

Suppose particle P is at a distance x from O at


* =
and has speed v [see Fig. 3-10]. Then the
force F = Bv 2 i where B >
is a constant of
proportionality. By Newton's law,

m-rri
dt

Integrating,

when

0,

2
= Bv
^ i

or

-5v2

(6)

is

Since

Thus

(J)

1/v = Bt/m + c v
we have c = -1/iv
1

which

= dt

F = -Bv 2 i

Fig. 3-10

/3t

or

wv
tr
Bv t +

the speed.

From W,

f=

.r?+ m

Bv

'
.

Then

f
J

i.

=
(*

-*-* =
Jf Bv t + m
,

=
Bv

dt

Jf t + m/Bv

or

'

CHAP.

3]

MOTION IN A UNIFORM FIELD. FALLING BODIES AND PROJECTILES

Since

at

(c)

From

-^ln(-^V

c2

0,

(+

Thus

= f'( i+

ln

^)-f te)

_ k _ d ( mv \ =
~ dt
dt\pv t + mj

PmVQ
(f3v t

u\

+ m)*

Note that although the speed of the particle continually decreases,

it

never conies to

rest.

(a) speed and (b) acceleration of the particle of Problem 3.12 as a


function of the distance x from O.

Determine the

Method

From

1.

parts

and

(a)

(6)

of Problem 3.12,

+ m\

m In

/fiv

fiv Q t

s=jln(^J

and the acceleration

is

From

2.

dv

Pn

dt

equation

(1)

when x =

0,

c3

= /8v
m -=
ax
=

In v

v
v

-** /m

and

Thus

(4.)

we have
dv

n 2
= Bv
= mv-jdx

v = x.
m

In (v/v

Pvl _ 2Bx/m

dt

m-j- -77
dx dt

dt

= 0,

/3v t

of Problem 3.12.

dv dx

dv

i>

_ Br/m dx

of Problem 3.12

m-77 =
or since

or

given in magnitude by

which can also be obtained from equation

Method

mv

"

and

Then

3.14.

^)

(a),

3.13.

71

lnv

Integrating,

= Bx/m

or v

VQe

fix/m

c3 .

Since

- ^/.

Suppose that in Problem 3.5 we assume that the projectile has acting upon it a force
due to air resistance equal to pv where /? is a positive constant and v is the instantaneous velocity. Find (a) the velocity and (b) the position vector at any time.
(a)

The equation of motion

in this case is

cPt

m-j-g

Dividing by
be written as

= mgk

e Pt/m

Integration yields
initial

Using

eJ

and multiplying by the integrating factor

{ e fit/m y } =

The

dv
m ~Jt^~
Py =

or

/?v

velocity or velocity at

this in (2)

we

v
t

0/m dt

eP t/m ,

C)
the equation can

-gefit/m^

- e& t/m k

cx

(2)

v Q sin a k

(8)

is

Vo

v o c s o]

v Q cos a

find
ci

#k

v<>

sin a

^-k

-I

MOTION IN A UNIFORM FIELD. FALLING BODIES AND PROJECTILES

72

Thus

becomes on dividing by

(2)

(b)

Replacing v by dr/dt in

(4)

v sin a

k)e~^ m

and integrating, we

~"g-( v 0COSa

^(1

Using

(6) in (5),

at

m9

v sinak)e-^"

^ e-flt/m)k

<* jl

v
f-(t

<

(5)

(*)

'

find

sin a k)(l

- e-"t/m) - 2M

-/3t/m

_ 2?M

Prove that the projectile of Problem 3.14 attains a limiting velocity and
Method

Ca

/?

J8

we

(4)

0,

mv Q
= -^(cosa
j +

3.15.

e~W)k

find

fi

Since

(7)

find its value.

1.

Refer to equation (4) of Problem 3.14. As t increases, e~ t/m approaches zero.


velocity approaches a limiting value equal to v
= (mg/(3)k.
llm

Method

e^ t/m ,

cos a

(v

[CHAP.

Thus the

2.

If the projectile is to

from equation

(1)

approach a limiting velocity its limiting acceleration must be zero.


of Problem 3.14, -mgk - 0v llm =
or v lim = -(mg/p)k.

CONSTRAINED MOTION
3.16. A particle P of mass m

slides

Thus

without rolling

down a f rictionless inclined plane AB of angle a


[Fig. 3-11]. If it starts from rest at the top A
of the incline, find (a) the acceleration, (b) the
velocity and (c) the distance traveled after

time
(a)

t.

N/

N. pjf/

Since there is no friction the only forces acting


= mgk and the reon P are the weight
action force of the incline which is given by the
normal force N.

W = -mgk
mg

cos a e 2

from the top A of the inclined plane, we have


by Newton's second law
m-j-zisei)
since the resultant equal to

W+N

is

mg

sin a

d?s/d&

Thus the acceleration down the


(6)

Since

ds/dt

is

the speed,

dv/dt

on integrating. Using the


speed at any time t is

incline at

(2)

velocity is

the incline.

ve t

e^ as indicated
g

<J>

in Fig. 3-11.

From

(I)

we have

sin a

any time

or

flrsina

initial condition

(g sin ajte^

raflrsinae!

(2)

a constant equal to g sin

is

a.

can be written

The

Fig. 3-11

+ N =
-

si" a i

<^.

Let e x and e2 be unit vectors parallel and


perpendicular to the incline respectively. If we
denote by s the magnitude of the displacement

d2

inff

at

(g sin a)t
t

0,

ct

we have

cx

so that the

(g sin a)t

which has magnitude

(3)

(g sin a)t

in the direction e t

down

CHAP.

MOTION IN A UNIFORM FIELD. FALLING BODIES AND PROJECTILES

3]

(c)

Since

ds/dt,

can be written

(5)

ds/dt

on integrating. Using the


required distance traveled

condition

initial
is

,,

or

(g sin a)t

3.17.

2
%(9 sin a)t

at

we

0,

c2
c2

find

(a)

Since 8
I

(6)

W
/\

The speed

from equation

at B, the time t to reach the bottom is

2
^(g sin a )r

or

at

is

so that the

w,

AB

2
-|(p sin a)t

of the incline in Problem 3.16 is I, find (a) the time


speed at B.
particle to reach the bottom B of the incline and (&) the
If the length

73

(4)

taken for the

of Problem 3.16 given by

}/2l/(g sin a).

given from

of Problem 3.16 by

(5)

(g sin )r

V2tf* sin a.

MOTION INVOLVING FRICTION


3.18.

Work Problem

3.16

if

the inclined plane has

a constant coefficient of friction


(a)

N>

ju..

In this case there is, in addition to the forces


and N acting on P, a frictional force f [see Fig.
3-12] directed up the incline [in a direction opposite to the motion] and with magnitude

= pmg
v.mg

liN

ping cos a

i.e.

Then equation

(1) of Problem 3.16 is replaced

cP(*e t )

dt2

+ N +

<Ps/dt*

or

mg

sin a ti

mg

'

\/

e2

^>V/\^

*)'

cos a e 2

(2)

m-

vjW

W = -mgk

CO

cos a

Fig. 3-12

by

=
flr(sin

sin a x e x

mflr

/*

(S)

y.mg cos a ej

cos a)

g(sin a - ? cos a) provided


acceleration down the incline has the constant magnitude
great that the particle will
/icosa or tana > /* [otherwise the frictional force is so
not move at all].

Thus the
sin a

(6)

(c)

>

Replacing d2s/dt2 by dv/dt in U) and integrating as in part


speed at any time t to be
v = gr( S in a /t COS a)t
Replacing v by

<*/<**

in (5)

and integrating as
8

3.19.

^g(sina

(6)

in part (c) of

(i

cos

of Problem 3.16,

we

find the
(K x

lJ

Problem

3.16,

we

find

a)t 2

(6)

straight line OA [Fig. 3-13].


object slides on a surface of ice along the horizontal
comes to rest after
At a certain point in its path the speed is v and the object then 2
is v /2gx
traveling a distance xo. Prove that the coefficient of friction

An

Let * be the instantaneous distance of the


from O and suppose that at
mass
=
=
and dx/dt = v
0, x
time t

object of

Three forces act on the object, namely (1) the


= mg, (2) the normal force N of the
weight
ice surface on the object, and (3) the frictional

mg

force

Fig. 3-13

f.

By Newton's

second law

we

have,

if

is

the instantaneous speed,

= W + N +
w ^i
dt

(1)

MOTION IN A UNIFORM FIELD. FALLING BODIES AND PROJECTILES

74

But

N = -W

since

and the magnitude of


dv

Method

Write

1.

f is

m-^i =

-fimgi

dv dx
dx"di

-m

Integrating, using the fact that

when x

(4)

From

2.

we

(2)

see that the object

dv

(1)

becomes

= -**

Tx

we

0,

= -mx +

find

v\l1

or

(-4)

(i.e.,

or

and noting that x

(7)

to rest

ngt

t = 0, we
t- frgfi

at

dx/dt

or

comes
v

Substituting this into

figt

we

fimgi,

figdx

v*/2

Integrating again, using the fact that x

(7)

v*/2gx Q

(5)

have, on integrating and using the fact that v

From

becomes

-figx Q

Method

-fig

dt

or

at

v*/2

dv
-^ =

or

so that

as

(2)

v dv

since

= pN = ymg

Then

Then

[CHAP.

at

0,

figt

(6)

find
(7)

= 0) when
t v //xg

we

obtain the required result.

STATICS IN A UNIFORM GRAVITATIONAL FIELD


3.20.

particle of

mass

a and b from pegs

m is

and

suspended in equilibrium by two inelastic strings of lengths


B which are distant c apart. Find the tension in each string.

mgk

Fig. 3-14

Fig. 3-15

Let
denote the weight of the particle and T t and T 2 the respective tensions in the strings
of lengths a and b as indicated in Fig. 3-14. These forces are also indicated in Fig. 3-15 and are
assumed to lie in the plane of unit vectors j and k. By resolving T 1 and T2 into horizontal and
vertical components it is clear that

Tx

sin a

Tx

cos a

where T 1 and T2 are the magnitudes of T x


angles at A and B. Also we have
Since the particle

is in

equilibrium

if

.and

T2

T2

sin

respectively and

p k

+ T2

cos

where a and

ft j

/J

if

the net force acting on

it is

zero,

(T2 cos p

cos a

cos a)j

are the respective

= mgk.

and only

= T + T2 + W
= 2 sin a k T

T2

j,

+ T2

sin

/?

(2\ sin a

+ T2

+ T2

sin

cos

/? j

mgk

- mg)k

we have

CHAP.

3]

MOTION IN A UNIFORM FIELD. FALLING BODIES AND PROJECTILES

From

this

we must have

T2

cos

/3

we

Solving simultaneously,

- T

cos a

0,

+ T2

sin a

_ mg cos p
~ sin + )'

my

j,

sin

(<*

The angles a and

can be determined from the law

/?

a2

COS -1

sin

ft

- mg =

find

From

75

c2

cos a

+ /?)

(<*

of cosines as
b2

b2

c2

a2

26c

2ac

these the tensions can be expressed in terms of a,

b, c.

MISCELLANEOUS PROBLEMS
3.21.

An

inclined plane [Fig. 3-16] makes an


angle a with the horizontal. A projectile is
launched from the bottom A of the incline
with speed vo in a direction making an
angle jB with the horizontal.
(a)

Prove that the range R up the incline


given by
2v 2 sin (/? a) cos /?

R =

(b)

is

g COS z a

Prove that the

maximum

Fig. 3-16

range up the incline

g(l

(a)

As

achieved

is

in

Problem

3.5,

when

/?

equation

=
(6),

The equation of the

(v

cos

(v

(v

sin

(v

/?)

is

(1)

\gt 2

(2)

y tan a

(3)

see that the projectile's path

sirt /3)t

lgt2 }k

line in the yz plane] is

\gt 2

2v (sin

sin a)

{(v Q sin p)t

/?)*,

we

cos/?)j

incline [which is

Using equations (2) in


values of t where

a/2.

the position vector of the projectile at any time

or

tt/4

given by

Umax

and

is

/?

cos a

and

cos

[(v

and the

(3)t]

cos

/3

incline intersect for those

tana

sin a)

2v sin

g cos a

(/J

a)

g cos a

t =
gives the intersection point A. The second value of t yields point B
the required point. Using this second value of t in the first equation of (2), we find
that the required range R up the incline is

The value

which

is

R =

y sec a

2v sin
{v

cos

/3)
flf

(b)

Method

1.

The range

(/?

o]

2v sin

(/3

a)

cos 2 a

COS a

can be written by using the trigonometric identity


sin

cos

B =

R =

{sin (A

+ B) +

vl

g cos 2 a

{sin (2/3

sin (A

a)

- B)}

sin a}

cos

/?

MOTION IN A UNIFORM FIELD. FALLING BODIES AND PROJECTILES

76

This is a maximum when


value of this maximum is
(1

g cos z a

1,

i.e.

#(1

20

-a =

T/2

or

a/2

g(l

fi

tt/4,

and the

sin a)

sin 2 a)

(1

sin a)

sin a)

2.

The required result can


finding maxima and minima.
3.22.

- o) =

be obtained by the methods of differential calculus for

also

Two

particles of masses mi and


2 respectively are
connected by an inextensible string of negligible mass
which passes over a fixed frictionless pulley of negligible mass as shown in Fig. 3-17. Describe the motion
by finding (a) the acceleration of the particles and
(b) the tension in the string.
Let us first isolate mass m,. There are two forces acting

'mWWXW

it:
= x gk, and (2) the force due to
(1) its weight
yg
the string which is the tension T = Tk. If we call a = dk
the acceleration, then by Newton's law

on

m
Next we

ak

mass

m gk

Tk

m2

(1)

m2 g

There are two forces acting


2
on it:
its weight
m 2 g = m^k, and (2) the tension
(1)
T = Tk [the tension is the same throughout the string
since the mass of the string is assumed negligible and inextensible].

of ra2

is

isolate

Since the string

(1)

is

Wig

inextensible, the acceleration

a = ak. Then by Newton's law

m2ak =
From

and

(2)

m2gk

Fig. 3-17

Tk

(2)

we have
m^a

Solving simultaneously,

we

-m 2 a

T =

2m m2
m + m2

T,

find

m ~ m2
g,
mi + m2
i

Thus the

particles move with constant acceleration, one particle rising and the other falling.
In this pulley system, sometimes called Atwood's machine, the pulley can rotate. However,
since it is frictionless and has no mass [or negligible mass] the effect is the same as if the string
passed over a smooth or frictionless peg instead of a pulley. In case the mass of the pulley
is not negligible, rotational effects must be taken into account and are considered in Chapter 9.

3.23.

particle

of

mass

m rests at the top A

frictionless fixed sphere of radius b.

of a

The par-

displaced slightly so that it slides (without rolling) down the sphere, (a) At what posi-

ticle is

it leave the sphere and (b) what will


speed be at this position?
The particle will slide down a circle of radius a
which we choose to be in the xy plane as indicated in
Fig. 3-18.
The forces acting on the particle are:

tion will
its

= mgj, and (2) the reaction force


weight
of the sphere on the particle normal to the sphere.

(1) its

Method
(a)

v,

R
Method

sin (2/3

[CHAP.

1.

Let the position of the particle on the circle be


measured by angle
and let v i and t be unit
vectors.
Resolving
into components in directions r x and $ lt we have as in Problem 1.43,
page 24,

Fig. 3-18

CHAP.

3]

MOTION IN A UNIFORM FIELD. FALLING BODIES AND PROJECTILES

=
=

+ (W #!)#,
r^ + (mg\

(W'r,)r,

( wgrj

= -mg

f x)*!

N = Nt

Also,

F = ma = m[(r
(2V

ra(r-r 2 )

While the particle

is

on the

Substituting

(4)

= g

sin

e,

we

(1)

= -mg

2r'e)

cos e

into the first equation of (S),

be = g

(3)

can be written

it

-*S (Bin#)

we

mg(Z

so that c x

Putting

sin 6

into

we

(4),

sin e

or

(5)

2)

if

Method

2.

is

we have v =

the speed,

By

the particle will

3 sin e

A +

be

K.E. at

+0

mgb

v2

or

Using the result of Problem


the radius of curvature

is

sin" 2/3

0,

i.e.,

(6)

2^/36

(7)

so that (7) yields

v2

= f&p

or

the conservation of energy, using the x axis as reference level,

P.E. at

find
e2

Then

and

find

N=

2/3

(4)

(2),

cos e

>
the particle stays on the sphere; but when
Now as long as
be just about to leave the sphere. Thus the required angle is given by
sin*

(2)

Substituting this into

b.

Now when * = tt/2,


6ff2 = 2gr(l - sin a)

N =

(6)

we have

26,

Tt\2j

c 1#

page

x]

we have

see that

d /e 2 \

1.49,

(r'o

N mg sin 0,

Multiplying the second equation by

be 2/2

m(r'e

circle (or sphere),

mbe 2 =

Integrating,

mflr

cos

+ 2re)$
sin tf)^ mg cose 9

re 2 )r x

W+N =
= N-mg sine,

mg

sin o r x

Using Newton's second law and the result of Problem

Thus

77

1.35,

page

P.E. at

mgb

2gb(l

20, together

P +

K.E. at

fymv 2

sin e

yj\bg.

we have

sin e)

()

with Newton's second law, we have, since

6,

w+n

= = Cf.-I'-) =
= (N mg sin e)r mg
x

cos e 6 X

Using only the r t component, we have


v 2 /b

From

(8)

Method

1.

N mg sine

and (9) we find N = mg(3 sin e - 2) which yields the required angle sin" 1
The speed is then found from (8).

(5)

() as in

MOTION IN A UNIFORM FIELD. FALLING BODIES AND PROJECTILES

78

[CHAP.

Supplementary Problems
UNIFORM FORCE FIELDS AND LINEAR MOTION OF
FREELY FALLING BODIES
3.24.
An object of mass m is dropped from a height H
negligible, then

is

3.25.

Work Problem
tude v

3.26.

3.27.

3.24 if the object is

Ans.

(y/v*

(a)

ground

(a) in

above the grou nd.


a time y/2H/g and

Prove that if air resistance


(b) with speed y/2gH.

thrown vertically downward with an

+ 2gH -

y/v*

(b)

)/g,

initial velocity of

magni-

+ 2gH

Prove that the object of Problem 3.3, page 67, returns to the earth's surface (a) with the same
speed as the initial speed and (6) in a time which is twice that taken to reach the maximum height.

ball

which

Ans.

(a)

thrown upward reaches

is

starting point,

3.28.

will reach the

it

With what speed was

(a)

80 ft/sec,

(6)

maximum

its
it

thrown?

height of 100 ft and then returns to the


How long does it take to return?

(6)

5 sec

which

is thrown vertically upward reaches a particular height


after a time t x on the
time t2 on the way down. Prove that (a) the initial velocity with which the ball was
thrown has magnitude $g(r t + t2 ) and (6) the height
= $griT2

ball

way up and a

3.29.

3.30.

3.31.

In Problem 3.28,

is

the

maximum

Ans. ^g(r t

height reached?

t2 ) 2

Two objects are dropped from the top of a cliff of height H. The second is dropped when the first
has traveled a distance D. Prove that at the instant when the first object has reached the bottom,
the second object is at a distance above it given by 2yDH D.

An
Ans.

from

elevator starts

160 lb

3.32.

what

man
(a)

rest

and attains a speed of 16 ft/sec in 2 sec. Find the weight of a


(a) moving up
(6) moving down.

in the elevator if the elevator is

200

lb,

120 lb

(6)

mass 3 kg moving in a straight line decelerates uniformly from a speed of 40 m/sec


m/sec in a distance of 300 m. (a) Find the magnitude of the deceleration. (6) How much
further does it travel before it comes to rest and how much longer will this take?
particle of

to 20

Ans.
3.33.

(a)

2 m/sec 2 ,

(6)

100 m; 10 sec

In Problem 3.32, what is the total work done in bringing the particle to rest
Ans. 2400 newton meters (or joules)
of 40 m/sec?

from the speed

MOTION OF PROJECTILES
3.34.

projectile is launched with a muzzle velocity of 1800 mi/hr at an angle of 60 with a


horizontal and lands on the same plane. Find (a) the maximum height reached, (6) the time
to reach the maximum height, (c) the total time of flight, (d) the range, (e) the speed after
1 minute of flight, (/) the speed at a height of 32,000 ft.

Ans.
3.35.

(a)

(a)

What

15.5 mi,

the

is

velocity 1 mi/sec

Ans.
3.36

A
is

3.37.

165 mi,

cannon has

# max and

its
(&)

71.4 sec,

(c)

maximum range
and
(6)

what
41.25 mi
(6)

is

maximum range

142.8 sec,

(d)

35.7 mi,

(e)

934 mi/hr,

(/)

1558 mi/hr

possible for a projectile fired from a cannon having muzzle


the height reached in this case?

given by

the time of flight

is

R max

Prove that

(a)

the height reached in such case

^R m& J2g.

launch a projectile from the ground so as to hit a given point on the ground
at a distance less than the maximum range. Prove that there are two possible angles
for the launching, one which is less than 45 by a certain amount and the other greater than
45 by the same amount.
It is desired to

which

3.38.

(a)

(6)

is

projectile

Prove that it must have


having horizontal range R reaches a maximum height
2
2
(6) at an angle with
(a) an initial speed equal to y/g(R + 16H )/8H and

been launched with

-1 (4H/y/R 2
the horizontal given by sin

16Jf? 2

).

CHAP.

3.39.

MOTION IN A UNIFORM FIELD. FALLING BODIES AND PROJECTILES

3]

projectile is launched at

79

an angle a from a

above sea level. If it falls into


of height
from the base of the
the sea at a distance
cliff, prove that its maximum height above sea
cliff

level is

H+

Ctan'q
+ Ptan)

_,_

Fl S- 3 " 19

4(H

MOTION IN A RESISTING MEDIUM


3.40.

Assuming that air resistance


is thrown vertically upward with speed v
object of weight
proportionality is k, prove
of
constant
the
that
and
velocity
instantaneous
to
the
proportional
is
that (a) the object will reach a maximum height of

An

and that

(6)

Wkv

K2g

K 2g

the time taken to reach this

maximum

Kg

3.41.

3.42.

mass

is

Assuming
line under the influence of a constant force F.
a resisting force numerically equal to kv 2 where v is the instantaneous speed and k

moves along a straight


is

a constant, prove that the distance traveled in going from speed v x to v 2

(6)

mass

w
^

In

/F-kv\
F_

moves

If it starts

Can Problem

is

in a straight line acted upon by a constant resisting force of magni(a) how long will it take before coming to rest and
with a speed of v
Ana. (a) mv /F, (b) mv2 /2F
what distance will it travel in this time?

particle of

tude F.

3.45.

height

on a parachute falls from rest and acquires a limiting speed of 15 mi/hr. Assuming
resistance is proportional to the instantaneous speed, determine how long it takes to reach
air
that
Arts. 1.86 sec
the speed of 14 mi/hr.

is

3.44.

KV o

A man

that there

3.43.

ln ( 1
\

3.43 be

locomotive of mass

Explain.

worked by energy considerations?

travels with constant speed v along a horizontal track, (a) How long
come to rest after the ignition is turned off, if the resistance
a + /3v 2 where v is the in stantaneous speed and a and /3 are constants?

will it take for the locomotive to


to the
(b)

3.46.

motion

What

is

is

given by

Ana.

the distance traveled?

(a)

y/mlp tan" 1

(v Q VJ/Z),

(6)

(m/2/3) In (1

+ prf/a)

moves along the x axis acted upon only by a resisting force which is proportional
the speed is
to the cube of the instantaneous speed. If the initial speed is v and after a time r
v prove that the speed will be v in time 5t.

particle

3.47.

Find the
(6) v
.

3.48.

total distance traveled

Ana.

(a)

r,

(6)

by the particle of Problem 3.46

(a)

the time to reach the highest point

(6)

the

mv
height

is

(a)

|v

>

iv

Prove that for the projectile of Problem

maximum

in reaching the speeds

sina
tj

is

page

3.14,

-j In

f
(

2
mpg In
,

71,
/?v

1 H

/
I

sina\
)

Joraa\
1 H

CONSTRAINED MOTION AND FRICTION


3.49.

weight of 100 lb slides from rest down a 60 incline of length 200 ft starting from the top.
Neglecting friction, (a) how long will it take to reach the bottom of the incline and (6) what is
Ana. (a) 3.80 sec, (&) 105.3 ft/sec
the speed with which it reaches the bottom?

MOTION IN A UNIFORM FIELD. FALLING BODIES AND PROJECTILES

80

3.50.

3.51.

Work Problem
(a)

3.49 if the coefficient of friction is 0.3.

Ans.

(a) 4.18 sec,

(6)

[CHAP.

95.7 ft/sec

With what speed should an object be thrown up a smooth incline of angle a and length
from the bottom, so as to just reach the top and (6) what is the time taken?

I,

starting

Ans.
3.52.

3.53.

(a) y/2gl sin a,

(b) s/2l/(g sin a)

If it takes a time t for an object starting from speed v


that the coefficient of friction is v Q/gr.

What

force

is

on an icy surface to come to

rest,

needed to move a 10 ton truck with uniform speed up an incline of 30


Ans. 5.87 tons

prove

the

if

coefficient of friction is 0.1 1


3.54.

3.55.

A mass rests on a horizontal piece of wood. The wood is tilted upward until the mass
begins to slide. If the angle which the wood makes with the horizontal at that instant
prove that the coefficient of friction is n = tan a.

just
is

a,

A 400 kg mass on a 30 inclined plane is acted upon by a force of 4800 newtons at angle 30
with the incline, as shown in Fig. 3-20.
Find the acceleration of the mass if the incline (a) is
frictionless,
Ans. (a) 5.5 m/sec 2 (6) 5.0 m/sec2
(6) has coefficient of friction 0.2.
,

Fig. 3-21

Fig. 3-20
3.56.

Work Problem
Ans.

5.5

(a)

3.55 if the force of 4800

m/sec 2

2.6

(6)

newtons acts as shown in Fig.

3-21.

m/sec 2

STATICS IN A UNIFORM GRAVITATIONAL FIELD


3.57.

A 100 kg weight is suspended vertically from the center of a rope as shown in Fig.
Ans. T = 100 kg wt = 980 nt
Determine the tension T in the rope.

3-22.

y///////////////^^^^^

Fig. 3-24

Fig. 3-23

3.58.

AB and AC are ropes attached to the ceiling CD and wall BD at C and B respectively.
If the ropes AB and AC make angles e x and e 2 with the
weight
is suspended from A.
wall and ceiling respectively, find the tensions T t and T2 in the ropes.
In Fig. 3-23,

Ans.

3.59.

T,

W cos
cos (0x

W sin e

62

e2

'

T2

cos

(e 1

e2

in equilibrium on the inclined plane


Find the magnitude of the force F needed to keep mass
of Fig. 3-24 if (a) the plane is smooth, (6) the plane has coefficient of friction p.
.

AnS

'

{a)

_
mg
F =

sin a

I^T'

...

(6)

__

mg (sin a

fi

c^sl

cos a)

CHAP.

3.60.

MOTION IN A UNIFORM FIELD. FALLING BODIES AND PROJECTILES

3]

81

needed to pull a train weighing 320 tons from rest to a speed of 15 mi/hr
and (a) the track is horizontal, (6) the track is
inclined at an angle of 10 with the horizontal and the train is going upward? [Use sin 10 = .1737,
Ans. (a) 17.4 tons, (6) 129.6 tons
cos 10 = .9848.]

How much

force

is

in 20 seconds if the coefficient of friction is 0.02

3.61.

Work Problem

3.62.

3.60(6) if the train is

mass

train of

Wfl^sin a

(i

cos a)

Ans. 3.6 tons

incline.

down an inclined plane of angle a and coefficient of friction fi


Prove that the force needed to stop the train in a time t is given by

coasting

is

with constant speed v Q

going down the

mt>

/r.

MISCELLANEOUS PROBLEMS
3.63.

stone

is

dropped down a well and the sound of the splash is heard after time t. A ssuming the
2
2
is c, prove that the depth of the water level in the well is (y/c + Igcr c) /2g.

speed of sound

3.64.

A projectile is launched downward from the top of an inclined plane of angle a in a direction
making an angle y with the incline. Assuming that the projectile hits the incline, prove that
(a)
v
'

the ranee

given by

is

R =

2i>o sin

3.65.

i? max

~"

a)

incline is

y cos (y

9 cos 2 a

n Ksin a)

,,,,.,
(6) the maximum
.

and that

.,

range down the

g(l

A cannon is located on a hill which has the shape of an inclined plane of angle a with the horizontal.
A projectile is fired from this cannon in a direction up the hill and making an angle with it. Prove
/3

that in order for the projectile to hit the

3.66.

3.67.

3.68.

3.69.

3.70.

3.71.

hill

Is it possible to solve equation (1),

page

33,

M/ 3 2_sincog2a2a

by the method of separation of variables?

Explain.

When launched at angle 9 1 with the horizontal a projectile falls a distance D x short of its target,
while at angle 2 it falls a distance D 2 beyond the target. Find the angle at which the projectile
should be launched so as to hit the target.

An object was thrown vertically downward. During the tenth second of travel it fell twice as far
Ans. 16 ft/sec
as during the fifth second. With what speed was it thrown?

gun of muzzle speed v is situated at height h above a horizontal plane. Prove that the angle
at which it must be fired so as to achieve the greatest range on the plane is given by
-1
e = % cos
gh/(vl + gh).
In Fig. 3-25, AB is a smooth table and masses ra x
and w2 are connected by a string over the smooth
peg at B. Find (a) the acceleration of mass m 2
and (6) the tension in the string.

-JJ

m2 m
^T+nT2 g m2>mi

m2

(o)
-

'

jm^
Wi +

3.73.

tan

Suppose that two projectiles are launched at angles a and /3 with the horizontal from the
same place at the same time in the same vertical plane and with the same initial speed. Prove
that during the course of the motion, the line joining the projectiles makes a constant angle
with the vertical given by (a + /3).

AnS

3.72.

horizontally we must have

Work Problem

Fig>3 . 25

m2
3.71 if the table

AB

has

coefficient of friction

ft.

The maximum range of a projectile when fired down an inclined plane is twice the maximum
fired up the inclined plane. Find the angle which the incline makes with the horizontal.

range when

Ans. sin -1 1/3

MOTION IN A UNIFORM FIELD. FALLING BODIES AND PROJECTILES

82

3.74.

Masses

[CHAP.

and ra2 are located on smooth inclined planes


and a 2 respectively and are connected by
an inextensible string of negligible mass which passes
over a smooth peg at A [Fig. 3-26]. Find the acceleram-^

of angles a t

tions of the masses.

Ans. The accelerations are in magnitude equal to

Wi

m2 sin a 2
9
m + m2

sin a x

Fig. 3-26

3.75.

Work Problem
Wj

sin

<*!

Ans.

3.76.

3.74 if the coefficient of friction between the masses

m2

nm x cos a
mi + m2

sin a2

and the

incline is

ju.

[im2 cos a 2

Prove that the least horizontal force F needed to pull


over an obstacle
a cylinder of radius a and weight
of height b [see Fig. 3-27] is given in magnitude by

Wy/b(2a-b)/(a 3.77.

3.78.

6).

Explain mathematically why a projectile fired from


cannon A at the top of a cliff at height H above the
ground can reach a cannon B located on the ground,
while a projectile fired from cannon B with the same
muzzle velocity will not be able to reach cannon A.

m hangs from an inextensible string OA.


by a horizontal string AB so that OA makes
an angle a with the vertical. Find the tension in each string.
Ans. Tension in AB = mg tan a; in OA = mg sec a
In Fig. 3-28 the mass

It is pulled aside

3.79.

3.81.

3.82.

3.83.

y////////////.

acted upon by a resisting


it to travel a distance x
t = Ax 2 + Bx + C where A, B and C are constants.
is given by
Prove that the magnitude of the resisting force is proportional
to the cube of the instantaneous speed.
particle

moving along the x axis

force which

3.80.

Fig. 3-27

is

such that the time

is

for

projectile is to be launched so as to go from A to B


[which are respectively at the bases of a double inclined
plane having angles a and /3 as shown in Fig. 3-29] and
If the distance
just barely miss a pole of height H.
between A and B is D, find the angle with the horizontal
at which the projectile should be launched.

particle of mass
moves on a frictionless inclined
plane of angle a and length I. If the particle starts
from rest at the top of the incline, what will be its
speed at the bottom assuming that air resistance is equal
to kv where v is the instantaneous speed and k is constant?

Fig. 3-29

Problem 3.23 the particle P is given an initial speed v Q at the top of the circle
(or sphere), (a) Prove that if v ^ yfgb, the angle e at which the particle leaves the circle is given
by sin -1 ( + v^/Zgb). (b) Discuss what happens if v Q > ygb.
Suppose that

in

above sea level.


is situated at the top of a vertical cliff overlooking the sea at height
should be the least muzzle velocity of the cannon in order that a projectile fired from it
will reach a ship at distance D from the foot of the cliff?

cannon

What

3.84.

In Problem 3.83,

(a)

how long would

velocity on reaching the ship?

it

take the projectile to reach the ship and

(6)

what

is

the

CHAP.

3]

3.85.

MOTION IN A UNIFORM FIELD. FALLING BODIES AND PROJECTILES

83

uniform chain of total length a has a portion


b < a hanging over the edge of a smooth table
AB [see Fig. 3-30]. Prove that the time taken for

<

the chain to slide off the table


is

3.86.

If

y/a/g\n(a

the

table

y/a 2

if it

starts

Fig. 3-30

Problem 3.85 has

in

V
A

weight

(i

coefficient

3.87.

from rest

-b 2 )/b.

+ fig

of

friction

+ V 2

In

hangs on one side of a smooth

6(1

p,

[6(1 + fi~
+ n) an

prove

[9(W 2

3.88.

mass

[see Fig. 3-31].

-W )-W a]/W
2

Two monkeys of equal weight are hanging from opposite ends of a


rope which passes over a smooth fixed pulley of negligible mass.
The first monkey starts to climb the rope at a speed of 1 ft/sec
while the other remains at rest relative to the rope. Describe the
motion of the second monkey.

is

W////////A

Fig. 3-31

1 ft/sec.

Prove that

+ 19^)6/81.

if friction is negligible

vertical circle starting

3.91.

taken

Prove that the particle of Problem 3.23 will land at a distance from the base of the sphere
given by (4\^90

3.90.

time

1.

Ans. The second monkey moves up at the rate of

3.89.

the

fixed pulley of neg-

A man

himself
of weight
2 pulls
up so that his acceleration relative to the fixed pulley is a.
Prove that the weight W^ moves upward with acceleration given by

ligible

that

from

the time taken for a particle to slide down any chord of a


is the same regardless of the chord.

rest at the top of the circle

AB of Fig. 3-32 and point P where AB and


are in the same vertical plane. Find a point Q on
AB such that a particle starting from point P will
reach Q in the shortest possible time.
Given line

[Hint.

3.92.

Use Problem

Show how

to

3.90.]

work Problem 3.91 if line AB is reCan it be done for a space

placed by a plane curve.


Explain.
curve?

3.93.

Find the work done in moving the mass from the top of the incline of Problem 3.18 to the bottom.
Ans. mgl(sm a

3.94.

3.95.

n cos a)

The force on a particle having electrical charge q and which is moving in a magnetic field of intensity
or strength B is given by F = g(v X B) where v is the instantaneous velocity. Prove that if the
particle is given an initial speed v in a plane perpendicular to a magnetic field B of constant
strength, then it (a) will travel with constant speed v Q and (6) will travel in a circular path
of radius mv /qB. Assume that gravitational forces are negligible.
Prove that the period, i.e. the time for one complete vibration, of the particle of Problem 3.94
Ans. 2irm/qB
independent of the speed of the particle and find its value.

is

3.96.

3.94 if B is constant and the particle is given an initial speed v in a plane which
not necessarily perpendicular to the magnetic field. Can we define a period in this case? Explain.

Work Problem
is

MOTION IN A UNIFORM FIELD. FALLING BODIES AND PROJECTILES

84

3.97.

If a particle of electrical charge q and mass m moves with velocity v


having electric intensity E and magnetic intensity B the force acting on
is

given by

F =

Suppose that
respectively.

whose equation

in

an electromagnetic

it,

called the Lorentz force,

field

+ vXB)

B and E are constant and in the directions of the negative y and positive z axes
Prove that if the particle starts from rest at the origin, then it will describe a

cycloid in the yz plane

where

g(E

[CHAP.

qBt/m,

= mE/qB 2

and

is

b(e

sin

e),

6(1

cos e)

the time.

is

3.98.

(a) An astronaut of 80 kg wt on the earth takes oft" vertically in a space ship which achieves
a speed of 2000 km/hr in 2 minutes. Assuming the acceleration to be constant, what is his apparent
weight during this time? (b) Work part (a) if the astronaut has 180 lb wt on the earth and the
space ship achieves a speed of 1280 mi/hr in 2 minutes.
Ans. (a) 117 kg wt, (6) 268 lb wt

3.99.

In Problem 3.82,

3.100.

how far from

the base of the sphere will the particle land?

that a force F
cot a ^ fii > n 2
while Wi not move relative to

that

is on top of weight
The
2 which is in turn on a horizontal plane.
t
between
Suppose
x and
2 is ^ while that between
2 and the plane is fi 2
inclined at angle a to the horizontal is applied to weight W^.
Prove that if
then a necessary and sufficient condition that
2 move relative to the plane

In Fig. 3-33 weight


coefficient of friction

2 is

COS a

n 2 sin a

W
W

< F g

COS a

jii

Sin a

Fig. 3-33

3.101.

Discuss the results in Problem 3.100

3.102.

Give a generalization of Problem 3.100.

3.103.

if

any of the conditions are not

Describe the motion of the particle of Problem 3.97

if

E and B are

satisfied.

constants, and have the

same

direction.
3.104.

bead of mass

vertical

is

located on a parabolic wire with its axis


in Fig. 3-34 and

and vertex directed downward as

whose equation

cz

is

x2

If the coefficient of friction is

above the x axis at which the


Ans. \p?c
particle will be in equilibrium.
fi,

3.105.

find the highest distance

Work Problem

3.104

circle of radius 6

3.106.

the parabola
is

is

replaced by a vertical

tangent to the x axis.

Fig. 3-34

weight
is suspended from 3 equal strings of length I which are attached to the 3 vertices
of a horizontal equilateral triangle of side s. Find the tensions in the strings.

Ans. Wl/^/912
3.107.

if

which

- 3s 2

Work Problem
having n

sides.

3.106 if there are

n equal

strings attached to the

vertices of a regular polygon

CHAP.

3.108.

MOTION IN A UNIFORM FIELD. FALLING BODIES AND PROJECTILES

3]

85

rope passes over a fixed pulley A of Fig. 3-35. At one end


At the other end of the
of this rope a mass
l is attached.
rope there is a pulley of mass
2 over which passes another
Prove that the accelrope with masses
1 and ra 2 attached.

eration of the

mass

is

given by

m 1 M2 m2M Am m2
9
+ m^Mi + M2 + 4m xm2

3m 2M2 m^M x
(m x

3.109.

S7\

KM
d

with an engine having constant


automobile of weight
instantaneous power "P, travels up an incline of angle a.
Assuming that resistance forces are r per unit weight, prove
that the maximum speed which can be maintained up the

W(r +

Fig. 3-35
sin a)

An

automobile of weight
moves up an incline of angle a, powered by an engine having
constant instantaneous power eP. Assuming that the resistance to motion is equal to kv per unit
weight where v is the instantaneous speed and k is a constant, prove that the maximum speed

which

3.111.

An

incline is

3.110.

is

possible on the incline

is

(y/W2

sin 2 a

4k

WP W sin <x)/2kW.

chain hangs over a smooth peg with length a on one side and length

6,

where

< b < a, on the


/
/
/v a+V&
In
>

other side. Prove that the time taken for the chain to slide off

3.112.

Prove that a bead

form of a

which

is

bottom

in the

given by

U+b
\ -
j

placed anywhere on a vertical frictionless wire [see Fig. 3-36] in the

cycloid

will reach the

is

b(e

sin e),

6(1

cos e)

same time regardless of the starting point and

Ans. vy/b/g
y

Fig. 3-36

find this time.

Chapter

The SIMPLE

J,,

HARMONIC

OSCILLATOR and the


SIMPLE PENDULUM

THE SIMPLE HARMONIC OSCILLATOR

m lies on a friction-

In Fig. 4-l(a) the mass

less horizontal table indicated

by the x axis.
attached to one end of a spring of negligible
mass and unstretched length I whose other end
is fixed at E.
It is

If

-nm^

m is given a displacement along the x axis

(a)

and released, it will vibrate or


back and forth about the equilibrium

[see Fig. 4-1(6)]

oscillate

position 0.

To determine the equation of motion, note


that at any instant when the spring has length
I + x
[Fig. 4-1(6)] there is a force tending to restore
to its equilibrium position. According

+x

-nmr^

to Hooke's law this force, called the restoring


force, is proportional to the stretch x

and

is

given by

Fig. 4-1

FR = - K xi
where the subscript

(1)

stands for "restoring force" and where is the constant of proportionality often called the spring constant, elastic constant, stiffness factor or modulus of
elasticity and i is the unit vector in the positive x direction. By Newton's second law we have
2

d (xi)
m ~aW

= ~ KX1

mx +

or

kx

(2)

This vibrating system is called a simple harmonic oscillator or linear harmonic oscillator.
This type of motion is often called simple harmonic motion.

AMPLITUDE, PERIOD AND FREQUENCY


OF SIMPLE HARMONIC MOTION
If

dx/dt

we

solve the differential equation (2) subject to the initial conditions


at t = 0, we find that

For the case where


Since cos

tat

A = 20,

= A cos o>t
m = 2 and

varies between

A graph of x vs.

where

o>

Problem

4.1.

1 and +1, the mass

oscillates

between x

appears in Fig.

8,

4-2.

86

= A and

s/K/m

see

(3)

= A and

A.

CHAP.

THE SIMPLE HARMONIC OSCILLATOR AND THE SIMPLE PENDULUM

4]

87

Fig. 4-2

The amplitude of the motion


equilibrium position.

is

the distance

and

is

the greatest distance from the

The period of the motion is the time for one complete oscillation or vibration [sometimes called a cycle] such as, for example, from x = A to x = A and then back to
x = A again. If P denotes the period, then

p =

2ttU

The frequency of the motion, denoted by /,

2ir^/mU
the

is

f = P = 2^ =
In the general case, the solution of

x
write

and

(6) in

the

number

of complete oscillations or

We have

cycles per unit time.

where

{U)

cos mt

2^\m

(5)

(2) is

+ B

sin

where

o>t

are determined from initial conditions.

= VkTm

As seen

in

Problem

(6)

4.2,

we can

form
x

and where

= C cos (<at <)

C =

y/A 2

+ B2

where
and

<f>

<o

y/T/m

(7)

tan' 1 (B/A)

(8)

The amplitude in this case is C while the period and frequency remain the same as in
(4) and (5), i.e. they are unaffected by change of initial conditions. The angle
is called
<f>

the phase angle or epoch chosen so that

</>

If

v.

0,

(7)

reduces to

(3).

ENERGY OF A SIMPLE HARMONIC OSCILLATOR


If T is the kinetic energy, V the potential energy and
simple harmonic oscillator, then we have

T = %mv 2
and

V =

E = \mv + \K x
2

$kx 2
2

E= T+V

the total energy of a

(9)

(10)

See Problem 4.17.

THE DAMPED HARMONIC OSCILLATOR


In practice various forces may act on a harmonic oscillator, tending to reduce the
magnitude of successive oscillations about the equilibrium position. Such forces are sometimes called damping forces. A useful approximate damping force is one which is proportional to the velocity and is given by
dx
FD = -Pv = -Pvi = -jB-^i
(11)

THE SIMPLE HARMONIC OSCILLATOR AND THE SIMPLE PENDULUM

88

[CHAP. 4

where the subscript D stands for "damping force" and where /? is a positive constant
called the damping coefficient. Note that FD and v are in opposite directions.

we assume the damping force (11), the


now called a damped harmonic oscillator, is

If in addition to the restoring force

of motion of the harmonic oscillator,

d2 x
m-T7z2
dt

kx
*"

dx

d2x

^ dt

'

on applying Newton's second law. Dividing by

/?/m

n dx
/3~rr

+
m^ns
""
dt2

or
WA

/?
M
dt

2y,

and

kItti

kX

equation
given by
(12)

calling
<o

(13)

this equation can be written

where the dots denote, as

+ 2y# +

2
o>

(U)

usual, differentiation with respect to

t.

OVER-DAMPED, CRITICALLY DAMPED AND

UNDER-DAMPED MOTION
Three cases arise in obtaining solutions

Case

1,

Over-damped motion,

>

<o ,

to the differential equation (14).


2

i.e.

/?

>

4ra

In this case (14) has the general solution

e' yt (Ae at

+ Be~ at

and where the arbitrary constants

Case

Critically

2,

where

and

2
2
y '= w ,

damped motion,

yV -

>

can be found from the


i.e.

4K

15 )

initial conditions.

In this case (14) has the general solution

where
Case

3,

and

B are found from initial

Under-damped or damped

e~ yt (A

Bt)

(16)

conditions.

oscillatory motion,

<

<o

i.e.

/3

<

4*<ra

In this case (14) has the general solution

= e~ yt (A sin \t + B cos \t)


= Ce~ yt cos (\t-<t>)
where
2
2
where C = V^ + ^ called the amplitude and
x

and

A
<,

V>

2
i

17 )

called the phase angle or epoch,

are determined from the initial conditions.


In Cases 1 and 2

damping is so large that no


and the mass m simply

oscillation takes place

returns gradually to the equilibrium position


x = 0. This is indicated in Fig. 4-3 where we
have assumed the initial conditions x = x
dx/dt 0. Note that in the critically damped
returns to the equilibrium position
case, mass
in
the over-damped case.
faster than

Critically

damped motion,

y*

= a?

Over-damped motion, y*

In Case 3, damping has been reduced to such


an extent that oscillations about the equilibrium
position do take place, although the magnitude
of these oscillations tend to decrease with time
as indicated in Fig. 4-3. The difference in times

Under-damped motion, y2
Fig. 4-3

<

u*

>

CHAP.

THE SIMPLE HARMONIC OSCILLATOR AND THE SIMPLE PENDULUM

4]

between two successive maxima [or minima] in the under-damped [or damped
motion of Fig. 4-3 is called the period of the motion and is given by

p =
and the frequency, which

is

\A>

\/4kWI

the reciprocal of the period,

_
~ P ~

V^ ?"
7

k_

47rm

2tt

?![

2tt

oscillatory]

lg

-p

given by

is

V^m - p

_
~

89

2
y

47rm

2tt

'

Note that if p - 0, (18) and (19) reduce to (4) and (5) respectively. The period and
are sometimes called the natural period and natural
frequency corresponding to /? =
frequency respectively.

The period P given by (18) is also equal to two successive values of t for which
cos(A <) = 1 [or cos(Ai <j>) = 1] as given in equation (17). Suppose that the values
of x corresponding to the two successive values t n and t n +i = t n + P are x n and x n +i respecThen

tively.

= e-^/e-^ +P) =

xjxn+i

The quantity
which

a constant,

is

is

In

e^ p

(20)

= yP

(xjx n +i)

(21)

called the logarithmic decrement.

FORCED VIBRATIONS
Suppose that in addition to the restoring force
on the mass m a force F(t)i where

= F

F(t)

Then the

motion

differential equation of

Or

where

The general

2yi

solution of (24) is found

X
[which has already been found and
(24).

is

particular solution of (24)

is

Now,

as

we have

dx
/?

-7T

+ Fq cos at

f Q COS at

K /m,

2yX

(23)

(2U)

= FJm

(25)

o>

(26)

given by (15), (16) or (17)] to any particular solution of


given by [see Problem 4.18]

tan d,

(22)

by adding the general solution of

vV-<o
where

oy

w2

p/2m,

we impress

is

kX -

force fSvi

cos at

dx
m -rp

kx\ and damping

2 2
)

cos (at

* ya

=
a

+ 4yV
.

tt

/,)

(27)

(^5)

seen, the general solution of (26) approaches zero within a short time

and we thus call this solution the transient solution. After this time has elapsed, the motion
of the mass m is essentially given by (27) which is often called the steady-state solution.
The vibrations or oscillations which take place, often called forced vibrations or forced
oscillations, have a frequency which is equal to the frequency of the impressed force but
lag behind by the phase angle 4>.

THE SIMPLE HARMONIC OSCILLATOR AND THE SIMPLE PENDULUM

90

[CHAP.

RESONANCE
The amplitude of the

steady-state oscillation (27)

oA

given by

is

(29)

VV-a>2 2 + 4 yV
)

assuming y ^ 0, i.e. p 0, so that damping is assumed to be present. The maximum value


of cA in this case occurs where the frequency /2tt of the impressed force is such that

a2

2
aR

-2y 2

w2

(30)

assuming that y < | 2 [see Problem 4.19]. Near this frequency very large oscillations may
occur, sometimes causing damage to the system. The phenomenon is called resonance and
the frequency ajl-n

The value

is

of the

called the frequency of resonance or resonant frequency.

maximum

amplitude at the resonant frequency

cA,

2y\A,

The amplitude

(29)

graph of cA

(31)

can be written in terms of a R as

aA

is

VV-a

2
)

4yV-y

(32)
2
)

vs. a 2 is

shown in Fig. 4-4. Note that the graph is symmetric around the
resonant frequency and that the resonant frequency, frequency with damping and natural
frequency (without damping) are all different. In case there is no damping, i.e. y =
or
all of these frequencies are identical.
In such case resonance occurs where the
/? = 0,
frequency of the impressed force equals the natural frequency of oscillation. The general
solution for this case is

cos

<at

+ B

sin

<at

75 sin at

(33)

From the last term in (33) it is seen that the oscillations build up with time until finally
the spring breaks. See Problem 4.20.
Resonant frequency

qA

v///////////////(

Frequency with damping

Natural frequency
without damping)

Fig. 4-4

Fig. 4-5

THE SIMPLE PENDULUM


A simple pendulum consists of a mass m
of length

pendulum

end of a massless string or rod


mass m, sometimes called the
pulled aside and released, the resulting motion will be oscillatory.

[which always remains straight,

bob, is

Calling

[Fig. 4-5] at the

i.e.

rigid].

If the

the instantaneous angle which the string

differential equation of

motion

is

[see

Problem

4.23]

makes with the

vertical,

the

CHAP.

THE SIMPLE HARMONIC OSCILLATOR AND THE SIMPLE PENDULUM

4]

91

assuming no damping forces or other external forces are present.


For small angles [e.g. less than 5 with the vertical], sin 9 is very nearly equal to
9 is in radians, and equation (3U) becomes, to a high degree of approximation,

where

0,

-f

This equation has the general solution

where
t

0,

and

+ Bsin^/gllt

Vgllt

cos

For example,

are determined from initial conditions.

then

is

0o,

at
(37)

that of simple harmonic motion. The period


,

P =

is

Vgllt

cos

6o

In such case, the motion of the pendulum bob


is given by

and the frequency

if

(36)

(38)

2-xyJTig

given by

fir.

{39)
[see

Problems 4.29 and

4.30]

dB

iir/2

fl

we can show

If the angles are not necessarily small,


that the period is equal to

P =

^^

VI -

W sin

where k = sin (9 /2). For small angles this reduces to (38).


For cases where damping and other external forces are considered, see Problems 4.25
and 4.114.

THE TWO AND THREE DIMENSIONAL HARMONIC OSCILLATOR


Suppose a particle of mass m moves in the xy plane
under the influence of a force field F given by

F = - Kl a?i where

and

(U)

2 yj

Fj

In this case the equations of motion of


given by

m dP

= ~K

x
'

= A

Fig. 4-6

cos -y/ajm

subjected to the force


various curves which

+ B

sin yj.jm

t,

= A 2 cos

to be determined

from the

yj*.jm

+ B2

y/ajm

initial conditions.

(A3)

The mass

often called a two-dimensional harmonic oscillator. The


describes in its motion are often called Lissajous curves or figures.

F = -k
2,

sin

field (41) is

These ideas are easily extended to a three dimensional harmonic


which is subject to a force field given by

< lf k

-/c 2l/j

= ~ K *y
d&

where Ai, B\, A%, B 2 are constants

where

m
F2 =

are

and have solutions


x

= Kjzi

are positive constants.

are positive constants.

x\

2 y]

zk

oscillator of

mass

(U)

THE SIMPLE HARMONIC OSCILLATOR AND THE SIMPLE PENDULUM

92

[CHAP.

Solved Problems
SIMPLE HARMONIC MOTION AND THE
SIMPLE HARMONIC OSCILLATOR
4.1.
A particle P of mass 2 moves along the

x axis attracted toward origin O by a force


numerically equal to Sx [see Fig. 4-7]. If it is initially at rest
(a) the differential equation and initial conditions describing the

whose magnitude

is

x = 20,
find
motion, (b) the position of the particle at any time,
(c) the speed and velocity of the particle at any time,
and (d) the amplitude, period and frequency of the
at

-8a?i

vibration.
(a)

Let r

of

xi
xi

d2
is

8xi.

be the position vector of P. The acceleration


d2 x
~ ~d+2*' ^ ne net f rce acting on P is

Then by Newton's second law,

Fig. 4-7

d2 x.

2-^1 =
which

The general

solution of (1) is

When

d2x

or

-8jbi

0,

20

so that

Then

= A

A =

20.

The

so that on putting

0,

dx/dt

(1)

initial conditions

are

(2)

+ B

sin 2

(3)

+ B

sin 2t

(4)

at

cos It

Thus
20 cos It

dx/dt

dx/dt

20,

..

^p-+ 4x =

the required differential equation of motion.

is

x
(6)

jijjC^i)

we
a;

-40

sin 2t

find

B=

+ 2B

cos 2t

Thus

0.

(5)

(5)

becomes

20 cos 2i

(0)

which gives the position at any time.


(c)

From

dx/dt

(6)

= -40

which gives the speed at any time.

sin It

^i
dt
(d)

4.2.

(a)

Amplitude

Show

20.

Period

2tt/2

VA + B
2

and

<j>

frequency of the function in


(a)

cos at

i.e.

(b)

+B

sin at

\l

w.

tan

Frequency

cos at
-1

velocity

is

+B

sin

(S/A).

1/period

1/v.

can be written as

>t

cos

(6)

+ B2 (

W^

cos at
2

VA 2 + B 2 cos (t - *) =

<(>

cos w

sin

<f>

sin

to*)

Ccos(wt-0)
,

i.e.

of

<p

tan

<p

which

= 5/A
lies

or

= C - y/A^TB 2

Period

2a-/.

tan- 1 /A,

between 0 and 180,

<p

value

</>)

sin w

VA 2 + B 2

\/A 2

+ # 2 (cos

+ B2

= maximum

(U -

Find the amplitude, period and

(a).

cos

Amplitude

given by

-40sin2ti

= A/ VA 2 + 2 and sin = B/y/A 2 + B 2


=
C VA 2 + 2 We generally choose that value
g ^ v.

where
and

that the function

where C =

The

Frequency

w/2*-.

CHAP.

4.3.

4]

THE SIMPLE HARMONIC OSCILLATOR AND THE SIMPLE PENDULUM

Work Problem
30,

4.1 if

the left with

(b) to

= 20 but is moving (a) to the right with speed


Find the amplitude, period and frequency in

at x

is initially

93

speed 30.

each case.
(a)

The only

difference here

by dx/dt = 30 at t
Problem 4.1 becomes

dx/dt

that the condition

is

Then from

0.

which gives the position of

20 cos 2t

at any time.

at

Problem

of

(5)

This

of Problem 4.1

we

4.1

B=

find

15,

is

replaced

and

(3)

15 sin 2t

may

of

CO

be written [see Problem

4.2]

as
\

V(20)

I
1/(20)2+

(15)2

25{|cos2i

+f

where

which can be found from

is

V(20)2

sin 2t

(15)2

\
J

- 0)

25 cos (2t

= f

sin

f,

(2)

15

(15) 2

sin2}

cos

The angle

cos 2t

20

(2)

often called the phase angle or epoch.

Since the cosine varies between 1 and +1, the amplitude = 25. The period and frequency are the same as before, i.e. period = 2w/2 = w and frequency = 2/2?r = II v.

(b)

at t
In this case the condition dx/dt =
at t = 0. Then B 15 and the position

which as

in part (a)

cos f

20 cos It

4.4.

replaced by

dx/dt

= -30

15 sin 2t

=
=

sin

25{|cos2t- f

sin2t}

cos 2t

25{cos

sin ^ sin 2i)

25 cos (2t

0)

= f

The amplitude, period and frequency are the same as in part (a). The only difference
We often describe
is
$ = <p + w.
the phase angle. The relationship between ^ and
by saying that the two motions are 180 out of phase with each other.

is in

this

is

can be written

where

of Problem 4.1

given by

is

<f>

spring of negligible mass, suspended vertically from one end, is stretched a


distance of 20 cm when a 5 gm mass is attached to the other end. The spring and
mass are placed on a horizontal frictionless table as in Fig. 4-l(a), page 86, with the
suspension point fixed at E. The mass is pulled away a distance 20 cm beyond the
equilibrium position O and released. Find (a) the differential equation and initial
conditions describing the motion, (b) the position at any time t, and (c) the amplitude,
period and frequency of the vibrations.
(a)

The gravitational force on a 5 gm mass [i.e. the weight of a 5 gm mass] is 5g = 5(980) dynes =
Then since 4900 dynes stretches the spring 20 cm, the spring constant is
4900 dynes.
Thus when the spring is stretched a distance x cm beyond the
K = 4900/20 = 245 dynes/cm.
equilibrium position, the restoring force is 245xi. Then by Newton's second law we have,
if

xi

is

the position vector of the mass,


5

The
(b)

initial conditions

The general

are

solution of (1) is

Using the conditions


(c)

*g*> = _245zi

(*)

we

find

x = 20 cos It we see that:


vib/sec or l/2v cycles/sec.

From

=
x

20,

dx/dt

= A

A=

20,

or

cos It

B=

amplitude

^+ 49*

(2)

sin It

($)

so that

(1)

at

+ B

20 cm;

period

20 cos

It.

2tt/7 sec;

frequency

7/2ir

THE SIMPLE HARMONIC OSCILLATOR AND THE SIMPLE PENDULUM

94

4.5.

mass m moves along the x axis, attracted toward a fixed point O


by a force proportional to the distance from O. Initially the particle is at
distance x from
and is given a velocity v away from 0. Determine (a) the
position at any time, (b) the velocity at any time, and (c) the amplitude, period,
frequency, and maximum speed.

particle of

on

it

(a)

The force

of attraction toward

kx'i

where k is a
Then by Newton's

is

positive constant of proportionality.

second law,
d?x.

Solving

we

(1),

We

also

kxi

or

have the

cos y/ic/m

+ B

at

we

find,

at

is,

using

The amplitude

From

(7),

From

(3),

is

or

(0)

(3)

Thus

sin yic/m

+ B y/lc/m

B=

that

(5),

V~i</ni t

+ B

V*/

using

#0 cos

at

(4)

cos

yV

vWk. Thus

vk/w

v Q \Zm/K sin

(5)

(4)

becomes
(8)

yx^+mvUic

-tt

the period
is

tan -1 (v /x

(7)

0)

yW/c

( #o y/x/m sin VxTm

\/x 2

^Jit/m

V^n +

(7)

(8)

+ mv

2 /ic

cos

yVm

sin (yfic/mt

k?/w. sin {\[7hn

0)

<j>)

) i

(3)

yxJ+mv*/K.

by

P=

is

cos (vk/ot

(7),

dx

given from

the speed

sin

A =

that

(2),

find,

=
(c)

dx/dt

'

velocity

(2)

x cos V ii/m

where

The

can be written

4.2, this

(6)

we

a;

Using Problem

(1)

Fig. 4-8

= x Q yjulm

dx/dt

so that

dx/dt

sin yjn/m

using

From

m =0

initial conditions

From

F = Kxi

KX

find

= A

4.6.

[CHAP.

2srV'ic/m.

maximum when

The frequency

is

sin (y/T/mt -</>)

1/P

2iry/m/K.

1; this speed

is

v^ 2 + kb 2 /.
,

mass 20 kg moves with simple harmonic motion on the x axis. Initially


located at the distance 4 meters away from the origin x = 0, and has
2
Find (a) the
velocity 15 m/sec and acceleration 100 m/sec directed toward * = 0.
position at any time, (b) the amplitude, period and frequency of the oscillations, and

An object
(t = 0) it

of
is

when

(c)

the force on the object

(a)

If x denotes the position of the object at time

Now

4,

dx/dt

-rr/10

sec.

t,

then the

-15, d 2 x/dt 2

= -100

conditions are

initial

at

(1)

for simple harmonic motion,

= A
Differentiating,

we

find

dx/dt

d2 x/dt 2

cos

u>t

+ B

sin at

+ Ba cos at
= Aw 2 cos ut - Ba 2 sin at
Aw

sin at

(3)
(4)

CHAP.

THE SIMPLE HARMONIC OSCILLATOR AND THE SIMPLE PENDULUM

4]

Using conditions
simultaneously,

in {2), (S)

(1)

we

A=

find

4,

and

we

(4),

find

5 = 3

5,

4 cos 5t

A,

-15

-100

Bw,

95

= -Aw 2

Solving

so that

3 sin 5t

(5)

which can be written

(6)

From

(c)

Magnitude of acceleration

we

(tf)

where

amplitude

see that:

Force on object

4.7.

5 cos (5t #)

5 m,

cos ^

period

=^

= 100 cos 5t + 75
= (mass) (acceleration) = (20 kg)(75
d2 x/dt2

sin 5*

20 lbwt object suspended from the end of a verspring of negligible mass stretches it 6 inches.
(a) Determine the position of the object at any time
if initially it is pulled down 2 inches and then released, (b) Find the amplitude, period and frequency of the motion.

the spring.
the object.

Position

is

(6)

75 m/sec 2

at

5/2jr vib/sec.

tt/10.

1500 newtons.

mm^x?

tical

Let D and E [Fig. 4-9] represent the position of the


end of the spring before and after the object is put on

m/sec2 )

frequency

2jt/5 sec,

(a)

sin

D
.5

the equilibrium position of

ft

Choose a coordinate system as shown in Fig. 4-9


so that the positive z axis is downward with origin at

zft

the equilibrium position.

By
\

ft,

Hooke's law, since 20 lb wt stretches the spring


40 lb wt stretches it 1 ft; then 40(.5 + z) lb wt

+ z)

Thus when the object is at


position F there is an upward force acting on it of
magnitude 40(.5 + z) and a downward force due to its
weight of magnitude 20. By Newton's second law we
stretches

it

(.5

ft.

Fig. 4-9

thus have

H^k

Solving,

Now

at

20k

= A

z
t

0,

=%

and

dz/dt

40(.5

4.8.

From

(2):

amplitude

Work Problem

ft,

period

2jt/8

W2 +

+ B sin St
=
A , B =

initial velocity of

=
=

^ cos

(1)

and

v/4 sec,

frequency

4/v vib/sec.

down 3 inches (instead of 2 inches)


2 ft/sec downward.

In this case the solution (1) of Problem 4.7 still holds but the initial conditions are: at
dz/dt = 2. From these we find A = \ and B = \, so that

= I

cos St

sin Bt

y/2/4 cos (St

Thus amplitude = \/2/4 ft, period = 2a-/8 = jt/4 sec, frequency = 4Ar
and frequency are unaffected by changing the initial conditions.

\ and

4.9.

64,

4.7 if initially the object is pulled

and then given an


z

or

cos St

thus

0;

(b)

+ z)k

0,

ir/4)

vib/sec.

Note that the period

uniform angular speed a around a circle of radius 6. Prove


projection on a diameter oscillates with simple harmonic motion of period
about the center.

particle travels with

that
2tt/g>

its

Choose the circle in the xy plane with center at the origin O as in Fig. 4-10 below.
the projection of particle P on diameter
chosen along the x axis.

AB

Let

be

THE SIMPLE HARMONIC OSCILLATOR AND THE SIMPLE PENDULUM

96

If the particle is initially at B,

I BOP

have

The projection Q of

b cos ut

P
r

from O at any time

Then the

ut.

on the x axis

then in time

position of
b sin ut
is

we

at time

[CHAP.

will
t

is

(1)

at distance

b cos ut

(2)

From (2) we see that the projection Q


with simple harmonic motion of period 2ir/w about

oscillates

t.

Fig. 4-10

the center O.

THE DAMPED HARMONIC OSCILLATOR


4.10.

in Problem 4.1 the particle P has also a damping force whose magnitude
numerically equal to 8 times the instantaneous speed. Find (a) the position and
(b) the velocity of the particle at any time, (c) Illustrate graphically the position of
the particle as a function of time t.
y

Suppose that
is

(a)

In this case the net force acting on

8a;i

Fig. 4-11]

doc

dt

ond law,

d2 x

is

Then by Newton's

8-rri.

-8i

[see
-

sec-

dx
di

8^i

Fig. 4-11

i*

This has the solution [see Appendix, page 352, Problem C.14]

x
t = 0,
x - 20 and
the position at any time t.

When

(b)

The

velocity

V
(c)

is

0;

and

40,

^
.

20e~ 2t

(1

+ 2t)

gives

-80e-i

The graph of a; vs. is shown in Fig. 4-12. It is


seen that the motion is non-oscillatory. The particle approaches O slowly but never reaches it.
This is an example where the motion is critically
Fig. 4-12

damped.

4.11.

a;

given by

dx

dx/dt

= e-(A + Bt)
thus A = 20, B =

gm

moves along the x axis under the influence of two forces:


O which in dynes is numerically equal to 40 times
from
the instantaneous distance
O, and (ii) a damping force proportional to the
instantaneous speed such that when the speed is 10 cm/sec the damping force is
200 dynes. Assuming that the particle starts from rest at a distance 20 cm from O,
(b) find
(a) set up the differential equation and conditions describing the motion,
period
and
amplitude,
the
determine
the position of the particle at any time, (c)
graph
the
motion.
frequency of the damped oscillations, and (d)
(i)

(a)

particle of

mass 5

a force of attraction to origin

Let the position vector of the particle

as indicated in Fig. 4-13.


tion (directed toward O) is
r

xi

damping force /
where /?
(S dx/dt

attrac-

-20(dx/dt)i

-40i

The magnitude

be denoted by

Then the force of

(1)

proportional to
Then
is constant.
the speed, so that /
since / = 200 when dx/dt = 10, we have /? = 20 and
and
/ = 20 dx/dt. To get , note that when dx/dt >
x >
the particle is on the positive x axis and moving to
of the

40*i

is

O
Fig. 4-13

CHAP.

41

THE SIMPLE HARMONIC OSCILLATOR AND THE SIMPLE PENDULUM


Thus the resistance force must be directed toward the

the right.
plished

if

- 20

97

This can only be accom

left.

This same form for f is easily shown to be correct


[see Problem 4.45].
< 0, dx/dt <

(2)

>

if

<

dx/dt

0,

0,

<

dx/dt

0,

>

0,

Hence by Newton's second law we have


d*x.
b

-20%
dt

dP

dx
d x
1M + *M +
z

Since the particle starts from rest at 20

(3)

SX

cm from

dx/dt

20,

40*1

O,

04)

we have

at

(5)

where we have assumed that the particle starts on the positive


just as well assume that the particle starts on the negative side,
(b)

e at

a solution of

is

a2

Then the general

i(-4 V16

20

at

we

0,

find

Thus by

dx/dt

(c)

From

at

- 32) = -2

2%

e~ 2t (A cos2t

from

(6)

+B

A=

that

e~ 2t (20 cos 2t

sin2t)

20,

+B

(6)

i.e.,

sin 2t)

(7)

(d)

The graph

shown

is

0,

+ 2B

we have from

20e-2'(cos 2t

cos 2t)

B=

(8),

sin 2t)

(-2e~2t)(20 cos 2t
20.

Thus from

20\/2 e" 2 * cos (2t

+B

we

(7)

sin 2t)

(8)

obtain

ir/4)

(9)

4.2.

amplitude

(9):

(e- 2 t)(-40 sin 2*

a;

using Problem

zero as

20].

differentiation,

dx/dt
Since

solution is

x
Since

which case x

in

if

(-4)

4a

side of the x axis [we could

= 20V2 e _2t cm,

in Fig. 4-14.

period

2jt/2

tt

sec,

frequency

Note that the amplitudes of the

1/V vib/sec.

oscillation decrease

toward

increases.

20V2

Fig. 4-14

4.12.

Find the logarithmic decrement in Problem


Method

The maxima

1.

(or

minima) of * occur where dx/dt


dx/dt

when
occur

n-/2, 3s-/2, 5xr/2,

e -2(ir)/ e -2(27r) > etc., i.e. e 2v .

- -80e- 2t

sin2

0.

From

(9)

of Problem 4.11,

the minima
....
The maxima occur when t = 0, v, 2v,
;
....
The ratio of two successive maxima is e -2 (0)/ e -2(7r) or
Then the logarithmic decrement is 8 =ln(e2w ) = 2w.

0, ir/2, w, 3n-/2, 2ir, 5ir/2,

when

4-11.

THE SIMPLE HARMONIC OSCILLATOR AND THE SIMPLE PENDULUM

98

Method
t

n+

(9) of Problem 4.11, the difference between two successive values of


for which cos(2t tt/4) 1 (or 1) is w, which is the period. Then

n+

Method

From

3.

4.13.

ra

and

(13), (18)

5,

and

e 27r

\n(x n /x n +

20y/2e- 2tn+i

since

t,

denoted by

yP
k

20,

^ irm

\(

2 ^ft

[Problem 4.11, equation

40

2b-

we have

pages 88 and 89,

(21),

1)

(3)],

2w.

Determine the natural period and frequency of the particle of Problem

4.11.

The natural period is the period when there is no damping. In such case the motion
given by removing the term involving dx/dt in equation (3) or (4) of Problem 4.11. Thus
d 2 x/dt2
Then:

4.14.

tn

20\/2e-2tn

Xn

Then

2.

From
and

[CHAP.

natural period

Sx

or

2irl2y[2 sec

a;

irly[2 sec;

= A

+ B

cos 2y[2t

sin 2y/2t

= y[2h

natural frequency

is

vib/sec.

For what range of values of the damping constant in Problem 4.11 will the motion
(a) overdamped, (b) underdamped or damped oscillatory, (c) critically damped?

be

Denoting the damping constant by p, equation

Then the motion


(a)

Overdamped

if

(/3/5)

(b)

Underdamped

if

(p/5)

(c)

Critically

n dx
-**

..

40 * 1

dtff

Problem 4.11

of

(3)

_
'

d#

replaced by

is

dx
_
+ P5Tt +8x

2x

is:

[Note that this

4.15.

2 x.

>

32,

i.e.

/
p > 20 v 2.

p < 20^2.
the case for Problem 4.11 where p

is

damped

if

<

32,

(p/5)

i.e.

32,

i.e.

20.]

20V2.

Solve Problem 4.7 taking into account an external damping force given numerically
in lb wt by pv where v is the instantaneous speed in ft/sec and (a) /? = 8, (b) p = 10,
(c)

12.5.

The equation of motion


20 d 2 z

I^
(a)

If

/8

8,

then

d 2 z/dt 2

Using the conditions


2

(6)

If

10,

is

then

or

SB dz

_.

64,

+ 12.8 dz/dt + 642 = 0. The solution is


z = e~ 6At (A cos4.8 + J? sin 4.8t)

1/6,

dz/dt

-?-e-6.4t(3 C0S 4.8

damped
d2 z/dt2

^ + f^+
d2z

dz.
= 20k-40(.5 + ,)k-/3 |k

The motion

is

at

4 sin 4.8*)

oscillatory with period

Udz/dt

64z
z

we

0,

Jg

2?r/4.8

~8

A =

%,

'

4t

J3

1/6,

cos < 4 8t
-

2/9 so that

~ 53 8

'>

5V/12 sec.

= 0. The solution
= e~ 4t (A+Bt)

Solving subject to the initial conditions gives

A=

find

is

B=f

then

- \e~ 4t

(1

+ U).

CHAP.

4]

THE SIMPLE HARMONIC OSCILLATOR AND THE SIMPLE PENDULUM


The motion

(c)

If

/?

damped

is critically

d2 z/dt 2

12.5 then

20dz/dt

any decrease

since

64z

/?

would produce oscillatory motion.

The solution

0.

is

= Ae-u + Be-

A =

Solving subject to initial conditions gives

The motion

in

99

1/6,

B=

-1/24; then

- \e~^ - ^e~ 16t

overdamped.

is

ENERGY OF A SIMPLE HARMONIC OSCILLATOR


4.16.

Prove that the force F = - K xi acting on a simple harmonic oscillator


servative, (b) Find the potential energy of a simple harmonic oscillator.
(a)

(a)

VXF

We have

d/dx

d/dy

d/dz

given by

so that

is

con-

conservative.

is

kx

(6)

The

potential or potential energy

is

~ KXl
Then dV/dx

responding to

4.17.

dV/dy

kx,

0,

we

dV/dz

0,

find

where

F--VV

or

fdV. ,BV. +,8V.k


J
*T

-(jr + ^

V=

from which
so that

V=

fax

fax

+ c. Assuming V =

cor-

2.

Express in symbols the principle of conservation of energy for a simple harmonic


oscillator.

By Problem

4.16(6),

we have

Kinetic energy
or

fafiv

which can also be written, since v


Another method.

The

Potential energy

fax

as

dx/dt,

differential equation for the

Total energy

fan{dx/dt) 2

+ fax2 =

E.

motion of a simple harmonic oscillator

is

md2 x/dt 2 = kx
Since

dx/dt

v,

this

can also be written as

dv
=
m,-
~ KKXx
li
dt =
Integration yields

fanv

+ fax 2 =

dx =
m dv
~ KX
dxlt

r
or

>

i.e.

dv
dx

mv-=-

= kx

E.

FORCED VIBRATIONS AND RESONANCE


4.18.

Derive the steady-state solution (27) corresponding to the differential equation


on page 89.
The

differential equation is

,,
x + 2ya? +

u2x

(24)

/ cos at

(1)

c 2 sin at

(*)

Consider a particular solution having the form


X

Ci

cos at

THE SIMPLE HARMONIC OSCILLATOR AND THE SIMPLE PENDULUM

100

where

and

cx

c 2 are to be determined.

(-a

*?!

2yaC2

(a 2

or

Solving these simultaneously,

we

Cl

w 2 )cj

w 2Cl

by Problem

page

4.2,

= 2ya/(a2 2
(<o

2yac2

(<o

/o

(a 2

tan

(a)

<p

a2

1.

C2

<o

'

It is

(a 2

(a 2

<o

2
),

co

2 2
)

2ya sin at

maximum when

<j>

V( 2

Using

v.

2 c2

2
<* )c

(4)

<o

2 2
)

(5)

4y 2 a2

2ya sin

at]
(

in

~ a2

2 2

<o

\/>

/o/^yV^Y 2

<o

2 2

2.

find as required

0)

0,

maximum

is

<o

2 2
)

4y 2 2

(1)

- 2(<o 2 - 2y 2 )a 2 + w 4
- 2(w 2 - 2y 2 )a 2 + 2 - 2y 2 2 +
2 - 2
[a 2 - ( 2 - 2y 2 2 + 4y 2
y

=
=
=

4y 2a 2

The function

= 0,
= V" 2 2y 2

For

4.20.

the resonant fre-

that the value of this

(b)

the denominator [or the square of the denominator]

a minimum.

is

To

a4
a4

(a>

(a>

)]

2(a 2

(a 2

a>

Vw 2 2y 2 where

w2 2 +

4y 2a 2 has a

2 )2a

y2

d2 U/da 2

gives the

-4(<o 2

< ^w 2

- 2y 2 <

minimum

8y 2 a

<o

(<o

- 2y 2

2
)

a2
is

w 2 2y 2 , and the
given from (i) by

=
For

0.

minimum
a(a 2

or

maximum when

or
<o

+ 2y 2 =
)

Now

dPU/da 2

(7)

<f>)

)-

$Tda
a

(at

maximum where

is

Problem 4.18

cos (at

4y 2 a2 cos

4y 2 a 2

- 2y and

we

(5),

2
)

This is a minimum where the first term on the last line is zero, i.e. when
value is then 4y 2 (w 2 y2 ). Thus the value of the maximum amplitude

i.e.

e)

minimum, write
(a 2

Method

(3)

4y 2 a 2

in

(7)

that the amplitude in Problem 4.18

The amplitude

cos at

(a 2

cos at

/o [("

/o/vV find this

/ cos at

92,

quency is determined from a


amplitude is / /(2y\/ w 2 -y 2 ).
Method

2/ yw

_
~

V( 2

Prove

find

w 2 c 2 ) sin at

2yac x

2yaC x

we

(1),

2yac 1

- a 2 c2 -

-/,

a2
+ 4y 2

2 2
<o

_
~

4.19.

into

(2)

becomes

(2)

where

(a 2 c

12a 2
a

4<o 2

8y 2

= Vw 2 - 2y 2

d2 U/da 2

8(a> 2

- 2y 2 >
)

0.

Thus

value.

(a) Obtain the solution (33), page 90, for the case where there
impressed frequency is equal to the natural frequency of the

is

no damping and the


(b) Give a

oscillation,

physical interpretation.
(a)

find

Now

Substituting

cos at

t)

- 2 Cl + 2yac 2 +

from which

Thus

u 2c

[CHAP.

The case

{23) or (24),

to be considered is obtained

page

89.

We

by putting y

or

(3

and a

in equations

thus must solve the equation

a2 x

/ cos

tot

(*)

CHAP.

THE SIMPLE HARMONIC OSCILLATOR AND THE SIMPLE PENDULUM

41

To

the general solution of

we add

find the general solution of this equation

101

u 2x

(*)

to a particular solution of (1).

Now

the general solution of

is

(2)

To

+ B

would do no good

find a particular solution of (1) it

c t cos ut

to

sin ut

(#)

assume a particular solution of the form

c 2 sin ut

substitute (-4) [which is identical in form to {3)] into the left side of
must therefore modify the form of the assumed particular solution
Appendix C, the assumed particular solution has the form

since

when we

We

get zero.
in

cos ut

t{Ci

cos ut

Substituting

(5),

t( u 2 c x cos ut

(6)

and

into

(7)

cx

and

c2

quired particular solution


The general solution of (1)

= A

cos wt

+ B

is

sin

we would
As seen
w)

(c x

cos ut

2( uc t sin ut

(5)

to

obtain

c 2 sin ut)

(6)

(7)

uc 2 cos ut)

find after simplifying

2uc 2 cos ut

Thus the

uc 2 cos ut)

u 2 c 2 sin ut)

sin ut

f /2u.
is

(5)

we

(1),

2uc
from which

t( c x sin ut

(4).

c 2 sin ut)

see that this yields the required particular solution, let us differentiate

To

{1),

cos ut

re-

(f /2u)t sin ut.

therefore

<ot

{fJ2u)t sin

tot

(5)

(6) The constants A and 5 in (8) are determined


from the initial conditions. Unlike the case with
damping, the terms involving A and B do not become
small with time. However, the last term involving t
increases with time to such an extent that the spring
A graph of the last term shown
will finally break.
in Fig. 4-15 indicates how the oscillations build up

in

4.21.

Fig. 4-15

magnitude.

a force
vertical spring has a stiffness factor equal to 3 lb wt per ft. At =
in
hangs
which
weight
lb
a
=
to
6
applied
is
12 sin4, t ^
given in lb wt by F(t)
of
the
position
find
the
damping,
equilibrium at the end of the spring. Neglecting

weight at any later time

t.

Using the method of Problem

4.7,

we have by Newton's second

d2 z

= -Sz +

32 dt2
d?z

^-|
Solving

When

'

0,

and

dz/dt

= A
0;

cos At

then
z

162

+ B

A =
2

law,

12 sin At

64 sin 4t

sin 4t

(1)

8t cos At

B = 2 and
sin At - 8t cos At
0,

(2)

physically the
t gets larger the term St cos At increases numerically without bound, and
spring will ultimately break. The example illustrates the phenomenon of resonance. Note that
the natural frequency of the spring (4/2*r = 2/w) equals the frequency of the impressed force.

As

THE SIMPLE HARMONIC OSCILLATOR AND THE SIMPLE PENDULUM

102

4.22.

Work Problem

4.21 if

F(t)

In this case the equation

Problem

of

(1)

d
and the

solution of (1)

2 z/dt 2

4.21

becomes

16z

160 cos 5t

(1)

Using conditions

(2)

in (5),

8(cos 4t

dz/dt

0,

at

sin 4*

(2)

8 cos 6t

(8)

is

we

t^O.

conditions are

initial

The general

Gt,

30 cos

[CHAP.

A =

find

cos 6t)

+ B

cos4
8,

and

8{cos(5-)

cos(5

)}

16 sin

sin 5t

The graph of z vs. t is shown by the heavy curve of Fig. 4-16. The dashed curves are the curves
= 16 sin t obtained by placing sin 5t = 1. If we consider that 16 sin t is the amplitude of
sin5, we see that the amplitude varies sinusoidally. The phenomenon is known as amplitude
modulation and is of practical importance in communications and electronics.
z

Fig. 4-16

THE SIMPLE PENDULUM


4.23.

Determine the motion of a simple pendulum of length


vibrations and no resisting forces.

and mass

Let the position of


at any time be determined by s,
the arclength measured from the equilibrium position
[see Fig. 4-17].
Let o be the angle made by the pendulum
string with the vertical.
If T is a unit tangent vector to the circular path of
the pendulum bob m, then by Newton's second law

m dP T =
or, since

sin e

(1)

le,

<&8

to

-mg

For small vibrations we can replace sin e by B so that


a high degree of accuracy equation (2) can be replaced by

dH
dP + 5<

(3)

which has solution

= A

Taking as

initial conditions

coay/gTlt

de/dt
e

From

this

we

see that the period of the

=
=

at
e

cos

pendulum

+ B
t

sinVFT^*
0,

we

VoU t
is 2iry/l/g.

find

A=

assuming small

CHAP.

4.24.

THE SIMPLE HARMONIC OSCILLATOR AND THE SIMPLE PENDULUM

4]

Show how to obtain the equation (2) for the


the principle of conservation of energy.

103

pendulum of Problem 4.23 by using

= I - I cos e - 1(1 - cos o). Then by the conserva= OC potential energy as a horizontal plane through
the
level
for
reference
[taking
the
tion of energy
the lowest point O] we have

We

from Fig. 4-17 that

see

B +

Potential energy at

Id,

Differentiating both sides of

mgl

4.25.

(2)

sin e h

agreement with equation

Work Problem

cos

e)

Total energy

constant

= E

^m(ds/dt) 2

(1)

becomes

this

mgl{\

in

Kinetic energy at

mgl(l
Since

AC

OA

4.23

if

(2)

cos

+ \ml2 (de/dt) 2 = E

e)

with respect to

t,

we

+ ml2

(2)

find

or

(g/l) sin

of Problem 4.23.

a damping force proportional to the instantaneous velocity

is

taken into account.


In this case the equation of motion

of Problem 4.23

(1)

2s

m dd^ T = -mgsmeT Using

le

and replacing

sin o

hy

Case

1.

is

Case

2.

p 2 /4m2 <

is

Case

3.

B 2 /4m2

e~-^ t/2m

damped

the case of

is

dt

-g

sin e

ds
-B ^

+ B smut)

(A cosut

the case of critically

B 2/4m 2

>

where w

y/g/l

B 2 lkm2

underdamped motion.

oscillations or

g/l

e~^ t/2m (A+Bt)

damped motion.

g/l
e

This

g/l

This

or

arise:

This

replaced by

for small vibrations, this becomes

dt*

Three cases

ds

Tt

is

2s

e -3t/2m(^ e \t

J5 e -xt)

where

V)8

/4m2

g/l

the case of overdamped motion.

In each case the constants A and B can be determined from the initial conditions. In Case 1
there are continually decreasing oscillations. In Cases 2 and 3 the pendulum bob gradually returns
to the equilibrium position without oscillation.

THE TWO AND THREE DIMENSIONAL HARMONIC OSCILLATOR


4.26.

Find the potential energy for


harmonic oscillator.
(a)

In this case the force

is

(a)

the

two dimensional and

given by

F = k^xi

VXF =

Since
a function

0,

the force field

such that

the three dimensional

(b)

is

conservative.

F = VV. We

K 2 yj

Thus a potential does

exist,

thus have

dV

dV

av,

i.e.

there exists

THE SIMPLE HARMONIC OSCILLATOR AND THE SIMPLE PENDULUM

104

from which

dV/dx

dV/dy

Kl x,

dV/dz

K 2 y,

V =

We

F = k

we have

In this case

then find as in part

potential energy

+ ^ K2 y2

particle

in general

moves
it

xy plane in a force field given by F


move in an elliptical path.

has mass

equation of motion

ra, its

= F =

m-jp
or, since

xi

+ fa z 2

1k 22/2

in the

will

If the particle

the required potential energy.

is

is

V = fax* +

4.27.

This

K 2 y] - K S zk which is also conservative since V X F = 0.


- k x, 8V/dy = K 2 y, dV/dz = k 3 z from which the required

xi

dV/dx

(a),

or

1^*2

choosing the arbitrary additive constant to be zero.


(b)

[CHAP.

d
m^i
oc

+ yj,

Then

ic

ct

ct

i/

= - KX

Prove that

is

-kxi

K yj

- K xi -

+ m-~j

= - K xi - K yj.

(l)

K yj

d^ij
= - Ky
m-^

(2)

These equations have solutions given respectively by


x

= A

cos yj kItyi

+ A2

sin

V^M

t,

t
the particle
= ai + bj and movin g with velocity dr/dt =
B = b, A 2 Vis/mU, B 2 = v 2 y/m/K and so

Let us suppose that at

is

located

vxi

+ v 2}.

cos y/ n/m

+ B2

sin

V'/c/m

(3)

at the point whose position vector


Using these conditions, we find A t =

is

a,

where

v{\fmTic,

a cos wt

dx cy
r= 3
ad be

Squaring and adding, using the fact that


(dx
(62

Now

d2 )x 2

cy)*

2(cd

cos 2 ut

(ay

an

sin ut

bx) 2
(a*

(4)

sin at

we

find,

(4)
if

ad

= be,

ay bx
j
=
ad be
j

+ sin 2 w =

+ ab)xy +

we

1,

find

= (ad- be) 2
+

c 2 )y*

(ad

- be) 2

(5)

the equation

Ax 2 + Bxy + Cy 2 = D
is

b cos ut

Solving for sin ut and cosut in

v 2 y/m/K.
cos ut

or

c sin wt,

ellipse if

B -4AC <
2

To determine what

(5)

is,

B - 4AC =
2

a parabola

0,

we

where

see that

4(cd

if

A =

+ ab) 2 -

provided ad = be. Thus in general the path


the ellipse reduces to the straight line ay

b2

4(b 2

A>0, C >

0,

D>

B 2 4AC = 0, and a hyperbola if B 2 -4AC>0.


+ d2 B = -2(cd + ab), C = a2 + c 2 so that
,

+ d?)(a2 + c 2 =

is

an

bx.

ellipse,

and

if

-4(ad -

be) 2

<

A -C

it is

a circle.

If

ad

be

MISCELLANEOUS PROBLEMS
4.28.

cylinder having axis vertical floats in a liquid of density a. It is pushed down


and released. Find the period of the oscillation if the cylinder has weight
and cross sectional area A.

slightly

CHAP.

THE SIMPLE HARMONIC OSCILLATOR AND THE SIMPLE PENDULUM

4]

105

Let RS, the equilibrium position of the cylinder, be distant


from the liquid surface PQ at any time t. By Archimedes'
principle, the buoyant force on the cylinder is (Az)a. Then by
z

Newton's second law,

W d?z

= Azo

rrs"
9 d&

d?z

gAo

dt*

Solving,

c t cos

and the period of the

yJgAolW t

oscillation is

^JgAaJW t

c 2 sin

2vyW/gAo.

Fig. 4-18

Show that if the assumption of small vibrations


simple pendulum is

4.29.

d<f>

41 S.

where k

VI ~k 2 sin 2

The equation of motion


[equation (34), page 91]

sin

(<9

/2)

*/,

simple pendulum

for a

not made, then the period of a

is

d2 e

small vibrations

if

are not assumed

is

(1)

dt*

Let

de/dt

Then

u.

dH
d&
and

_
~

du

du

Integrating

(2)

we

Now when

u2

we

so that

(2g/l)(cos e

cos e

or

and

Thus

de/dt

P/4 at

(5)

let

cos0 o )

cos e

we must use

U)

to

the minus sign in

0,

(4)

Vcos

where

0,

identity

C e

cos e

cos e

the period,

is

de
e

(5)

cos e

2 sin 2 (*/2)

1,

with a similar one replacing

can be written

P
Now

^(2g/l)(cose

de

Vcos

Making use of the trigonometric

can be written

we have

[T

(3)

motion where the bob goes from

-us

at

(S)

= - y/(2g/l)(cos o

Separating the variables and integrating,

to one fourth of the period, then

de/dt

cos

~(g/l) cos e

restrict ourselves to that part of the

which represents a time equal


so that it becomes

by

de

obtain
-r-

du

= -fsin*

de

Since

_
~

de dt

becomes

(1)

If

du de

dt

vir

sin (e/2)

d*

Vsin
sin

(<?

(0 o /2)

/2) sin

sin 2 (<?/2)

(7)

THE SIMPLE HARMONIC OSCILLATOR AND THE SIMPLE PENDULUM

106

Then taking the

differential of both sides,

(7)

we

when

see that

2 sin

0,

<f>

if

we have small

as

<p

d<f>

we have already

cos

<p

sin 2

,7r/2

d<f>

<f>

<j>

jt/2.

Hence

(6)

becomes,

d_

<VJr-=2=
\ Vl -

k 2 sin 2

vibrations,

k2

and when

0;

_
*

(e /2)

Vl -

as required,

Note that
becomes

sin (e /2) cos

sin (e /2),

de

Also from

(0/2) de

\ cos
or calling k

[CHAP. 4

if

i.e.

equal to zero very nearly, then the period

is

VJC^

(8)

(9)

^VI

seen.

The integral in (8) is called an elliptic integral and cannot be evaluated exactly in terms of
in terms
elementary functions. The equation of motion of the pendulum can be solved for
of elliptic functions which are generalizations of the trigonometric functions.

4.30.

Show

that period given in Problem 4.29 can be written as

The binomial theorem


<!+*).>
If

!,

this

states that if

= k 2

<

then

1,

&=$* + "Vffr" - +

p.

can be written

d + aO-i/2 =
Letting

\x\

sin 2

<

AyjUg

we

to v/2,

find

djp

r
yl

ir/2

+ ^f* - f^frf* 3 +

and integrating from


/*ir/2

-\ x

f
-j

2.vw{i +

k 2 sin 2o^
,o

tf>

+ |

A;

sin 2

+ 2T4 **

sin4

1
"

d(f>

'
'
|

(i)'*. + (!)'* +
}
(Hfcf)'"' +

where we have made use of the integration formula

/2

The term by term integration

4.31.

bead of mass

is

is

a
"0^

2
sin a,

possible since

constrained to

l*3'5---(2w-l)
2
|fc|

<

.4.6---(2n)

ff

1.

move on a

frictionless wire in the

shape of a

cycloid [Fig. 4-19 below] whose parametric equations are

a(cj>

sin

</>),

a(l

cos

</>)

CO

a vertical plane. If the bead starts from rest at point O, (a) find the
speed at the bottom of the path and (b) show that the bead performs oscillations
with period equivalent to that of a simple pendulum of length 4a.

which

lies in

CHAP.

THE SIMPLE HARMONIC OSCILLATOR AND THE SIMPLE PENDULUM

4]

(a)

Let

and

let s

107

be the position of the bead at any time t


be the arclength along the cycloid measured from point O.

By

the conservation of energy, measuring

AB

potential energy relative to line


through the
minimum point of the cycloid, we have

P + K.E. at P = P.E. at O + K.E.


mg(2a-y) + im(ds/dt) 2 = mg(2a) +

P.E. at

Thus

At
(b)

(ds/dt) 2

the lowest point

From

part

Then

(ds/dt) 2

(a),

{ds/dt) 2

2a 2 (l

cos

2
<f>)^>

When

<

equation of

0,

2w,

a2 (l

cos

$2

or

2
<f>)

2y/ga.

P =

P/2

$2

where

sin 2

<f>

is

y/g/at

see

cos

2
<f>)]>

cx

(4)

Hence from the second

the period.

2w-ya/g

the same as that of a simple pendulum of length

is

2a2 (l

Thus

<f>

Airya/g

a2

g/a.

and

For some interesting applications

4.32.

-\j2g(2a)

</>)

(t),

and the period

cos
= yfgja,

2ga{l

when

0;

ds/dt

is

(dy/dt) 2

dtp/dt

or

But

2gy.

(dx/dt) 2

(2)

2gy

2a the speed

at

Problems

4a.

4.86-4.88.

particle of mass
is placed on the inside
of a smooth paraboloid of revolution having

equation cz = x 2 + y 2 at a point P which is at


height
above the horizontal [assumed as the

xy plane]. Assuming that the particle starts


from rest, (a) find the speed with which it
reaches the vertex 0, (b) find the time r taken,
and (c) find the period for small vibrations.
It is convenient to choose the point P in the yz
plane so that x =
and cz = y 2 By the principle
of conservation of energy we have if Q is any point
on the path PQO,
.

where

8 is the

P.E. at

P +

mgH

arclength along

P.E. at

Q +

m(0) 2

mgz

K.E. at

OPQ

or

(a)

Putting

(6)

We

0,

we

have, since x

fds\
\dtj
Thus

(I)

= 2g(H-z)

and

fdx\*
\dtj

can be written

^7
dt

decreasing with

see that the speed

cz

fdyY
\dtj
(1

$m(ds/dt) 2

y2
,

"*"

(1)

= s/2g(H z)

ds/dt
is

K.E. at

measured from O. Thus


(ds/dt) 2

using the negative sign since s

Fig. 4-20

is

t.

y]2gH at the vertex.

fdz\
\dt)

_
~

fdyY.W/dy
\dtj

"*"

c2 \dt

<*+Wt)

+ 4y 2/c 2 )(dy/dt) 2 = 2g(H - y 2 /c). Then

= - y/2gc -=
Vc +
2

or
4y2

V2^c

dt

+ 4i/2
yfcH - y*
V"c2
,

dy

THE SIMPLE HARMONIC OSCILLATOR AND THE SIMPLE PENDULUM

108

Integrating, using the fact that

= V^H

and thus y

at

while at

[CHAP.

t,

0,

we have
T

f
I
y2gc
,

dt

A)

Letting

yJcH cos

-^

and

this

(9,

J^

Vc2 +
^/ c

2
2
1== r \^H Vc + %
F
I
VcH - 2/2
v20C J

4y2

dy

#~

or

2
2/

<fy

the integral can be written

Vc 2 + 4c#

can be written

v/2

where

cos 2 9 de

jiJ
=

fc

sin 2

d*

y/T^W^Jde

+ AH)

y/AH/{c

Vc 2 + 4cH - AcH

<

(3)

(4)

The integral in (5) is an elliptic integral and cannot be evaluated in terms of elementary
functions. It can, however, be evaluated in terms of series [see Problem 4.119].
particle oscillates back

The

(c)

and forth on the inside of the paraboloid with period given by

4t

m r

/2

VI -

fe

sin 2

(5)

cZg

For small vibrations the value of & given by &) can be assumed so small so as to be zero
for practical purposes. Hence (5) becomes

P =

2w^fJc

The length of the equivalent simple pendulum

is

+ A~H)j29
I

^(c

AH).

Supplementary Problems
SIMPLE HARMONIC MOTION AND THE SIMPLE HARMONIC OSCILLATOR

gm moves along the x axis attracted toward the point O on it by a force


equal to 60 times its instantaneous distance x cm from O. If the
numerically
in
and (c) frequency of
particle starts from rest at x = 10, find the (a) amplitude, (6) period
vib/sec
(c)
Vb/2jt
sec,
Ans. (a) 10 cm, (6) 2jt/Vb
the motion.

4.33.

particle of

mass 12

dynes which

is

O of 20 cm/sec, determine
the particle of Problem 4.33 starts at x = 10 with a speed toward
O for the first time.
reaches
the
particle
when
its amplitude, period and frequency. (6) Determine
(b) 0.33 sec
vib/sec;
=
frequency
=
sec,
=
V5/2*period
cm,
2^/V5
Ans. (a) Amplitude
(a) If

4.34.

6^

force proportional
moves on the x axis attracted toward the origin O on it with a
x - 2.5 cm
reaches
and
=
cm
x
at
rest
5
from
starts
it
to its instantaneous distance from O. If
t after it starts,
(6) the speed
any
time
at
position
the
(a)
find
2
sec,
after
time
first
for the
vibration, (d) the maximum acceleration,
at x = 0, (c) the amplitude, period and frequency of the

4 35

particle

(e)

the

Ans.

maximum

speed.

a = 5cosM/6);
cm/sec

(a)

(e) 5?r/6

(6)

5W6

cm/sec;

(c)

cm, 12

sec,

1/12 vib/sec;

(d)

5^/36 cm/sec2

axis, prove that (a) the acceleration


a particle moves with simple harmonic motion along the x
velocity
is numerically greatest in the
the
path,
the
(6)
of
ends
is numerically greatest at the
path, (d) the velocity is zero
the
of
middle
middle of the path, (c) the acceleration is zero in the
at the ends of the path.

4 36

If

4 37

particle

'
'

and

its

moves with simple harmonic motion

maximum

Ans. v/2

sec,

acceleration

2/v vib/sec

is

80 ft/sec 2

in a straight line.

Its

maximum

speed

is

20 ft/sec

Find the period and frequency of the motion.

CHAP.

4.38.

THE SIMPLE HARMONIC OSCILLATOR AND THE SIMPLE PENDULUM

4]

moves with simple harmonic motion.

particle

equilibrium position
4.39.

A
8

particle

cm and

is

If

its

acceleration

A, prove that the period of the motion

is

109

at distance

from the

2tt^D/A.

moving with simple harmonic motion has speeds of 3 cm/sec and 4 cm/sec at distances
cm respectively from the equilibrium position. Find the period of the motion.

Ans. Av sec
4.40.

An

8 kg weight placed on a vertical spring stretches it 20 cm. The weight is then pulled down
a distance of 40 cm and released, (a) Find the amplitude, period and frequency of the oscillations.
(b) What is the position and speed at any time?
Ans. (a) 40 cm, 2ir/l sec, 7/2tt vib/sec

(b)

40 cos It cm,

= 280

sin It cm/sec

4.41.

mass of 200 gm placed at the lower end of a vertical spring stretches it 20 cm. When it is in
equilibrium, the mass is hit and due to this goes up a distance of 8 cm before coming down again.
Find (a) the magnitude of the velocity imparted to the mass when it is hit and (6) the period of
the motion.
Ans. (a) 56 cm/sec, (6) 2jt/7 sec

4.42.

kg mass at the end of a spring moves with simple harmonic motion along a horizontal straight
with period 3 sec and amplitude 2 meters, (a) Determine the spring constant. (6) What is the
maximum force exerted on the spring?
Ans. (a) 1140 dynes/cm or 1.14 newtons/meter

line

(6) 2.28

4.43.

When

10 5 dynes or 2.28 newtons

a mass

hanging from the lower end of a vertical spring i s set into motion,
Prove that the period when mass m is added is Py/l + m/M.

with period P.

THE DAMPED HARMONIC OSCILLATOR


4.44.
(a) Solve the equation
d2 x/dt2 + 2 dx/dt + 5x at t
and (6) give a physical interpretation of
Ans. (a) x J. e -t(10 cos 2* -5 sin 2i)
4.45.

Verify that the damping force given by equation


position

4.46.

4.47.

(2)

of Problem 4.11

is

5,

oscillates

dx/dt

= -3

correct regardless of the

particle.

A 60 lb weight hung on a vertical spring stretches it 2 ft. The weight is then pulled down 3 ft
and released, (a) Find the position of the weight at any time if a damping force numerically
equal to 15 times the instantaneous speed is acting. (6) Is the motion oscillatory damped, overdamped or critically damped?
Ans. (a) x - 3e-(4+l), (6) critically damped
Work Problem
Ans.

4.48.

and velocity of the

subject to the conditions


the results.

it

(a)

4.46

4e~ 2t

if

the

e~ 8t

damping force
overdamped

is

numerically 18.75 times the instantaneous speed.

(b)

In Problem 4.46, suppose that the damping force is numerically 7.5 times the instantaneous speed.
Prove that the motion is damped oscillatory. (6) Find the amplitude, period and frequency of
the oscillations, (c) Find the logarithmic decrement.
(a)

Ans.

(b)

Amplitude

= 2^3 e~^

ft,

period

ir/yfz sec,

frequency

= yfzh

4.49.

Prove that the logarithmic decrement is the time required for the
an oscillation to reduce to 1/e of this value.

4.50.

The natural frequency of a mass vibrating on a spring


damping is 16 vib/sec. Find the logarithmic decrement.

4.51.

is

vib/sec;

maximum

20 vib/sec, while
Ans. 3/4

(c)

2^/V3

amplitude during

its

frequency with

Prove that the difference in times corresponding to the successive maximum displacements of a
oscillator with equation given by (12) of page 88 is constant and equal to

dampe d harmon ic
4n-m/\/4/cm
4.52.

Is

the

/3 2

difference

oscillator the

in

same as

times between successive minimum displacements of a damped harmonic


in Problem 4.51? Justify your answer.

THE SIMPLE HARMONIC OSCILLATOR AND THE SIMPLE PENDULUM

110

[CHAP.

FORCED VIBRATIONS AND RESONANCE


4.53.

The position of a

particle moving along the x axis is determined by the equation d2 x/dt2 + Adxldt +
20 cos 2t. If the particle starts from rest at x = 0, find (a) x as a function of t, (b) the
amplitude, period and frequency of the oscillation after a long time has elapsed.

8*

Ans.

4.54.

(a)

(b)

Amplitude

cos 2t

+ 2 sin 2t - e~ 2t (cos 2t + 3 sin 2t)


= yfh, period = v, frequency =

1/v

Give a physical interpretation to Problem 4.53 involving a mass at the end of a vertical spring.
What is the natural frequency of such a vibrating spring? (c) What is the frequency of the
impressed force?
Ans. (b) y/2/ir, (c) 1/tt
(a)
(b)

4.55.

The weight on a vertical spring undergoes forced vibrations according to the equation
d2 x/dt 2 + 4x = 8 sin at where x is the displacement from the equilibrium position and w >
is a
constant. If at t = 0, x =
and dx/dt = 0, find (a) x as a function of t, (b) the period of
the external force for which resonance occurs.
Ans.

4.56.

(a)

(6)

=
=

(8 sin ut

or

4w

sin 2t)/(4

period

<o

if

^ 2;

sin 2t

2t

cos 2t

if

it

A vertical spring having constant 17 lb wt per ft has a 32 1b weight suspended from it. Ah
A damping
external force given as a function of time t by F(t) = 65 sin4t,
is applied.
force given numerically in lb wt by 2v, where v is the instantaneous speed of the weight in ft/sec,
(a) Determine the
is assumed to act.
Initially the weight is at rest at the equilibrium position,
position of the weight at any time. (6) Indicate the transient and steady-state solutions, giving
physical interpretations of each, (c) Find the amplitude, period and frequency of the steady-state

i0

[Use

solution.

Ans.

32 ft/sec 2 .]

4e -t cos4t

Amplitude

(c)

y/Vf

period

ft,

cos4t

frequency

v/2 sec,

2/tr

vib/sec

cm by a force of 50 dynes. A mass of 10 gm is placed on the lower end


After equilibrium has been reached, the upper end of the spring is moved
up and down so that the external force acting on the mass is given by F(t) = 20 cos at, t0.
(a) Find the position of the mass at any time, measured from its equilibrium position. (6) Find the
value of (o for which resonance occurs.

of

spring
the

Ans.

4.58.

+ sin 4t 4 cos 4*
-t
Transient, 4e
cos4t; steady-state, sin4i
=

(a)
(6)

4.57.

(a)

is

stretched 5

spring.

(20 cos ut)/(l

- u 2 - 20
)

cos

(b)

t,

A periodic external force acts on a 6 kg mass suspended from the lower end of a vertical spring
having constant 150 newtons/meter. The damping force is proportional to the instantaneous speed
of the mass and is 80 newtons when the speed is 2 meters/sec. Find the frequency at which
Ans. 5/6v vib/sec
resonance occurs.

THE SIMPLE PENDULUM


4.59.

4.60.

Find the length of a simple pendulum whose period is 1 second. Such a pendulum which registers
Ans. 99.3 cm or 3.26 ft
seconds is called a seconds pendulum.
Will a pendulum which registers seconds at one location lose or gain time when
another location where the acceleration due to gravity is greater? Explain.

it

is

moved

to

Ans. Gain time

4.61.

simple pendulum whose length is 2 meters has its bob drawn to one side until the string makes
an angle of 30 with the vertical. The bob is then released, (a) What is the speed of the bob as
(c) What
(6) What is the angular speed at the lowest point?
it passes through its lowest point?
is the maximum acceleration and where does it occur?

Ans.

(a)

2.93 m/sec,

(6)

1.46 rad/sec,

(c)

2 m/sec2

CHAP.

4.62.

4.63.

THE SIMPLE HARMONIC OSCILLATOR AND THE SIMPLE PENDULUM

4]

HI

Prove that the tension in the string of a vertical simple pendulum of length I and mass m
by mg cos e where a is the instantaneous angle made by the string with the vertical.

is

given

seconds pendulum which gives correct time at a certain location is taken to another location
it is found to lose T seconds per day. Determine the gravitational acceleration at the second
location.
Ans. g(l T/86,400) 2 where g is the gravitational acceleration at the first location

where

4.64.

What

is

the length of a seconds pendulum on the surface of the moon where the acceleration due
approximately 1/6 that on the earth?
Ans. 16.5 cm

to gravity is

4.65.

simple pendulum of length I and mass


hangs vertically from a fixed point O. The bob is given
initial horizontal velocity of magnitude v
Prove that the arc through which the bob swings
in one period has a length given by 41 cos -1 (1 v 2/2gl)

an

4.66.

Find the minimum value of v

in

vertical circle with center at 0.

Problem 4.65
Ans.

order that the bob will

in

make a complete

2~\fgl

THE TWO AND THREE DIMENSIONAL HARMONIC OSCILLATOR


4.67.

mass 2 moves

xy plane attracted to the origin with a force given by


the particle is placed at the point (3, 4) and given a velocity
of magnitude 10 in a direction perpendicular to the x axis, (a) Find the position and velocity of
the particle at any time. (6) What curve does the particle describe?
particle of

F = 18*i 50?/j.

Ans.

4.68.

4.69.

(a) r

Find the

At

3 cos 3t

[4

in the

cos ht

+2

sin 5] j,

= 9

energy of the particle of Problem

total

sin St

[10 cos 5t

- 20

sin 5t] j

Ans. 581

4.67.

two dimensional harmonic oscillator of mass 2 has potential energy given by V = S(x 2 + 4y 2 ).
and velocity of the oscillator at time t =
are given respectively by
r = 2i j and v = 4i + 8j, (a) find its position and velocity at any time t >
and (6) determine the period of the motion.
If the position vector

Ans.

(a)

(2 cos At

+ sin 4t)i + (sin St - cos 8)j,

(4

cos 4t

-8

sin 4*)i

+ (8

cos 8i

+8

sin 8*)j

(6) s-/8

4.70.

Work Problem

V=

4.69 if

8(x 2

+ 2y 2).

Is there

a period defined for the motion in this case?

Explain.

4.71.

of mass
moves in a 3 dimensional force field whose potential is given by
^k(x 2 + 4y 2 + 16z 2 ). (a) Prove that if the particle is placed at an arbitrary point in space
other than the origin, then it will return to the point after some period of time. Determine this
time. (6) Is the velocity on returning to the starting point the same as the initial velocity? Explain.
particle

V=

4.72.

Suppose that

in

Problem 4.71 the potential

to the starting point?

is

V=

%k(x 2

+ 2y 2 + 52

).

Will the particle return

Explain.

MISCELLANEOUS PROBLEMS
4.73.

A
is

vertical spring of constant k having natural length I is supported at a fixed point A.


placed at the lower end of the spring, lifted to a height h below
and dropped.

the lowest point reached will be at a distance below

4.74.

4.75

Work Problem

4.73 if

Given the equation


if

damping proportional

mx + /3x + kx =

E = \mx 2 + \kx 2

decreases with time.

then

E=

given by

to the instantaneous velocity is

mass

taken into account.

damped oscillations of a harmonic oscillator. Prove that


Thus show that if there is damping the total energy E

for

-p'x 2

What happens

A
Prove that
+ mg/ K + y/m 2 g 2/ K 2 + 2mgh/n.

to the

energy lost? Explain.

THE SIMPLE HARMONIC OSCILLATOR AND THE SIMPLE PENDULUM

112

4.76.

(a)

A =

where

(6)

4.77.

4.78.

Prove that

Use
and

cos (at

+ A\ + 2A A 2

y/A\

+ A2

4> x )

cos

(^ -

cos (at

<p 2 )

cos (at

<fi

</>)

+ A2
+ A2

sin^t

/ A-l
<p 2 ),

^'(^cos^

[CHAP. 4

sin^> 2 \
'

cos <t>J

demonstrate that the sum of two simple harmonic motions of the same frequency
in the same straight line is simple harmonic of the same frequency.
to

(a)

Give a vector interpretation to the results of Problem 4.76.


equal.
Discuss Problem 4.76 in case the frequencies of the two simple harmonic motions are not
motion simple harmonic? Justify your answer.

Is the resultant

4.79.

and y from two mutually perpendicular

particle oscillates in a plane so that its distances x


axes are given as functions of time t by

= A

cos (at

= B

X ),

cos (at

</>

2)

the rectangle defined by


(a) Prove that the particle moves in an ellipse inscribed in
b (&) p r ove that the period of the particle in the elliptical path is 2ir/a.
y
4.80.

Suppose that the particle of Problem 4.79 moves so that


X

where

assumed

is

= A

COS (at

+ 0i),

be a positive constant which

to

COS (at

assumed

is

+ et +

Illustrate

Problem 4.80 by graphing the motion of a particle which moves


x

4.82.

+ n-/4),

3 cos(2t

in the

path

4 cos (2.4t)

which is on a frictionless
In Fig. 4-21 a mass
points A and B by
fixed
to
table is connected
two springs of equal natural length, of negligible mass and spring constants /q and /c 2 reis displaced horizontally
spectively. The mass
that the perio d of
Prove
released.
then
and

oscillation is given

4.83.

2)

much smallertiian a. Vrvve


rectangle x - A, y - B.

to be

that the particle oscillates in slowly rotating ellipses inscribed in the


4.81.

= A,

by

P=

2v-\/'m/(K 1

K2

-nmr-

m -^TOKT^-

A
Fig. 4-21

k 2 ).

spring having constant k and negligible mass has


one end fixed at point A on an inclined plane of
at the other end, as indicated
angle a and a mass

If the

in Fig. 4-22.

mass

m is pulled down

a distance

below the equilibrium position and released, find


the displacement from the equilibrium position at any
time if (a) the incline is frictionless, (6) the incline
has coefficient of friction /i.
x

4.84.

particle

located at

4 85.

4.86.

moves with simple harmonic motion along the x


*

a, b

and

respectively.

Fig. 4-2

axis.

At times

Prove that the period of oscillation

is

2tQ an dJ*o

cos -i (a

i*

)/26*

location is taken to another locati or where


seconds pendulum giving the correct time at one
rod be lengthened or shortened in
pendulum
the
must
By how much
it loses 5 minutes per day.
order to give the correct time?

is susvertical pendulum having a bob of mass


pended from the fixed point O. As it oscillates, the
A [or OC\
string winds up on the constraint curves OB
ABC is a
as indicated in Fig. 4-23. Prove that if curve
the same
be
will
oscillation
of
period
the
cycloid, then

penregardless of the amplitude of the oscillations. The


The
pendulum.
cycloidal
a
called
is
dulum in this case
of
curves ODA and OC are constructed to be evolutes

the cycloid.

[Hint.

Use Problem

4.31.]

Iig.4-A

CHAP.

4.87.

THE SIMPLE HARMONIC OSCILLATOR AND THE SIMPLE PENDULUM

4]

A bead slides down a frictionless wire located in a vertical plane. It is desired to find the
shape of the wire so that regardless of where the bead is placed on the wire it will slide under
the influence of gravity to the bottom of the wire in the same time. This is often called the
tautochrone problem.
Prove that the wire must have the shape of a cycloid.
Use Problem

[Hint.
4.88.

4.31.]

Prove that the curves

ODA

OC

and

of Problem 4.86 are cycloids having the

same shape as the

ABC.

cycloid
4.89.

113

simple pendulum of length I has its point of support moving back and forth on a horizontal line
from a fixed point on the line is A sin ut, t ^ 0. Find the position of the
pendulum bob at any time t assuming that it is at rest at the equilibrium position at * = 0.
so that its distance

4.90.

Work Problem

4.91.

4.92.

4.89 if the point of support moves vertically instead of horizontally and


the rod of the pendulum makes an angle O with the vertical.

vertical elastic spring of negligible weight

and having its upper end fixed, carries a weight


lower end. The weight is lifted so that the tension in the spring is zero, and then it is
released. Prove that the tension in the spring will not exceed 2W.

its

having constant
has a pan on top of it with
on it [see Fig. 4-24]. Determine the largest frequency with which the spring can vibrate so that the weight
will remain in the pan.
vertical spring

a weight

4.94.

at

A particle of mass
moves in a plane under the influence of forces of attraction toward fixed
points which are directly proportional to its instantaneous distance from these points.
Prove
that in general the particle will describe an ellipse.

W at

4.93.

if

/c

spring has a natural length of 50 cm and a force of 100 dynes


required to stretch it 25 cm. Find the work done in stretching
the spring from 75 cm to 100 cm, assuming that the elastic limit
is not exceeded so that the spring characteristics do not change.
is

Ans. 3750 ergs


4.95.

A
x

moves

particle

= Acosw,

in the

=B

xy plane so that its position is given by


Prove that it describes an arc of a

cos 2ut.

parabola.
4.96.

A
y

Fig. 4.24

particle

= Bcos(w 2 t +

rational or not.
4.97.

4.98.

4.101.

4.102.

2 )-

Find the period of a simple pendulum of length


vertical is

(a)

30,

(6)

60,

(c)

meter

if

the

maximum

d2 x/dt2
(a)

its

Ay,

position

angle which the rod

90.

A simple pendulum of length 3 ft is suspended vertically from a fixed point. At t = the bob is
given a horizontal velocity of 8 ft/sec. Find (a) the maximum angle which the pendulum rod
makes with the vertical, (b) the period of the oscillations.
Ans.

4.100.

<f>

The position of a particle moving in the xy plane is described by the equations


d 2 y/dt2 = -4x. At time t =
the particle is at rest at the point (6, 3). Find
and (b) its velocity at any later time t.

makes with the


4.99.

in the xy plane so that its position is given by


x = A cos (u^ + 9^),
Prove that the particle describes a closed curve or not, according as a t /u 2 is
In which cases is the motion periodic?

moves

(a)

cos" 1 2/3

41

48',

(6)

1.92 sec

Prove that the time averages over a period of the potential energy and kinetic energy of a
simple harmonic oscillator are equal to 2tt 2A 2IP 2 where A is the amplitude and P is the period
of the motion.

A cylinder of radius 10 ft with its axis vertical oscillates vertically in water of density 62.5 lb/ft3
with a period of 5 seconds. How much does it weigh?
Ans. 3.98Xl0 5 lbwt
A

V=

particle moves in the xy plane in a force field whose potential is given by


x2
xy y 2 .
If the particle is initially at the point (3, 4) and is given a velocity of magnitude 10 in a direction
parallel to the positive x axis, (a) find the position at any time and (6) determine the period of

the motion

if

one exists.

THE SIMPLE HARMONIC OSCILLATOR AND THE SIMPLE PENDULUM

114

4.103.

4.104.

4.105.

[CHAP.

t
the particle is at the
In Problem 4.96 suppose that a x /a 2 is irrational and that at
particular point (x ,y ) inside the rectangle denned by x = A, y B. Prove that the point
(x ,y ) will never be reached again but that in the course of its motion the particle will come
arbitrarily close to the point.

particle oscillates on a vertical frictionless cycloid with its vertex downward. Prove that the
projection of the particle on a vertical axis oscillates with simple harmonic motion.

A mass of 5 kg at the lower end of a vertical spring which has an elastic constant equal to
20 newtons/meter oscillates with a period of 10 seconds. Find (a) the damping constant, (6) the
Ans. (a) 19 nt sec/m, (6) 3.14 sec
natural period and (c) the logarithmic decrement.

4.106.

4.107.

mass of 100 gm is supported in equilibrium by two identical


springs of negligible mass having elastic constant equal to
50 dynes/cm. In the equilibrium position shown in Fig. 4-25
the springs make an angle of 30 with the horizontal and are
100 cm in length. If the mass is pulled down a distance of
2 cm and released, find the period of the resulting oscillation.
thin hollow circular cylinder of inner radius 10 cm is fixed
A particle is placed on the inner
frictionless surface of the cylinder so that its vertical distance
so that its axis is horizontal.

above the lowest point of the inner surface is 2 cm. Find


and
(a) the time for the particle to reach the lowest point
(b) the period of the oscillations which take place.

4.108.

cubical box of side a and weight

period of vibration

4.109.

vibrates vertically in water of density

spring vibrates so that

Ans.

0,

dx/dt

at

r*

yrriK

4.110.

Work Problem

4.111.

its

equation of motion

find

0,

it

F(u) sin

V /m

(t

4.109

is

kx

F(t)

as a function of time

u)

t.

du

damping proportional

if

spring vibrates so that

its

at
v
If x = 0, x
resonance will occur.

d2 x/dt2

kx

(a) find x at

0,

taken into account.

to dx/dt is

equation of motion

is

5 cos

any time

cot

and

2 cos 3wt
(6)

determine for what values of a

the
at its lower end. At t =
mass
that
its distance
so
vertically
move
to
made
suddenly
spring is in equilibrium and its
position of the mass
from the original point of support is given by A sin a t, t 0. Find (a) the
occurs.
resonance
which
for
of
<o
values
the
at any time and (b)

vertical spring having elastic constant k carries a

upper end

4.113.

2 x/dt*

^o

4 112

Prove that the

{2ir/a)^<jg/W.

is

md
If

o.

(a)

Solve d*x/dt*

sin

cos

is

where x

0,

dx/dt

at

0,

and

(6)

give a physical

interpretation.

4.114.

where damping and external forces are


Discuss the motion of a simple pendulum for the case
present.

CHAP.

4.115.

4.116.

THE SIMPLE HARMONIC OSCILLATOR AND THE SIMPLE PENDULUM

4]

115

Find the period of small vertical oscillations of a cylinder of radius a and height h floating
with its axis horizontal in water of density a.

vertical spring

having

elastic constant

2 newtons per meter has a 50

gm

weight suspended

force in newtons which is given as a function of time t by F(t) = 6 cos4 t, t ^


is applied.
Assuming that the weight, initially at the equilibrium position, is given an upward
velocity of 4 m/sec and that damping is negligible, determine the (a) position and
(6) velocity
of the weight at any time.

from

it.

4.117.

In Problem 4.55, can the answer for = 2


the limit as u - 2? Justify your answer.

4.118.

An

by taking

oscillator has a restoring force acting on it whose magnitude is kx ex 2 where


compared with k. Prove that the displacement of the oscillator [in this case often
anharmonic oscillator] from the equilibrium position is given approximately by

where

4.119.

be deduced from the answer for

and

Prove that

if

</>

= A

cos (ut-<f>)

A 2

^-{cos2(ut

is

small

called

an

d>)-3}

OK

are determined from the initial conditions.

the oscillations in Problem 4.32 are not necessarily small, then the period

is

given by

Chapter 5

CENTRAL FORCES
and PLANETARY MOTION

CENTRAL FORCES
Suppose that a force acting on a particle of

mass

(a)

is

such that [see Fig.

it is

always directed from

away from
(b)

5-1]:

toward or

a fixed point O,

magnitude depends only on the distance


r from O.

its

call the force a central force or central


as the center of force. In symforce field with
bols F is a central force if and only if

Then we

F = f(r)n =
where

ri

The
f(r)

<

f(r)r/r

Fig. 5-1

(1)

r/r is a unit vector in the direction of

r.

central force is one of attraction toward


respectively.
or f(r) >

or repulsion from

according as

SOME IMPORTANT PROPERTIES OF CENTRAL FORCE FIELDS


If a particle

moves

The path or

1.

a plane.

are valid.
in a central force field, then the following properties

orbit of the particle

This plane

is

must be a plane

curve,

often taken to be the xy plane.


of the particle is conserved,

i.e.

the particle

See Problem

moves

in

5.1.

constant. See Problem 5.2.

2.

The angular momentum

3.

drawn
The particle moves in such a way that the position vector or radius vector
the
words,
other
In
times.
equal
in
areas
equal
to the particle sweeps out
from
time rate of change in area
See Problem 5.6.

is

constant.

This

is

i.e. is

sometimes called the law of areas.

EQUATIONS OF MOTION FOR A PARTICLE


IN A CENTRAL FIELD
the motion of a particle in a central force field takes place in a plane. Choosing this
plane as the xy plane and the coordinates of the particle as polar coordinates (r, 6), the equations of mo-

By Property

1,

tion are found to be [see

Problem

m(r-r) =

5.3]
(2)

f{r)

m(r'e+ 2r9) =

where dots denote


time

(r,)

(8)

differentiations with respect to

t.

116

Fig. 5-2

CHAP.

CENTRAL FORCES AND PLANETARY MOTION

5]

From

equation

we

(3)

find

r2 6

This

is

117

related to Properties 2

constant

(4)

and 3 above.

IMPORTANT EQUATIONS DEDUCED FROM


THE EQUATIONS OF MOTION
The following equations deduced from the fundamental equations

and

(2)

(3)

often

prove to be useful.

h2

=
3

1
-!

f(r)

du

where u =

(K\

'

1/r.

d0 2

r\de)

mh2

POTENTIAL ENERGY OF A PARTICLE IN A CENTRAL FIELD

A central force field is a conservative


This potential which depends only on r
given by
=

V(r)

hence it can be derived from a potential.


apart from an arbitrary additive constant,

field,
is,

-jf(r)dr

(g)

This is also the potential energy of a particle in the central force


additive constant can be obtained by assuming, for example, 7 =
as r-* oo.

field.

at r

The arbitrary

or

V -*

CONSERVATION OF ENERGY
By

using

im^ + r^e

),

(8) and the fact that in polar coordinates the kinetic energy of a particle
the equation for conservation of energy can be written

im(r2 + r2 6 2 )
or

E is the total

energy and

mh
and

- f

\m(r2 + r2 6 2 )

where

also as

In terms of u

is

Vfdr\

f(
=

1/r,

we can

f2

f(r)

S) ~

dr

J ^ dr

(P)

(io)

equation (10) can also be written as

(4),

= E

V(r)

Using

constant.
2

is

also write equation (9) as

f^V
KdeJ

^
+ u2 _
=
.

2(E~V)

mh2

(I 3 )

DETERMINATION OF THE ORBIT FROM THE CENTRAL FORCE


If the central force field is prescribed,
;d, i.e. if f(r) is given, it is possible
possi
to determine
the orbit or path of the particle. This orbit
rbit can be obtained in the form

r{0)
(

CENTRAL FORCES AND PLANETARY MOTION

118

i.e.

r as a function of

9,

which are parametric equations

or to use equations

(4)

r{t),

(IS)

9{t)

terms of the time parameter

in

To determine the orbit in the form (U)


or (11). To obtain equations in the form (15),
(A)

or in the form
r

with

[CHAP.

and

t.

convenient to employ equations (6), (7)


sometimes convenient to use (12) together

is

it

it is

(5).

DETERMINATION OF THE CENTRAL FORCE FROM THE ORBIT


Conversely

if

the orbit or path of the particle, then we can find the correspondcentral
If the orbit is given by r = r(9) or u = u(9) where u = 1/r, the

we know

ing central force.


force can be found from

mh2

f(r)

de*

f(Vu)

or

2 /dr\ 2
,de)

[d2 r

-mh 2u2

d2u
de 2

(16)

+u

(17)

can also be
which are obtained from equations (6) and (7) on page 117. The law of force
obtained from other equations, as for example equations (9)-(13).

many

important to note that given an orbit there may be infinitely


which the orbit is possible. However, if a central force field exists
It is

it is

force fields for

unique,

i.e.

it is

the only one.

CONIC SECTIONS, ELLIPSE, PARABOLA AND HYPERBOLA


and a

Consider a fixed point

Suppose that a point

in the plane of

from

to its distance

from point

line

Then the curve described by P


polar coordinates (r, 9) by

AB

is

XV

Focus

<

\^ A'

/
/

^ SFig. 5-3

[See Fig. 5-4 below.]

is

Note that the major axis


length 2a.

Dir

the center of the ellipse and CV = CU = a


axis, then the equation of the ellipse can be written as

r/\

is called

value of the eccentricity.

If

\p

a focus, the line AB is


called a directrix and the ratio e is called the
The curve is often called a conic
eccentricity.
section since it can be obtained by intersecting
Three
a plane and a cone at different angles.
possible types of curves exist, depending on the

Ellipse:

^-

(18)

COS 9

distance
.

11

^ ^v

See Problem 5.16.

The point

in Fig. 5-3.
its

always equal to the positive constant

given in

is

AB distant D from 0, as shown


and AB moves so that the ratio of

fixed line

is

is

the length of the semi-major

a(l-e2 )
i

(19)

cos e

the line joining the vertices

and

of the ellipse

and has

CHAP.

CENTRAL FORCES AND PLANETARY MOTION

5]

If b is the length of the

119

semi-minor axis

[CW or CS in Fig. 5-4] and c is the distance


CO from center to focus, then we have the
important result

\Za 2

b2

ac

(20)

circle can be considered as a special case


of an ellipse with eccentricity equal to zero.
2.

Parabola:

[See Fig. 5-5.]

The equation

of the parabola is

(21)

cos

We can consider a parabola to be a


limiting case of the ellipse (19) where -* 1,
which means that a - o [i.e. the major
axis becomes infinite] in such a way that
a(l - 2 ) = p.
Hyperbola:

>

Fig. 5-5

[See Fig. 5-6.]

The hyperbola consists of two branches


as indicated in Fig. 5-6. The branch on the
left is the important one for our purposes.

The hyperbola

is

lines of Fig. 5-6


totes.

The

is called

asymptotic to the dashed

which are

intersection

asymp-

asymptotes
The distance CV = a

the center.

from the center C

called its

C of the

to vertex

is

called the

semi-major axis [the major axis being the


distance between vertices V and U by analogy with the ellipse]. The equation of the
hyperbola can be written as
r

a(<2

=
1

- l)
cos 9

(22)

Fig. 5-6

Various other alternative definitions for conic sections may be given. For example, an
can be defined as the locus or path of all points the sum of whose distances from two
fixed points is a constant. Similarly, a hyperbola can be defined as the locus of all points
the difference of whose distances from two fixed points is a constant. In both these cases
the two fixed points are the foci and the constant is equal in magnitude to the length of
ellipse

the major axis.

SOME DEFINITIONS IN ASTRONOMY


A solar system is composed of a star [such

as our sun] and objects called planets which


revolve around it. The star is an object which emits its own light, while the planets do
not emit light but can reflect it. In addition there may be objects revolving about the
planets. These are called satellites.

In our solar system, for example, the moon is a satellite of the earth which in turn is a
planet revolving about our sun. In addition there are artificial or man-made satellites
which can revolve about the planets or their moons.

CENTRAL FORCES AND PLANETARY MOTION

120

[CHAP.

The path of a planet or satellite is called its orbit. The largest and smallest distances
of a planet from the sun about which it revolves are called the aphelion and perihelion
respectively. The largest and smallest distances of a satellite around a planet about which
it

revolves are called the apogee and perigee respectively.

revolution of a body in an orbit is called its period. This is


sometimes called a sidereal period to distinguish it from other periods such as the period

The time for one complete

of earth's motion about

its axis, etc.

KEPLER'S LAWS OF PLANETARY MOTION

/^

Before Newton had enunciated his famous laws


of motion, Kepler, using voluminous data accumulated by Tycho Brahe formulated his three laws
concerning the motion of planets around the sun

^N^pianet

[see Fig. 5-7].


1.

Every planet moves


ellipse

2.

in

an orbit which

is

an

with the sun at one focus.

Fig. 5-7

The radius vector drawn from the sun

to

any planet sweeps out equal areas

in

equal times (the law of areas, as on page 116).


3.

of the periods of revolution of the planets are proportional to the cubes


of the semi-major axes of their orbits.

The squares

NEWTON'S UNIVERSAL LAW OF GRAVITATION


law and equations (16) or (17), Newton was able to deduce his
famous law of gravitation between the sun and planets, which he postulated as valid for any
objects in the universe [see Problem 5.21].
Newton's Law of Gravitation. Any two particles of mass mi and m 2 respectively and

By

using Kepler's

first

distance r apart are attracted toward each other with a force

F
where

is

_ Gmim2

ri

^3)

a universal constant called the gravitational constant.

using Newton's law of gravitation we can, conversely, deduce Kepler's laws [see
Problems 5.13 and 5.23]. The value of G is shown in the table on page 342.

By

ATTRACTION OF SPHERES AND OTHER OBJECTS


using Newton's law of gravitation, the forces of attraction between large objects
such as spheres can be determined. To do this, we use the fact that each large object is
composed of particles. We then apply the law of gravitation to find the forces between
particles and sum over these forces, usually by methods of integration, to find the resultant
force of attraction. An important application of this is given in the following

By

uniform spheres of masses mi and m 2 respectively


which do not intersect are attracted to each other as if they were particles of the same

Theorem

mass situated

5.1.

Two

solid or hollow

at their respective geometric centers.

Since the potential corresponding to

-^^ri

(24)

CHAP.

5]

CENTRAL FORCES AND PLANETARY MOTION

121

V = -Q*

{25)

is

it is

between objects by

also possible to find the attraction

then using

F = -y7.

finding the potential and

first

See Problems 5.26-5.33.

MOTION IN AN INVERSE SQUARE FORCE FIELD


As we have seen, the planets revolve in elliptical orbits about the sun which is at one
focus of the ellipse. In a similar manner, satellites (natural or man-made) may revolve around
planets in elliptical orbits. However, the motion of an object in an inverse square field of
attraction need not always be elliptical but may be parabolic or hyperbolic. In such cases
the object, such as a comet or meteorite, would enter the solar system and then leave but
never return again.
The following simple condition
an

in

terms of the

energy

total

determines the path of

object.
(i)

if

(ii)

if

(iii)

if

E<
E=
E>

Other conditions

in

the path

is

an

the path

is

a parabola

the path

is

a hyperbola

ellipse

terms of the speed of the object are also available.

See Problem 5.37.

In this chapter we assume the sun to be fixed and the planets do not affect each other.
Similarly in the motion of satellites around a planet such as the earth, for example, we
assume the planet fixed and that the sun and all other planets have no effect.

Although such assumption is correct as a first approximation, the influence of other


planets may have to be taken into account for more accurate purposes. The problems of
dealing with the motions of two, three, etc., objects under their mutual attractions are often
called the two body problem, three body problem, etc.

Solved Problems

CENTRAL FORCES AND IMPORTANT PROPERTIES


5.1.

Prove that

if

a particle moves in a central force

field,

then

its

path must be a plane

curve.
Let

F =

f(r) r x

be the central force


r

since r 1

is

field.

X F =

Then
f(r) r

rx

(1)

a unit vector in the direction of the position vector

r.

Since

F = mdv/dt,

this

can be

written

X dx/dt =

(2)

|(rXv) =

(3)

or
Integrating,

where h

is

we

find

a constant vector.

X v

= h

Multiplying both sides of

rh =

(-4)

(4)

by

(5)

using the fact that r (r X v) = (r X r) v = 0. Thus r is perpendicular to the constant vector h,


and so the motion takes place in a plane. We shall assume that this plane is taken to be the
xy plane whose origin is at the center of force.

CENTRAL FORCES AND PLANETARY MOTION

122

5.2.

Prove that for a particle moving

in a central force field the

[CHAP.

angular

momentum

is

conserved.

From

equation

of Problem 5.1,

(4)

we have
X v = h

where h

is

Then multiplying by mass m,

a constant vector.

w(r X

= mh

v)

(1)

Since the left side of (1) is the angular momentum, it follows that the angular
conserved, i.e. is always constant in magnitude and direction.

momentum

is

EQUATIONS OF MOTION FOR A PARTICLE IN A CENTRAL FIELD


5.3.

Write the equations of motion for a particle in a central


By Problem 5.1 the motion of the particle takes place in a

field.

plane. Choose this plane to be


the xy plane and the coordinates describing the position of the particle at any time t to be
polar coordinates (r, e). Using Problem 1.49, page 27, we have

(mass) (acceleration)

net force

f( r ) r i

m{(r
Thus the required equations

re 2 )r 1

m(r e

Show

that r 2 6

Method

(3)

(r 2 e)

m (r

2re)

is

Method

and so

By Problem

Then from equation

1.49,

(4)

5.5.

2rre)

d_

~r

di

(r 2 e)

(1)

rt

and

frj

rbei

of Problem 5.1

$x

X v =

f(r

i.e.

rx)

ro(r

X oj = r2 tfk

(2)

the unit vector in a direction perpendicular to the


Using h = fik in (2), we see that
in the direction r X v.

where k

rk

plane of motion [the xy plane],


r2 e

27, the velocity in polar coordinates is

page

h
X

(8)

a constant.

2.

2r'e)

(2)

f{r)

r2

since

dt

where h

of Problem 5.3 can be written

m(r

Thus

re 2 )

(*)

a constant.

h,

Equation

1.

2r'e)0i}

by

of motion are given

m(r -

5.4.

{r'o

is

h.

Prove that r2 & = 2A where A is the time rate


at which area is swept out by the position
vector

r.

time At the particle moves from


The area AA swept out by the
position vector in this time is approximately half the
area of a parallelogram with sides r and Ar or [see

Suppose that

M to N

in

[see Fig. 5-8].

Problem

1.18,

page

15]

AA = A|rXAr|
Dividing by Ai and letting
lim
At-0

AA
At

A-*0,
Ar
lim - r X
At
At-tO 2
I

|r

v|

Fig. 5-8

CHAP.

CENTRAL FORCES AND PLANETARY MOTION

5]

A = i|rXv| =

i.e.,

using the result in Problem

Thus r2 o - 2A, as

5.4.

\r2

required.

A = Ak = i(rXv) =
is

5.6.

The vector quantity

^(r2 *)k

often called the areal velocity.

in a central force field the areal velocity is constant.

Prove that for a particle moving


By Problem

5.4,

r2

The

result

\hk =

r2 6k

often stated as follows:

is

center, then the radius vector

This result

times.

Then the

a constant.

A =

5.7.

123

is

Show by means

sometimes called the law of areas.

r
is

d2u

1/u

that the differential equation for the

- f(l/u)

+ u
,

do 2
5.4 or equation (S) of

r2 o

Substituting into equation

1/u,

we have
-tt

dt

From

this

we

we

5.3,

/r3

,du\

d (

dr

we have
hu 2

(1)

find
)

(2)

f(r)

de\r

du\ do
Te)d-t

_
~

(5)

"

W^
t,2..2

d2u

//\
(4)

see that (2) can be written

m(-h 2 u 2 d2 u/do 2 - h2 u?) =


or,

h/r2

d (

= Tt = dt\- h Te) =

5.3,

h_ dr_
_vdu
dr cfo _
_
=
= -z _
= -*~
~ -^^r
do
de dt ~ r2 do

dr

or

Problem

of

(2)

mh u2

Problem

m(fif

^h, a constant vector

a particle moves in a central force field with O as


to the particle sweeps out equal areas in equal

If

path of the particle in a central field

Now

is

drawn from O

of the substitution

From Problem

areal velocity

^+

as required,

= "

f(l/u)

(5)

^f

POTENTIAL ENERGY AND CONSERVATION OF ENERGY


FOR CENTRAL FORCE FIELDS
5.8.

Prove that a central force field is conservative and


potential energy of a particle in this field.

(a)

Method
If

find the corresponding

1.

we can

that the field

find the potential or potential energy, then we will have also incidentally
exists, it must be such that
if the potential
conservative.

Now

is

F-rfr

where F

f(r) r x

is

the central force.

F
since

(b)

Since

dr

dr

We

f(r) r t

V
= -dV

dr

f(r)-

r dr.

we can determine

(1)

have

such that

-dV =

f(r)dr

proved

dr

f(r)

dr

CENTRAL FORCES AND PLANETARY MOTION

124

V = -

for example,
it

follows that the field

Method

Method

VXF=

can show that

The

1.

but this method

directly,

tedious although straightforward.

is

Method

is

2.

The equations of motion for a

Multiply equation

(1)

by

r,

equation

Then integrating both

sides,

we

re 2

r2

"*"

1.49,

page

27]

r2 * 2

f(r)dr

- E

2re)

by Problem

5.3,
(1)

f(r)

(2)

re and add to obtain

r o

lm^(r2 +

This can be written

rre 2 )

r 2 'e 2 )

f(r)r

(3)

f f(r) dr

U)

= E

(5)

obtain
i

m(r2

r2

'e

f(r)

dr

that the differential equation describing the motion of a particle in a central

can be written as

h2
2r

From Problem

5.9

^\

we have by

have

also

Substituting

we

+ 1*]-ff(r)dr = E

the conservation of energy,

_
~

{2) into (1),

l\dej

|m(r2

r2 e 2 )

(5)
e

h/r 2

If

(b)

Use

dr

dt

= E
dr

~d6

= E

l[() 2+r2]-J'w *

(a) to

(1)

find

">-J'"*

u=l/r, prove

(a)

f(r)

_ dr do _
~ Tedt ~

dr

since

[Problem

particle in a central field are,

by

(2)

m(r r

We

is

a constant.

m(r 6

field

im(rHrV) -

or

m(r

Show

y2

s0 tnat

reOi

+ V = E

imt) 2

rr t

in a central force field.

principle of conservation of energy can be expressed as

Then the

5.11.

represents the potential or potential energy.

(2)

velocity of a particle expressed in polar coordinates

5.10.

(2)

Write the conservation of energy for a particle of mass

where

2.

We

5.9.

conservative and that

is

dr

f(r)

[CHAP.

that v 2

f2

= ^2 {(d%/^) 2 + u2 }.

prove that the conservation of energy equation becomes


(du/dOf

v?

= 2(E-V)/mh2

CHAP.

CENTRAL FORCES AND PLANETARY MOTION

5]

From

(a)

equations

(i)

v2

From

(b)

and

r2

we have

of Problem 5.7

(3)

'

r2^2

hHdu/de) 2

(l/u2 )(hu2 2
)

the conservation of energy [Problem 5.9] and part

x mv 2

'

= i m (f2 + r e 2)

-V

= e

hu 2

hdu/de.

2 {(du/de) 2

u2

Thus
}

(a),

(du/de) 2

or

125

+ u2 = 2(E - V)/mh2

DETERMINATION OF ORBIT FROM CENTRAL FORCE,


OR CENTRAL FORCE FROM ORBIT
5.12.

Show

that the position of the particle as a function of time

can be determined

from the equations


t

where

J*

h/r2

Then assuming the

- f

h2 /r2 )

m + m- Jf

r2

or

positive square root,

5.13.

Show

that

if

5.9,

= E

-%
r

G(r)'

[G(r)]-v*dr

h/r 2

is

find

dt

as

r2 de/h

and integrating.

denned by

= -Kir2

K>0

an inverse square law of attraction, then the path of the particle

i.e.

Method

is

a conic.

1.

In this case f(l/u)

we

the law of central force


/(r)

2 2

y/Gir)

and so separating the variables and integrating, we

The second equation follows by writing

-^^
2m r

we have
dr/dt

r2 d6

energy of Problem

dr

f(r)

dr

f(r)
JS '

dr

f(r)'
JK

in the equation for conservation of

%m(r2

= | f

m + m Jf

G(r)
v '

Placing

[G(r)]-"*dr,

= Ku2

Substituting into the differential equation of motion in Problem 5.7,

find

d2u/de 2

+ u = K/mh 2

(l)

This equation has the general solution

= A

cos 8

4-

sin

+ K/mh 2

(2)

or using Problem 4.2, page 92,

1,e "

It is

= K/mh 2 + C

cos

{o

cos

(e

- 0)

0,

in

K/mh 2 + C

always possible to choose the axes so that

<j>

<p)

which case we have

K/mh 2 + Ccoss

{$)

CENTRAL FORCES AND PLANETARY MOTION

126

[CHAP.

This has the general form of the conic [see Problem 5.16]

Then comparing

(5)

and

(6)

we

or
Since

2.

= -K/r 2 we

/(r)

we assume

If

5.10,

= C

e/p

m/i2 /K,

f(r)

-K/r +

as

dr

V -

that

-> ,

from which

To

Obtain the constant

show that the

Cj

and so

E + r

\^^

conic

of

an

is

we

find the solution

where

is

E and (b) thus


E < 0, E = 0,

-Ku

(1)

in

1.

The

potential energy

is

v = - f

f(r)

dr

(K/r2) dr

-K/r

where we use u = \/r and choose the constant of integration so that


equation (5) of Problem 5.13,
u - 1/r = K/mh 2 + C cos e

Thus from Problem

5.11(6) together

(Cs^e^ +

c2

or

C >

assuming
(6)

(5)

terms of the total energy


parabola or hyperbola according as

Problem 5.13

ellipse,

(11)

respectively.

Method
(a)

then

(10)

By separating variables and integrating [see Problem 5.66]


expressed in terms of the energy E.

E>

(9)

cx

-ff/r

2r

(a)

(8)

page 124, we find

mh 2 /dr\ 2 +r2
4
dej

5.14.

(7)

= mh 2 C/K

V =
Using Problem

(6)

COS

(e/p)

have

a constant.

c x is

1/p

tf/mft 2

V = -
where

=
COS 6

see that

1/p

Method

with

2E

^p + rf

if

Method

this

this with

2.

we

+
,

Now from
(2)

2K

^p(^p + C cos.
.

V^ ^

c =

or

(-4)

of

Problem

5.16,

E=

2E
-^

in part (a), the equation of the conic


1

From

0.

we have

K
V

cose)
i^+C
K2

V =

K2 +

2lT

0.

Using the value of C

Comparing

(1),

lim

see that the conic is

and a hyperbola

The value of C can

if

an

E > 0,

{V
L

we

becomes
2ffmft 2

see that the eccentricity is

^1+^

ellipse if 7

since in

(4)

#2

<

[but greater than -K/2mfc}, a parabola


e-1 and e>l respectively.
e < 1,

such cases

also be obtained as in the second

method of Problem

5.13.

CHAP.

5.15.

CENTRAL FORCES AND PLANETARY MOTION

5]

Under the

127

moves

influence of a central force at point 0, a particle


force.

in a circular orbit

which passes through O. Find the law of


Method

1.

In polar coordinates the equation of a circle of radius a


passing through O is [see Fig. 5-9]

Then

since

de 2

~~

d2

1/r

2a cos

we have

(sec 8)/2a,

du

sec e tan e

de

2a

(sec fl)(sec 2 e)

tan

(sec 6

fl)(tan e)

2a

sec 3 9

sec 9 tan 2 9

2a
Fig. 5-9

Thus by Problem

5.7,

f(l/u)

cPu

-mh 2 u2

^-f

+u

{sec 3

mh2u2

2a
-8mh 2 a2 u5

,.

Thus the force


Method

2.

is

2a cos

mh

2
-7- \
r4

2a

cos e

sec e

mh2u2

1)}

2a

2 sec 3 e

power of the distance from O.

fifth

page 118, we have

in equation (16),

2a

one of attraction varying inversely as the

Using

sec 8 tan 2 e

8m h 2 a2
IS

/(")

(tan 2

sec

/ sec 3 9

-mh 2 u2

2a cos

sin 0) 2

(2a

2a cos

SaHnh2

4amh 2
r4 cos 9

CONIC SECTIONS. ELLIPSE, PARABOLA AND HYPERBOLA


5.16.

Derive equation

page 118, for a conic section.

(18),

Referring to Fig.

page 118, by

5-3,

definition of a conic section

rid

or

we have

for any point

on

it,

r/e

(1)

eD

(2)

Corresponding to the particular point Q, we have

p/D

But

Then from

(2)

and

(3),

if

5.17.

>

is

a circle

r cos

if

=
e

r(l

0,

or

r cos 9

= r (1 +

cos

8)

(5)

eliminating D,

an

cos

or

9)

r
1

ellipse if

<

<

1,

(4)

COS

a parabola

if

and a hyperbola

1.

Derive equation
r

we have on
p

The equation

(19),

page 118, for an

ellipse.

Referring to Fig. 5-4, page 119, we see that when


using equation (4) of Problem 5.16,

= OU. Thus

0,

= OV

and when

v,

CENTRAL FORCES AND PLANETARY MOTION

128

OV =
But since 2a

is

p/(l

OU =

e),

2a

Thus the equation

of the ellipse

p/(l

or

from which

a(l

in Fig. 5-4,

From Problem

5.17,

page 119,

equation

(5)

From Problem

5.17,

equation

(3)

and the

is

From

An

5.20.

e)

and

that

(a)

- e) =

2a

(*)

(3)

U)

COS 9

- e ),

a(l

equation of

4^
1

^
1

OU =

(b)

^r^
16

+ e).

(1),

a(l

"e

( 1

(i)

(1),

a<1

(a)

e>

Fig. 5-4, page 119,

we have

= CO = CV - OV - a-a(l-e) =

From

are as in Problem 5.19 and b


2
c = \Za - b\ (b) b = a^/l -

is

[see

Prdblem

Fig. 5-4, page 119, and the definition of an ellipse,

CZ__

= QV =

VE

VE

= __^

139].

using

(1)

and the result of Problem

OW

= eCE =

e(CV

(OW) 2 = (OC) 2 + (CW) 2

From Problem

5.19

and part

(a),

a2

ys = ^=^
e

(i)

divided by the distance from

5.19,

+ VE) =

or a2

we have

or

VE

OW/CE -

Then

ae.

the length of the semi-minor axis, prove

Also since the eccentricity is the distance from O to


to the directrix AB [which is equal to CE], we have

or,

page

5.73(c),

ellipse.

(6)

p/(l

and the second equation of

analogous result holds for the hyperbola

If a

(a)

a where c is the distance from the center to the focus of the


the length of the semi-major axis and e is the eccentricity.

Prove that
a

first

^+

ou =

5.19.

OV =

(a)

OV =
(b)

- e2

o(l

(a)

(-0

is

Prove that

p/(l-e)

the length of the major axis,

OV + OU =

5.18.

[CHAP.

b2

&2

e[a

+ c2

+ a2 e 2

(a-c)/e]

i.e.

or

= V^2 -

ea

&2

a-c =

= ay/1- e2

KEPLER'S LAWS OF PLANETARY MOTION AND


NEWTON'S UNIVERSAL LAW OF GRAVITATION
5.21.

Prove that

if

a planet

is

to revolve

around the sun

in

an

elliptical

path with the sun

inversely as
at a focus [Kepler's first law], then the central force necessary varies
the square of the distance of the planet from the sun.
distance from the sun,
If the path is an ellipse with the sun at a focus, then calling r the

we have by Problem

5.16,

CHAP.

CENTRAL FORCES AND PLANETARY MOTION

5]

where

<

1.

Then the central force


f(l/u)

on substituting the value of u in

is

From

5.22.

= ^mh2 u 2/p

u)

we have on

(2)

= -mh

f(r)

(1)

given as in Problem 5.7 by

= -mh2 u 2 (d2 u/de 2 +


(1).

^costf

cos o

129

(2)

replacing u by 1/r,

= -Kir 2

2 /pr 2

(S)

Discuss the connection of Newton's universal law of gravitation with Problem 5.21.
Newton arrived at the inverse square law of force for planets by using Kepler's
law and the method of Problem 5.21. He was then led to the idea that perhaps all objects
of the universe were attracted to each other with a force which was inversely proportional to the
square of the distance r between them and directly proportional to the product of their masses.
This led to the fundamental postulate
Historically,

first

GMm

_
where
is

is

Equivalently, the law of force

the universal gravitational constant.

same as

the

(1)

(3)

of

Problem

5.21

where

K = GMm

5.23.

(2)

The squares of the periods of the various planets are


proportional to the cubes of their corresponding semi-major axes.
Prove Kepler's third law:

are the lengths of the semi-major and semi-minor axes, then the area of the
Since the areal velocity has magnitude h/2 [Problem 5.6], the time taken to sweep
over area vab, i.e. the period, is

a and

If

ellipse is nab.

_
~

vab
h/2

Now

by Problem 5.17 equation

(1)

and

(2)

we

Problem

= aVl ~

Then from

(3),

5.20(6),

<?,

On

GM = gR
m

Calculate the

mass

From Problem
6.38 X 10 8 cm,
g 1.32

10 25

we have
(2)

2irw>i 2 a^ 2 IK^ 2

P2 =

or

4ir 2

maVK

to the cubes of the

semi-major axes.

is

i.e.
r R where R
equal to the weight

GMm/R 2 = mg

5.25.

5.13 equation (8),

'

the earth's surface,

on an object of mass

and Problem

= a(l-e 2 = mhVK

Thus the squares of the periods are proportional

Prove that

,^\
K

find

P =

5.24.

2vab

is

the radius, the force of attraction of the earth


is the mass of the
of the object. Thus if

mg

GM =

or

gR 2

of the earth.

5.24,

GM = gR 2

980 cm/sec 2

and

M - gR /G.
2

or

6.67

Taking

the

X 10~ 8 cgs units, we

radius
find

of

the

earth as
27 gra =

M = 5.98 X 10

lb.

ATTRACTION OF OBJECTS
5.26.

Find the force of attraction of a thin uniform rod of length 2a on a particle of

mass

placed at a distance b from

its

midpoint.

CENTRAL FORCES AND PLANETARY MOTION

130

[CHAP.

Choose the x axis along the rod and the y axis


perpendicular to the rod and passing through its
center O, as shown in Fig. 5-10. Let a be the mass
per unit length of the rod. The force of attraction
dF between an element of mass o dx of the rod and
is, by Newton's universal law of gravitation,

Gmadx

dF

x2

(sin e

b2

Gmox dx

(X 2

from Fig.

since

5-10,

_,

b tanfl

j)

Gmvb dx

62)3/2

sin e

_
~

X2

+ 62)3/2

x/y/x 2

Fig. 5-10

b 2,

cos $

b/y/x 2

Gmox dx

=
Let

cos e

)^ 2
(x
x 2 +ftb 22)3/2
(

_.

2j

b2

Then the

(x 2

ft

2 3'2
)

Xa

+ b 2 )*' 2
x

total force of attraction is

Gmabdx

Gmabdx
(x 2

Then when

in this integral.

0,

0;

dx
(X 2

+ 62

and when

3 /2
)

a,

tan -1

(a/6).

Thus the integral becomes


tan

-2Gmabj

(a/b)

Since the mass of the rod

is

b se( 2 Q d$

M = 2aa,

this

<s>)

3/ 2

by/a2

'

+b 2

can also be written as

GMm

^,

2Gmcra

2
2
(b
- sec

by/a 2

+b 2

Thus we see that the force of attraction is directed from


and of magnitude 2Gmaa/by/a 2 + b 2 or GMmfby/a 2 + 6 2

to

center of the rod

the

5.27.

mass

lies

on the perpendicular through the center of a uniform thin circular

and at distance
between the plate and the mass m.
plate of radius a

Method

from the

center.

Find the force of attraction

1.

Let n be a unit vector drawn from point P where


is located to the center O of the plate.
Subdivide the
circular plate into circular rings [such as ABC in
Fig. 5-11] of radius r and thickness dr. If a is the mass
per unit area, then the mass of the ring is a{2wrdr).
S ince all points of the ring are at the same distance
y/r2 + b 2 from P, the force of attraction of the ring on

m will

\Vr2 +

fe

be
,_

Ga{2vr dr)m
r2

cos

62

<f>

Go 2irr dr mb
(

r2

4. ,2)3/2

"

(1)

where we have used the fact that due

to

symmetry the

resultant force of attraction is in the direction n.


integrating over all rings from r =
to r = a,
find that the total attraction is

= 2vGomb

To evaluate the integral, let r 2 + 6 2


and u = yja 2 + b 2 when r a, the result

Fig. 5-11

-r

so that
is

By
we
r dr
(

r2

r dr

-j.

(2)

&2)3/2

= u du. Then

since

u=

when

CHAP.

CENTRAL FORCES AND PLANETARY MOTION

5]

v^+b*

IwGomb n
If

we

a be the value of $

let

when

udu

2irGom n

F = lirGom n
Thus the force

is

from

directed

y/a2

+b 2

can be written

this

a,

131

to the center

(1

cos a)

(3)

cos a).

of the plate and has magnitude 2irG<rm(l

Method 2.
The method of double integration can also be used. In such case the element of area at A is
rdrde where 6 is the angle measured from a line [taken as the x axis] in the plane of the
circular plate and passing through the center O. Then we have as in equation (1),
Ga(r dr do)mb

r2

-(-

J,2)3/2

and by integrating over the circular plate

Gamb

/* S
^0 =
r=0

5.28.

r dr de
(r 2

Xa

2irr

6 2 ) 3/2

(r 2

dr

2TrGom n

6 2 )3/ 2

(1

cos a)

uniform plate has its boundary consisting of two concentric half circles of
inner and outer radii a and b respectively, as shown in Fig. 5-12. Find the
attraction

force of

mass

on a

of the plate

located at the center 0.

convenient to use polar coordinates


of area of the plate [shaded
in Fig. 5-12] is dA = r dr de, and the mass is
ardrde. Then the force of attraction between
It

(r, e).

is

The element

dA and O

Fig. 5-12

is

_ _
Thus the

(cos e

XC

G{ardrde)m

^va2

),

The method of

5.29.

sin e

j)

+
.

..

sin en

j)

(cos o

sine

we have

= 2Mh(b 2 a2

de

2Gaw

In

Problem

and the force can be written

AGMm
fb\
^2-^2) ^ (^-j,
.

single integration can also be used

into circular rings as in

j)

X=o

r=a

Gam In

M o(vb

(cos e

total force of attraction is

)=0

Since

G{ar dr de)m

by dividing the region between r

and

5.27.

Find the force of attraction of a thin spherical


m at a distance r > a from its center.

shell of radius

a on a particle

of

mass

Let
such as

be the center of the sphere.

ABCDA

of Fig. 5-13 below

Subdivide the surface of the sphere into circular elements

by using parallel planes perpendicular to OP.

The area of the surface element

ABCDA

as seen from Fig. 5-13

2^{a sin e)(a de)

2va2 sin

is

de

since the radius is a sin e [so that the perimeter is 2ir(a sin )] and the thickness is ade.
is 2rra 2 a sin e de.
if a is the mass per unit area, the mass of

ABCDA

Then

CENTRAL FORCES AND PLANETARY MOTION

132

[CHAP.

Since all points of ABCDA are at the same distance w = AP from P, the force of attraction of
is
the element ABCDA on

G(2va2 a

de)m

sin 6

cos

in*

<p

(1)

where we have used the fact that from symmetry the

net force will be in the direction of the unit vector

Now from Fig.


PE
PO - EO

from P toward O.

AP
Using

in

(2)

(1)

ade
a cos

e
(2)

AP

together with the fact that by the

cosine law

w2 =
we

5-13,

a2

r2

2ar cos

(3)

find

G{2ira 2 a sin e de)m{r

dF

(a 2

r2

2ar cos

a cos
0)

e)

3/2

Fig. 5-13

Then the

total force is

We

2TrGa 2 am

(r

+ a) 2

so that

w = r + a.

a cos e) sin e
2ar cos 0) 3/2 de

=
'

a COS

(4)

a2

2ar sin
-f-

r2

w
if

given by (3) in place of 6.


Also when 6 = ir,
r > a.

Work Problem

5.29

)2

x:

if

<

a2

r2

2r

dw

w2

de

_ w 2\

2ar

irGaam n

When

we have

becomes

(4)

r2

In addition,

2w dw

5.30.

can evaluate the integral by using the variable


2ar + r 2 = (r a) 2 so that w = r a

= 0,
a 2 + 2ar + r2 =

Then

(r

(a 2

"J.

&TrGa2om n

r2

a.

However, in evaluating the


In this case the force is also given by (4) of Problem 5.29.
yields w 2 = (a r) 2
we note that on making the substitution (3) of Problem 5.29 that e =
w a r if r < a. Then the result (4) of Problem 5.29 becomes

integral

or

F
Thus there
means that

5.31.

= wGaom

Jo

dw

no force of attraction of a spherical shell on any mass placed inside.


such case a particle will be in equilibrium inside of the shell.

will be
in

Prove that the force of attraction in Problem 5.29


the spherical shell were concentrated at its center.
The mass of the

shell is

M = AttcPo.

Thus the force

is

is

the

same as

if all

the

F = (GMm/r 2 )n, which

This

mass of

proves the

required result.

5.32.

Find the force of attraction of a solid uniform sphere on a mass m placed outside
of it and (b) prove that the force is the same as if all the mass were concentrated
(a)

at its center.
(a)

We
of

can subdivide the solid sphere into thin concentric spherical

from the center and dp is the


attraction of this shell on the mass m is

any of these

force of

shells

shells.

thickness, then

If p is the distance
5.29 the

by Problem

CHAP.

CENTRAL FORCES AND PLANETARY MOTION

5]

d
where

to

is

the
a

mass per unit volume.

d P )m

Then the

total

AirGomn f a

M ^waPo,

We

is

(2)

can be written as

first

potential

Then the

dV due

element

to the

G{2tto, 2 o

ABCDA

Problem

5.130].

finding the potential due to the

sin e

is

do)m

G{2va2 a

__

V 2 +

If

>

a this yields

If

<

it

the force

de)m

2ar cos

< a

the force

+ y)2

r2

2ar cos

-,/(.-

4:irGa 2 cnn

GMm

r)2

is

F
if

sin e

V = kwGaom

yields

> a

V a2 +

2irGaam ^/
(

if

r2

total potential is

2irGa2 am

GMm \

-VV = -V

GMm
,2

is

F =
in

[see

F = (GMm/r2 )n, which

mass were concentrated at

the

all

if

can also use triple integration to obtain this result

dV

and

from

distribution.

The

Then

force obtained by integrating

(*)

same as

the

Derive the result of Problems 5.29 and 5.30 by

mass

(1)

G{% va s )am n

Xa>^ dp

Since the mass of the sphere is


shows that the force of attraction
the center.

(6)

is

5.33.

G<x(4t7 P 2

133

-VV =

agreement with Problems 5.29 and

-V(-4s-Gaam) =

5.30.

MISCELLANEOUS PROBLEMS
5.34.

An

is projected vertically upward from the earth's surface with initial


Neglecting air resistance, (a) find the speed at a distance
above the
earth's surface and (6) the smallest velocity of projection needed in order that the
object never return.

object

speed

(a)

Vo.

Let r denote the radial distance of the object at time t


from the center of the earth, which we assume is fixed
[see Pig. 5-14].
If
is the mass of the earth and R is
its radius, then by Newton's universal law of gravitation
and Problem 5.29, the force between m and
is

GMm
(1)

rj is a unit vector directed radially outward from


the earth's center in the direction of motion of the object.

where

If v is the speed at

ond law,

time

t,

we have by Newton's

sec-

Fig. 5-14

CENTRAL FORCES AND PLANETARY MOTION

134

GMm r
r2

[CHAP.

dv
i

Tt

This can be written as

dv dr
dr dt

Then by integrating, we

GM

= GM/r +

v 2 /2

find

dv
dr

or

r2

ct

tt)

Since the object starts from the earth's surface with speed v
so that

<?!

vjj/2

GM/.R.

Then

2GM (i-i) +
Thus when the object

at height

is

we have v = v

when r =

becomes

(4)

(5)

above
earth' surface,
ve the earth's

i.e.

= R + H,

2GMH
R(R + H)

=
=

V
v

Using Problem

2GMH

V
^jvl
,

-R(R + H)

can be written

5.24, this

2gRH

!~2

H -*

As

(b)

oo,

the limiting speed

(6)

\<~R+H

becomes

vX ~ 2GM/R
since

lim /p
H-t (it

=
+ /3)

v^ 2 - 2gR

or

The minimum

1.

(6)

initial

speed occurs where

= V2GM/R =

(7)

(7)

is

zero or where

y/2gR

(8)

This minimum speed is called the escape speed and the corresponding velocity
escape velocity from the earth's surface.

5.35.

Show
is

that the magnitude of the escape velocity of an object

5.36.

called the

earth's surface

about 7 mi/sec.

From
we

from the

is

find

equation

(*)

of Problem 5.34,

Taking

y/2gR.

32 ft/sec 2

and

R=

4000 mi,

6.96 mi/sec.

Prove, by using vector methods primarily, that the path of a planet around the sun
is an ellipse with the sun at one focus.
Since the force

F between

the planet and sun

we have

_ w
~ m

dv

dv

-=r

(4),

since r

rr.,

dv
-37

at

rr-

= rX

at

{1)

ri

r2

r2

,-.

(2)

we have
r

av^

GMm

_
~

GM
x-r

dt

Also, by Problem 5.1, equation

Now

dt

is

(y

-r-r,.

at

X v

= h

(3)

Thus from

(S),

\
f dr,
r
lX (,
+ -)
(j

dr,

= ^r.x-J

W)

CHAP.

CENTRAL FORCES AND PLANETARY MOTION

5]

From

135

(2),

GM

dv

But

(4)

since

is

above and equation


a constant vector,

X h

-jr

at

drA

{("-^>-<'.-")-i

page

(7),

*A

[f

= - GM
using equation

dr i

or,

= GM ^r

5.

at

(v

so that

h)

^r i

d
|(vxh) = GM-
v X h

Integrating,

= GM r +

from which
r

where

c is

(v

= GM t

h)

r1

(v

h)

(r

v)

hh

h2

h2

[see

Problem

GMr +

GM +

which is the equation of a conic.


required result is proved.

c cos e

(8)

Since the only conic which

of Problem 5.13,

(4)

of Problem 5.14 and

mh2
n
2
= -g^ = a(l-e

E -

from which

v2

or

27],

is

a closed curve

elliptical

is

an

ellipse,

the

path in an inverse square

a
of Problem 5.17,

(3)

we have

2Emh2 \

K2~J

...

(*)

(2)

V=

Kir,

(3)

Kf2
a
m \r +h)

(*)

can similarly show that for a hyperbola,

v2

-*

in either (3) or (4)]

= 2K/mr

(man-made) satellite revolves about the earth at height H above the


Determine the (a) orbital speed and (b) orbital period so that a man in the

artificial

surface.

satellite will
(a)

-(---)
m \r a I

while for a parabola [which corresponds to letting

An

page

-K/2a

Thus by the conservation of energy we have, using

5.38.

1.72(a),

re cos

the angle between c and r x

the semi-major axis.

We

= GMr +
e is

h2/GM
(c/GM) cos

m \r
is

re cos 6

Prove that the speed v of a particle moving in an


field is given by

where a

and

c,

h2

and so

By

rr x

an arbitrary constant vector having magnitude

Since

5.37.

GMr +

be in a state of weightlessness.

Assume that the earth is spherical and has radius R. Weightlessness will result when the
centrifugal force [equal and opposite to the centripetal force, i.e. the force due to the cen-

CENTRAL FORCES AND PLANETARY MOTION

136

[CHAP.

acceleration] acting on the man due to rotation of the satellite just balances his
attraction to the earth. Then if v is the orbital speed,

tripetal

mv l _
R+H ~
If

GMm _
(R

gR*m

+ H)* ~

(R

small compared with R, this

is

(b)

is

2tt(R

+ H)

2u(R

+ H)

^7

5.39.

is

+ H)g

(a)

y/(R

distance traveled in one revolution


time for one revolution, or period

Thus

If

R
_
~ R+H

y/Rg approximately.

Orbital speed

Then from part

or

+ H)*

small compared with R, this

is

R+H
1
R )V
:

2iry/R/g approximately.

Calculate the (a) orbital speed and (b) period in Problem 5.38 assuming that the
height
above the earth's surface is small compared with the earth's radius.

Taking the
4.92 mi/sec

5.40.

and

earth's
(b)

P=

radius
2-n-^R/g

and

4000 miles

as

hr

1.42

32 ft/sec 2

we

find

(a)

= y/Rg -

85 minutes, approximately.

Find the force of attraction of a solid sphere of radius a on a particle of mass


a distance b < a from its center.
By Problem
shell containing

shown dashed

at

5.30 the force of attraction of any spherical


in its interior [such as the spherical shell

in Fig. 5-15] is zero.

Thus the force

of attraction on
is the force due to a
sphere of radius b < a with center at O. If a is the mass
per unit volume, the force of attraction is
G(f7r&3)<rm/&2

= (^Gam)b

Thus the force varies as the distance

from the mass

to the

center.

Fig. 5-15

Supplementary Problems
CENTRAL FORCES AND EQUATIONS OF MOTION
5.41.

Indicate which of the following central force fields are attractive toward origin O and which
are repulsive from O.
(a)
F =
F = Kxjyfr,
> 0; (c) F = r(r- ljr^r2 + 1);
(6)
(d) F = sin wr r v

-4^;

Ans. (a) attractive; (6) repulsive; (<;) attractive


2n < r < 2n + 1, attractive for 2n+l<r<2n

5.42.

Prove that

5.43.

Give an example of a force

in

if

<r<
where

repulsive

1,

0, 1, 2,

if

rectangular coordinates the magnitude of the areal velocity

field directed

toward a fixed point which

is

>

r
.

is

1;

(d)

repulsive for

^{xy

yx).

not a central force

field.

CHAP.

5.44.

5.45.

Derive equation
If
its

5.46.

CENTRAL FORCES AND PLANETARY MOTION

5]

page 117.

(7),

a particle moves in a circular orbit under the influence of a central force at


speed around the orbit must be constant.

(K

ma2 v\

Show

(a)

that the differential equation for the orbit in Problem 5.46

do 2

(1

y)u

Solve the differential equation in

(b)

sin 2 a)

mr3

dt2

5.48.

prove that

its center,

particle of mass m moves in a force field defined by F = -Kr^r3


If it starts on the positive
x axis at distance a away from the origin and moves with speed v in direction making angle
a with the positive x axis, prove that the differential equation for the radial position r of the
particle at any time Ms

d2 r

5.47.

137

(a)

where

y'

is

given in terms of u

ma2 v2

1/r

by

sin2 a

and interpret physically.

particle is to move under the influence of a central force


always constant and equal to v
Determine all possible orbits.

field

so that its orbital

speed

is

POTENTIAL ENERGY AND CONSERVATION OF ENERGY


5.49.

Find the potential energy or potential corresponding to the central force

F = -Krjr*,

(a)

Ans.
5.50.

-K/2r2

(a)

(b)

F=

(b)

a/r

{air2

+ 0/r)r lf

p/2r2

(c)

(c)

\Kr2

F = Krr u
(d) 2ypr,

(d)
(e)

F=

r x /yfF,

(e)

fields

F=

sin a-r

defined

by

r^

(cosr)/ir

Find the potential energy for a particle which moves in the force field F = Krjr2 (b) How
is done by the force field in (a) in moving the particle from a point on the circle
r = a >
to another point on the circle r = 6 > 0?
Does the work depend on the path? Explain.
Ans. (a) -Kir, (6) K(a-b)/ab
(a)

much work

5.51.

Work Problem

5.52.

particle of mass
moves in a central force field defined by F = Krjr*. (a) Write an equation
for the conservation of energy. (6) Prove that if
is the total energy supplied to the particle,

5.50 for the force field

F = -Krjr.

Ans.

(a)

-K\nr,

-K In (a/6)

(6)

then

5.53.

its

speed

given by

= yjKlmr2 + 2E/m.

A particle moves in a central force field defined by F = Kr2 r v It starts


o n the circle r = a.
(a) Prove that when it reaches the circle
r =
V2#(a3 -

5.54.

is

6 3 )/3w

and that

(6)

its

speed will be

the speed will be independent of the path.

mass m moves in a central force field F = Kxjrn where K and n are constants.
from rest at r = a and arrives at r =
with finite speed v
(a) Prove that we must
n < 1 and K > 0. (b) Prove that v = y/2Ka i - n/m{n- 1). (c) Discuss the physical sig-

particle of

It starts

have

nificance of the results in

5.55.

from rest at a point

By

differentiating

both

(a).

sides

of

equation

(IS),

page 117, obtain equation

(6).

DETERMINATION OF ORBIT FROM CENTRAL FORCE OR


CENTRAL FORCE FROM ORBIT
5.56.

5.57.

particle of mass
moves in a central force field given in magnitude by f(r) = Kr where
a positive constant. If the particle starts at r a, e =
with a speed v in a direction
perpendicular to the x axis, determine its orbit. What type of curve is described?

is

(a)

Work Problem

(6)

Discuss the cases

5.56 if the speed is v

0,

ir

in

a direction making angle a with the positive * axis.

and give the physical

significance.

CENTRAL FORCES AND PLANETARY MOTION

138

5.58.

[CHAP.

particle moving in a central force field located at r =


describes the spiral
that the magnitude of the force is inversely proportional to r3

e~

Prove

5.59.

Find the central force necessary to make a particle


describe the lemniscate r 2 = a 2 cos 2d [see Fig. 5-16].
Ans.

5.60.

5.61.

force proportional to r~ 7

a*cos2

Obtain the orbit for the particle of Problem 5.46 and


describe physically.

Prove that the orbits r


possible for the case of

e~ e and r = 1/8 are both


an inverse cube field of force.

Explain physically how this

5.62.

r2

(a)

Show

that

if

Fig. 5-16

is possible.

the law of force

given by

is

F =

r4 cos

r2 cos 3 e

then a particle can move in the circular orbit r 2a cos 8.


(b) What can you conclude about
the uniqueness of forces when the orbit is specified?
(c) Answer part (b) when the forces are
central forces.

5.63.

What

central force at the origin O is needed to make a particle move around O with a speed
inversely proportional to the distance from O.
(b) What types of orbits are possible in
such case?
Ans. (a) Inverse cube force.
(a)

which

is

5.64.

Discuss the motion of a particle moving in a central force

5.65.

Prove that there

5.66.

is

field

given by

F =

(a/r2

+ )8/r3 )r

no central force which will enable a particle to move in a straight

Complete the integration of equation (12) of Problem


of the same problem.
[Hint. Let r = 1/u.]

5.13,

1.

line.

page 125 and thus arrive at equation

(5)

5.67.

Suppose that the orbit of a particle moving in a central force field is given by 8 = e(r). Prove
mh 2 [2e' + re" + r2 (e') 3
that the law of force is
where primes denote differentiations with
5
3
]

respect to

5.68.

(a)

,.

r.

Use Problem

inversely as r 3

show that if
Graph the orbit in

5.67 to
(6)

(a)

1/r,

the central force

is

one of attraction and varies

and explain physically.

CONIC SECTIONS. ELLIPSE, PARABOLA AND HYPERBOLA


5.69.

The equation
(c)

of a conic

is

12
-^,

the length of the major axis,

(d)

Graph the conic, finding (a) the foci,


cos 8
the length of the minor axis, (e) the distance

(6)

the vertices,

from the center

to the directrix.

5.70.

Work Problem

5.69 for the conic

24

=
3

'

5 cos 8

5.71.

Show

5.72.

Find an equation for an ellipse which has one focus at the origin,
Ans. r = 9/(5 + 4 cos 8)
and its major axis of length 10.

that the equation of a parabola can be written as

sec 2 (8/2).

its

center at the point (4,

0),

CHAP.

5.73.

CENTRAL FORCES AND PLANETARY MOTION

5]

In Fig. 5-17, SR or TN is called the minor axis of


the hyperbola and its length is generally denoted
by 26. The length of the major axis
is 2a,
while the distance between the foci O and O' is 2c

%\

x/'

>^

VU

[i.e.

the distance from the center

or O'

(a)

Prove that

(b)

Prove that

= +
= aVe 2 1 where
a2

b 2.

Prove that

re's
\f
y

e is

>^
c

etc.

Compare with

results for

'

\x

//

the eccen-

tricity.
(c)

n.,

m_\

/ y / /Cx U \
/yy
^H\

is C].

c2

//

\\lxrX \

to a focus

139

^>

the ellipse.

Fig. 5-17

5.74.

Derive equation

5.75.

In rectangular coordinates the equations for an ellipse and hyperbola in standard form are given by

(22),

page 119, for a hyperbola.

*2
tf

2
+,y2 =

v2

x2

and

V ~V

where a and b are the lengths of the semi-major and semi-minor axes. Graph these
equations, locating vertices, foci and directrices, and explain the relation of these equations to
equations [19), page 118, and (22), page 119.
respectively,

5.76.

Using the alternative definitions for an ellipse and hyperbola given on pages 118-119, obtain the
equations (19) and (22).

5.77.

Prove that the angle between the asymptotes of a hyperbola

KEPLER'S LAWS AND NEWTON'S


5.78.

is

2 cos -1

(1/e).

LAW OF GRAVITATION

Assuming that the planet Mars has a period about the sun equal to 687 earth days approximately,
mean distance of Mars from the sun. Take the distance of the earth from the sun as

find the

Ans. 140 million miles

93 million miles.

5.79.

Work Problem

5.78 for

(a)

225 earth days respectively.


5.80.

Jupiter and

Ans.

(a)

(b)

Venus which have periods of 4333 earth days and

484 million miles,

Suppose that a small spherical planet has a radius of 10

What would

be the acceleration due to gravity at


on this planet if he weighed 80 kgwt on earth?
(a)

5.81.

km

its

(b)

67 million miles

and a mean density of 5 gm/cm 3


(6) What would a man weigh
.

surface?

due to gravity on the surface of a spherically shaped planet P is g P while its


density and radius are given by aP and R P respectively, prove that g P = %irGR P eP where G
the universal gravitational constant.

If the acceleration

mean
is

5.82.

If L, M, T represent the dimensions of length,


gravitational constant.
Ans. L3 _1 T~ 2

5.83.

mass and

time, find the dimensions of the universal

Calculate the mass of the sun using the fact that the earth is approximately
it and makes one complete revolution about it in approximately 365 days.

150

5.84.

10 6 kilometers

Ans. 2 X 10 30 kg

from

Calculate the force between the sun and the earth if the distance between the earth and the sun is
taken as 150 X 10 6 kilometers and the masses of the earth and sun are 6 X 10 24 kg and 2 X 10 30 kg
respectively.
Ans. 1.16 X 10 24 newtons

ATTRACTION OF OBJECTS
5.85.

5.86.

Find the force of attraction of a thin uniform rod of length a on a mass


Ans. GMm/b(a
the same line as the rod and distance 6 from an end.

outside the rod but on

+ b)

In Problem 5.85 determine where the mass of the rod should be concentrated
Ans. At a point in the rod a distance y/b(a + b)
same force of attraction.

so as to give the
6

from the end

[CHAP.

CENTRAL FORCES AND PLANETARY MOTION

14 ()

5.87.

Find the force of attraction of an infinitely long thin uniform rod on a mass
Arts. Magnitude is 2Gmafb
from it.

5.88.

angle ^.
is in the form of an arc of a circle of radius 6 and central
circle
of
the
center
the
placed
at
on
mass
a
wire
the
of
attraction
of
force
the

uniform wire

2Gam
r

6V

where
and yp
5.89.

is

at distance b

magnitude by

mass of the wire and a

the

is

sin

Prove that
is

given in

(vfr/2)

the mass per unit length. Discuss the cases

v/2

7r.

AB

is a thin rod of length 2a and


In Fig. 5-18,
a mass located at point C a distance 6 from the
Prove that the force of attraction of the rod
rod.

is

on

has magnitude

^^

D/

Ua + (3)

sin

ao

a direction making an angle with


given by
_ / cos/3 + cosoA
tan 1

the

in

rod

/^

Discuss the case

5.90.

5.91.

s'

^\ \
/^

\v

^ r

VsiniS-sinay

lem

\^^~~ ~~' '-"""


E ^A~^ /?

2a

and compare with Prob-

/?

Fig. 5-18

5.26.

prove that the rod of Problem 5.89 can be


which has its
replaced by a wire in the form of circular arc BEG [shown dashed in Fig. 5-18]
attraction is toward
the
of
direction
the
that
Prove
E.
at
rod
the
tangent
to
and
is
at
C
center
the midpoint of this arc.

By comparing Problem

hemisphere of mass

force of attraction

Ana.

(a)

GMm/2a2

(a)

if

36

(b)

with Problem

5.89

5.88,

located at
and radius a has a particle of mass
the hemisphere is a thin shell, (6) the hemisphere

its
is

center.

Mm/2a2
hemisphere

having outer radius a and inner radius

5.92.

Work Problem

5.93.

and planets
Deduce from Kepler's laws that if the force of attraction between sun
planet.
particular
2
the
of
independent
be
magnitude by ym/r then y must

5.91 if the

Find the

solid.

is

shell

b.

is

given in

5 94

cone has height

placed at

its

and radius a.
. ^
magnitude

vertex has

Prove that the force of attraction on a particle of mass


\
1
/ 24- W2 /

GMm
^ A1 ~

"

5.95.

Find the force of attraction between two non-intersecting spheres.

5 96

distance a
placed outside of a uniform solid hemisphere of radius a at a
force of attraction is
the
that
Prove
center.
its
through
base
the
to
on a line perpendicular

particle of

mass

is

given in magnitude by GMm{\f2

5.97.

Work

(a)

Problem

(b)

5.26,

l)/a2

Problem

5.27,

and

(c)

Problem

5.94

by

first finding

the potential.

MISCELLANEOUS PROBLEMS
5.98.

particle

is

projected vertically

maximum

upward from the

height

reached above the earth's surface

(a)

Prove that the

(b)

Discuss the significance of the case where v\

(c)

Prove that

if

is

small, then

it is

earth's surface with initial speed v

2gR.

equal to v%/2g very nearly.

is

H=

v 2 R/(2gR

- v\).

CHAP.

5.99.

CENTRAL FORCES AND PLANETARY MOTION

5]

(a)

Prove that the time taken

maximum

reach the

to

R+

HJ

(b)

5.100.

(a)

Prove that

Prove that
resistance

if

if

is

very small compared with R, then the time in

an object

negligible

is

height of Problem 5.98

R + H wo _JR-H
2R
\R + H

[H
~

^~2tUr
dropped

is

it will

to

141

very nearly ^2H/g.

(a) is

the earth's surface from

hit the earth with a speed

is

a height H, then

if

- ^2gRH/(R + H) where R

air
is

the radius of the earth.


(b)

(a) for the cases where


H
Take the radius of the earth as 4000 miles.

Calculate the speed in part


respectively.

100 miles

and

H=

10,000 miles

5.101.

Find the time taken for the object of Problem 5.100 to reach the earth's surface in each of
the two cases.

5.102.

What must be the law of force if the speed of a particle in a central force field
proportional to r~ n where n is a constant?

5.103.

What velocity must a space ship have in order to keep it in an orbit around the earth at a
distance of (a) 200 miles, (6) 2000 miles above the earth's surface"
>9

5.104.

An

is thrown upward from the earth's surface with velocity v


Assuming that
and that air resistance is negligible, find its velocity on returning.

object

to earth

5.105.

(a)

What

is

the

work done by a space ship of mass

in

is

it

to

be

returns

moving from a distance a above the

earth's surface to a distance 6?

5.106.

(6)

Does the work depend on the path? Explain.

(o)

Prove that it is possible for a particle to move in a


field whose law of force is /(r).

(b)

Suppose the particle of part


it

will return to the orbit,

w
but

(c)

5.107.

5.108.

is

4U

i.e.

Illustrate the result in


stability can occur.

(6)

'

{a)

by considering
(c) For
n <

Ans.

were suddenly stopped

GmM(a

b)/ab

circle of radius

a in any central force

is displaced slightly from its circular orbit.


the motion is stable, if

If the moon were suddenly stopped in its


assuming that the earth remained at rest?

If the earth

(a)

(a)

af

.l
unstable otherwise.

Ans.

3/(a)

f(r)

>

1/r"

there

is

and deciding for which values of n


stability.

how long would it take to


Ans. About 4 days 18 hours

orbit,

in its orbit,

Prove that

how long would

it

take for

it

fall

to

the earth

to fall into the

sun?

Ans. About 65 days


5.109.

Work Problem

5.110.

Find the velocity of escape for an object on the surface of the moon. Use the fact that the
acceleration due to gravity on the moon's surface is approximately 1/6 that on the earth and
that the radius of the moon is approximately 1/4 of the earth's radius.
Ans. 1.5 mi/sec

5.111.

An

page 133, by using energy methods.

object is dropped through a hole bored through the center of the earth. Assuming that the
resistance to motion is negligible, show that the speed of the particle as it passes through the
center of the earth is slightly less than 5 mi/sec.
[Hint.

5.112.

5.34,

Use Problem

5.40,

page

136.]

In Problem 5.111 show that the time taken for the object to return

is

about 85 minutes.

CENTRAL FORCES AND PLANETARY MOTION

14 2

5.113.

Work Problems

and 5.112

5.111

if

the hole

[CHAP.

straight but does not pass through the center of

is

the earth.
5.114.

Discuss the relationship between the results of Problems 5.111 and 5.112 and that of Problem 5.39.

5.115.

How

5.116.

Prove Theorem

5.117.

Discuss Theorem 5.1

5.118.

5.119.

5.120.

would you explain the fact that the earth has an atmosphere while the moon has none?

the spheres intersect.

space ships move about the earth on the same elliptical path of eccentricity e. If they are
separated by a small distance D at perigee, prove that at apogee they will be separated by the

Two

- e)/(l + c).

Explain how you could calculate the velocity of escape from a planet. (6) Use your method to
Ans. (6) 5 km/sec, or about 3 mi/sec
calculate the velocity of escape from Mars.

(a)

5.122.

Work Problem

5.125.

if

Find the force of attraction between a uniform circular ring of outer radius a and inner radius
and a mass m located on its axis at a distance 6 from its center.

5.121.

5.124.

page 120.

Explain how you could use the result of Problem 5.27 to find the force of attraction of a solid
sphere on a particle.

distance D(l

5.123.

5.1,

5.121 for

(a)

Jupiter,

(b)

Venus.

Ans.

(a)

about 38 mi/sec,

(6)

about 6.3 mi/sec

Three infinitely long thin uniform rods having the same mass per unit length lie in the same plane
and form a triangle. Prove that force of attraction on a particle will be zero if and only if the
particle is located at the intersection of the medians of the triangle.
Find the force of attraction between a uniform rod of length a and a sphere of radius
do not intersect and the line of the rod passes through the center.

Work Problem

5.124 if the rod

is

situated so that a line

b if

drawn from the center perpendicular

they

to the

line of the rod bisects the rod.

5.126.

A
If

the

4^-2(a

5.127.

a revolves in a circular orbit about a planet of radius b with period P.


between their surfaces is c, prove that the mass of the planet is
distance
shortest

satellite of radius

+ b + c)3/GP 2

one complete
Given that the moon is approximately 240,000 miles from the earth and makes
earth.
the
of
mass
the
find
approximately,
revolution about the earth in 27 days

Ans. 6 X 10 24 kg
5.128.

Discuss the relationship of Problem 5.126 with Kepler's third law.

5.129.

Prove that the only central force

5.130.

Work Problem

5.131.

5.32,

field

F whose

page 132, by using

divergence

is

zero

is

an inverse square force

field.

triple integration.

is placed
uniform solid right circular cylinder has radius a and height H. A particle of mass
end. Prove that the
on the extended axis of the cylinder so that it is at a distance D from one
force of attraction is directed along the axis and given in magnitude by

^^{H
+ Va2 + Z)2- Va2 + (D + H)2}
aH
2

5.132.

5.133.

Suppose that the cylinder of Problem 5.131 has a given volume.


when the particle is at the center of one end of the cylinder is a

Work

(a)

Problem 5.26 and

(&)

Problem

5.27

Prove that the force of attraction

maximum when alH =

(9

assuming an inverse cube law of attraction.

- y/ll).

CHAP.

CENTRAL FORCES AND PLANETARY MOTION

5]

5.134.

Do the results of Problems 5.29 and 5.30 apply


Explain.

5.135.

What would

5.136.

there

is

an inverse cube law of attraction?

be the velocity of escape from the small planet of Problem 5.80?

spherical shell of inner radius a and outer radius 6 has constant density
gravitational potential V(r) at distance r from the center is given by

2ua(b 2

V(r)

27r(j(b 2
47tct(6 3

5.137.

if

143

If Einstein's

theory of relativity
of a planet becomes

-a2

- lr - 4a
a3 )/3r
2

3 /3r

<

<

>

<

Prove that the

a.

taken into account, the differential equation for the orbit

is

d 2u

mh2

do 2

'

y = 3K/mc2 c being the speed of light, (a) Prove that if axes are suitably chosen, then
the position r of the planet can be determined approximately from

where

/K
+mh cos
aO
2

where

, L92
yK/mh

(b) Use (a) to show that a planet actually moves in an elliptical path but
that this ellipse slowly
rotates in space, the rate of angular rotation being 2iryK/mh 2
(c) Show that in the case of
Mercury this rotation amounts to 43 seconds of arc per century. This was actually observed, thus
offering experimental proof of the validity of the theory of relativity.
.

5.138.

Find the position of a planet in its orbit around the sun as a function of time
where it is furthest from the sun.

5.139.

At apogee

5.140.

measured from

of 200 miles from the earth's surface, two space ships in the same elliptical path are
500 feet apart. How far apart will they be at perigee 150 miles assuming that they drift without
altering their path in any way?

particle of mass
is located on a perpendicular line through the center of a rectangular
plate
of sides 2a and 26 at a distance
from this center. Prove that the force of attraction of the plate
on the particle is given in magnitude by

GMm Sin _j
.

ab

ab

\^(a 2

+ D 2 )(b 2 + D 2

5.141.

Find the force of attraction of a uniform infinite plate of negligible thickness and density
a
on a particle at distance D from it.
Ans. 2iroGm

5.142.

Points where r =
are called apsides [singular, apsis], (a) Prove that apsides for a central
force field with potential V(r) and total energy
are roots of the equation V(r) + h2/2r2 = E.
(b) Find the apsides corresponding to an inverse square field of force,
showing that there are
two, one or none according as the orbit is an ellipse, hyperbola or parabola.

5.143.

particle

moving

Find the law of


5.144.

5.145.

in a central force field travels in

force.

a path which
Ans. Inverse fourth power of r.

is

the cycloid

Set up equations for the motion of a particle in a central force field if it takes place
where the resistance is proportional to the instantaneous speed of the particle.

satellite

has

its

largest and smallest orbital speeds given by v max and

that the eccentricity of the orbit in which the satellite moves

is

i;

Prove that

if

is

y/v max vmin

in

medium

Prove

equal to

the satellite of ProbLem 5.145 has a period equal to

path having major axis whose length

a(l-cose).

min respectively.

max
5.146.

t,

then

it

'

vr

moves

in

an

elliptical

MOVING
COORDINATE SYSTEMS

Chapter 6

NON-INERTIAL COORDINATE SYSTEMS


particles
In preceding chapters the coordinate systems used to describe the motions of
importance,
practical
of
were assumed to be inertial [see page 33]. In many instances
however, this assumption is not warranted. For example, a coordinate system fixed in
the earth is not an inertial system since the earth itself is rotating in space. Consequently
relative to the earth
if we use this coordinate system to describe the motion of a particle
the motion of
consider
to
we obtain results which may be in error. We are led therefore
particles relative to

moving coordinate systems.

ROTATING COORDINATE SYSTEMS


In Fig. 6-1 let XYZ denote an inertial coordinate
system with origin O which we shall consider fixed
in space. Let the coordinate system xyz having the
same origin O be rotating with respect to the XYZ
system.

Consider a vector A which is changing with


time. To an observer fixed relative to the xyz system
the time rate of change of A = Aii + A 2 j + A 3k is
found to be

dA

dAi

dt

~df

dA,
dt

dAt
(1)

dt

indicates the derivative


where subscript
moving (xyz) system.

in the

However, the time rate of change of A relative


XYZ system symbolized by the subscript
be [see Problem 6.1]
found
to
is

to the fixed

where

a>

is

dA

dA

dt

dt

called the angular velocity of the xyz

DERIVATIVE OPERATORS
Let D F and D M represent time
Then we

Fig. 6-1

o>

(2)

system with respect to the

derivative operators in the fixed and

XYZ

system.

moving systems.

can write the operator equivalence

DF

D M + oX

(3)

the fixed and moving


This result is useful in relating higher order time derivatives in
systems. See Problem 6.6.

144

CHAP.

MOVING COORDINATE SYSTEMS

6]

145

VELOCITY IN A MOVING SYSTEM


If, in

particular, vector

is

the position vector r of a particle, then

_
~

dr
dt f

dr

+ *>Xr

dt

DFr = DMx +

or

gives

(2)

<o

a)

(5)

Let us write

=
=
=

vp|F

vp(M

vmif

Then

(-4)

= D Fr = velocity of particle P relative to fixed system


M = D M r = velocity of particle P relative to moving system
= velocity of moving system relative to fixed system.

dr/dt F
\

dr/dt
\

x r

or

(5)

can be written

v pif

v P|M

v M|F

ACCELERATION IN A MOVING SYSTEM


2
If D F = d2/dt 2 F and D^ = d2 /dt
M are second
2

(6)

derivative operators with respect to

and moving systems, then application of

in the fixed

D F2 r = D M2 r +

(Z>

<o)

Problem

(3) yields [see

2 X

D Mr +

6.6]

r)

(7)

Let us write

= D Fr =

P relative to fixed

aP|F

a P|M

= d 2 r/dt 2 M = D^r = acceleration of particle P relative


= D m<*) X r + 2a. X D M r + X X r)
= acceleration of moving system relative to fixed system

acceleration of particle

a Mir

Then

d2 r/dt 2

can be written
*P|F

CORIOLIS
The

moving system

(o,

(7)

to

system

last

**P|M

"*"

aM|F

(#)

AND CENTRIPETAL ACCELERATION


two terms on the right of

acceleration respectively,

(7)

are called the Coriolis acceleration and centripetal

i.e.,

Coriolis acceleration

2o

Centripetal acceleration

The second term on the right

of

(7) is

D Mr

2<o

x vM

(9)

r)

(10)

sometimes called the linear acceleration,

i.e.,

/j

Linear acceleration

(D^a)
M ' x r
v

(~
\dt

M/

Z>M o> is called the angular acceleration. For many cases of practical importance
the rotation of the earth] <a is constant and D M <a = 0.

and

The quantity <o x

(o>

r) is

(11)
[e.g.

in

often called the centrifugal acceleration.

MOTION OF A PARTICLE RELATIVE TO THE EARTH


Newton's second law
(7)

we

is strictly applicable only to inertial systems. However, by using


obtain a result valid for non-inertial systems. This has the form

mD^r = F - m(D M a) X r - 2m(u x D M r) - mx( B


where F

the resultant of
fixed or inertial system.
is

all

Xr)

(12)

forces acting on the particle as seen by the observer in the

MOVING COORDINATE SYSTEMS

146

[CHAP.

In practice we are interested in expressing the equations of motion in terms of quantities


by an observer fixed on the earth [or other moving system]. In such case
determined
as
and write (12) as
subscript
the
may
omit
we

m ddtv

m(m X

r)

m[ x

2m(o> x v)

(13)

r)]

For the case of the earth rotating with constant angular o> about
(13) becomes
2
2m(<o X v) - m[ X (o> x r)]
m 2

its

axis,

*>

and

(U)

dt

CORIOLIS

AND CENTRIPETAL FORCE

Referring to equations (13) or

we

(14)

often use the following terminology

Coriolis force

Centripetal force

= m[ x

Centrifugal force

= -m[ x

2ra( x

r)

x
(

2ra(o>

v)

r)]

r)]

MOVING COORDINATE SYSTEMS IN GENERAL


In the above results we assumed that the coordinate systems xyz and XYZ [see Fig. 6-1] have common origin O. In case they do not have a common
origin, results are easily obtained from those already
considered.

Suppose that

is

the position vector of origin

Then

[see Fig. 6-2].

relative to origin

if

R and

denote the velocity and acceleration of Q relative


to O, equations (5) and (7) are replaced respectively

by

D Fr = R +
=

R +

+ <oXr

ZXr

dr
dt

<

xr

(15)
Fig. 6-2

and

Dl*

R + D M2 r +

Similarly equation (14)

is

^r

m dt

+S+

(D M <o) x

2. x

iXr + 2Xv +

DMr +

. x ( x

)X(a Xr)

r)

(16)

replaced by

F - 2m(u X

v)

m[<a

x(<aX

r)]

mR

(17)

THE FOUCAULT PENDULUM


Consider a simple pendulum consisting of a long string and heavy bob suspended
from a f rictionless support. Suppose that the bob is displaced from its equilibrium
position and is free to rotate in any vertical plane. Then due to the rotation of the earth,
the plane in which the pendulum swings will gradually precess about a vertical axis. In the

vertically

northern hemisphere this precession is in the clockwise direction if we look down at the
earth's surface. In the southern hemisphere the precession would be in the counterclockwise direction.

Such a pendulum used for detecting the earth's rotation was


in 1851 and is called Foucault's pendulum.

first

employed by Foucault

CHAP.

MOVING COORDINATE SYSTEMS

6]

147

Solved Problems

ROTATING COORDINATE SYSTEMS


6.1.

An

observer stationed at a point which is fixed relative to an xyz coordinate system


with origin O [see Fig. 6-1, page 144] observes a vector A = Ad + A 2j + Ask. and

calculates its time derivative to be

fri

+ ~dT^-

~rfj~i

Later, he finds that he

and his coordinate system are actually rotating with respect to an XYZ coordinate
system taken as fixed in space and having origin also at 0. He asks, "What would
be the time derivative of A for an observer who is fixed relative to the XYZ coordinate system?"

dA
If

fixed

dA

and rr

dt f

and

denote respectively the time derivatives of

dt
moving systems,

show that there

dA

dA

dt

dt

exists a vector quantity

relative to the
<*>

A
Hence such an

To the fixed observer the unit vectors i, j,k actually change with time.
observer would compute the time derivative as

dA

dA x

dt

dt

Since
j

and

dAl
dt \m

\f

from

dk
+ A 2 d\ + A ZTt
ft

di

dt

(1)

dk

di
d)
+ Aa 3 dt
+ ^1^7
+ A 2^
dt
dt
.

perpendicular to

is

di/dt

dj/dt

a 3k

dk/dt

a5i

0,

differentiation yields

Thus a 4

i *

dk

0,

i-^dt

jT+

dt

= a v

Similarly from ik
and a 6 = a 3 Then

,di

dj

ij
(3).

A^

(2)

and must therefore

lie

in

the plane

Then

k.

Similarly,

From

dA

3
-^k
+
dt

dt

a unit vector, di/dt

is

dAI
i.e.,

of

dA 2

^i
dt

such that

ji*
dt

di
=
+ ^k
dt
,

_
=

a 2k

(S)

a4 i

U)

a 6j

0-

and a 5

>dy

But

i -3ia 4

dt

from

= a 2 from jk =
;

(4)

and

di
-Tz'j

dt

.rfk+dl.i,
k
J'-^t+Tt*
dt
dt

0,

al

di/dt

= aj +

dj/dt

a 2k,

a 3k

dk/dt

a t i,

= a 2 i

atf

It follows that

di
=
^dt + A 2 ^ + A s -^
dt
.

(-iA 2

- a 2A 3 )i +

(iA l

-a sA s )j +

which can be written as

Then

where

if

<a

we

choose

u1i

a3

lf

+ 2 j + w 3k.

<o 2 ,

at

_ 2

<*i

Ai

A2

A3

this determinant

w3

W2

<0 3

<>

becomes

(a 2A!

+ a 3A 2)k

(6)

MOVING COORDINATE SYSTEMS

148

From

and

(2)

(6)

we

dt

6.2.

Let

D F and D M be

dA

is

moving system

the angular velocity of the

relative to the fixed system.

symbolic time derivative operators in the fixed and moving systems

DF
dA

DFA

definition

Then from Problem

D M + x

=
=

derivative in fixed system

derivative in

dt

DmA =

moving system

~di

6.1,

D F A = D M A + XA = (D M +
X
equivalence of the operators D F = D M +

which shows the

6.3.

dt

j?

Demonstrate the operator equivalence

respectively.

By

find, as required,

dA
The vector quantity

[CHAP.

o>

Prove that the angular acceleration

same

the

is

co

X)A

both

in

XYZ

and xyz coordinate

systems.
Let

A=

Since dm/dt

in

<o

is

Problem

Then

6.1.

da

da

~dt

~dl

to

da

~di

the angular acceleration, the required statement

is

proved.

VELOCITY AND ACCELERATION IN MOVING SYSTEMS


6.4.

Determine the velocity of a moving particle as seen by the two observers in


Problem 6.1.
Replacing

by the position vector r of the

particle,

dr
dt

dr
dt

we have
o,

(1)

If r is expressed in terms of the unit vectors i, j,k of the moving coordinate system, then the
velocity of the particle relative to this system is, on dropping the subscript M,

dr

dx

dt

dt

6.5.

velocity (3)

is

dz
dt

di 3

to the fixed

and the velocity of the particle relative

The

dy^

dr

dr

~di

dt

system

is

(2)

from

(1)

v
+ wXr
.

sometimes called the true velocity, while

(3)

(2) is

the apparent velocity.

An xyz coordinate system is rotating with respect to an XYZ coordinate system


having the same origin and assumed to be fixed in space [i.e. it is an inertial system].
The angular velocity of the xyz system relative to the XYZ system is given by
a)

time

2ti
t

t j

(2t

+ 4)k where

as observed in the

(a) the apparent velocity

is

the time.

and

(b)

The

position vector of a particle at

given by r = (t 2
the true velocity at time

xyz system

is

+ l)i t = 1.

6ij

4 3 k.

Find

CHAP.

MOVING COORDINATE SYSTEMS

6]

(a)

The apparent

any time

velocity at

is

dr/dt

At time
(6)

The true

this is

I.Xr

At time

this

6j

any time

velocity at

dr/dt

2i

(2ti

2ti

[2ti

6j

12t 2 k

12k.

is

- 6j +

12t%)

t2 )

(2t

+ 4)k] X

[(t 2

l)i

6j

4%]

is

2i

6j

12k

2-16
2-6

6.6.

149

34i

2j

2k

Determine the acceleration of a moving particle as seen by the two observers


Problem 6.1.
The acceleration of the particle as seen by the observer in the fixed XYZ system
Using the operator equivalence established in Problem 6.2, we have

in

is

D F x D F {D F r).

D F (D F r) = D F (D M r +

*>

r)

= (D M + X )(D M r + x r)
= D M {D M r + X r) + u X (D M r + .Xr)
= D^r + D M X r) + X D M r + X

a.

(a>

or since

DM

(<*

r)

(D M m)

r)

X (D M r),

- DmT +

Dlr

o>

+ 2.X

(D MW ) X r

(D M r)

X ( X

r)

(1)

If r is the position vector expressed in terms of i, j,k of the moving coordinate system, then
the acceleration of the particle relative to this system is, on dropping the subscript M,

<ffr

d?x.

dt 2

The acceleration of the

(Pr

6.7.

The apparent acceleration at any time


d fdr\

d2 r

(b)

this is

2i

2i

d*z

dt 2

dt 2

/dr

..

(g)

is

given from

this equals

24k

2i

40i

(4i

+
+

(b)

as

(1)

+ ttX(wXr)
(2) is

the apparent acceleration

the true acceleration of the particle in

is

d
eft

(2-

61

+ 12^)

2i

24*k

6j

+ 4k)}

24k.

The true acceleration at any time

At time

~ dt\dtj

dt2

At time

d2 y.

sometimes called the true acceleration, while

Find (a) the apparent acceleration and


Problem 6.5.
(a)

^ + ^ Xr + 2wX U^

dt 2 |f
F
(3) is

particle relative to the fixed system

d2 r

The acceleration

dt 2

is

- 2j + 12k) X (2i - 6j + 12k)


+ (2i - 2j + 2k) X (2i - 6j + 4k)
+ (2i - j + 6k) X {(2i - j +

24k

184j

(48i

- 24j - 20k) +

36k

(4i

6k)

(2i

- 4j - 8k) +

(-141

212j

+ 40k)

MOVING COORDINATE SYSTEMS

150

CORIOLIS
6.8.

acceleration and

From Problem

(c)

we

6.5

6.5,

find

From Problem

6.5

(a)

From

parts

(a)

we

and

the centripetal

(b)

1.

have,

=
=

2 X dr/dt
48i

24j

= (4i - 2j + 12k) X
- 20k

(2i

- 6j +

12k)

have,

=
=

Centripetal acceleration

(c)

the Coriolis acceleration,

their magnitudes at time

Coriolis acceleration

(b)

AND CENTRIPETAL ACCELERATION

Referring to Problem

(a)

[CHAP.

a.

-14i

=
+ 212j +
X

r)

(2i

- j + 6k) X

(32i

+ 4j - 10k)

40k

we have

(6)

Magnitude of Coriolis acceleration

Magnitude of centripetal acceleration

V(48)

V(-14)

(-24)2

(212)2

(_ 2 o)2

4\/205

2\/ll,685

40 )2

MOTION OF A PARTICLE RELATIVE TO THE EARTH


6.9.

Express Newton's second law for the motion of a particle relative to an XYZ
(6) Use (a) to find an equation of
coordinate system fixed in space (inertial system).
motion for the particle relative to an xyz system having the same origin as the XYZ
system but rotating with respect to it.
(a)

(a)

2
2
the mass of the particle (assumed constant), d r/dt F its acceleration
particle
as viewed in the
the
on
acting
all
forces
of
resultant
and
F
the
system
then Newton's second law states that

If

is

2
m ddtr \F =
2

(b)

Using subscript
Problem 6.6,

d?r

(1),

-2
dt \m

dt2

Substituting this into

(1)

denote quantities as viewed in the moving system,

to

we

we have from

....
w dt
+.X(.Xr)
+iXr+2.X^I
dt \m
.

()

find the required equation

F - m(iXr) - 2w(. xfjl


) - m[. X
dt \m

=
m^
dt 2 m

in the fixed
fixed system,

(.

(*)

r)]

provided it is clear that all quantities except F are as


can drop the subscript
the
determined by an observer in the moving system. The quantity F, it must be emphasized, is
If we do remove the subscript
resultant force as observed in the fixed or inertial system.
and write dr/dt = v, then (3) can be written

We

d2v

6.10.

= F - w(i X r) - 2m( X

Calculate the angular speed of the earth about


Since the earth makes one revolution
86,400 sec, the angular speed is
w

The

actual time for one revolution

=
is

[2tt

86^400

v)

m[ X

(*>

(4)

r)]

its axis.

radians] about its axis in approximately 24 hours

7.27

X 10-5

closer to 86,164 sec

rad/sec

and the angular speed

7.29

X lO" 5

rad/sec.

CHAP.

MOVING COORDINATE SYSTEMS

6]

151

MOVING COORDINATE SYSTEMS IN GENERAL


6.11. Work Problem 6.4 if the origins of the XYZ
and xyz systems do not coincide.
Let R be the position vector of origin Q of the
xyz system relative to origin O of the fixed (or inertial)

XYZ

system

ticle

dr\
dt \m

Now
= R+r

XYZ

The velocity of the parmoving system is, as before,

[see Fig. 6-3].

relative to the

dx.

dt

dt

-i

dy

"rrj

dt

dz

(1)

dt

the position vector of P relative to O is


and thus the velocity of P as viewed in the

system
dp
dt

dv

is

di

(R

+ r)
\

"

H\f +

= * +

dv
dt

di

Xr

using equation (3) of Problem 6.4. Note that R is the


this reQ with respect to O. If R =
duces to the result of Problem 6.4.

velocity of

6.12.

Work Problem

XYZ

6.6 if the origins of the

Referring to Fig.

Fig. 6-3

and xyz systems do not coincide.

6-3, the acceleration of the particle

as before,
d?r

dt2

Since the position vector of

XYZ

system

&x i+

dfr

relative to

is

= R + r,

dt 2

d*yd?z
dt2

K1)

dt 2

'

the acceleration of

as viewed in the

dt 2

R +

S + ^Xr + 2.X*+X(
dt2

using equation (S) of Problem 6.6. Note that R


R=
this reduces to the result of Problem 6.6.

Work Problem

6.9 if the origins of the

The

position vector of the particle


required equation of motion is

dt

dt

is

XYZ

relative

the acceleration of

Xr)

tt

Using the result

(2)

of Problem 6.12 in

d2 r
dt 2

where F

6.14.

is

mR

= F

the force acting on

(1),

If

and xyz systems do not coincide.

to

the

fixed

(XYZ) system

is

p.

Then the

(1)

we

m(i X

(2)

with respect to O.

d?P
dt 2

(6)

is,

d2 r

dm,

J^+'i

dt 2

(a)

moving system

relative to the

is

d 2p

6.13.

dt2

dt 2

obtain

r)

2m(<*

v)

m[ X

(o>

r)]

as viewed in the inertial system and where

r.

Find the equation of motion of a particle relative to an observer on the earth's


surface.

MOVING COORDINATE SYSTEMS

152

[CHAP.

We assume the earth

to be a sphere with center


rotating about the Z axis with
also use the fact that
angular velocity = wK.
the effect of the earth's rotation around the sun is
negligible, so that the XYZ system can be taken as

at

[Fig.

6-4]

We

an

inertial system.

Then we can use equation (2)


For the case of the earth, we have

Problem

6.12.

R =

of

o>

(1)

(o>

X R)

(2)

-2E,

F =

first equation arising from the fact that the rotation of the earth about its axis proceeds with constant angular velocity, the second arising from the
fact that the acceleration of origin Q relative to O

the

is the centripetal acceleration, and the third arising


from Newton's law of gravitation. Using these in
(2) of Problem 6.12 yields the required equation,

GM
p

dt*

assuming that other forces acting on

We

a)

(*

Fig. 6-4

X R)

2(<*

v)

X ( X

[such as air resistance,

etc.]

r)

(4)

are neglected.

can define

GM
g-p
as the acceleration due to gravity, so that

|^ =
Near the

earth's surface the last

term

(4)

in

tt

(*

X R)

(5)

becomes

2(

(6)

can be neglected, so that to a high degree of

v)

r)

(6)

approximation

2(

v)

(7)

In practice we choose g as constant in magnitude although it varies slightly over the earth's
surface. If other external forces act, we must add them to the right side of equations (6) or (7).

6.15.

Show that if the particle of Problem 6.14 moves near the earth's surface, then the
equations of motion are given by

where the angle A


From

Fig. 6-4

is

= 2(w

= g +

2>

cos A y
cos A x

m sin A

z)

2o>sinXy

the colatitude [see Fig. 6-4] and 90

-A

is

we have

K =
=
an(j s0

(K-i)i

(sin X)i

(K-j)j

Oj

= mK =

+ (K-k)k
(cos X)k

sin X

=
i

-sin X

w cos X

+
k

cos X

the latitude.

CHAP.

MOVING COORDINATE SYSTEMS

6]

Then

Xv

+ yj + zk)

(xi
i

w cos \

=
Thus from equation

( w cos X

= gk. +
Equating corresponding

coefficients of

(u cos \

z)j

a sin X

z)j

u sin X

( sin X

y)k

we have

2( cos \ x

2<o

sin X

2/

we

j,k on both sides of this equation,

i,

= 2w cos X y
= 2(u cos X x +
g + 2u sin X

An

2 cos X y

6.16.

y
y)\

of Problem 6.14

(7)

sin X

<o

153

find,

as required,
(1)

sin X z)

<o

(2)

(3)

initially at rest is dropped to the earth's surface from a height


mass
small compared with the earth's radius. Assuming that the angular speed
of the earth about its axis is a constant >, prove that after time t the object is
deflected east of the vertical by the amount %<gt 3 sin A.

object of

which

is

Method

1.

assume that the object is located on the z axis at * =


(i) and (2) of Problem 6.15 we have on integrating,

We

0,

0,

<o

sin X z)

[see Fig. 6-4].

From

equations

x
Since at

0,

0,

(5)

Then

2(o

cos X

j/,

0,

2<o

cos X y

0,

c it

cos X x

= we have c = 0,
= 2(w cos X x + w sin X

2(w

V
0,

ft

z)

c2

2u sin X

ft.

2a sin X

ft

c2

Thus
(i)

of Problem 6.15 becomes

g +

'z

2w sin X y

4u 2 sin X [cos X

a;

sin X (z

h)]

But since the terms on the right involving u 2 are very small compared with g we can neglect them
or
at t = 0, we have c 3 =
and write z = g. Integration yields z = gt + c 3 Since 1 =
.

= -gt

Using equation

(2)

and the
j/

Then neglecting the

first

first

=
=

term,

equation of

4w 2

cos 2

at

we have

0,

c4

=
2/

Then

since

at

0,

c5

2/)

2w sin

= ^ug

(!), ()

y
z

find

t3

c4
t

2.

Integrating again,

c5

so that, as required,

^ug

sin X

t3

(3)

2.

Integrating equations

we

Integrating,

ag sin X

sin X

of Problem 6.15

X gt

t2

wg sin X

(2)

( 2u sin X)(gt)

sin X gt.

and y

Method

1/

we have y = 2
y

Since y

equation

(1) in

(2(0 cos X)(2w cos X

(2)

and

()

of Problem 6.15,

we have

= 2w cos X + c
= 2(w cos X x + sin X z) +
= gt + 2w sin \y + c z
2/

(o

c2

MOVING COORDINATE SYSTEMS

154

Using the fact that at t =


= 2o>h sin X, c 3 = 0. Thus

0,

and

0;

[CHAP.

h,

0,

we have

ex

0,

c2

2w cos X

i/

2(<o cos X
gt + 2w
we

Integrating these

w sin X

2)

2uh sin X

using the above conditions,

find,

2w cos X
y

sin X

2aht

du

J" y
sin X 2w

igt 2

cos

XI

x du

2w sin X

dw

(5)

y du

2w sin X

(6)

unknowns are under the integral sign, these equations are called integral equations.
method called the method of successive approximations or method of iteration to
obtain a solution to any desired accuracy. The method consists of using a first guess for x, y, z
Since the

We

shall use a

under the integral signs in (4), (5) and (6) to obtain a better guess. As a first guess we can try
under the integral signs. Then we find as a second guess
x 0, y = 0, z =

x
Substituting these in
x

Using these

0,

in (4), (5)

2cofct

and

1/

and

(5)

(4),

0,

2wfti sin X,

gt2

and neglecting terms involving

(6)

sin X

2<o

sin X(7ii

^g&) ^ug&

<o

2
,

we

and again neglecting terms involving w 2 we

(6)

[ugt3

0,

sin X,

i9t

find the third guess


z

sin X,

&*

find the fourth guess

Since this fourth guess is identical with the third guess, these results are accurate up to terms
involving w 2 and no further guesses need be taken. It is thus seen that the deflection is
y = j^ugt3 sin X, as required.
,

6.17.

Referring to Problem 6.16, show that an object dropped from height h above the
earth's surface hits the earth at a point east of the vertical at a distance
%oih sin A yjlhlg.

From
and

(2)

z-h-

Problem 6.16 we have on integrating, z=_-gt 2 + c. Since z-h at t - 0, c-h


\g&. Then at z = 0, h = gt 2 or t = V^ft/fl'. Substituting this value of t into (5)
of

we

of Problem 6.16,

find the required distance.

THE FOUCAULT PENDULUM


6.18.

Derive an equation of motion for a simple


pendulum, taking into account the earth's
rotation about its axis.
Choose the xyz coordinate system of Fig. 6-5.
Suppose that the origin O is the equilibrium position
of the bob B, A is the point of suspension and the
length of string AB
is T, then we have

=
=

(T-i)i

cos a
'

-T

is

(T-j)j

+ T

x\

If the tension in the string

I.

cos p

Ti

+ T

y\
l

Since the net force acting on


tion of motion of B is given

m^J-

(T-k)k

k
z
(1)

is

by

T + mg - 2m( X

cos y

T
+ mil

v)

T + mg,

[see

the equa-

Problem

m<*

6.14]
r)

(*)

Fig. 6-5

CHAP.

MOVING COORDINATE SYSTEMS

6]

If

we

neglect the last term in

= gk

put g

(2),

and use

(1),

155

then

(2)

can be written in component

form as

mx =
my =

+ 2muy cos \
T(y/l) 2mu(x cos \ + z sin X)
= T(l z)/l mg + 2mJy sin X

m*z

6.19.

T(x/l)

(S)
(4)
(5)

that the bob of the simple pendulum in Problem 6.18 undergoes small
oscillations about the equilibrium position so that its motion can be assumed to take
place in a horizontal plane, simplify the equations of motion.

By assuming

Making the assumption that the motion of the bob takes place in a horizontal plane amounts
assuming that z and z are zero. For small vibrations (l z)/l is very nearly equal to one.
Then equation (5) of Problem 6.18 yields

to

T mg + 2muy

= mg 2muy

or
Substituting

(1)

and

into equations (3)

gx
= ^j-

gy
^j-

sin X

(1)

we

of Problem 6.18 and simplifying,

(4)

sin X

2axy sinX

2ayy

^y

+
,

sin X

obtain
,>

2wj/cosX

(2)

/ff

2 <oa; cosX

(3)

These differential equations are non-linear because of the presence of the terms involving xy
However, these terms are negligible compared with the others since , x and y are
and yy.
small. Upon neglecting them we obtain the linear differential equations

6.20.

gx/l

gy/l

2uy cos X

(-4)

2(o* cos X

(5)

Solve the equations of motion of the pendulum obtained in Problem 6.19, assuming
suitable initial conditions.

Suppose that

initially the

A >

of magnitude

0,

To

bob

x
(-4)

and

(5)

K2 =
so that they

become

we

given a displacement from the z axis


initial conditions are
at

of Problem 6.19,

g/l,

K2 x +
= -K2 y -

it

=
is

convenient to place

a cos X

(2)

2ay

(8)

2ax

U)

Multiplying equation

convenient to use complex numbers.

(1)

(4)

by

and adding

to

(3),

find

Then

calling

iy

= -K2 (x + iy) +

u = x + iy,

this

u
If

A, y

x
y

It is also

0,

is

Then the

released.

is

it

0,

find the solution of equations

the yz plane and

is in

after which

Cey*

= -K2 u -

2iau

Now

since

V
a2

w 2 cos 2 X

=
is

-K2 (x + iy) -

2ia(x

+ iy)

can be written

where C and y are constants,

so that

2a (y-ix)

(-2i

or

this

u + 2iau +

(5)

becomes

2iay

+ K2 =

V -4a 2 - 4K2 )/2 =


/

small compared to
y

K2 u =

K2 = g/l,

= ia iK

-ia

Wa 2 + K2

we can

(6)

write
(7)

MOVING COORDINATE SYSTEMS

156

Then

solutions of the equation are (allowing for complex coefficients)

(C 1

and the general solution

where d> C 2 C 3
,

C4

+ iC2 )e- Ka - K

e i0

iy

(C x

(C^iCJe-*"- +

iC4 )e- + K>t

(C 3

(C 3

+ tC4 )e- + K)t

+ ism 6,

costf

+ iy,

e~ ie

cos e

sin

(9)

can be written

(8)

- K)t -

t'C 2 ){cos (a

(5)

Using Euler's formulas

are assumed real.

and the fact that u

and

>t

is

[CHAP.

sin (a

- K)} +

+ iC4 ){cos ( + #) -

(C 3

sin (a

+ K)t)

Equating real and imaginary parts, we find

- #)* + C 2

d cos

-dam (a -K)t

Using the

C 3 = -d.

a cos X

0,

we

0,

we

sin (a

from

find

from

find

K)t

+ X)t

sin (a

+ C4

cos (a

(10)

that

(10)

+ K)t

(11)

d+C

or

that

(10)

/Vff/l- wcosX

a\

small compared with

is

+ K)t + C4

cos (a

-K)t - C 3

at

at

X_
C
"
U +

c*

- K)t + C 3

cos (a

Similarly, using

Now since
C4 = C2

+ C2

condition

initial

sin (

C2

"

Vx^77+
we

-\fgjl,

w cosx,

have, to a high degree of approximation,

Thus equations

Using the
find

C2 =

(10)

-d

initial

sin (a

condition

Thus

-|A.

and

(1)

y
and

- K)t

d cos a + K + C sin
+ d sin + K) + C cos
)

Similarly using

0-

(a

d=

yields

+ K)t

(a

=A

- K)t + |A
\A cos (a - K) + \A

= \A

sin (a

sin (a

+ K)t

cos (a

+ X)t
at

(15)
t

0,

we

= A

2/

= A COSK*

a;

= A

cosy/gilt sin(ucosXt)

j/

= A

cos

cos

Kt sm at

#)*
"1

COS at

(*)

i-

6.21.

(12)

become

(15)

e.,

cos (a

(1*)

0,

- K)t -

sin (a

- K)t + C 2

or

become

(ii)

cos ( - X)t + C2

V^

cos (w cos

Give a physical interpretation to the solution

H)

(15) of

Problem

6.20.

In vector form, (15) can be written

w here
is

x\
i

2/j

= A

sin ( cos X)t

cos
j

V'gilt n

cos (w cos \)t

a unit vector.
[namely,
is very small compared with the period of n
pendulum
the
physically
Thus
vector.
turning
slowly
a very
axis.
axis which is slowly rotating (or precessing) about the z

The period of cos yfgllt [namely, 2ttV^]


2tt/((ocosX)].

It follows

oscillates in a plane

that

through the

is

and the bob is at y = A. After a time t = 2^/(4 w cosX), for example,


j
as
the rotation of the plane is proceeding in the clockwise direction
that
J-V^j
so
n
2
the
In
cosX>0].
[where
viewed from above the earth's surface in the northern hemisphere
southern hemisphere the rotation of the plane is counterclockwise.
provide laboratory
The rotation of the plane was observed by Foucault in 1851 and served to
axis.
its
about
earth
the
of
evidence of the rotation

Now

at

l\/2i

0,

n =

CHAP.

MOVING COORDINATE SYSTEMS

6]

157

MISCELLANEOUS PROBLEMS
6.22.
The vertical rod AB of Fig.

6-6 is rotating with


constant angular velocity <>. A light inextensible
string of length I has one end attached at point O
of the rod while the other end P of the string has a
mass
attached. Find (a) the tension in the string
and (b) the angle which string OP makes with the
vertical when equilibrium conditions prevail.
Choose unit vectors i and k perpendicular and parallel
respectively to the rod and rotating with it. The unit vector
j can be chosen perpendicular to the plane of i and k.
Let
= I sin e i I cos e k
r

be the position vector of

with respect to O.

Three forces act on particle


(i)

The weight, mg =

(ii)

The centrifugal

mgk
Fig. 6-6

force,

-m{a X

(iii)

The

tension,

When

T =

the particle

is in

=
=

r)}

sin e

-m{[tok]

m{[wk] X

+ T

[I

sin e

(al sin e j)}

cos B k])}

= ma2

sin

Then

+ muH sin 6 T sin e + T cos k =


2
(mu sin d T sin $)i + (T cos 6 mg)k =
tow2 sin 6 T sin
-

i.e.,

or

T
Solving

and

(1)

(2)

rod

AOB

simultaneously,

OP with mass
pendulum.

Since the string


called a conical

cos 6

([wk]

equilibrium, the resultant of all these forces is zero.

mgk.

6.23.

we

cos 6

find

at

mg
(a)

(1)

T = mu2

(2)
l,

(6)

cos -1 (g/u2 l).

describe the surface of a cone the system

is

sometimes

[Fig. 6-7] rotates in a vertical plane [the yz plane]

about a horizontal
perpendicular to this plane [the x axis] with constant angular
velocity w. Assuming no frictional forces, determine the motion of a particle P of
mass
which is constrained to move along the rod. An equivalent problem exists
when the rod A OB is replaced by a thin hollow tube inside which the particle can move.

axis through

Fig. 6-7

At time
the y axis.
i

= j X k.

increasing

t let r be the position vector of the particle and e the angle


made by the rod with
Choose unit vectors j and k in the y and z directions respectively and unit vector
Let rj be a unit vector in the direction r and e 1 a unit vector in the direction of

e.

MOVING COORDINATE SYSTEMS

158

[CHAP.

There are three forces acting on P:


(i)

The weight, rag

= mgk mg

(ii)

The centrifugal

force,

-ra[o

(iii)

(*

mg

rx

sin

=
=
=

r)]

ra-^To2

dt

=
=

m-7r^r 1

or

It follows

that

N = mg cos

mg
(ma

a,

ra^k +

mg

at

ra<o 2 rr x

2V0J

+ mu 2 rr +
+ (N mg cos ff)^

cos

sin 0)r x

wt

if

d2 r/d2

we assume that

perpendicular to the rod since there are

is

mg

0j

iVOj

and

we have

a constant,

rr t )]

d2 r/d2
Since

(i

ra[coi(wi rrj.) rr (wi wi)]


ra[0 wVrj] = ra-uVr!

sine r t

2r

9X

-ra[wi

The reaction force N = N8 t of the rod which


no frictional or resistance forces.

Then by Newton's second law,


d2 x

If

cos

0,

(i)

sin

we assume

w2r

dr/dt

u2r

at

-sr sin u

we

find

0.

Then

(i)

becomes
(2)

or in terms of hyperbolic functions,


r

6.24.

r cosht

--^jsinhwt

(-4)

that under suitable conditions the particle of Problem 6.23 can oscillate
the
rod with simple harmonic motion and find these conditions, (b) What
along
the particle if the conditions of (a) are not satisfied?
to
happens
(a)

(a)

(6)

Show

The particle will oscillate with simple harmonic motion along the rod if and only if r =
and v = 0/2<o. In this case, r = (g/2<*2 ) sin at. Thus the amplitude and period of the simple
harmonic motion in such case are given by g/2u> 2 and 2n-/w respectively.
If

(g/2u)

ur

then

r-

e _wt

(g/2a 2 )

harmonic after some time. Otherwise the mass

6.25.

+ ^-sinw*

smut and

the motion

is

approximately simple

will ultimately fly off the rod if it is finite.

projectile located at colatitude X is fired with velocity v in a southward direction


t.
at an angle with the horizontal, {a) Find the position of the projectile after time
original
the
of
the
east
toward
deflected
is
(b) Prove that after time t the projectile

vertical plane of motion

by the amount

Xtz

^<*g sin

(a)

We

Assuming the

use the equations of Problem 6.15.

x
Also, the initial velocity is

=
v

0,

v cos

a,

w^o cos (a

0,

cos a

0,

X)

sin a

projectile starts at the origin,

at

sin

we have
U)

so that

at

(2)

CHAP.

MOVING COORDINATE SYSTEMS

6]

Integrating equations

and

(2)

(1),

we

of Problem 6.15,

(3)

2oj

2(w

gt + 2w

cos X

159

obtain on using conditions

v cos a

cos X x

(3)

w sin X

sin X y

(2),

z)

(4)

v sin a

(5)

Instead of attempting to solve these equations directly we shall use the method of iteration
or successive approximations as in Method 2 of Problem 6.16. Thus by integrating and using
conditions (1), we find

c y du

2wcosX

(v

cos X

sin a)t

x du

2u sin X

gt 2

2w sin X

cosa)(

(t;

(6)

du

(7)

y du

(8)

Jo

As a first guess we use x = 0, y = 0,


become, neglecting terms involving w 2

under the integral signs.

Then

(6),

(7)

and

(8)

To obtain a better guess we now use


(8),

(v

cos a)t

(v

sina)

(9)

(10)

and

(9), (10)

- \g&

(11)

under the integral signs

(II)

in (6), (7)

and

thus arriving at
X

= uv

COS a)t

(V

cos (a

sin a)t

(i)

(12)

\)t 2

^wgt3

sin X

(13)

2
Iflrt

(14)

where we have again neglected terms involving


Further guesses again produce equations
2
(12), (13) and (14), so that these equations are accurate up to terms involving
w2 .

(6)

From

equation (13)

by the amount

6.26.

we

see that the projectile

^ugts sin X

cos (a

wi;

X)

t2 .

is

deflected

If

toward the east of the xz plane


Problem 6.16.

this agrees with

Prove that when the projectile of Problem 6.25 returns to the horizontal,
the distance


o>vl sin
;r-

to the west of that point

where

(3

it

cos a cos A

would have landed assuming no axial rotation

projectile will return to the horizontal


(v

Using

this value of

sin a)t

^gt2

in equation (13) of

will be at

sin a sin A)

of the earth.

The

it

when

or

Problem

6.25,

=
t

we

0,

i.e.,

(2v

sin a)/g

find the required result.

MOVING COORDINATE SYSTEMS

160

[CHAP.

Supplementary Problems
ROTATING COORDINATE SYSTEMS. VELOCITY AND ACCELERATION
= 2i 3j + 5k
An xyz coordinate system moves with angular velocity
6.27.

relative to a fixed or
*
coordinate system having the same origin. If a vector relative to the xyz system
A = sin t i cos t j + e -t k, find (a) dA/dt relative to
is given as a function of time t by
the fixed system, (b) dA/dt relative to the moving system.
inertial

Ans.

6.28.

XYZ

(a)

(6 cos t

(b)

cos

t i

- 3e-*)i +
+ sin t j

Find d2A/dt2 for the vector

(6 sin t
e _t

2e-*)j

(3 sin t

2 cos

e-')k

of Problem 6.27 relative to

(a)

the fixed system and

(6)

the moving

system.

Ans.

(a)

cos

(6

sin t

(b)

6.29.

+ Ue-^i + (40 cos * - 23 cos t + 16e~')k


+ cos t j + e _t k

+
i

45 sin

(10 sin

6 sin

lle-^j

xyz coordinate system is rotating with angular velocity o = 5i 4j 10k relative to a fixed
coordinate system having the same origin. Find the velocity of a particle fixed in the xyz
system at the point (3, 1, 2) as seen by an observer fixed in the XYZ system.

An

XYZ

Ans. 18i-20j
6.30.

Discuss the physical interpretation of replacing

Problem

(6)

6.31.

+ 17k

6.6,

by

<*

-*>

in

(a)

Problem

page 148, and

6.4,

page 149.

Explain from a physical point of view

expect the result of Problem

why you would

6.3,

page 148,

to be correct.
6.32.

xyz coordinate system rotates with angular velocity = cos t i + sin t j + k with respect to a
XYZ coordinate system having the same origin. If the position vector of a particle is given
by r = sin t i - cos t j + tk, find (a) the apparent velocity and (6) the true velocity at any
(6) (t sin t + 2 cos *)i + (2 sin t - t cos t)j
Ans. (a) cos t i + sin t j + k
time t.

An

fixed

6.33.

Determine (a) the apparent acceleration and


Problem 6.32.
Ans. (a) -sin ti + cos t j
(6) (2t cost - 3 sin

CORIOLIS

the

(6)

t)i

(3

true
cos

acceleration

2t sin

)j

the

of

(1

particle

of

t)k

AND CENTRIPETAL ACCELERATIONS AND FORCES

the northern hemisphere, (a) Would the path of the ball,


when it is not
if the Coriolis force is taken into account, be to the right or to the left of the path
taken into account as viewed by the person throwing the ball? (6) What would be your answer
Ans. (a) to the right, (6) to the left
to (a) if the ball were thrown in the southern hemisphere?

6.34.

6.35.

What would

6.36.

Explain why water running out of a vertical drain will swirl counterclockwise in the northern
hemisphere and clockwise in the southern hemisphere. What happens at the equator?

6.37.

6.38.

ball

in

be your answer to Problem 6.34

if

the ball were thrown at the north or south poles?

In Problem 6.37, where would the centrifugal force be


(a)

at the equator,

(6)

maximum,

(a)

(a)

35.0

kg wt,

(6)

17.5

minimum?

(a)

the equator

(b)

colatitude 30.

kg wt

flows northward with a speed v at colatitude X. Prove that the left


river of width
of the river will be higher than the right bank by an amount equal to

where w
(6)

(6)

at the north and south poles.

(2D uv cos\)(g 2

6.41.

(a)

Find the centrifugal force acting on a train of mass 100,000 kg at


Ans.

6.40.

thrown horizontally

on the earth's surface is a


Prove that the centrifugal force acting on a particle of mass
velocity vector o> and
angular
to
the
perpendicular
and
earth
the
vector (a) directed away from
2
(6) of magnitude mu> R sin X where X is the colatitude.

Ans.
6.39.

is

is

Prove that the result in part

If the river of

the angular speed of the earth about

Problem

how much higher

(a) is

all

1'2

its axis.

practical purposes equal to (2Duv

cos \)/g.

km wide and flows at a speed of 5 km/hr at colatitude 45,


Ans. 2.9 cm
bank be than the right bank?

6.40 is 2

will the left

for

V cos^)-

bank

CHAP.

6.42.

6.43.

MOVING COORDINATE SYSTEMS

6]

161

An

automobile rounds a curve whose radius of curvature is p. If the coefficient of friction


prove that the greatest speed with which it can travel so as not to slip on the road is Vwff-

Determine whether the automobile of Problem 6.42

(a) p

500

ft,

(6)

50

will slip if the speed is 60 mi/hr,

is

.05

/j,

and

Discuss the results physically.

ft.

MOTION OF A PARTICLE RELATIVE TO THE EARTH


6.44.

An

If air resistance is neglected,


is dropped at the equator from a height of 400 meters.
far will the point where it hits the earth's surface be from the point vertically below the
initial position?
Ans. 17.6 cm toward the east

object

how

6.45.

Work Problem 6.44 if the object


(a) 15.2 cm toward the east

is

dropped

(a)

at colatitude 60

and

at the north pole.

(6)

Ans.
6.46.

An

object

returns
6.47.

An

it

thrown vertically upward at colatitude X with speed v

is

will be at a distance

westward from

its

Prove that when

starting point equal to (av\ sin X)/Sg 2

How

object at the equator is thrown vertically upward with a speed of 60 mi/hr.


position will it land?
Ans. .78 inches

it

far from

its initial

6.48.

6.49.

With what speed must the object of Problem 6.47 be thrown in order that
Ans. 406 mi/hr
on the earth which is 20 ft from its original position?

An

object

thrown downward with initial speed v


by the amount

is

it

return to a point

Prove that after time

the object

is

deflected east of the vertical

wv sin X
6.50.

Prove that

if

surface, then

t2

6.52.

Prove that

a.

mgly/l 2

a2

If

t3

the object of Problem 6.49 is thrown downward from height h above the earth's
will hit the earth at a point east of the vertical at a distance

Suppose that the mass


radius

sin X

it

2
^JP^ (Vv + 2gh

6.51.

^ug

(a)

of a conical

the

speed

)H^v 2Q + 2gh +

pendulum of length

is

ayfg/^/l

and

2v

moves
(6)

the

path

is

in

a horizontal

tension

in

the

circle

of

string

is

an object

is

dropped to the earth's surface prove that

its

a semicubical parabola.

THE FOUCAULT PENDULUM


6.53.

why

Explain physically

when viewed from above

the plane of oscillation of a Foucault pendulum should rotate clockwise


the earth's surface in the northern hemisphere but counterclockwise in

the southern hemisphere.


6.54.

How

long would

revolution if the

Ans.
6.55.

(a)

take the plane of oscillation of a Foucault pendulum to


is located at (a) the north pole,
(6) colatitude 45,

it

pendulum

23.94 hr,

(6)

33.86 hr,

(c)

make one complete


(c)

colatitude 85?

92.50 hr

Explain physically why a Foucault pendulum situated at the equator would not detect the
rotation of the earth about its axis. Is this physical result supported mathematically? Explain.

MOVING COORDINATE SYSTEMS IN GENERAL


6.56.

xyz coordinate system rotates about the z axis with angular velocity o> = cos t i + sin t j
XYZ coordinate system where t is the time. The origin of the xyz system
has position vector R = ti j + t2k with respect to the XYZ system. If the position vector of
a particle is given by r = (Zt + l)i 2tj + 5k relative to the moving system, find the (a) apparent

An

relative to a fixed

velocity

and

(6)

true velocity at any time.

apparent

and

6.57.

Determine (a)
Problem 6.56.

6.58.

Work (a) Problem 6.5, page 148, and (b) Problem 6.7, page 149,
xyz system relative to the origin of the fixed XYZ system is R =

the

acceleration

(6)

the

true

acceleration

if
t2 i

of

the

particle

in

the position vector of the

2tj + 5k.

MOVING COORDINATE SYSTEMS

162

[CHAP.

MISCELLANEOUS PROBLEMS
6.59.

Prove that due

mass

at

to the rotation of the earth

colatitude

is

m^(g -

u2

about

sin 2 x) 2

its

axis the app arent weight of an object of

w 2#

sjn x

cos X )2

where R

is

the*

radius

of

the earth.
6.60.

Prove that the angle


given by

is

tan

/?

g
6.61.

6.62.

6.63.

makes with the true

vertical

sin X cos X

u 2R sin 2 X

'

Explain physically why the true vertical and apparent vertical would coincide at the equator
and also the north and south poles.

A stone is twirled in a vertical circle by a string of length 10 ft. Prove that it must have a
speed of at least 20 ft/sec at the bottom of its path in order to complete the circle.
A

car

[Fig. 6-8] is to

circular

loop

Assuming the track


at which it must

particle of

go completely around the vertical


a without leaving the track.
frictionless, determine the height

of radius

H
6.64.

which the apparent vertical at colatitude

/?

mass

is

start.

m is constrained to move on a friction-

a which rotates about a fixed


diameter with constant angular speed w. Prove that the
particle will make small oscillations about its equilibrium
position with a frequency given by 2vau/-\/ a2 w4 g 2
less vertical circle of radius

6.65.

Discuss what happens in Problem 6.64

6.66.

hollow cylindrical tube


of length 2a [Fig. 6-9]
rotates with constant angular speed about a vertical
axis through the center O.
particle is initially at rest

if

yfg/a.

Fig. 6-8

AOB

CJ>

in the

tube at a distance & from O.

tional forces, find (a) the position


the particle at any time.

6.67.

(a)

How

long will

it

Assuming no

and

(6)

frac-

the speed of

take the particle of Problem 6.66 to


(&) what will be its speed as

~B

come out of the tube and


it

6.68.

6.69.

6.70.

Ans.

leaves?

In (a

y/a2

b2

Fig. 6-9
)

Find the force on the particle of Problem 6.66 at any position

in the tube.

A mass, attached to a string which is suspended from a fixed point, moves in a horizontal circle
having center vertically below the fixed point with a speed of 20 revolutions per minute.
Find the distance of the center of the circle below the fixed point.
Ans. 2.23 meters
A

particle on a frictionless horizontal plane at colatitude X is given

northward
6.71.

(a)

direction.

Prove that

it

an

initial

speed v

describes a circle of radius v /(2w cos X) with period

The pendulum bob of a conical pendulum describes a horizontal circle of radius


pendulum is I, prove that the period is given by 4ir 2 y/l 2 a2 /g.

a.

n7(<o

in

cos

X).

If the length

of the

6.72.

6.73.

move on a circular wire of radius a and coefficient n is given an initial


Assuming no other forces act, how long will it take for the particle to come to rest?

particle constrained to

velocity v

(a) Prove that if the earth were to rotate at an angular speed given by y2g/R where R is its
would be
radius and g the acceleration due to gravity, then the weight of a particle of mass
the same at all latitudes. (6) What is the numerical value of this angular speed?

Ans.

(b) 1.74

X 10~3

rad/sec

CHAP.

6.74.

6.75.

6.76.

MOVING COORDINATE SYSTEMS

6]

163

so that no

tank containing water rotates about its axis with constant angular speed &>
water spills out. Prove that the shape of the water surface is a paraboloid of revolution.

Work

Problem 6.16 and

cylindrical

(a)

Problem

(6)

6.17,

accurate to terms involving w 2 .

Prove that due to the earth's rotation about its axis, winds in the northern hemisphere traveling
to a low pressure area are rotated in a counterclockwise sense when
viewed above the earth's surface. What happens to winds in the southern hemisphere?

from a high pressure area

6.77.

Prove that in the northern hemisphere winds from the north,


and west are deflected respectively toward the west,
north, east and south as indicated in Fig. 6-10. (b) Use this to
(a)

east, south

explain the origin of cyclones.

6.78.

Find the condition on the angular speed so that a particle will


describe a horizontal circle inside of a frictionless vertical cone
of angle

a.

6.79.

Work Problem

6.80.

The period of a simple


the ceiling of a train
PVp~9/y/v*

6.81.

6.82.

6.78 for a hemisphere.

pendulum is given by P. Prove that its period when it is suspended from


moving with speed v around a circular track of radius p is given by

+ p 2g2.

Work Problem

6.25 accurate to terms involving w 2

A thin hollow cylindrical tube OA inclined at angle a with the


horizontal rotates about the vertical with constant angular speed
w [see Fig. 6-11]. If a particle constrained to move in this tube
is initially at rest at a distance a from the intersection O of the
tube and the vertical axis of rotation, prove that its distance r
from O at any time t is r = a cosh (ut cos a).

6.83.

Work Problem

6.84.

Prove that the particle of Problem 6.82


between the distances from O given by

6.82 if the rod has coefficient of friction

sin a / l

w2

tan a
tan a + n

assuming tan a <


6.85.

jit

and

is

in stable equilibrium

+ n tan a
\ tan a fi
Fig. 6-11

l//x.

A train having a maximum speed equal to v is to round a curve with radius of curvature p. Prove
that if there is to be no lateral thrust on the outer track, then this track should be at a

av\l yjv\

+ P 2g 2

where a

is

the distance between tracks.

directed toward the west and at angle a with


terms involving 2 are neglected, then the time taken to reach

projectile is fired at colatitude X with velocity v

the horizontal. Prove that


the maximum height is

if

2uv sin X sin a cos a

v sin a

Compare with the case where w =


6.87.

/i.

sin a ( 1

w2

height above the inner track given by

6.86.

In Problem 6.86, prove that the

0,

ff

i.e.

maximum
2

v sin 2 a
2g

Compare with the case where

<o

that the earth does not rotate about

0.

height reached

2uv% sin X

sin 2

g*

is

a cos a

its axis.

MOVING COORDINATE SYSTEMS

164

6.88.

Prove that the range of the projectile of Problem 6.86


Vq sin 2a

[CHAP.

is

(8 sin 2

uVq sin a sin X

6)

3^2

Thus show that if terms involving co 2 and higher are neglected, the range will be larger, smaller
or the same as the case where <o = 0, according as a > 60, a < 60 or a 60 respectively.

6.89.

If a projectile is fired with initial velocity v x i


v 2 j + v 3k from the origin of a coordinate system
fixed relative to the earth's surface at colatitude X, prove that its position at any later time t
will be given by

vxt

uv 2 t 2 cos X

v2 t

at 2 ^! cos \

v3t

gt 2

neglecting terms involving

6.90.

6.91.

Work Problem

co

v 3 sin

X)

^ugt 3 sin X

uv 2 t 2 sin X

2.

6.89 so as to include

terms involving

co

but exclude terms involving

co

3.

An object of mass m initially at rest is dropped from height h to the earth's surface at colatitude X.
Assuming that air resistance proportional to the instantaneous speed of the object is taken into
account as well as the rotation of the earth about its axis, prove that after time
deflected east of the vertical by the amount
" X

2t

[(g

^3

~ 2hp 2 )(l - e-Pt) +

phte-Bt

pgt

+ \g$ 2 t 2

the object

is

neglecting terms of order w 2 and higher.

6.92.

Work Problem

6.93.

6.91,

obtaining accuracy up to and including terms of order u 2

and angle a located at colatitude X is so situated that


I
under the influence of gravity from north to south. If the
particle starts from rest at the top, prove that it will reach the bottom in a time given by
frictionless inclined plane of length

a particle placed on

it

would

slide

2ul sin X cos a

y g sin a

and that

its

speed at the bottom

yj2gl sin a

neglecting terms of order

6.94.

co

3g

is

f ul sin a cos a sin X

2.

(a) Prove that by the time the particle of Problem 6.93 reaches the bottom
a deflection of magnitude
2Zco

\\
g

-5to
(&)

the east or west respectively according


Discuss the case where cos (a + X) = 0.

21

as

cos (a

(c)

Use the

X)

X)

is

greater than

result of

(a)

to

or less than zero.


arrive at the result

of Problem 6.17.

6.95.

Work Problems

6.93

and 6.94

if

have undergone

...

COS (a

sin a

will

it

the inclined plane has coefficient of friction

(i.

SYSTEMS

Chapter 7
of

DISCRETE AND CONTINUOUS SYSTEMS


Up to now we have dealt mainly with the motion

PARTICLES

which could be considered


which we are concerned
or point mass. In many practical cases the objects with
Such systems
be considered as collections or systems of particles.

as a particle

of an object

can more realistically

particles can be considered as separated


are called discrete or continuous according as the
from each other or not.
finite number
For many practical purposes a discrete system having a very large but
system can
continuous
a
Conversely
system.
continuous
of particles can be considered as a
particles.
of
number
finite
but
large
of
a
consisting
be considered as a discrete system

DENSITY
convenient to

For continuous systems of particles occupying a region of space it is often


volume density or briefly density.
define a mass per unit volume which is called the
of particles, then the density can
At
volume
a
of
mass
Mathematically, if AM is the total
be defined as
<t

aM
hm At
,.

m
v-U

at^o

the density
a function of position and can vary from point to point. When
uniform.
density or simply
is a constant, the system is said to be of uniform
When the continuous system of particles occupy a surface, we can similarly define a
occupy a line [or curve]
surface density or mass per unit area. Similarly when the particles
we can define a mass per unit length or linear density.

The density

RIGID

is

AND ELASTIC BODIES

distances between
In practice, forces applied to systems of particles will change the
bodies. In some
elastic
individual particles. Such systems are often called deformable or
purposes
practical
for most
cases, however, deformations may be so slight that they may
in which
model
be considered non-existent. It is thus convenient to define a mathematical
regardless
same
the
the distance between any two specified particles of a system remains
of rigid bodies is
mechanics
The
body.
rigid
a
called
is
system
Such
a
forces.
applied
of
considered in Chapters 9 and 10.

DEGREES OF FREEDOM
of coordinates required to specify the position of a system of one or
particles is called the number of degrees of freedom of the system.

The number

Example

A
the

more

1.

particle

number

moving freely

position.
in space requires 3 coordinates, e.g. (x, y, z), to specify its

of degrees of freedom

is 3.

165

Thus

SYSTEMS OF PARTICLES

166

Example

[CHAP.

2.

system consisting of

Thus the number

position.

particles

moving freely

of degrees of freedom

in space

3N

requires

coordinates to specify

its

is 32V.

A rigid body which can move freely in space has 6 degrees of freedom,
are required to specify the position. See Problem 7.2.

i.e.

6 coordinates

CENTER OP MASS
Let

mu m

2,

n, r2
.

be the position vectors of a system of

rN

.,m N respectively

The center

mass or centroid of the system of

of

+ m2r2 +
m + m2 +

mxxx

where

in place of

^m

is

particles

of

masses

the total

+ mNrN
+ mN

_
~

We

mass of the system.

particles is defined as that point

having position vector


r

[see Fig. 7-1].

(2)

sometimes use

or simply J)

..2

Fig. 7-1

Fig. 7-2

For continuous systems of


volume density

is

o-,

particles occupying a region

of space in which the

the center of mass can be written

rdi

or

J
where the integral

is

taken over the entire region


f

then

(3)

xi

yj

zk,

dr

[see Fig. 7-2].

xvi

y vj

If

we

write

z vk

can equivalently be written as

_
~

and

where the

X
total

mass

=
is

S WyXy
M

JaXdr
%
M

>

_
~

2)

-,

'

yv

'

aydr

<V

*J CD
*"K

JaZdr

(5)

given by either

M
M

=
=

^m
\

<rdr

(6)

(7)

CHAP.

The

SYSTEMS OF PARTICLES

7]

integrals in

(3),

or

(5)

167

can be single, double or triple integrals, depending on

(7)

which may be preferable.


In practice it is fairly simple to go from discrete to continuous systems by merely
replacing summations by integrations. Consequently we will present all theorems for
discrete systems.

CENTER OF GRAVITY
If a

system of particles

is in

a uniform gravitational

field,

the center of mass

is

some-

times called the center of gravity.

MOMENTUM OF A SYSTEM OF PARTICLES


If v, = dxjdt = r is the velocity of m the total momentum of the
v,

] rav

can show

[see

Problem

This

is

is

^m

rv

(8)

Mi

(9)

that

7.3]

M^

Mv =

where v = di/dt

defined as

v=l

v=l

We

is

system

the velocity of the center of mass.

expressed in the following

Theorem

The

7.1.

total

M of the

the total mass

of a system of particles can be found


the velocity v of the center of mass.

momentum

system by

by multiplying

MOTION OF THE CENTER OF MASS


Suppose that the internal forces between any two particles of the system obey Newton's
Then if F is the resultant external force acting on the system, we have [see

Problem 7.4]
,_
,
third law.

F
This

is

ft

M w>

M dt

<

I0)

expressed in

Theorem 7.2. The center of mass of a system of particles moves as


and resultant external force were applied at this point.

if

the total mass

CONSERVATION OF MOMENTUM
Putting

F=

in (10),

we

find that

^m
v=l

\v

constant

(11)

Thus we have

Theorem
then the total
is

7.3.

on a system of particles is zero,


conserved. In such case the center of mass

If the resultant external force acting

momentum remains

constant,

i.e. is

either at rest or in motion with constant velocity.

This theorem
tion of

is

Theorem

often called the principle of conservation of


2-8,

page 37.

momentum.

It is

a generaliza-

SYSTEMS OF PARTICLES

168

[CHAP.

ANGULAR MOMENTUM OF A SYSTEM OF PARTICLES


The quantity

O
is called

the total angular

2m

momentum

[or

{rXv v )

moment

of

(12)

momentum]

of the system of particles

about origin 0.

THE TOTAL EXTERNAL TORQUE ACTING ON A SYSTEM


If F is the external force acting on particle

force

Fv

or torque about 0.

then

v,

x F

is

called the

moment

of the

The sum
N

A
is called

2r

X F

(iS)

the oaZ external torque about the origin.

RELATION BETWEEN ANGULAR MOMENTUM AND


TOTAL EXTERNAL TORQUE
If we assume that the internal forces between any two particles are always directed
along the line joining the particles [i.e. they are central forces], then we can show as in

Problem 7.12 that

da

dt

Thus we have

Theorem 7.4. The total external torque on a system of particles is equal to the time
rate of change of the angular momentum of the system, provided the internal forces
between particles are central forces.

CONSERVATION OF ANGULAR MOMENTUM


Putting

A=

in (U),

we

find that

^m

(rv

v)

constant

(15)

Thus we have

Theorem

7.5.

If the resultant external torque acting

then the total angular

momentum remains

constant,

i.e.

on a system of particles

is

is zero,

conserved.

This theorem

is often called the principle of conservation of angular


generalization of Theorem 2.9, page 37.

momentum.

It is the

KINETIC ENERGY OF A SYSTEM OF PARTICLES


The

total kinetic

energy of a system of particles is defined as


\ N
j N
T = Jm,t)J =

sXm,^

(16)

WORK
in

If Tv is the force (external and internal) acting on particle v, then the total work done
moving the system of particles from one state [symbolized by 1] to another [symbolized

by

2] is

W12

v=l

yWr

*s

(17)

CHAP.

As

SYSTEMS OF PARTICLES

7]

in the case of a single particle,

Theorem
where the

7.6.

The

total

kinetic energy

is

we can prove

169

the following

work done in moving a system of particles


2\ to another where the kinetic energy is T2 ,
W12

= T 2 -T,

from one

state

is

(is)

POTENTIAL ENERGY. CONSERVATION OF ENERGY


When
energy

external and internal, are conservative, we can define a total potential


of the system. In such case we can prove the following

all forces,

Theorem 7.7. If T and V are respectively the


energy of a system of particles, then

T+V =
This

is

total kinetic

energy and

constant

total potential

(19)

the principle of conservation of energy for systems of particles.

MOTION RELATIVE TO THE CENTER OF MASS


It is often useful to describe the motion of a system of particles about [or relative to]
the center of mass. The following theorems are of fundamental importance. In all cases
primes denote quantities relative to the center of mass.

of

Theorem 7.8. The total


mass is zero. In symbols,

linear

momentum

N
Y,

v=X

of a system of particles about the center

mv<

^m

rv

(20)

v=l

Theorem 7.9. The total angular momentum of a system of particles about any point
equals the angular momentum of the total mass assumed to be located at the center of mass
plus the angular momentum about the center of mass. In symbols,

2m

Mv +

v=l

v (r'
v

v'v )

(21)

Theorem 7.10. The total kinetic energy of a system of particles about any point O
equals the kinetic energy of translation of the center of mass [assuming the total mass
located there] plus the kinetic energy of motion about the center of mass. In symbols,

|5>X

= ^Mv2 +

(22)

Theorem 7.11. The total external torque about the center of mass equals the time rate
of change in angular momentum about the center of mass, i.e. equation (lb) holds not
only for inertial coordinate systems but also for coordinate systems moving with the
center of mass. In symbols,

da'

-W

If motion is described relative to points other than the center of mass, the results
in the

above theorems become more complicated.

IMPULSE
If

is

the total external force acting on a system of particles, then

SYSTEMS OF PARTICLES

170

[CHAP.

dt

(U)

St.
h

is called

the total linear impulse or briefly total impulse.

As

in the case of one particle,

we can prove
Theorem

7.12.

Similarly

if

The
is

total linear

impulse

is

momentum.

equal to the change in linear

the total external torque applied to a system of particles about origin O,

then

t2

A eft

is called

the total angular impulse.

Theorem

7.13.

The

total

We

(25)

can then prove

angular impulse

is

equal to the change in angular

momentum.

CONSTRAINTS. HOLONOMIC AND NON-HOLONOMIC CONSTRAINTS


Often in practice the motion of a particle or system of particles is restricted in some
way. For example, in rigid bodies [considered in Chapters 9 and 10] the motion must be such
that the distance between any two particular particles of the rigid body is always the same.
As another example, the motion of particles may be restricted to curves or surfaces.

The

limitations on the motion are often called constraints.

If the constraint condition

can be expressed as an equation


<(ri,r 2 ,

...,rw

t)

(26)

connecting the position vectors of the particles and the time, then the constraint
holonomic. If it cannot be so expressed it is called non-holonomic.

is

called

VIRTUAL DISPLACEMENTS
Consider two possible configurations of a system of particles at a particular instant
which are consistent with the forces and constraints. To go from one configuration to
the other, we need only give the vth particle a displacement Sr* from the old to the new
position. We call Sr, a virtual displacement to distinguish it from a true displacement
[denoted by dr v which occurs in a time interval where forces and constraints could be
changing. The symbol 8 has the usual properties of the differential d; for example,
S(sin0) = cos0 80.
]

STATICS OF A SYSTEM OF PARTICLES.


PRINCIPLE OF VIRTUAL WORK
In order for a system of particles to be in equilibrium, the resultant force acting on each
where F v 8r v is called
particle must be zero, i.e. F = 0. It thus follows that F, 8r v =
then
have
the virtual work. By adding these we

2Fv8r =

(27)

v=l

If constraints are present,

then

we can
F

write

= F a) +
l

F< c)

(28)

where F< a) and F< c) are respectively the actual force and constraint force acting on the vth
particle. By assuming that the virtual work of the constraint forces is zero [which is true
for rigid bodies and for motion on curves and surfaces without friction], we arrive at

CHAP.

SYSTEMS OF PARTICLES

7]

Theorem 7.14.
work of the actual

system of particles

forces

is

zero,

in equilibrium if

is

171

and only

if

the total virtual

if

i.e.

F< 0)

8r

(29)

v=l

This

is

often called the principle of virtual work.

EQUILIBRIUM IN CONSERVATIVE FIELDS.


STABILITY OF EQUILIBRIUM
The results for equilibrium of a particle in a conservative force field [see page 38]
can be generalized to systems of particles. The following theorems summarize the basic
results.

Theorem

If

7.15.

coordinates q v q 2

V is the total potential of a system of particles depending on


then the system will be in equilibrium if

W^
Since the virtual

work done on the system

"

".

(31) is equivalent to the principle of virtual

Theorem
is

7.16.

8o2

'

system of particles

>

()

is

".

work.
be in stable equilibrium

will

if

the potential

a minimum.
In case

depends on only one coordinate, say q


v

dV

sufficient conditions are

dW

Other cases of equilibrium where the potential

not a

is

minimum

are called unstable.

D'ALEMBERT'S PRINCIPLE
Although Theorem 7.14 as stated applies to the statics of a system of particles, it can be
restated so as to give an analogous theorem for dynamics. To do this we note that
according
to Newton's second law of motion,
F

or

(so)

where p v is the momentum of the vth particle. The second equation amounts to saying
that a moving system of particles can be considered to be in equilibrium under
a force
F - p v i.e. the actual force together with the added force -p which is
often called the
reversed effective force on particle v. By using the principle of virtual work we
can then
arrive at
,

Theorem

7.17.

system of particles moves in such a way that the

2 (F^
With

this theorem,

which

as a special case of statics.

is

a)

p.)

8r

total virtual

work

often called D'Alembert's principle,

we can

consider dynamics

SYSTEMS OF PARTICLES

172

[CHAP.

Solved Problems

DEGREES OF FREEDOM
7.1.

Determine the number of degrees of freedom


(a) a particle moving on a given space curve; (b)
plane;

(a)

five particles

(c)

rigid rod

moving

moving freely

in space;

in

each of the following cases:


moving freely in a
two particles connected by a

five particles
(d)

freely in a plane.

The curve can be described by the parametric equations x = x(s), y = y(s), z = (s) where
s is the parameter. Then the position of a particle on the curve is determined by specifying
one coordinate, and hence there is one degree of freedom.
particle requires two coordinates to specify its position in the plane. Thus 5 2 = 10
coordinates are needed to specify the positions of all 5 particles, i.e. the system has 10 degrees
of freedom.

(b)

Each

(c)

Since each particle requires three coordinates to specify


degrees of freedom.

(d)

Method 1.
The coordinates of the two particles can be expressed by {x lf y{) and (x 2 ,y 2 ),
of 4 coordinates. However, since the distance between these points is a constant a
of the rigid rod],

we have

(x 1

x2 2 +
)

(l/i

2/2) 2 =

Thus there are 4

expressed in terms of the others.

ft2

its position,

5*3 = 15

the system has

i.e.

a total

[the length

so that one of the coordinates can be

1 =

degrees of freedom.

Method 2.
The motion is completely specified if we give the two coordinates of the center of mass
and the angle made by the rod with some specified direction. Thus there are 2 + 1 = 3 degrees
of freedom.

72.

Find the number of degrees of freedom for a rigid body which (a) can move freely
in three dimensional space, (b) has one point fixed but can move in space about
this point.
(a)

Method

1.

If 3 non-collinear points of a rigid

body are fixed

of

9.

(x x

in space, then the rigid

Let these points have coordinates (x lt y x zj, {x 2 y 2


Since the body is rigid we must have the relations

in space.

- x2 2 +

(y x

- y2 2 +
)

- z2
(x 3 - x

(z t

2
)

=
+

constant,

z 2 ), (x 3 ,

y3

- x 5 2 + (y 2 - y 3 2 +
+ (z 3 - z 2 = constant
(x 2

body

is

also fixed

z 3 ) respectively,

{z 2

- z3

2
)

a total

constant,

(y 3 - y x )
t)
x)
hence 3 coordinates can be expressed in terms of the remaining 6. Thus 6 independent
coordinates are needed to describe the motion, i.e. there are 6 degrees of freedom.

Method
To

2.

An axis through this point


rotation about the axis
specify 2 ratios of the direction cosines of this axis.
can then be described by 1 angular coordinate. The total number of coordinates required,
3 + 2 + 1 = 6.
i.e. the number of degrees of freedom, is
is fixed if

(6)

one point of the rigid body requires 3 coordinates.

fix

we

The motion is completely specified if we know the coordinates of two points, say (x^y^Zj)
and (x 2 ,y 2 ,z 2 ), where the fixed point is taken at the origin of a coordinate system. But since
the body is rigid we must have
x\

+ y\ +

z\

from which

constant, x\

3 coordinates

+ yl + z\=

constant,

(x x

- x 2 2 + (y - y 2 2 +
)

can be found in terms of the remaining

3.

{z t

- z2 2 =
)

constant

Thus there are 3 degrees

of freedom.

CENTER OF MASS AND MOMENTUM OF A SYSTEM OF PARTICLES


7.3.

Prove Theorem 7.1, page 167: The total momentum of a system of particles can be
of the system by the velocity v of the center
found by multiplying the total mass

of mass.

CHAP.

SYSTEMS OF PARTICLES

7]

The center of mass

Then the

7.4.

total

is

by

momentum

2 rn v
^
v

2 i^v v 2

is

definition,

173

,,

M dr/dt

= Mx.

Prove Theorem
as

if

7.2, page 167: The center of mass of a system of particles moves


the total mass and resultant external force were applied at this point.

Let F be the resultant external force acting on particle


v due to particle \.
We shall assume that f = 0,
force on itself.

while

particle

By Newton's

second law the total force on particle

F" +
where the second term on the

dPy
f

the internal force on


does not exert any

is

J2

~dt

f x is

particle

i.e.

dP>

(w

r ">

left represents the resultant internal force

on particle

due to

all

other particles.

Summing

over

2F
Now

we

in equation (1),

2 2if.

find
tv \

m
d&

2m

rv I

according to Newton's third law of action and reaction,


left of {2) is zero. If we then write

(2)

f x

so

Xl>

that the double

summation on the

2 F

and

^^ m

1Y1

(2)

becomes

Since

is

the total external force on

all

rv

(3)

d2 r

= M-r^

(4)

particles applied at the center of

mass

f,

the required

result is proved.

7.5.

system of particles consists of a 3 gram mass located at (1, 0, -1), a 5 gram mass
1, 3) and a 2 gram mass at (3, -1, 1). Find the coordinates of the center of mass.

at (-2,

The

position vectors of the particles are given respectively

rt

Then the center


f

of

mass

3(i

is

k,

r2

7.6.

r3

3k,

3i

+k

- k) +

5(-2i

of the center of

+ j + 3k) +
+5+2

2(3i

- j + k)

10

+
,

3
10

+
,

7
5

mass are (J*.


A. 1)
1 0' 1
o
v

'

if the total momentum of a system is constant, i.e. is conserved, then the


center of mass is either at rest or in motion with constant velocity.

Prove that
The

total

momentum

of the system

is

given by

Then

7.7.

given by

Thus the coordinates

= -2i +

by

if

is

constant, so also

is

..

r2^r]

rff

dr/dt, the velocity of the center of mass.

why the ejection of gases at high velocity from the rear of a rocket will
the rocket forward.

Explain

move

Since the gas particles move backward with high velocity and since the center of mass does
not move, the rocket must move forward. For applications involving rocket motion, see Chapter 8.

SYSTEMS OF PARTICLES

174

7.8.

Find the centroid of a

solid region

Consider the volume element At v of the


this volume element is

[CHAP.

as in Fig. 7-3.
solid.

The
Ar

mass of

AM V =

o v At v

Ax v Ayv &z v

Ax v Ay v Az v

av

where a v is the density [mass per unit volume] and


Ax v Ay v Az are the dimensions of the volume element.
Then the centroid is given approximately by
,

2 r AM
2 AM

2r
2

<r

where the summation

2 * a Ax Ay Az
2 v ^ xv ^Vv ^ z

av Arv

is

At v

taken over

all

volume elements

Fig. 7-3

of the solid.

Ar

Taking the limit as the number of volume elements becomes infinite


or Ax v - 0, Ay v - 0, Az v - 0, we obtain for the centroid of the solid:

in

such a

way

that

dM

JJJ radxdydz

radr

dx dy dz
9.

where the integration


Writing

xi

is to

+ yj +

III

be performed Over %, as indicated.


f

zk,

xi

+ yj + zk,

can also be written in component form as

\\\ yadxdydz

x a dx dy dz

Find the centroid of the region bounded by the plane x


x = 0, y = 0, z = 0.
The

region, which

the results of Problem

is

a tetrahedron,

is

indicated in Fig. 7-4.

jJj zadxdydz
%

sss- dx dy dz

Jlf* dx dy dz

III adxdy dz

7.9.

this

+y+z =
To

a and the planes

find the centroid,

we use

7.8.

an
In forming the sum over all volume elements of the region, it is advisable to proceed in
orderly fashion. One possibility is to add first all terms corresponding to volume elements
and
contained in a column such as PQ in the figure. This amounts to keeping x v and y v fixed
adding over all z v Next keep * fixed but sum over all y v This amounts to adding all columns,
such as PQ, contained in a slab RS, and consequently amounts to summing over all cubes contained
This amounts to addition of all slabs such as RS.
in such a slab. Finally, vary x v
.

In performing the integration over %, we use these


same ideas. Thus keeping x and y constant, integrate
[base of column PQ] to z a x y [top
from z =
of column PQ]. Next keep x constant and integrate
with respect to y. This amounts to addition of columns
having bases in the xy plane [z = 0] located anywhere
from R [where y = 0] to S [where x + y = a or y =

At

= Axv Ayv Az

to y = a x.
and the integration is from y =
we add all slabs parallel to the yz plane, which
We
to x a.
amounts to integration from x

x],

Finally,

thus obtain

pa x
Jr>a
t

x=

pax y
a(xi

*A/

=
na

+ yj + zk) dz dy dx

*z =

paa x

s* xi
/*a

a dz dy dx
K

'x=Q

*^j/

Fig. 7-4

*^=0

Since a is constant in this case, it may be cancelled. The denominator without a


4
Thus the center
be a3 /6, and the numerator without a is (a /24)(i + j + k).
=
a/4.
=
=
a/4, z
a/4, y
f = (a/4)(i + j + k) or x

to

is

of

evaluated

mass

is

CHAP.

7.10.

SYSTEMS OF PARTICLES

7]

Find the centroid of a semi-circular region of radius


Method

Using rectangular coordinates.


Choose the region as in Fig. 7-5. The equation of the
since y ^ 0.

175

a.

1.

If a is the mass per unit area, assumed


are given by

constant,

JJ xdydx

x2

+ y2 =

a2 or y

y/a2

- x2

ss

Jtf^

=
dydx

ydydx
x

odA

xdy dx

ydydx

/.

is

then the coordinates of the centroid

ff dydx

<rdA

y/l^x*

xadA

circle

= a

dy dx

J-._ a

3/=0

V^? dydx

2a3 /3

4a

ira2 /2

3lT

"y=0

Note that we can write x immediately, since by symmetry the centroid is on the y axis.
The denominator for y can be evaluated without integrating by noting that it represents
the
semi-circular area which is %ira2
.

Fig. 7-5

Fig. 7-6

Method 2. Using polar coordinates.


The equation of the circle is r = a [see Fig. 7-6]. As before, we see by
symmetry that the
centroid must lie on the y axis, so that x = 0. Since
y = r sin e and dA=rdrde in polar

we can

coordinates,

write

yadA

(r sin 9 )

r dr de
2q3/3

r=0

dA

s%
7.11.

Sir

rdrde

Find the center of mass of a uniform solid hemisphere of radius


By symmetry the center of mass lies on the
z axis

[see Fig.

Aa

a.

Subdivide the hemisphere


of radius r, such as
ABCDEA. If the center G of such a ring is at
distance z from the center O of the hemisphere,
r 2 + z 2 = a2
Then if dz is the thickness of the
plate, the volume of each ring is
into

solid

7-7].

circular

plates

irr 2

and the mass

is

dz

ira(a 2

ir{a 2

z2

traz{a2

-z 2 )dz
Thus we have

dz.

z2

dz

z=

= la

Jz=0

va(a2

z2

dz

Fig. 7-7

SYSTEMS OF PARTICLES

176

[CHAP.

ANGULAR MOMENTUM AND TORQUE


7.12.

Prove Theorem 7.4, page 168: The total external torque on a system of particles is
equal to the time rate of change of angular momentum of the system, provided that
the internal forces between particles are central forces.
As

Problem

in equation (1) of

+ 2fx
x

f
Multiplying both sides of

(1

we have

7.4,

by r v X

dP v
-of

2 r X

(2)

Summing

over

we

in (4),

2 2rXf
vX
X

sum

in (5) is

which becomes on writing

X|/

since
(7)

is

CtC

2m

i
^

A = 2

*i/

XU

v)

(5)

+ *xXf x

rx

Xf x

i\^m
^
dt \f

XF

()

- r x X f vX

(r

"" v (r v

(7)

same direction as r rx it follows


Thus equation (5) becomes

Xv*"v )\
"

A =

or
"

d
dt

2 Wj/(r X v).

O =

F,

that the forces are central, i.e. f x has the


zero and also that the double sum in (5) is zero.

v (t v

j;

we suppose

2r
where

j-

according to Newton's third law,

-f x

rXf x

Then

(4)

composed of terms such as


rv

that

l>

l>

the double

(3)

v )}

^K(r,Xv,)}

f vX

(*)

find

2r,XF, +
Now

{1)

X ^(m v)

vv

d
^K(r,Xv

2 * X

r,XF, +

becomes

f x

X ^(mv)

Vv)

we have

r,XF v +
Since

d
{m "
Tt

WORK, KINETIC ENERGY AND POTENTIAL ENERGY


7.13.

page 169: The total work done in moving a system of particles


with kinetic energies 2\ and T 2 respectively is T2-T1.
another
from one state to

Prove Theorem

7.6,

pth particle in the system is

The equation of motion of the

Tv
Taking the dot product of both

F rv

tv \

Id,
Since

(2)

-^ (ra

'

>-.

r)

f i>x

over

in equation (5),

2^-r,

we

(1)

2 5t

2di

(2)

Id/

Id/{m

Summing

can be written

Tv'K

^(m,r,)

we have

sides with r,
==

Tv'K

2U

2\

^
2\

(3)

find

2F.r +

22U-r,

^(2,^)

U)

CHAP.

SYSTEMS OF PARTICLES

7]

Integrating both sides of

12

with respect to

(4)

2 ('rv 'r

<Zt

to

tx

( F v -l v dt +

ti

Using the fact that

dt

from

177

we

t2 ,

22

find
2

(
l

and the symbols

dr

t vX

-r v dt

and 2 for the states at times

and

tx

t2

respectively, this can be written

2 Cr

12

where Ti and T^ are the

'dr v

total kinetic energies at

12

22

( F v 'drv +

and

tx

fx

'

(5)

Since

respectively.

t2

T2 - T x

f fv 'dr v

(6)

the total work done (by external and internal forces) in


another, the required result follows.

moving the system from one

is

state to

It should be noted that the double sum in (5) indicating work done by the internal forces,
cannot be reduced to zero either by using Newton's third law or the assumption of central forces.
This is in contradistinction to the double sums in Problems 7.4 and 7.12 which can be reduced to zero.

7.14.

Suppose that the internal forces of a system of particles are conservative and are
derived from a potential

= V

Vxv (rw)

where
ticles

(a)

n = rv \ = V( x * ~ x ) 2 +
A and v of the system.

Prove that

Jf^-dr, = -

particle

(#*.

due to particle

Evaluate the double sum

22

(a)

The force acting on

BV Xv
hv

particle X
^

i vX

'drv

where

the distance between par-

f x is

the internal force on

of Problem 7.13.

f\v'dr v

dVXv

dVXv

in

~ ~J^ k

~S^d x VXv

-V x VXv

-ivX

producing the displacements drv and drx of particles

(2)

and X

is

i Xv

'drx

=
=

Then the

is

is

dVXv

The work done by these forces


respectively

2
)

Wxv.

~~B^ ~ ~d^

2 2 dVw

- zv

is

Wxv.
The force acting on

(z\

(r v \)

A.

(b)

particle

Vv) 2 +

v\

total

JdVXv

vKv

dVXv

dV Xv

dVXv

dVXv

~dVXv

work done by the internal forces

22fx-rfr

l,

is

-22<n\,
*
v

(s)

the factor on the right being introduced because otherwise the terms in the summation
would enter twice.

-]

SYSTEMS OF PARTICLES

178

(6)

By

integrating

(a),

we have

t Xy

dr v

of part

(3)

[CHAP.

-2

2 2)
lnt)
and
where y<

y2 mt)

22

--

J22v
t

7.15.

^ and

-r/(int)

U)

K2

denote the total internal potentials

2
at times

1/(int)

C*Vx

(5)

respectively.

Prove that if both the external and internal forces for a system of particles are
conservative, then the principle of conservation of energy is valid.
we have

If the external forces are conservative, then

- -VV

2 (\-dr

from which

where V[ ext) and y< ext) denote the

-2 fV,

(1)

T/(ext)
K2

v?

total external potential

2V
V

at times

tt

Using

and

respectively.

and equation

{2)

t2 - t x
where

of Problem 7.14(6) in equation

(4)

y{

v<

ext)

(int)

V (ext) +

Vi

ext)

v<

Int)

and

vj

Vo

y^

ext)

v - y2

(5),

+ y2(int)

Tt +
which

is

T2 + y 2

T + V

or

tx

and

find
(*)

are the respective total potential energies [external and internal] at times

from

we

of Problem 7.13,

(5)

y< int)

U)

We

t%.

constant

thus find
(5)

the principle of conservation of energy.

MOTION RELATIVE TO THE CENTER OF MASS


7.16.

be respectively the position vector and velocity of particle


\' = 0.
(b)
v
v r'v = 0,
the center of mass. Prove that (a)
v

Let

r^

and

\'

Let r^ be the position vector of particle v relative to O


and f the position vector of the center of mass C relative
Then from the definition of the center of mass,
to O.

where

2 mv

From
rv

Then substituting

(2)

^^m

into

'

(1),

Differentiating both sides of

rv

Fig. 7-8
v

we

(1)

we have

(3)

Fig. 7-8

(2)

find

2v m

from which

(b)

relative to

^m

^m
V

(a)

rv

with respect to

~
t,

(S)

we have

2 m* x =
v

CHAP.

7.17.

SYSTEMS OF PARTICLES

7]

Prove Theorem 7.9, page 169: The total angular momentum of a system of particles
about any point O equals the angular momentum of the total mass assumed to be
located at the center of mass plus the angular momentum about the center of mass.
Let r be the position vector of particle

mass C

O and

relative to

r'

relative to O, f the position vector of the center of

the position vector of particle

Differentiating with respect to

t,

we

find

i'

angular

total

a =

momentum

2m

v (r'v

(3)

(2)

to O, v is the velocity of particle v relative to

is

+ v)}

2 m,(r' X v) + 2 m (t X V

Xvp +

v)

2 m {l X v)

(3)

7.16,

K(<Xv)

J2^v<fXv

2 m Xv)

2 rn

\'V

2m

<

2 H, Mr X v)

v {f

Then

by Problem

2^{ + f)X( V

Now

\'

of the system about

Sw.fr.Xv,,)

tt)

where v is the velocity of the center of mass relative


O, and v'v is the velocity of particle v relative to C.

The

Then

relative to C.

<+*

*v

v (t

v)

-j

M{i X

v)

becomes, as required,

7.18.

179

2m(<Xv') + Af(fXv)

Prove Theorem 7.10, page 169: The total kinetic energy of a system of particles
about any point O equals the kinetic energy of the center of mass [assuming the
total mass located there] plus the kinetic energy of motion about the center of mass.
The

kinetic energy relative to

[see Fig. 7-8] is

Using equation

(2)

of Problem 7.16

we

(1)

can be written

^2^{(v + v;).(v + V ;)}

2 >

'

vv

2^v-v
*

since

find

rv

Thus

+ 2v'< +
v

h^^K'K
&
v

|(?^)v- + v.jsm.v;} + 2*^;,2

gMv2 + -

by Problem

7.16.

2 mv'

SYSTEMS OF PARTICLES

180

[CHAP.

IMPULSE
7.19.

Prove Theorem
momentum.
The

The

7.12:

total external force

by equation
_,

F
Then the

of

(4)

Problem

ifdH
M

is

-di?

equal to the change in linear

7.4 is

,.dv
lt

M*2

- M\_

total linear impulse is

CH

Ydt

X*
where p x

impulse

total linear

= Mx

and p2

Mv 2

dv
M ~^
dt

represent the total

momenta

at times

p2
tx

Pi

and

t2

respectively.

CONSTRAINTS. HOLONOMIC AND NON-HOLONOMIC CONSTRAINTS


7.20.

In each of the following cases state whether the constraint is holonomic or nonholonomic and give a reason for your answer: (a) a bead moving on a circular wire;
(b) a particle sliding down an inclined plane under the influence of gravity; (c) a
particle sliding down a sphere from a point near the top under the influence of gravity.
(a)

(6)

The constraint is holonomic since the bead, which can be considered a


to move on the circular wire.
The constraint

is

holonomic since the particle

is

constrained to

particle, is constrained

move along a surface which

is

in this case a plane.


(c)

the
is non-holonomic since the particle after reaching a certain location on
sphere will leave the sphere.
Another way of seeing this is to note that if r is the position vector of the particle
relative to the center of the sphere as origin and a is the radius of the sphere, then the
particle moves so that r2 a2 . This is a non-holonomic constraint since it is not of the
2
2
form (26), page 170. An example of a holonomic constraint would be r = a

The constraint

STATICS. PRINCIPLE OF VIRTUAL WORK. STABILITY


7.21.

Prove the principle of virtual work, Theorem 7.14, page 171.


For equilibrium, the net resultant force F v on each particle must
2F'8r

be zero, so that

(-0

But since F

= F (a) + F (c) where F^a)

pth particle, (1)

and F< c) are the actual and constraint forces acting on the

can be written

2Fa) -8r + 2Ftc), 8'v


If
(2)

we assume
is zero,

that the virtual

so that

work

2F

7.22.

is

Co)

dr v

(2)

of the constraint forces is zero, the second

we have

which

sum on

the left of

W
/

a\

the principle of virtual work.

are located on a frictionless double incline and


7-9
connected by an inextensible massless string passing over a smooth peg [see Fig.
have
must
we
equilibrium
for
that
show
to
work
below]. Use the principle of virtual

Two

particles of

masses mi and

sin a x

sin a2

where a x and

a 2 are the angles of the incline.

m2
mi

CHAP.

SYSTEMS OF PARTICLES

7]

Method

181

1.

Let r x and r2 be the respective position vectors


of masses
1 and
2 relative to O.

The actual forces (due


and m2 are respectively

= m xS

F< a)

to gravity) acting on

= m 2g

F< a)

A,

^\B

00
Wlj

S\

According to the principle of virtual work,

2Fj 0)
or

F'<

0)

Sr x

-rv

+ F 2a)

Sr2

Fig. 7-9

(*)

where 5r x and Sr2 are the virtual displacements of


Sr x

wi 2 g

8r t sin a x

m 2g

mg

or

Then

since the string is inextensible,

8r t

i.e.

(mrf sin a x

But

we must have

since 8r x is arbitrary,

Method

ai

and

Sr2

m2

m^ sin a )S^i

8r2

_ m2

sm a 2

wh

(1)

in (2),

(4)

Sri,

(4)

becomes

m2g

sin ax

Using

or

sin x

incline.

(3)

8r2 sin a 2

+ 8r2 =

down the

sin a 2

0>

i-e>

(5)

2.

When

it is not clear which forces are constraint forces doing no work, we can take into account
forces and then apply the principle of virtual work. Thus, for example, taking into account
the reaction forces R x and R2 due to the inclines on the particles and the tension forces T and T ,
x
2
the principle of virtual work becomes

all

(m x g + T x

+ Rj)

8r x

(m 2g + T2

+ R2

Sr2

<*)

Now since the inclines are assumed smooth [so that


the reaction forces are perpendicular to the inclines]

we have

Ri'Srj

R2 *8r2 =

0,

(7)

Also, since there is no friction at the peg, the tensions Tj and T2 have the same magnitude.
Thus
we have, using the fact that 8r x and Sr2 are directed

down the corresponding


8r2 = -8r lt

Ta

8r x

+ T2

and the fact that

= -T 8r - T2 8r2
= {T2 -T )8r =

Sr2

inclines

since

T x = T2

Then using

mg
x

as obtained in

7.23.

5r x

m2 g

(7)

Sr2

and

(5), (6)

(8)

becomes

=
Fig. 7-10

(3).

Use Theorem

7.15,

page 171, to solve Problem

7.22.

Let the string have length I and suppose that the lengths of string OA and OB on the inclines
[Fig. 7-9] are x and l x respectively.
The total potential energy using a horizontal plane
through O as reference level is

V =

-m x gx

sin a t

m2g(l x)

sin a 2

Then for equilibrium we must have

It should

as

is

= m

be noted that

also evident physically.

sin a x

is

+ m^g sm a 2 =

not a

minimum

sinaj

m2

sin a 2

in this case so that the equilibrium is not stable,

SYSTEMS OF PARTICLES

182

[CHAP.

D'ALEMBERTS PRINCIPLE
7.24.

Use D'Alembert's principle

We

(m x g-

This can be written

or

since the string

Sr2

= Srj,

r2

..
x

r x )8r x

= rV Thus

ra2 r2 )

+ r%

m^

m2

m^ sin a
m

sin a
m +
x

up or down

We

goes

down

up the

or

0,

with

incline

acceleration according as
in these cases goes

constant

sin a x < m^g sin a 2 respectively.


sin a 2 or
xg
respectively with the same constant acceleration.

> m<$

8r x

m2 r

sina 2

flr

we have

constant,

becomes, after dividing by

(2)

+ V2 =

rx

0,

)Sr2

'r 2

to
(1)

#>

(2)

8r2

TOjjf

Thus particle

m2

sina 2

7.22.

Problem 7.22

Sr a

n =

m g sin a

in equation (3) of

..

8r x

sinaj

wijflf

m2 V

and

rx

(ra 2

rx

inextensible so that

is

+ (m2g -

r x )'8r x

sin**!

(TOjfli

Now

introduce the reversed effective forces

obtain

motion of the masses in Problem

to describe the

Particle 2

can also use a method analogous to the second method of Problem

7.22.

MISCELLANEOUS PROBLEMS
7.25.

masses mi and m 2 move


so that their relative velocity is v and the veloc-

Two

particles having

If M = m\ + w 2
= miw2/(wi + ra 2 is the

mass is v.

ity of their center of

the total mass and


)
reduced mass of the system, prove that the total
2
2
kinetic energy is $Mv + $ixv
is

ju,

Let r x r2 and f be the position vectors with re,

spect to

mass C

of

mass

x,

mass

m2

and the center of

respectively.

From

the definition of the center of mass,

we have

m 1T + m2r2
and
t =
r
m + m2
v = f
m \ + m 2\ 2 =

i + 2
i

or using v x

ix

v2

r2 ,

If the velocity of

relative to

m2 is

v,

vi

so that

Solving

(1)

and

(2)

simultaneously,

we

h -

- v2 =

mv

total kinetic

energy

vx

v2
(*)

_ _v _ m v
Wi + m2

v2

is

l^ivf +
1

f2

Then the

K1 )

find

= V + , + ,'

vi

+ m2)v

then

= ^(r!-r2

(m x

m2V 2

!<-+*

mv
w
2

\\ 2

m
v
'"''

/_

l
1

^1%
+1^...

_
=

w
a^o

+
,

1
-/iV 2

CHAP.

7.26.

SYSTEMS OF PARTICLES

7]

183

Find the centroid of a uniform semicircular


wire of radius a.
By symmetry [see Fig. 7-12] the centroid of
the wire must be on the y axis, so that x = 0. If ois the mass per unit length of the wire, then if ds
represents an element of arc, we have ds = ado
so that

y a ds

(a sin e)(a de)

Jcr ads

f-

2a?

de

2a
Fig. 7-12

n-a

7.27.

Suppose that n systems of particles be given having centroids at f i, f 2


f and
masses
x
2 ... .,M n respectively.
Prove that the centroid of all the systems

total

Mif +
l

Mi +
M +

Let system 1 be composed of masses


system 2 be composed of masses m 21

m n m 12
m22
,

+ Mn rn
+ Mn

located at r n r 12
located at r21 , r 22 , ....
.

i
all

systems

is

inn2

respectively.

Then by

m21 r21 + w22r22 +

ra u r u

^12*12

Similarly

definition,

Mi

m21 + m22 +
mnl rnl + mn2rn2 +

But the centroid for

m n r n + m vFn +
m n + m 12 +
21*21 + w 22r22 +
*2

Mi +
let

at

is

M
'

mn2fn2 +

^nl^nl

located at

+ m 12r 12 +) + (m21 r21 + m 22r22 +


+
+ (mnl rMl + mn2rn2 +
(w u + m 12 +
) + (w 21 + m 22 + ) +
+ (mnl + mn2 H
Mjt + M2r2 +
+ Mnrn
M + M2 +
+ Af
(m n r u

7.28.

Find the centroid of a

solid of constant density


consisting of a cylinder of radius a and height

surmounted by a hemisphere of radius a

Af,

[see
'

Fig. 7-13].
Let f be the distance of the centroid of the solid from the
The centroid of the hemisphere of radius a is at distance fa +
from the base of the solid, and its mass is
= |^a 3 r [see Problem 7.11].
x

*-~.

is

Then by Problem

/"

2
I

ira 2 Ho.

''iliiip'

'

M
is

*r

>s>s>

base.

The centroid of the cylinder of radius a and height H


at distance H from the base of the solid and its mass

"V

M2

7.27,

+ ff) + {TraWo){%H)
~
+ 7ro2Hff
+ 8aH + 6m
8a + 12H

(|,ra3q)(fa

|ira3 a

32

^-

Fig. 7-13

Base of

s olid

SYSTEMS OF PARTICLES

184

7.29.

[CHAP.

is cut out of a circular region of radius


Find the centroid of the shaded region thus obtained.

circular hole of radius a/2

in Fig. 7-14.

a,

as

shown

y %ira2o

'x
x a

Fig. 7-15

Fig. 7-14

By symmetry

the centroid is located on the x axis, so that

0.

TraPo concentrated at its


can replace the circular region of radius a by the mass
1
centroid x x = a [Fig. 7-15]. Similarly, we can replace the circular hole of radius a/2 by the
= \ira% o concentrated at its centroid x 2 = fa. Then the centroid is located
negative mass
2

We

on the x axis at

Mx

+
Mi +
l

7.30.

Mx
M

(Tra 2 a)(a)

2 2

ira 2 a

(-%ira 2 a)(%a)

\ira% o

uniform rod PQ [see Fig. 7-16] of mass m and length L has its end P resting
against a smooth vertical wall AB while its other end Q is attached by means of an
on the wall. Assuming that
inextensible string OQ of length I to the fixed point
the plane of P, Q and O is vertical and perpendicular to the wall, show that

equilibrium occurs

if

sin a

V4L 2 -

V4L 2 -

sm/3

lV&
There is only one actual force, i.e. the weight rag of the
Other forces acting are the force of the wall on the
rod and the tension in the string. However, these are constraint forces and can do no work. This can be seen since
if P were to slide down the wall no work would be done,
because the wall is frictionless and thus the force due to
the wall on the rod is perpendicular to the wall. Also if Q
rod.

were

to drop,
string at Q.

it

could only

move perpendicular

to

the

Let r be the position vector of the center of mass C


case also the center of gravity] relative to 0. Also
let i and j be unit vectors in the horizontal and vertical
directions respectively so that r = xi + yj.
[in this

From

Fig. 7-16,

Then from

(1),

OQ = OP + PQ

(1)

OQ = OC + CQ

(2)

on taking the dot product with

OQi
Since

or

OP

Fig. 7-16

0, this

= OP'i + PQ'i

reduces to

OQ'i
I

i,

sin a

= PQ'i
= L

sin

/?

CHAP.

SYSTEMS OF PARTICLES

7]

Similarly on taking the dot product of both sides of

Now

cos a

mg

and

(S)

(A),

and

Since

8y

COS a 8a

sin a 8a

are constants and since

from

(7),

and

we have on

(P),

work becomes

sin

cos

iin

So;

=
=
_
~

fi

Lcos/JS/3

(5)

lLsin/?S/?

(9)

1 sin/?

2 cos

(10)

(l/L) sin a

= Vl - (WL

fi

2
)

sin2

Dividing by sin a and squaring both sides,

we

sin/?

(11)

(U)

V4L2-

/2

(IS)

solid consists of a cylinder of radius -a

H on

a hemisphere of radius a, as indicated in Fig. 7-17. Prove that the solid is in stable
equilibrium on a horizontal plane if and only if
\/2.

C is at a distance
of the hemisphere given by

7.28 the centroid

from the center

3q2

+ SaH + 6ff2
8a + 12H

_
~

6ff2

8a

3a2

+ 12H

Then the distance of the centroid C above the plane

CP =

CD + DP = CB cos e + BQ
6W - 3a2 cos + a
8a + 12H

sin2 a

Z2

as required.

By Problem

(IS)
J2

find

Ly/Z

a/H >

(12)

sin a

iVs

uniform
and height

()

sin2 a

VL2 -

V4L2-

d.

can be written

Vl -

7.31.

S/3

division,

so that

and from

(7)

has the same properties as the differential operator

sin a

(3),

(10)

8y

(i

sin

Thus equation

(6)

= L cos SfS
= 8y \L sin

cos a

Now from

(5)

these equations become

(5)

8yj

or

ZcosaSa

From

given by

is

we have

(4)

Sr

mg 8y =

becomes

this

(5),

j,

cos /?

the only actual force, the principle of virtual

is

Now from

since

+ \L

8xi

mg
Using

a virtual displacement of the center of mass


Sr

Since

with

(2)

= OC'j + CQ'j

OQ'j
or

185

is

CB

SYSTEMS OF PARTICLES

186

energy

so that the potential

aW

do 2
i.e.

7.32.

3a2

3a2

2 -6H2\
-=
M# /3a

or

sin 9

0,

8a

- 6#2
+ 12H

3a2 - 6ff2 \
8a + 12H

M#

cos 9

0=0

0.

>

ends suspended

its

fixed points at the

same

.B

hori-

//T

Find an equation for the

curve in which

it

A*lf-

J^*r

rium under the tensions of magnitude T and


T + AT due to the rest of the chain and also

*T

the weight ag As of the element of chain. Now


from Fig. 7-18 if the directions of the vectors
corresponding to T and T + AT make angles
of 9 and 9 + A9 with the x axis respectively,
we have as the condition for equilibrium [neglecting terms of order (A9) 2 and higher],

+ AT)

cos

(9

hangs.

Let A and B [Fig. 7-18] be the fixed points.


element of the chain of length As is in equilib-

(T

i.e.

a/H > V2.

or

zontal level.

An

'3a2

M^

uniform chain has

from two

6ff2

will be stable if

,=o

- 6H 2 >

n*

dV
=

-,.v
Equilibrium
occurs where

Then the equilibrium

(or potential) is

V
tp

[CHAP.

+ A9)i +

+ AT)

(T

(T

or

(T

sin (9

+ AT)

+ AT)

cos

sin (9

Ae

agAs
,,

/,

J
i

Fig. 7-18

+ A9)j -

+ A9)

{9

(T cos

+ A9) - T

sin 9

+ T

sin 9

j)

ag] As

cos 9

(1)

(2)

ag As

Equation (1) shows that the horizontal component T cos 9 must be a constant, which we shall take
as T which corresponds to the tension at the lowest point of the chain, where 9 = 0. Thus

Tcos9

From

(2)

we

find on dividing

(T

= Tn

{3)

by A9,

+ AT)

sin(fl

+ Afl)

As

sin9

A9

Taking the limit of both sides of U) as A9


d
d9

Using

(?)

to eliminate T, (5)

-* 0,

(T sin

we

9)

find

9)

Tn

where

ag

do

Thus from

Tjag.
(7)

and

Now

ds
d9

ag

(5)

becomes

d
j- (T tan

or

tt)

A~9

dx

de

9,

(6)

b sec 2 9

sec 2 9

dy
COS

ds

T9

-f-

ds

(7)

= sm9n

(8)

(*),

dx

dx ds

d9

ds d9

dy

dy ds

de

ds d9

(cos 9)(b sec 2 9)

b sec 9

(sin 9){b sec 2 9)

b sec 9

(9)

tan

(10)

CHAP.

SYSTEMS OF PARTICLES

7]

Integrating

(9)

and

(10)

with respect to

=
=

05

we

$,

find

6 In (sec 6

c2

b sec $

tan

From

(11)

(15)

and

we

(12)

find

Cj

c2

0,

we have

(1)

by

sec 2 e

tan 2

b In (sec

b sec

(15),

we

find

sec e

Adding

(15) to (17")

and using

(1-4),

y
This curve

is

we

Cj

(11)
(12)

i.e.

at

0,

and

b.

Then

Thus

0.

sec 6

But

Then dividing

e)

Let us assume that at the lowest point of the chain,

from

187

tan

(IS)

e)

(14)

ff

tan

tan

tan 6

(sec

e* /b

tf)(sec

(15)

tan

e)

(10)

e~*/ b

(17)

find

|(ex/b

e -*/b)

called a catenary [from the Latin,

6 cosh

(15)

meaning chain]

Supplementary Problems
DEGREES OF FREEDOM
7.33.

7.34.

Determine the number of degrees of freedom in each of the following cases: (a) a particle moving
on a plane curve; (6) two particles moving on a space curve and having constant distance between
them; (c) three particles moving in space so that the distance between any two of them is
always constant.
Ans. (a) 1, (b) 1, (c) 6

Find the number of degrees of freedom for a rigid body which


(6) has two points fixed but can otherwise move freely.

plane,

7.35.

(a)

moves

Ans.

parallel to a fixed

(a) 3,

(6)

Find the number of degrees of freedom for a system consisting of a thin rigid rod which can
freely in space and a particle which is constrained to move on the rod.
Ans. 4

move

CENTER OF MASS AND MOMENTUM OF A SYSTEM OF PARTICLES


7.36.

7.37.

7.38.

7.39.

ABCD

quadrilateral
has masses 1, 2, 3 and 4 units located at its vertices A( 1,-2,2),
B(S, 2, -1), C(l, 2, 4) and D(3, 1, 2). Find the coordinates of the center of mass.
Ans. (2, 0, 2)

A system consists of two particles of masses x and m2 Prove that the center of mass of the
system divides the line joining Wj to m 2 into two segments whose lengths are in the ratio m? to Wj.
A bomb dropped from an airplane explodes in midair.
then the center of mass describes a parabola.

Prove that

if

air resistance is neglected,

respectively have position vectors r t = 5ti 2t2j + (St 2)k,


r3 = (2* - 1)1 + ( 2 + 2)j - tfk where t is the time.
Find (a) the velocity of the center of mass at time t = 1 and (6) the total linear momentum
of the system at t = 1.
Ans. (a) 3i 2j k, (6) 181 12j 6k

Three particles of masses


r2

(2t

- 3)i +

(12

2, 1, 3

- 5t2 )j +

(4

+ 6* - 3*3)k,

SYSTEMS OF PARTICLES

188

7.40.

7.41.

Three equal masses are located at the vertices of a triangle.


located at the intersection of the medians of the triangle.

A
y

Prove that the center of mass

is

uniform plate has the shape of the region bounded by the parabola
x 2 and the line y
in the xy plane. Find the center of mass.

Ans. x

7.42.

[CHAP.

0,

= ^H

Find the center of mass of a uniform right circular cone of radius a


and height H.
Ans. That point on the axis at distance i? from the vertex.

7.43.

The shaded region of Fig. 7-19 is a solid spherical cap of height


cut off from a uniform solid sphere of radius a. (a) Prove that the
centroid of the cap is located at a distance f (2a H) 2 /(Sa H) from

AB.

the base

7.44.

Discuss the cases

(b)

0,

a and

H=

Fig. 7-19

2a.

Find the center of mass of a uniform plate bounded by


= sin x and the x axis.
Ans. x tt/2, y ir/&

7.45.

Find the center of mass of a rod of length I whose denfrom one end O.

sity is proportional to the distance

Ans. |Z from end


7.46.

Find the centroid of a uniform solid bounded by the


planes Ax + 2y + z = 8, x 0, y = 0, 2 = 0.
Ans. f

7.47.

j^(i

+ 2j + 4k)

bounded by the paraboloid of revoluand the plane z = H [see Fig. 7-20].


Ans. x = 0, y = 0, z = %H
Find the centroid.
uniform

tion

x2

solid is

+ y2 =

cz

Fig. 7-20

ANGULAR MOMENTUM AND TORQUE


7.48.

2, 3 and 5 move under the influence of a force field so that their


2
position vectors relative to a fixed coordinate system are given respectively by r x = 2ti 3j + t k,
2
total
the
(a)
l)k where t is the time. Find
*j + (2t
4k and r3 = < i
r3 = (t + l)i + 3tj
angular momentum of the system and (b) the total external torque applied to the system, taken

Three particles of masses

with respect to the origin.

Ans.

(a)
(b)

7.49.

- 12t)i + (6t 2 - lOt - 12)j +


-12i + (12* - 10)j + lOtk
(31

Work Problem

(21

7.48 if the total angular

5* 2 )k

momentum and

torque are taken with respect to the

center of mass.

7.50.

7.51.

7.52.

7.53.

7.54.

Verify that in (a) Problem 7.48 and (6) Problem 7.49 the total external torque
time rate of change in angular momentum.

is

equal to the

In Problem 7.48 find (a) the total angular momentum and (b) the total external torque taken
about a point whose position vector is given by r = ti - 2tj + 3k. Does the total external torque
equal the time rate of change in angular momentum in this case? Explain.

Verify Theorem

7.9,

page 169, for the system of particles of Problem

State and prove a theorem analogous to that of Theorem


torque applied to a system.

Is the

angular

momentum

conserved in Problem 7.38?

7.9,

Explain.

7.48.

page 169, for the

total external

CHAP.

SYSTEMS OF PARTICLES

7]

189

WORK, ENERGY AND IMPULSE


7.55.

Find the total work done by the force


positions at time

7.56.

Is the

work

of Problem 7.55 the

to be concentrated there?

7.57.

7.59.

of Problem 7.48 in
t

moving the

(a)

72.5,

(6)

same as that done on the center of mass assuming

7.61.

Verify Theorem

mass

all

and

(b) t

2.

30.5

in

Problem 7.48 at times

Find the total impulse applied to the system of Problem 7.48 from
Ans. lOi + 4k
the connection of your result with Problem 7.58.
Prove Theorem

their

Explain.

Find the total linear momentum of the system of particles


Ans. (a) 17i + 4j + 14k, (6) 27i + 4j + 18k
(6) t = 2.

7.60.

from

particles

Ans. 42

2.

Find the total kinetic energy of the particles in Problem 7.48 at times (a) t
Discuss the connection between your results and the result of Problem 7.55.
Ans.

7.58.

field

to their positions at time

7.13,

to

(a) t

and

and discuss

page 170.

7.13,

page 170, for the system of particles

in

Problem

7.48.

CONSTRAINTS, STATICS, VIRTUAL WORK, STABILITY AND D'ALEMBERT'S PRINCIPLE


7.62.

In each case state whether the constraint is holonomic or non-holonomic and give a reason for
your answer: (a) a particle constrained to move under gravity on the inside of a vertical paraboloid
of revolution whose vertex is downward; (6) a particle sliding on an ellipsoid under the influence
of gravity; (c) a sphere rolling and possibly sliding down an inclined plane; (d) a sphere rolling
down an inclined plane parallel to a fixed vertical plane; (e) a particle sliding under gravity on
the outside of an inverted vertical cone.
Ans. (a) holonomic, (6) non-holonomic, (c) non-holonomic, (d) holonomic, (e) holonomic

7.63.

and
[Fig. 7-21] has weights
lever
2
t
at distances a t and a 2 from the fixed support B.
Using the principle of virtual work, prove that a
necessary and sufficient condition for equilibrium is

ABC

ax

a2

W*

W*

IE
Og

7.64.

Work Problem

7.63 if one or

more additional weights

are placed on the lever.

7.65.

Fig. 7-21

An inextensible string of negligible mass hanging


over a smooth peg at B [see Fig. 7-22] connects one mass
frictionless inclined plane of angle a to another
x on a
Using D'Alembert's principle, prove that
mass
2
the masses will be in equilibrium if
2 =
1 sin a.

7.66.

Work Problem
tion

7.67.

7.65 if the incline has coefficient of fric-

Ans.

ft.

m2 =

m^sin

fi

cos a)

Fig. 7-22

mass m has its ends on a smooth wall and floor


The foot of the ladder is tied by an inextensible
rope of negligible mass to the base C of the wall so that the
ladder makes an angle a with the floor. Using the principle
of virtual work, find the magnitude of the tension in the rope.

ladder

AB

of

[see Fig. 7-23].

Ans.
7.68.

\mg

Work
tential
is

cot a

Problem 7.63 and (6) Problem 7.65 by using the poenergy method. Prove that the equilibrium in each case

(a)

unstable.

Fig. 7-

SYSTEMS OF PARTICLES

190

7.69.

7.70.

7.71.

[CHAP.

A thin uniform rod of length I has its two ends constrained to move on
the circumference of a smooth vertical circle of radius a [see Fig. 7-24].
Determine conditions for equilibrium.
Is the equilibrium of the rod of

solid

hemisphere of radius a

plane of angle

Explain.

Problem 7.69 stable or not?


is

located on a perfectly rough inclined

a.

<

-1

(a)

Prove that

(b)

Are there any other values of a for which equilibrium can occur?
Which of these, if any, yield stable equilibrium?

in stable equilibrium if

it is

sin

(3/8).

principle to obtain the equations of motion of masses

7.72.

Use D'Alembert's

7.73.

Work Atwood's machine problem

7.74.

Use D'Alembert's

Problem

[see

7.22,

Fig. 7-24

and

w2

of Problem 7.65.

page 180] by using D'Alembert's principle.

principle to determine the equations of motion of a simple pendulum.

MISCELLANEOUS PROBLEMS
7.75.

7.76.

7.77.

Prove that the center of mass of a uniform circular arc of radius a and central angle a
located on the axis of symmetry at a distance from the center equal to (a sin a)/a.
Discuss the cases

(a)

and

tt/2

Problem

in

7.75.

Work Problem
Ans. The

7.79.

circle of radius a is removed from a uniform circular plate


of radius b > a, as indicated in Fig. 7-25. If the distance between their centers A and B is D, find the center of mass.

Ans. The point at distance a 2D/(b 2


7.78.

(6)

is

- a2

below B.

7.77 if the circles are replaced

point at distance a3 D/(b 3

- a3

by spheres.

below B.

Prove that the center of mass does not depend on the origin
of the coordinate system used.

7.80.

Prove that the center of mass of a uniform thin hemispherical


a is located at a distance a from the center.

Fig. 7-25

shell of radius

7.81.

the angular momentum of a


Let the angular momentum of the moon about the earth be A. Find
mass. Assume the masses
system consisting of only the earth and the moon about the center of
Ans.
respectively.
m)
and
e A/(M e +
by
m
e
of the earth and moon to be given

7.82.

Does Theorem

page 170, apply

in case the

angular

momentum

is

taken about an arbitrary

Explain.

point?
7.83.

7.13,

In Fig. 7-26, AD, BD and CD are uniform thin rods


of equal length a and equal weight w. They are
smoothly hinged at D and have ends A, B and C on a
smooth horizontal plane. To prevent the motion
of ends A, B and C, we use an inextensible string
ABC of negligible mass which is in the form of an
equilateral triangle.

If a

weight

is

suspended

equal angles a with


from
the horizontal plane, prove that the magnitude of

so that the rods

the tension in the string


7.84.

make
is

cot

a.

is removed
7.83 if the weight
and suspended from the center of one of

Work Problem
from

%V% (W + 3w)

Fig. 7-26

the rods.
7.85.

Derive an expression for

an arbitrary

point.

(a)

the total angular

momentum and

(b)

the total torque of a system about

CHAP.

7.86.

7.87.

SYSTEMS OF PARTICLES

7]

Prove that the torque about any point P is equal to the time rate of
change in angular momentum about P if and only if (a) P is fixed in
space, (b) P coincides with the center of mass or (c) P is moving with
a velocity which is in the same direction as the center of mass.

Find the centroid of a

solid of constant density consisting of

circular cone of radius a


radius a [see Fig. 7-27].

Ans. At height (a 2

7.88.

7.89.

and height

+ H 2 )/(2a + H)

above O.

Work Problem

7.87 if the density of the cone

the hemisphere.

Ans. At height (a2

is

twice the density of


above O.

+ 2H 2 )/(a + H)

A hemisphere of radius a is cut out of a uniform solid cube of side


the center of mass of the remaining solid.

uniform chain of

in the

7.91.

a right

surmounted by a hemisphere of

Fig. 7-28

7.90.

191

middle

is

>

2a

Fig. 7-29

45kgwt

is

[see Fig. 7-28].

Find

Fig. 7-30

suspended from two fixed supports 15 meters apart.


Ans. 450 kg wt

If the

sag

20 cm, find the tension at the supports.

chain of length L and constant density a is suspended from two fixed points at the same
horizontal level. If the sag of the chain at the middle is at a distance
below the horizontal
line through the fixed points, prove that the tension at the lowest point of the chain is
a(L2 - 4D 2 )/8D.

7.92.

Three particles of masses


l ,m 2 ,m 3 are located at the vertices of a triangle opposite sides having
lengths a lt a2 ,a 3 respectively. Prove that the center of mass lies at the intersection of the angle
bisectors of the triangle if and only if mj/aj =
2 /a 2 =
3 /a 3

7.93.

Masses

7.94.

7.95.

m2

are on a frictionless circular cylinder connected by an inextensible string of


(a) Using the principle of virtual work, prove that the system
in equilibrium if
(b) Is the equilibrium stable?
Explain.
1 sin ai =
2 sin a 2

negligible
is

and
mass

Work Problem

[see Fig. 7-29].

7.93 if friction is taken into account.

Derive an expression for the total kinetic energy of a system of particles relative to a point
may be moving in space. Under what conditions is the expression mathematically
simplified? Discuss the physical significance of the simplification.

which

7.96.

Find the center of mass of a uniform plate shown shaded in Fig. 7-30 which is bounded by the
hypocycloid x 2 ' z + y 2' s = a2' 3 and the lines x = 0, y = 0. [Hint. Parametric equations for the
hypocycloid are x a cos 3 e, y = a sin 3 e.]
Ans. x y = 256a/315jr

SYSTEMS OF PARTICLES

192

7.97.

Let
(a)

m m2 m 3
1(

be

Prove that the

the

masses of three particles and

total kinetic

[CHAP.

v 12 v23 v 13 be their relative


,

energy of the system about the center of mass

m m2v\2 + m2m3V23 + m m
m + m2 + m
x

7.98.

7.99.

7.100.

Generalize the result in

(a).

A chain of variable density is suspended from two fixed points on the same horizontal level.
Prove that if the density of the chain varies as the horizontal distance from a vertical line
through its center, then the shape of the chain will be a parabola.
Discuss the relationship of Problem 7.98 with the shape of a suspension bridge.

A solid consists of a uniform right circular cone of vertex angle a on a uniform hemisphere of
the same density, as indicated in Fig. 7-31. Prove that the solid can be in stable equilibrium on
a horizontal plane if and only if a > 60.

Fig. 7-32

uniform solid [see Fig. 7-32] consists of a hemisphere of radius a surmounted by a cube of
side b symmetrically placed about the center of the hemisphere. Find the condition on a and b

Ans. alb

for stable equilibrium.


7.102.

> yWv-

Find the centroid of the area bounded by the cycloid


x

and the x
7.103.

Ans.

axis.

(ira,

a(e

sin

a(l

cos

e)

Prove that if the component of the torque about point P in any direction is zero, then the
component of angular momentum about P in that direction is conserved if (a) P is a fixed point,
(c) P is a point moving in the same direction as
(6) P coincides with the center of mass or
the center of mass.
the angular

momentum

In Problem 7.103,

7.105.

Prove that the virtual work due to a force


respond to all components of the force.

is

conserved only

is

equal to the

if

sum

(a),

(6)

or

(c)

occurs?

works which cor-

of the virtual

Explain.

2
2
uniform solid having the shape of the paraboloid of revolution cz = x + y
the xy plane, assumed horizontal. Prove that if the height of the paraboloid
< f c.
equilibrium is stable if and only if

7.108.

Explain.

Prove that it is impossible for one sphere to be in stable equilibrium on a fixed sphere which is
occur
perfectly rough [i.e. with coefficient of friction p = 1]. Is it possible for equilibrium to
at all?

7.107.

e),

5a/6)

7.104.

7.106.

is

,2
i v'x Z

Fig. 7-31

7.101.

velocities.

(6)

Work Problem

7.107

if

the xy plane

is

inclined at an angle a with the horizontal.

c
is

>

rests on

H, then the

CHAP.

7.109.

SYSTEMS OF PARTICLES

7]

193

In Fig. 7-33, AC and BC are frictionless wires in a vertical plane making angles of 60 and 30
respectively with the horizontal. Two beads of masses 3 gm and 6 gm are located on the wires,
connected by a thin rod of negligible mass. Prove that the system will be in equilibrium
when the rod makes an angle with the horizontal given by tan -1

(#y3

Fig. 7-33

7.110.

Prove each of the following theorems due to Pappus.


(a)

C in a plane is revolved about an axis in the plane which does not intersect
then the volume generated is equal to the area bounded by C multiplied by the distance
traveled by the centroid of the area.

If a closed curve
it,

(b)

an arc of a plane curve (closed or not) is revolved about an axis in the plane which does
it, then the area of the surface generated is equal to the length of the arc
multiplied by the distance traveled by the centroid of the arc.

If

not intersect

7.111.

Use Pappus' theorems


circular wire,

7.112.

(c)

to find (a) the centroid of a semicircular plate, (b) the centroid of a semithe centroid of a plate in the form of a right triangle, (d) the volume of a cylinder.

Find the (a) surface area and (6) volume of the doughnut shaped region obtained by revolving
a circle of radius a about a line in its plane at a distance b > a from its center.
Ans.

(a)

A^ab,

(b)

l^a^b

Chapter 8

APPLICATIONS to
VIBRATING SYSTEMS,
ROCKETS and COLLISIONS

VIBRATING SYSTEMS OF PARTICLES


If two or more particles are connected by springs [or interact with each other in some
equivalent manner], then the particles will vibrate or oscillate with respect to each other.

As seen

Chapter 4, a vibrating or oscillating particle such as the simple harmonic


bob of a simple pendulum, has a single frequency of vibration. In the case
of systems of particles, there is generally more than one frequency of vibration. Such
frequencies are called normal frequencies. The motions of the particles in these cases are
in

oscillator or

often called multiply -periodic vibrations.

mode

[i.e. a particular way in which vibration occurs, due to particular


example] in which only one of the normal frequencies is present is called
a normal mode of vibration or simply a normal mode. See Problems 8.1-8.3.

of vibration

initial conditions for

PROBLEMS INVOLVING CHANGING MASS. ROCKETS


Thus far we have

restricted ourselves to motions of particles having constant mass.


important class of problems involves changing mass. An example is that of a rocket
which moves forward by expelling particles of a fuel mixture backward. See Problems

An

8.4

and

8.5.

COLLISIONS OF PARTICLES
During the course of their motions two or more particles may collide with each other.
Problems which consider the motions of such particles are called collision or impact
problems.
In practice we think of colliding objects, such as spheres, as having elasticity. The
time during which such objects are in contact is composed of a compression time during
which slight deformation may take place, and restitution time during which the shape is
restored. We assume that the spheres are smooth so that forces exerted are along the
common normal to the spheres through the point of contact [and passing through their
centers]

can be direct or oblique. In a direct collision the direction of motion of both


along the common normal at the point of contact both before and after collision.
collision which is not direct is called oblique.
collision

spheres

is

Fundamental in collision problems is the following principle called Newton's


which is based on experimental evidence. We shall take it as a postulate.

collision

rule

Newton's collision rule. Let v 12 and v^ be the relative


common normal before and after impact. Then

along the

V 12

~
194

~~

eV 12

velocities of the spheres

CHAP.

8]

APPLICATIONS TO VIBRATING SYSTEMS, ROCKETS AND COLLISIONS

195

The quantity c, called the coefficient of restitution, depends on the materials of which the
the
and 1. If e =
objects are made and is generally taken as a constant between
collision is called perfectly inelastic or briefly inelastic.

If

the collision

is

called

perfectly elastic or briefly elastic.

is

In the case of perfectly elastic collisions the total kinetic energy before and after impact
the same.

CONTINUOUS SYSTEMS OF PARTICLES


For some problems the number of particles per unit length, area or volume is so large
that for all practical purposes the system can be considered as continuous. Examples are a
vibrating violin string, a vibrating drumhead or membrane, or a sphere rolling down an
inclined plane.

The basic laws of Chapter 7 hold for such continuous systems of particles. In applying
them, however, it is necessary to use integration in place of summation over the number
of particles and the concept of density.

THE VIBRATING STRING


Let us consider an elastic string such as a violin or piano string which is tightly
and x = I of the x axis [see Fig. 8-1]. If the
stretched between the fixed points x =
string is given some initial displacement [such as, for example, by plucking it] and is then
released, it will vibrate or oscillate about the equilibrium position.

Pig. 8-2

Fig. 8-1

denote the displacement of any point x of the string from the equilibrium
then the equation governing the vibrations is given by the
partial differential equation
If

we

let

Y(x,

position at time

t)

[see Fig. 8-2],

d2

Y =
2

dt

where

if

is

c<

d2

dx 2

the (constant) tension throughout the string and a

is

the (constant) density

[mass per unit length of string],

Tfa

The equation (1) holds in case the vibrations are assumed


at any point of the string is much less than one.

(2)

so small that the slope dY/dx

BOUNDARY-VALUE PROBLEMS
The problem of solving an equation such as (1) subject to various conditions, called
boundary conditions, is often called a boundary-value problem. An important method
for solving such problems makes use of Fourier series.

APPLICATIONS TO VIBRATING SYSTEMS, ROCKETS AND COLLISIONS

196

[CHAP.

FOURIER SERIES
Under

certain conditions [usually satisfied in practice and outlined below] a function

defined in the interval


has the series expansion

f(x),

f\ x )

where the

<x<

+ 21 and

o"

ttn

having period

cos ~~

,y

sm ~T~ )

Fourier

coefficients in the series, called

coefficients, are

(3)

given by

+ 2l
n-irX

cos
-j

21 outside of this interval,

aa?

s*y + 2l

sm

dx

(5)

7-

Such a

series is called the Fourier series of

For many problems

/(a;).

or I.

ODD AND EVEN FUNCTIONS


If

I,

certain simplifications can occur in the coefficients

(4)

and

(5)

as indicated

below:
1.

If f(-x)

f(x),

an

/(a;)

cos

bn

aaj,

(6)

In such case f(x) is called an even function and the Fourier series corresponding to
f(x) has only cosine terms.
2.

If

f(-x)

-f(x),

an

6n

0,

fX'^> sin

= ax

{7)

In such case f(x) is called an odd function and the Fourier series corresponding to
has only sine terms.
If f(x) is neither

even nor odd

its

f(x)

Fourier series will contain both sine and cosine terms.

Examples of even functions are x 4 Sx 6 + 4x 2 5, cos a;, e x + e~ x and the function shown
graphically in Fig. 8-3. Examples of odd functions are x 3 ,2x 5 5x 3 + 4, sinx, e x e~ x and
the function shown graphically in Fig. 8-4.
,

Examples of functions which are neither even nor odd are x 4 + x 3 x + cos x and the
function shown graphically in Fig. 8-5.
,

fix)

Fig. 8-3

Fig. 8-4

If a function is defined in the "half period" x =


then the function is known throughout the interval

Fig. 8-5

to

I <

x = I and is specified as odd,


x < I and so the series which

CHAP.

APPLICATIONS TO VIBRATING SYSTEMS, ROCKETS AND COLLISIONS

8]

197

contains only sine terms can be found. This series is often called the half range Fourier
to x = I which is specified as even
sine series. Similarly, a function denned from x =
has a series expansion called the half range Fourier cosine series.

CONVERGENCE OF FOURIER SERIES


Let us assume the following conditions on
f{x) is defined in y

1.

f(x)

and

its

<

f(x):

Hx)

< y + 21.

/(*i

+ 0)

derivative f'(x) are piece-

wise continuous in y < x < y + 21. [A


function is said to be piecewise continuous in an interval if the interval can
be divided into a finite number of subintervals in each of which the function
is continuous and bounded, i.e. there
B>
such that
is
a constant
B.
B < f(x) <
An example of such a
function is indicated in Fig. 8-6.]

/(* 2

At each point of discontinuity, for example, Xi [or x 2 in Fig. 8-6, f(x) has
from the right and left denoted respectively by f(xi + 0) and f{xi 0)

3.

+0)

Fig. 8-6

finite limits

[or f(x 2

+ 0),

f(X2~0)}.
4.

f(x)

has period

21, i.e.

f(x

+ 21) =

f(x).

These conditions if satisfied are sufficient to guarantee the validity of equation (3)
[i.e. the series on the right side of (3) actually converges to f(x)) at each point where f(x)
is

continuous.

by

[/(#

0) 4-

At each point where f(x) is discontinuous, (3) is still valid if f(x)


f(x 0)], i.e. the mean value of the right and left hand limits.

The conditions described above are known as Dirichlet

is

replaced

conditions.

Solved Problems

VIBRATING SYSTEMS OF PARTICLES


8.1.

Two

equal masses
are connected by
springs having equal spring constant k,
as shown in Fig. 8-7, so that the masses
are free to slide on a frictionless table
AB. The walls at A and B to which the
ends of the springs are attached are fixed.
Set up the differential equations of
motion of the masses.
Let x x i and x 2i [Fig. 8-8] denote the displacements of the masses from their equilibrium
positions C and D at any time t.

"

T5WKT

nit

J^RRKP

C ,#"

-OH5ffir>

Fig. 8-7

C
^1
-nmu^-

uni

*ii

"ChJ

x2i

Lj- ww^-

^^m^

P
Fig. 8-8

APPLICATIONS TO VIBRATING SYSTEMS, ROCKETS AND COLLISIONS

198

[CHAP.

Consider the forces acting on the first mass at P. There will be a force due to the spring on
the right given by K (x 2 i a^i) = K (x 2 - a^i, and a force due to the spring on the left given
by
KXyi. Thus the net force acting on the first mass at P is
k(x 2

way

In the same

)i

the net force acting on the second

mass at Q

x 2 )i

k(x x

KX t i
is

kx 2 i

Then by Newton's second law we have


d2

m-r^(x x \)

k(x 2

1 )i

KX t i

d2

=
m
m x2 =

or

8.2.

'x\

Find (a) the normal frequencies and


system in Problem 8.1.
(a)

Let x 1 = A t cos at,


after simplifying,

x2

= A2

cosut

k(x 2

~ 2Xi)

(l)

k(x 1

2x 2

(2)

the normal

(b)

and

in equations (1)

modes

of vibration for the

Then we

of Problem 8.1.

(2)

find,

(2K-mu 2 )A - KA 2 =

(1)

-icA 1

Now

if

Ay and

A2

are not both zero,


2k

(2/c

Solving for w 2

?ww 2 )(2c mw2


)

we

find

<o

Aktyi

(o

Then the normal

(2/c-m2)A

ffllO

2k

jc

is

(b)

Ww 2

k2

\/16k 2

m w4
2

<*>

4/cra<o 2

3/c

12/c 2 m2
giving

w2

and

/c/m

or

3/c/m

(.4)

system are given by


.

/ = 7T \\~
2v \ m

The normal frequencies are

(2)

we must have

(or natural) frequencies of the


1

and

\\\
m
1

3/c

(5)

2tt

characteristic frequencies

also called

and the determinant

(5)

and

(2).

called the characteristic determinant or secular determinant.

To find the normal mode corresponding


Then we find

to

let

yficfm,

w2

ic/m

in equations (i)

= A2

In this case the normal mode of vibration corresponds to the motion of the masses in the
[i.e. both to the right and both to the left] as indicated in Fig. 8-9.

same

direction

Normal mode corresponding

to

Normal mode corresponding

yj nlm

(1)

and

().

Then we

yZic/m

Fig. 8-10

Fig. 8-9

Similarly to find the normal

to

mode corresponding

to

find

A 2

ySK/m,

let

3ic/m

in equations

CHAP.

8]

APPLICATIONS TO VIBRATING SYSTEMS, ROCKETS AND COLLISIONS

199

of the masses in opposite


In this case the normal mode of vibration corresponds to the motion
left, and vice versa] as
the
to
moves
other
the
right
the
directions [i.e. when one moves to
indicated in Fig. 8-10 above.

working this problem we could also just as well have assumed,

In

x2
x2

8.3.

= B 2 sin a t
- C2 e
ibit

= A

xx

or

+B

cos at

= A2

x2

sin at,

cos at

+ B2

xt

= B sin at,
= C e*,
x

xx

or

sin at

is held at its equilibrium position while


to the right of its
magnitude a >
of
the second mass is given a displacement
of each mass
position
the
Find
released.
equilibrium position. The masses are then

Suppose that

at

any

Problem

in

8.1 the first

mass

later time.

Writing

Wl

and

^f^Jm

= V^c/w,

xx

x2

= D

the general motion of both masses

C cosu 1 t + C2
1

+ D2

coswx*

is

described by

sin ujt

+ C3

cos<o 2

+ C4

sin w 2 i

(-0

sinwjt

+ D3

cos u 2 *

+ D

sin a 2 t

(2)

where the coefficients are all constants. Substituting these into equation (1) or equation (2) of
t,
Problem 8.1 [both give the same results], we find on equating corresponding coefficients of cos Wl
sin a^t, cos u 2 t, sin a 2 t respectively,

=C

Dj

Thus equations

We now

(1)

and

x2

=
,

we
C + C3 =

these conditions
1

From

these

we

Thus equations

where a x

cos o^t

+ C2

sin u x t

Cxcosuit

+ C2

sin u x t

= V'/c/w,

0,

x2

a,

C - C3 =

xx

0,

x2

cos

<o

2<

cosu 2 t

(4)

\a,

C 2 W! + C4

a,

w2

+ C3
C3

+ C4
~ C4

sin a 2 t

(3)

sina 2 t

(4)

at

(5)

find respectively
0,

C =
and

D 4 = -C4

subject to the initial conditions

find
(5)

C 3 C4
Xl

From

D 3 = -C 3

can be written

(2)

xx

determine Cj,C 2

D 2 = C2

lf

C2 =

0,

<o 2

0,

C 2 o^ - C4 w 2 =

C 3 = -ia, C 4 =

()

give the required equations

xi

^(cos w x t

cos

o>

2 t)

(7)

x2

^a(cos u^t

cos

o)

2 t)

{8)

= v 3*/m.

Note that in the motion described by (7) and (8), both normal frequencies are present. These
equations show that the general motion is a superposition of the normal modes. This is sometimes
called the superposition principle.

CHANGING MASS. ROCKETS


8.4.

Derive an equation for the motion of a


rocket moving in a straight line.

be the total mass of the rocket at


time t. At a later time t + At suppose that
+ Am due to expulsion of a mass
the mass is
Am of gas through the back of the rocket.
Note that Am is actually a positive quantity

Let

o
Aw

v
1

+ Av

m + Am

since

Am

is

Fig. 8-11

assumed negative.

Let v and v + Av be the velocities of the rocket at times t and t + At relative to an inertial
system with origin at O. The velocity of the mass of gas ejected from the back of the rocket relative
is the velocity of the gas relative to the rocket.
to O is v + v where

APPLICATIONS TO VIBRATING SYSTEMS, ROCKETS AND COLLISIONS

200

Since the change in


Total

momentum

momentum
{(ra

where F

at

of the system
t

+ At

+ Am)(v + Av) +

Equation

+v

(-Am)(v

impulse

- mv

)}

at

At

(l)

can be written as

(1)

m Av
M ~
Then taking the

we

limit as At -> 0,

Am

vi,

= -v

F =

i,

Av

AT

Am

find

m dv
di ~
Fi, this

dm
-dT

becomes

dv
m Tt
+
.

8.5.

the net external force acting on the rocket.

is

Writing v

momentum

total

we have

equal to the impulse,

is

[CHAP. 8

dm
It

Find the velocity of the rocket of Problem 8.4 assuming that gas is ejected at a
constant rate and at constant velocity with respect to it and that it moves vertically

upward

in a constant gravitational field.

If the

gas

rocket at t can be written

ejected at constant rate

is

s\
(m -at)
/

Integrating,
If

at

we
t

F = -mg\

Since

0.

dv

0,

if

which
in

(2)

is

= -gt -

dv
^

or

v In

-at where

dm/dt

- at) +

(ra

-a, equation

= -9 +

the mass of the


of Problem 8.4

is
(S)

aV
^
o

{1)

cx

(2)

the rocket starts from rest, then

=
Thus

and

v
i.e.

m=m

then

0,

= -(m - a t)g

av

find

>

F - -mg)

(or

becomes

the speed at any time.

m +

v In

The

gt + v n

velocity

Note that we must have m - at >


which case the rocket will be out of

or

cx

0,

is

\n

v In

\m (

cx

at J

vi.

otherwise there will be no gas expelled from the rocket,

fuel.

COLLISIONS OF PARTICLES
Two masses mi and m 2 traveling
8.6.

in the same straight line collide. Find the velocities


of the particles after collision in terms of the velocities before collision.

Assume that the straight line is taken to be


the x axis and that the velocities of the particles
before and after collisions are v,,v 9 and vi, v,
1

respectively.

By Newton's

collision rule,

i-v =
By

e(v 2

-v-v

v2

page 194,

- Vl

momentum

Fig. 8-12

(1)

the principle of conservation of

Total

/~\

momentum,

after collision

total

momentum

m \[ + m2V2 = m \ + m2v2
x

before collision
(2)

CHAP.

APPLICATIONS TO VIBRATING SYSTEMS, ROCKETS AND COLLISIONS

8]

Solving

(1)

and

(2)

simultaneously,

(m x

V2

8.7.

201

Discuss Problem 8.6 for the case of

- em2 )v + m 2 (l + e)v 2
fltj + W&2
1

(l

(a)

(S)

+ e)vj + (m2 w 1 )v 2
m t + m2

a perfectly inelastic collision,

(b)

a perfectly

elastic collision.
(a)

Here we put

and

in (3)

of Problem 8.6 to obtain

(4)

m^Vi
vi

Wj +

mv
2

ra 2

v2

m^ + m2V
mj +

'

Thus after collision the two particles move with the same
were stuck together as a single particle.
(b)

Here we put

in (3)

vi

and

(ra x

(-4)

m2

velocity,

i.e.

they move as

if

they

of Problem 8.6 to obtain

m2)v + 2m2v 2
^ j,^
wii + m 2
1

v2

2m l \ l +
-

m,

m^v2
+ w2

(mj

These velocities are not the same.

8.8.

Show

that for a perfectly elastic collision of the particles of Problem 8.6 the total
kinetic energy before collision equals the total kinetic energy after collision.
Using the result of Problem

8.7(&),

we have

Total kinetic energy after collision

m )v! +
m +m

(m x

2 mi

8.9.

total kinetic

wv
i

2
i

+ ~m 2v2 2

2w 2v 2l

2%

C2m l v 1

(w2 m!)v2 ]

z
2

energy before

collision

Two

spheres of masses mi and ra 2 respectively, collide obliquely.


after impact in terms of their velocities before impact.

Find their

velocities

Let v 1 ,v 2 and vi,v 2 be the velocities of the


spheres before and after impact respectively, as
indicated in Fig. 8-13. Choose a coordinate system
so that the xy plane is the plane of v t and v 2 and
so that at the instant of impact the x axis passes
through the centers C t and C2 of the spheres.
,

By

the conservation of

m
From

v1

Fig. 8-13

+
we

m 2v 2 =

momentum, we have
w^vi

wi 2v 2

(1)

Fig. 8-13

see that

vi

v2

vi
v2
Substituting equations

(2)-(5)

in (1)

^iCcos *i

v 2 (cos *2

^i(cos

<Pi i

v 2 (cos

2 i

sin e j)
sin e % j)
sin j)
sin 2 j)

and equating

<f>

(3)

U)

(5)

<f>

coefficients

of

and

j,

we have

APPLICATIONS TO VIBRATING SYSTEMS, ROCKETS AND COLLISIONS

202

ra^

cos e i

m 2 v 2 cos B 2 =

m^ cos

<p x

mv

sin e l

m2v2 sin e

m^v'^ sin

</>!

By Newton's

m2 v'2 cos
m2 2 sin
i>

[CHAP.

(7)

<p 2

we have

collision rule,

Relative velocity after impact along x axis

e{relative

or

vi

which on using equations

{2)-(5)

v2

velocity before impact along x axis}


i

-e(v!

e(v l cos

v2

i)

(*)

becomes

v[ cos 0!

v 2 cos

<f>

v 2 cos

e 2)

()

Furthermore, since the tangential velocities before and after impact are equal,

or

Equation

From

(7)

is

(9)

we

V x COS 0!

and

(6)

V 2 COS
(12)

vi'j

v 2 *3

v2

(10)

v x sin

el

v[ sin

v 2 sin

e2

v'2

<f>

(J*)

sm 02

(18)

automatically satisfied by using equations (12) and

equations

Then using

Vi'j

and
vi

(15)

we

(13).

find

(wj

m 2 e)v

1 (l

+ e)Vi

+ (m2 wiie)^
+ m2

+ m2 (l + e)t> 2
+ m2

cos

cos

cos e 2

cos #2

find

m2 e)v

^i(cos 0i

(m l

~~

sin

X j)

cos

ra2 (l

+ e)v 2

cos

m + m2

sin e x

2 sin e 2

t>i

v2

v 2 (cos 02

m^l + e)i>!

sin $2

J)

+ (m2 m
wij + m2

cos *!

1 e)v 2

cos

2 i

t>

CONTINUOUS SYSTEMS OF PARTICLES


8.10.

Derive the partial differential equation


of a vibrating string.

(1),

page 195, for the transverse vibrations

Ax

Fig. 8-14

Let us consider the motion of an element of the string of length As, greatly magnified in
Fig. 8-14.

The forces acting on the element due to the remainder of the string are given by the tensions,
as shown in Fig. 8-14, of magnitude T(x) and T(x + A) at the ends x and x + As of the element.

CHAP.

APPLICATIONS TO VIBRATING SYSTEMS, ROCKETS AND COLLISIONS

8]

The net horizontal force

in direction

[T(x

The net

+ Ax)

vertical force in direction

[T(x

acting on the element

+ Ax) -

cos e(x

+ Ax)

sin 6(x

is

T(x) cos e(x)]i

acting on the element

+ Ax)

203

(1)

is

T(x) sin e(x)]j

(2)

we assume

that the horizontal motion in direction i is negligible, the net force (1) is zero.
Using the fact that the acceleration of the element is d 2 Y/dt2 approximately and that its mass is
a As where a is the mass per unit length, we have from (2) and Newton's second law,
If

a As
or, dividing

d2

-TT^- j

[T(x

+ Ax)

or

Taking the limit as Ax

_
~

As d*Y

Ax

dt 2

T(x

(3)

T(x

+ Ax) -

T(x) sin 6(x)

U)

Ax) sin e(x

+ As) -

T(x) sin e(x)

Ax

becomes

this

>Rs)
Since

sin 6{x

Ax

/AY\*d 2 Y
\Ax J dt2

-* 0,

+ Aas)

'

"d 2 Y

Vl +

U2

tan

sin 9

(5)

T(x) sin e(x)] j

by Ax j,
G

equation

+ Ax)

sin 6 (x

dx(

Tsin

&

dY/dx

Vl +

tan2 e

(dY/dx) 2

can be written
I

2
+ (&Y\ PY2

\8xJ

TdY/dx__

_j_

dx

dt

Vl +

(6)

(dY/dx)*

To simplify
is

this equation we make the assumption that vibrations are small so that the slope dY/dx
small in absolute value compared with 1. Then we can neglect (dY/dx) 2 compared with 1 and

W becomes
If

we

further assume that the tension

constant, (7) becomes

d2

constant throughout the string and that a

is

where

c2

8.11.

(8)

dx

Unless otherwise specified, when

T/a.

refer to equation

we

also

C2
c2
idr
2

dt 2

is

deal with the vibrating string

we

shall

(8).

Derive the equation of Problem 8.10

if

the string

is

horizontal and gravity

is

taken

into account.
In this case we must add to the right hand side of equation
on the element due to gravity
mg = a As gj

The

effect of this is to replace equation (8) of


d2

dt2

Problem 8.10 by
C2

FOURIER SERIES
8.12.

Graph each of the following


f

<)/<*>

<x <

= |_3

-5

functions.

<*<0

Peri d

10

&Y _
dx 2

(3)

of Problem 8.10 the force

APPLICATIONS TO VIBRATING SYSTEMS, ROCKETS AND COLLISIONS

204

[CHAP. 8

/(*)

~*~

Per iod

3
1

-25

-20

-15

-5

-10

--

10

15

--

20

25

Fig. 8-15

Since the period is 10, that portion of the graph in 5 < x < 5 (indicated heavy in
Fig. 8-15 above) is extended periodically outside this range (indicated dashed). Note that f(x)
is not defined at x = 0, 5, 5, 10, 10, 15, 15, etc. These values are the discontinuities of f(x).

^
<

sin x
(b)

f{x)
77

x
x

^
<

-nr

Period

2tt

2tt

/<*)
J

-St

/
/

\.

2w

2r

3a-

Av

Fig. 8-16

Note that

Refer to Fig. 8-16 above.

8.13.

Prove

sin

y- dx

kirX

j
j ax

sin

8.14.

Prove

(a)

(b)

where
(a)

kirX

j dx

m-n
m-rrX

cosjdx

si
sin

m-rrX
5 cos j- dx

C
\

+ B)}.

m = n,

m = n,

B =

by Problem

n-irx

sin ( kir)

m-rrX
i sin i dx

m= n
m=n

{cos (A

8.13,

cos ( kir)
+ j
kit

t~
kir

kir

sin

any of the values

cos (A

if

continuous everywhere.

is

trigonometry: cos A cos

if

7 sin
kir

TlirX

'

cos kir

Kir

x and

fc= 1,2,3,

if

-j

kvx

fltrX

cos

From

Similarly

kirX

m and n can assume

Then,

kir

kir

j- dx =

cos

j- cosj,

cos

f(x) is defined for all

1, 2, 3, ...

- B) +

cos (A

+ B)},

sin

sin

B =

^{cos (A

- B) -

CHAP.

8]

APPLICATIONS TO VIBRATING SYSTEMS, ROCKETS AND COLLISIONS

m = n,

If

J.sm-r sm-r
mirx

We

have

sin

C
J_
If

sin

rl
o,-~
sin

8.15.

gj
-

2mrx\
(l-cos
)d*
/..

If
2

and

J_

sin

r
i

8.13, if

(m + n)irx
1 -)**
+ *i
.

2mrx
i
sm

respectively.

Then by Problem

(m n)vx

sin(A+B)}.

nvX j
m
~ -y^=x cos
dx

results of parts (a)

-J^l+COSjd*
'-i

-r

these integrals are equal to 2/ and

mirx
nirx
_~
cos
dx

y,

dx

A cosB = ^{sin(A-B) +

m = n,

The

dx

m==

if

replaced by

nwx

-i

Note that

205

we have

j^ COS COS

(6)

do;

m + n,
=

remain valid even when the limits of integration

(6)

I, I

are

21 respectively.

If

A +

f[x)

2,

nCosy

h bn

smJ J

prove that by making suitable assumptions concerning term by term integration of


infinite series, that for

(a)

(a)

an

n=

\^ f{x)co^dx

(b)

bn

=
\

f(x)sin^dx,

(c)

A = ^.

Multiplying

by cos

-|-

and integrating from -Z

J%(*)coB*pfe

am l

Thus

Multiplying

A +

f( x)

(6)

1, 2, 3,

(1)

by

sin^p

/(). in Spcfc

I,

6n

using Problem 8.14,

sin^

(i)

we have

7 J

(f)

m=

'^

cos|

da;

and integrating from -I

cos^f da

if

if

to

I,

m=

1, 2, 3,

using Problem 8.14, we have

J' sin^fd*

to

J^^cos^f

6m Z

Si

{* sin^cos^fd,

(Jf)

sin^sin^f,

APPLICATIONS TO VIBRATING SYSTEMS, ROCKETS AND COLLISIONS

206

Thus

(c)

bm

Integration of

(1)

from

m=

Putting

The above

to

sinj^-dx

f(x)

if

m=

[CHAP.

1,2, 3, ...

using Problem 8.13, gives

Z,

/()<*

2AI

we

in the result of part (a),

when

results also hold

or

= tIf

find

f(x)

^J

the integration limits I,

dx

f(x)

dx

and

so

are replaced by

o
-~
A _

y,

+ 21.

parts above we have assumed interchange of summation and integration.


assumption is not warranted, the coefficients a m and b m as obtained above are
called Fourier coefficients corresponding to f(x), and the corresponding series with these values
of am and b m is called the Fourier series corresponding to f(x). An important problem in this
case is to investigate conditions under which this series actually converges to f(x). Sufficient
conditions for this convergence are the Dirichlet conditions given on page 197.

Note that

Even when

8.16.

(a)

in all

this

Find the Fourier

coefficients

corresponding to the function

-5<z<0
0<z<5

TO

f(x)

'

[3

(b)

Write the corresponding Fourier

(c)

How

should f(x) be defined at x

of f(x)

10

series.

-5, x

and x = 5

in order that the

Fourier

in Fig. 8-17 below.

shown

is

-5 ^ x ^ 5?

series will converge to f(x) for

The graph

Period

Period

-10

10

Fig. 8-17

(a)

Period

21

10 and

Choose the interval y to y

5.

+ 2l

an

=
=

(0)

V n^r

an

0,

bn

to 5, so that y

r5

-r

/(*)

yj7

3 /
-I

5 \

nir

cos

Jf

(3)

cos^f

fa|

n#0

if

'5

If

f cos^f dx
f^sin^f =
|

-5

21 as

5.

|J_5 /(*)cos^

f(x)cos^dx

\f

sm-y-

mrx\
>

5
I

5
/-5
r

..

ovx
-=-

cos

da;

das

3(1

3
g

gj

3.

/(as)

sin

-g-

cos

da;

rnr)

das

jT cos^ dx

Then

CHAP.

8]

(b)

APPLICATIONS TO VIBRATING SYSTEMS, ROCKETS AND COLLISIONS


The corresponding Fourier
a

( ~ cos nirX
(n

^
v<

+
i

series is

nirx\
sm
j

bn

3
2

i.

nwx
^cos nv) sm-g-

3(1

=
Since f(x) satisfies the Dirichlet conditions,

"x +

points of continuity and to

"x

6 /

+
,

(c)

207

^(

we can say

~ ^

irx

Sin -5-

If

we

3tr

-5-

+
.

5vx
-T +

Sm

'

that the series converges to f(x) at

(3

+ 0)/2

At *

at points of discontinuity.

which are points of discontinuity, the series converges to


the graph.

Sin

-5,

as

3/2

all

and

5,

from

seen

redefine f(x) as follows,


'3/2

= -5

-5 < x <

f(x)

3/2

<

<

.3/2

then the series will converge to f(x) for

8.17.

show that

If f(x) is even,

(a)

Letting x

/()

an

(a)

cos^ cfe

5 ^

x
x

<
=

Period

10

5
5
5.

^^ +

/() cos

f J"

cos^dx,

f(x)

| J"

bn

(b)

J'

/(*)

0.

cos^ dx

u,

/"/<*) cos*|*<fc

ljV*)cos(^)d%

since by definition of an even function

f(u)

lJ*V)cos^fdM

fff(x)cos^dx

Then

f(u).

*n
If

we make
\

y//()coB^d

+|J
=

/(^)sin^f d*

= -u

/(*)eos^te

/(*)sin^<*s +

|J

the transformation

2p ifa

-j f f(u)Bin^du

fix) sin

J*

in the first integral

f{-u) sin

(=f^)

f(x)sm?fLdx

on the right of

<fc*

\f

-]

(I),

we

f(- u) sin

(1)

obtain

<*

-\ f f(x)sin^dx

(2)

where we have used the fact that for an even function f(-u) = f(u) and in the last step that
dummy variable of integration u can be replaced by any other symbol, in particular x. Thus
from (1), using (2), we have

the

~lj
8.18.

Expand
(a)

f(x)

x,

<x<

2,

M sm j dx

f( x >

in a half

n-n-x

range

(a)

f
I

,.

/(*)

mtx ,
sinz-dx
.

sine series,

(b)

cosine series.

Extend the

definition of the given function to that of the odd function of period 4


Pig. 8-18 below. This is sometimes called the odd extension of /(). Then 21
1
4,

shown

2.

in

APPLICATIONS TO VIBRATING SYSTEMS, ROCKETS AND COLLISIONS

208

[CHAP.

/(*)

/
/

-1
-6

71
/-4

-2

/
if

O
7
'y

Fig. 8-18

Thus a n ~

and
bn

1J

j dx

/() sin

5
{w(^-)-(k
,,

Then

/(a;)

4
cos nir sin
2i
n = 1 %7T
v<

-4 /

(b)

nirx

sm-y- dx

~0

77-3

sin

nirx \

sin

-g-

cosjt

%7ra;

2ttx
-21 S in^-+
.

3jra;

gSin^-

Extend the
This

is

definition of f(x) to that of the even function of period 4


the even extension of f(x). Then 21 = 4, 1 = 2.

shown

in Fig. 8-19 below.

Fig. 8-19

Thus

bn

0,
,.

rura \

\71tt

(cos

("
a;

sm-j-

/ 2

(*)

nirx

1177

/ 4
-5\n2

M
(1)
v

cosy- dx

HrC

ir

if

1)

M/V

If

Then

0,

= r
I

/(a?)

asda;

2.

>
2V

It

-^-^(cosrair

COS

wx

T+
,

32 COS

should be noted that the given function f(x)

by the two different

series in (a)

and

(6).

1)

cosr

SwX
"gx,

1
+ g2COS^~ +
5irx
,

<

<

2,

is

represented equally well

CHAP.

APPLICATIONS TO VIBRATING SYSTEMS, ROCKETS AND COLLISIONS

8]

209

SOLUTIONS OF VIBRATING STRING PROBLEMS


8.19.

Find the transverse displacement of a vibrating string of length


if

the string

is initially

given a displacement f(x) from

its

I with fixed endpoints


equilibrium position and

then released.
Let the transverse displacement of any point x of the string at time t be Y(x, t). Since the
and x I of the string are fixed, we must have Y(0, t) =
x
and Y(l, t) = 0. Since
the initial displacement is f(x), we have Y(x, 0) = f(x); and since the initial velocity is zero, we
have Yt (x, 0) =
where Y t denotes the partial derivative with respect to t. We must thus solve
the boundary-value problem
ends

&X.
a<2

Y(0,t)

Y(l,t)

0,

Assume a solution to (1) of


t.
Then substituting into

only on

d2Y

form

(1),

using

Y(x,0)=f(x),

0,

X"

(x,0)

(2)

where X depends only on x and T depends


d2 X/dx 2 and T" to denote d2 T/dt 2 we have

Y = XT

the

dx*

to denote

X T" =

c2

X" _
X ~

X" T

T"
c2

(3)

Since one side depends only on x and the other side depends on t while x and
the only way in which (2) can be valid is if each side is a constant, which

-X 2

are independent,
shall take as

we

Thus

- _ x2
II
2
~

*? X ~
X" + \*X =

or

T"

0,

\ z cH

These equations have solutions

X = A
Thus a

solution

is

= XT =

t)

the first condition in

so that A t =
[since
do not want]. Thus

B B2

b,

Using the

second

1,2,3, ...

Thus

= A 2 cos Xct + B 2

sin Xx,

+ B

{A x cos \x

t)

+ B2

{A 2 cos Xct

the second factor

if

Y(x,

B A2

Bi

sin \x)(A 2 cos Xct

zero then the solution

is

sin Xx (6 cos Xct

sin Xx (A 2 cos Xct

+ B2

t\
t)

differentiating with respect to

*\
/
{x,t)

sin Xct)

(i)

a sin

is

identically zero,

which we

sin Xct)

Xct)

(5)

a.

sin

nwx
j- (
I

b cos

nirct

that

+
.

sin \l

a sin

_
-

t,

this

or

nvct\

j
j

v
Y

+ B2

sin Xct)

condition of (2 ) in (5), we see


= mr/l and the solution so far is

v(
x (x,

By

sin Xct

we have

(2),

on writing

cos \x

given by

Y(x,

From

Xl

n-ir

where

(6)

becomes

nirX (
sin^

nircb

nvct

T-sn-T

nirca
nirct\
r cos-r J

so that the fourth condition in (2) gives

Y
from which a

0.

Thus

(6)

(x,0)

in^(p)

becomes

v/~
Y(x,

*\
t)

u
b
sin

n7rX

-j-

cos

n-n-ct
y

(7)

APPLICATIONS TO VIBRATING SYSTEMS, ROCKETS AND COLLISIONS

210

[CHAP.

To satisfy the third condition of (2) we use the fact that solutions of (7) multiplied by constants
as well as sums of solutions are also solutions [the superposition theorem or principle for linear
differential equations]. Thus we arrive at the solution
Y(x,

ja
t)

T7-/

b n sin

2i

n=l

Using the third condition of

{2) in (8)

nvx
j

nvct

cos

,.

(8)

we must have
00

Y(x,0)

But

this is

sin^

(9)

simply the expansion of f(x) in a Fourier sine series and the coefficients are given by

bn

Thus the

2
n=l

f(x)

solution

T1

f( x )

sin^ dx

Jo

given by

is

Y(x,t)

The method of

8.20.

^1

Y XT

assuming

solution

/(x)sin^^jsin^cos^

j|j*
is

(10)

often called the method of separation of variables.

string with fixed ends is picked up at its


center a distance
from the equilibrium
position and released. Find the displacement at any position at any time.

From
ment

we

Fig. 8-20

of the string

is

see that the


given by

initial displace-

0<x^l/2

(2Hx/l

Y(x,0)

NOW

\ 2H{l . x)/l

|/2 Si

f(x) sin

ff

l/2

< I

%K

sin

v2

2Hx

2H
C
I
jX

nirx

sm -j

dx

(t

mrx

sc)

sm r~ dx

w "-/ 2 )
n2

in equation ilO) of

Problem

8.19,

find

.
Y{x,

JX
t)

SH

^
2i

sin imr/2)

nvx

nirct

sm ~T~ cos r~
.

z2

Sirct
3rrx
1
vet
SH f 1
^2\Y2Sm T c0S -r ~ sm ~r cos ~r +

n9

n=l
.

8.21.

-j dx

on using integration by parts to evaluate the integrals. Using this

we

n fl

bn

f(x)

-n-X

1
52

5n-aj

sin_

5vct

r cos ~r

"

Find the normal frequencies and normal modes for the vibrating string
Problem 8.20.
The normal mode corresponding
the solution of Problem 8.20,

to the lowest

i.e.,

8H
r4

The frequency

is

normal frequency

is

given by the

_._. vet

sm =-

cos

,
<>

given by f x where

2Wi =

or

= ~

\7

first

term

in

in

CHAP.

8]

APPLICATIONS TO VIBRATING SYSTEMS, ROCKETS AND COLLISIONS

211

Since the cosine varies between 1 and +1, the mode is such that the string oscillates as in
Fig. 8-21 from the heavy curve to the dashed curve and back again.

Fig. 8-21

Fig. 8-22

The next higher frequency

mode corresponding

given by the

is

to

the next term

in

the

series which, except for sign, is

81?

3irx

Zirct
^2 sin cos-y-

In this case the frequency

is

2W
The mode

is

given by

or

fs

Yl

2^-

indicated in Fig. 8-22.

The higher normal frequencies are given by

f*

The amplitudes of modes corresponding

/a

fl

2z\7'

'"

2i\7>

to the even frequencies

/4

2!\a'

21

are zero, so that these frequencies are not present.

\ 7'

*"

In a general displacement, however, they

would be present.
Because of the fact that all higher normal frequencies are integer multiples of the lowest
normal frequency, often called the fundamental frequency, the vibrating string emits a musical
note. The higher frequencies are sometimes called overtones.

8.22.

Find the transverse displacement of a vibrating string of length I with fixed endpoints if the string is initially in the equilibrium position and is given a velocity
distribution defined by g(x).
In this case

we must

solve the boundary-value problem


a2

r _

Y(0,t)

Y(l,t)=zO,

0,

C2

3<2

<py
CO

5*2

Y(x,0)

0,

(x,0)

g(x)

The method of separation of variables and application of the first two conditions of
as in Problem 8.19,
V(rr
X
\x,

+\
t)

nirX
J j
sin
I

However,

in this case if

we apply

w~
we

x{x,

\"

niTCt

"+
:

_
=

*\
t)

first

t)

the third condition of

Y(x,

To satisfy the fourth condition


arrive at the solution

( 6U cos
wo

a sin

nvX

(2)
.

sin

a ai
sin
"
"

we

(2\;

vields

% =
C \

""

find

0,

so that

nvct
-^~-

note that the superposition principle applies, so that

2,

=i

an sin f sin
'

we

APPLICATIONS TO VIBRATING SYSTEMS, ROCKETS AND COLLISIONS

212

From

this

we have by

differentiation with respect to

v
Y

N
(x, t)

2i

n=X

or

(x, 0)

g{x)

mrcan

7
I

Thus the required

C
I

solution

is,

v/ i\
Y(x,t)

cos

on using

S
2

sin

~T
l

nirx

smj ax

g(x)

nirc

<*)

in

(-4)

an

or

| Jf

mrx

ain-

ir()

nvx
sm

<te|

sin
si

(o)

-.

Find the transverse displacement of a vibrating string of length I with fixed endpoints if the string initially has a displacement from the equilibrium position given
by f(x) and velocity distribution given by g(x).
The solution to the given problem
Thus the required solution is

Y(x,t)

"I7

show that

v,

The solution to
become infinite and
plane respectively].

dx

sm

nirx

cos
WB

nirct
I

nirX

n=l

sin
W
J/g(x)-sm-^-dx>
I

sm

r-

nrrct

on to a fixed horizontal plane. If


rebound with velocity cv.

it

hits the plane

will

problem can be obtained from the results of Problem 8.6 by letting


2
while v x = v [where subscripts 1 and 2 refer to the particle and
Then the respective velocities after impact are given by
v2

lim

lim

remains

of the solutions to the Problems 8.19 and 8.22.

this

m 2 -+o

Thus the

it

sum

the

/(*) sin^j^-

MISCELLANEOUS PROBLEMS
8.24. A particle is dropped vertically
with velocity

is

8.25.

dx
sm7
I

g(x)

see that

n T?%

8.23.

we

series

n ct
~

w=l

Then by the method of Fourier

t,

nwX
sm ~t~
=
2
nircan

[CHAP.

velocity of

v(

Vo

the particle

lim

m 2 -+oo

{{mjm^ e}v
r
1 + {m 1 /m 2
.,

lim

after

ev

(7n 1

/m2)(l + e)v

r
;

impact

is

r-

The

-ev.

velocity

the

of

plane of course

zero.

Suppose that the particle of Problem 8.24 is dropped from rest at a height H above
the plane. Prove that the total theoretical distance traveled by the particle before
coming to rest is given by #(1 + e 2 )/(l - e 2 ).
Let v be the speed of the particle just before it hits the plane. Then by the conservation of
= + mgH or v 2 - 2gH. Thus by Problem 8.24 the particle rebounds with
energy, \mv 2 +
speed ev and reaches a height {ev) 2/2g = e2 H. It then travels back to the plane through the
2
distance e 2H. Thus on the first rebound it travels through the distance 2e H.

By

similar reasoning
distance 2e4 H, 2e6 , ....

H + 2e H
2

we

+2e*H + 2e6#+

using the result

find that

Then the

on the second, third,

rebounds

it

total theoretical distance traveled before

= H+

+ r + r2 + rs +--- ~

2*ff(l

1/(1

- r)

+ e2 + e* +
if

\r\

<

1.

= H+

travels through the


coming to rest is

^^ = #(fz")

CHAP.

8.26.

APPLICATIONS TO VIBRATING SYSTEMS, ROCKETS AND COLLISIONS

8]

Two

particles having

masses

are traveling on the x axis

and

213

(assumed

with velocities vd and V\i


respectively. Suppose that they collide

frictionless)

and that after the

collision (impact) their


are v 2 i and V2 i respectively.
Prove that the velocities of the center of
mass before and after collision are equal.

velocities

By

the conservation of

Total

momentum

before impact

or

Let x and

The

(mx

8.27.

mv 2 \ +

MV

mv 2 +

MV2

be the respective coordinates of the particles.

velocity of the center of

velocity of the center of

particle of

length

momentum

mass before impact

starts initially
to reach the

mass

L which

(mv,

down

2i

is

given

is

+ MVJ/im + M)

mass after impact

slides

after impact

Then the center of mass

+ MX)/(m + M).

rx

The

MV

mVl
f

total

+ MVxi

mt^i

by

Fig. 8-23

momentum,

is

f2

= (mv 2 + MV2 )/(m + M). Thus ^ =

a frictionless incline of angle

<*,

mass

*f

2.

and

on a horizontal frictionless plane [see Fig. 8-24]. If the particle


from rest at the top of the incline, prove that the time for the particle
is

bottom

is

given by

2L(M + m si n 2 a)
(M + m)g sin a

Choose a fixed vertical xy coordinate


system as represented in Fig. 8-24. Let R be
the position vector of the center of mass C
of the incline, A the (constant) vector from
C to the top of the incline, and s the position
vector of the particle relative to the top of
the incline. Then the position vector of particle
with respect to the fixed coordinate
system is R + A + s. Since the only force
acting on the particle is the weight mg of the
particle, we have by Newton's second law
applied to the particle,

d2 ,

m-jp

(R+A. + s) = mg

dt*

Writing

s,

(2)

(1)

and

-^-j

dt 2
s,

where

ss lt

Sl

becomes

d2 X

Multiplying by

Fig. 8-24

(2)

dt 2

R- Xi+Yj,

direction of

this

d%
dt 2

Sl

is

a unit vector down the incline in the

-91

becomes

d2 X
dt2

Sl

' *

X
d2

cPs
dt 2

Sl

'

Sl

ffSi

'

COSa

d s
+__

9 sin a

()

APPLICATIONS TO VIBRATING SYSTEMS, ROCKETS AND COLLISIONS

214

[CHAP.

Since the net horizontal force acting on the system consisting of the particle and incline is zero, the
momentum in the horizontal direction before and after the particle starts sliding is zero. Then

total

M^-i
at

+ m~CR + A + s)'i =
dt

This can be written as

m -j- cos a

(M + m) -j-

Differentiating

with respect to

(4)

(M + m)g

subject to the conditions

8.28.

(M + m)g

sin a

M + m sin

at

ds/dt

0,

+ m)g sin a]
I + m sin 2 a

2
J

(5)

dt2

(6)

we

0,

find

C(M

when

cPs

sin a

M + m m cos

~~

dt 2

which,

find

yields

(3)

(j2 s

(6)

m cos a
M+m

_
~

dt2

Integrating

(-4)

and solving for d2 X/dt2 we

d2 X

Substituting into

t2

yields the required time.

L,

Solve the vibrating string Problem 8.19


The boundary-value problem

t)

taken into account.

is

is

d2

F(0,

gravity

if

Y(l, t)

0,

0,

d2

Y(x,

Y
=

0)

f(x),

(x, 0)

(2)

Because of the term g the method of separation of variables does not work
order to remove this term, we let

Y(x,t)
in the equation

Thus we

and conditions.

d2

Z(0,

The equation

t)

\fr(0)

Z(l,

0,

and conditions

(4)

d 2Z

f(l)

(3)

f(x)

Z(x, 0)

0,

become similar

(5)

In

find

t)

Z(x,t)

in this case.

f(x)

/(*),

(x, 0)

to

problems already discussed

0,

=
if

(5)

we

choose $

such that
efy"

In this case

(4)

and

*(0)

0,

dt 2

Now from
we

(6)

obtain c2

we
0,

t)

find
c

(6)

become

(5)

Z(0,

f(l)

=
f"

0,

gl/2c 2 .

Z(l,

g/c 2

t)

=
or

Thus

0,

Z(x, 0)

$(x)

Kn

dx 2

f{x)

= gx 2 /2c 2 + c

- f(x), Z
x

+ c2

(x, 0)

and since

(8)

^(0)

0,

f(l)

0,

CHAP.

8]

APPLICATIONS TO VIBRATING SYSTEMS, ROCKETS AND COLLISIONS


The

solution to equation

Z(x,

t)

with conditions

(7)

| f

as in Problem 8.19,

(8) is,

- *(*)}

[/(*)

8.29.

t)

^2

||

[/<*)

- JL

_ fa)

X2

^
A

sin

dx

sin

and thus
Y(x,

215

da

sin

nirx

cos

nirct

2^

sin

** +

cos

J?L

(jB

_ lx)

Assume that a continuous

string, which is fixed at its endpoints and vibrates


replaced by
particles of mass
at equal distances from each other.
Determine the equations of motion of the particles.

transversely,

is

We assume that the particles are connected to each other by taut, elastic strings
having constant tension T [see Fig. 8-25].
We also assume that the horizontal distances
between particles [i.e. in the direction of the
unit vector i] are equal to a and that the
transverse displacement [i.e. in the direction
of the unit vector j] of particle v is Y We
v
assume that there is no displacement of any
particle in direction i or i.
.

Let us isolate the rfh particle.


and {v + l)st particles. We have

The forces acting on

Transverse force due to

{v

- l)st

Transverse force due to

{v

Then by Newton's second law the

Si
d*

particle

l)st particle

due to the

fY Y

T
Y

fY
T

on particle

(v

- l)st

\
) j

\
)

-*(*^=0i-r(*^)i

=
d?Y

this particle are those

total transverse force acting

m -W v =
le

Fig. 8-25

-(Y*-i-2Y v + Yv +

(^-i-2^+^ +

1)

i)

(1)

To take into account the fact that the endpoints are fixed, we assume two particles
corresponding to v =
and r =
for which Y = 0, YN + 1 = 0. Then on putting = 1 and
v =
in equation (1), we find

N+l

^
8.30.

^-2Y + Y
i

^(YN _ -2YN

?s =

^>

Obtain the secular determinant condition for the normal frequencies of the system
of particles in
Let

Problem

Y v = A v cosat

8.29.

in equations (1)

and

(2)

of Problem 8.29.

-A_! + (2- ma /T)A v - A v + 1


a2

(2

ma?IT)A x

- A2 =

Putting

~A N _ +

0,

mtuP/T

cA 2

-A =

(2

Then after simplifying we

2,

.,

N- 1

- ma^/T)A N =

-A 2

0,

-A +
1

0,

...,

(l)

(2)

(3)

these equations can be written

cA 1

find

-A N _ +
1

cA N

APPLICATIONS TO VIBRATING SYSTEMS, ROCKETS AND COLLISIONS

216

Then

if

we wish

Av

solutions such that

of the coefficients be zero,

require that the 2Vth order determinant

i.e.,

-1

-1

-1

-1

AW

we must

= 0,

[CHAP.

..

..

-1

..

..

..

The normal frequencies are obtained by solving

-1

-1

-1

this equation for the

values of w 2

Although we have used Y v = A v cos cot, we could just as well have assumed Yv = B v sin wt
or Yv = A v cos at + B v sin at or Y v = Cv e iat The secular determinant would have come out to be
the same [compare the remarks at the end of Problem 8.2(6)].
.

8.31.

Prove that the normal frequencies in Problem 8.30 are given by

By expanding
we have

2T

<D

cos

ma

solve the difference equation


Substituting this into (1),

determined.

p2
call

2 cos

cp

(e

A!

1,

if

we

Thus

1.

(3)

A N = p N where p
~
on dividing by pN 2

find

take

is

a constant to be

cos

No -

Vc 2 -4

then

0,

Thus

assume that

(1),

we

c2

(I)

are formally satisfied

(2)

A2

c,

= 2 in (i), we see that equations


Putting
conditions consistent with (1) and (2) are

we

...,N

= cAN -i - A N _ 2

Ax

Also,

If

1,

the determinant A N of Problem 8.30 in terms of the elements in the first row,

AN

To

a=

N+l

cos 6

,i0

sin e

solutions of the difference equation are


ifl)N

e iN0

cos

Ne +

(e~ ie ) N

and

sin Nff

e~ Nie

sin

No

Since constants multiplying these solutions and sums of solutions are also solutions [as in
the case of linear differential equations], we see that the general solution is

= G

AN

Now from

equations

(3)

we have

=
This

is

equal to zero

of Problem 8.30,

we

when

sin

cos

At

1,

1)6

Ne +

sin

Ne +

(N +

cos

H sin Ne
so that

2 cos e

Ne

cos e

sin

(4)

G=
(N +

1,

H = cot

e.

Thus

l)e
(5)

sine

sins

or

aw/(N +1), a

l,...,N.

Thus using

find
o

2T

fl1

ma\

*
COS

N+
t.

.,

(6)

CHAP.

8.32.

APPLICATIONS TO VIBRATING SYSTEMS, ROCKETS AND COLLISIONS

8]

Solve for

From
Problem

Problem 8.30 and thus

in

find the transverse displacement

217

Yv of particle

equations (1) and (2) of Problem 8.30 we have [on using the normal frequencies
and the superscript a to indicate that the A's depend on a],

(6)

v.

of

8.31

cos^^

-Ail\ + 2AJ">

- A%\ =

(1)

together with the end conditions

K =
The equation
and we find

(1)

subject to conditions

(2)

can be solved in a manner exactly like that of Problem

(2)

A^
I

A^l, =

0,

Csin

(a)

av*

N+

Yv = B v

where Ca are arbitrary constants. In a similar manner if we had assumed


remarks at the end of Problem 8.30] we would have obtained

Thus

sin ut

[see

avir

_ D

>()

8.31,

sin

N+l

by

solutions are given

Ca

sin

N+l cos

and since sums of solutions are also

Da

and

u>t

solutions,

sin

N+l

sin ut

we have

*v

The constants Ca and

Da

sin

2i
a=l

iV

\Ca cos

oxt

+ Da

sin t)

are determined from initial conditions.

The analogy with the continuous vibrating string

is

easily seen.

Supplementary Problems
VIBRATING SYSTEMS OF PARTICLES
8.33.

8.34.

Find the normal frequencies of the vibrations


and masses are all different.

Two

equal masses

in

Problem

8.1,

page 197,

if

the spring constants

on a horizontal frictionless

shown in Fig. 8-26 are connected by equal


springs. The end of one spring is fixed at A
and the masses are set into motion, (a) Set up the

table as

equations of motion of the system. (6) Find the


normal frequencies of vibration, (c) Describe the
normal modes of vibration.

a. (
8.35.

8.36

Work Problem
Two

3^ JI
\m
Air

/,z
'

equal masses

Work Problem

Q'O'OQ^

*
'

QOOQ ^

* 8 -26

and masses are

different.

are attached to the ends of a spring of constant k which is on a horizontal


masses are pulled apart and then released, prove that they will vibrate
2iryfm/2tc.

8.36 if the masses are different

Ans. 2irV^/K where

Fi

If the

with respect to each other with period

8.37.

JI
41
4v
y m

8.34 if the spring constants

frictionless table.

Qm

^JL~^

M M J{M
1

+ M2

and equal

to

and

respectively.

APPLICATIONS TO VIBRATING SYSTEMS, ROCKETS AND COLLISIONS

218

8.38.

In Fig. 8-27 equal masses

and

B by means

constant tension T and length


from the equilibrium position

F2

and

and

to

of elastic strings of
If the

I.

AB

displacements

of the masses are

show that the equations

respectively,

lying on a horizontal

frictionless table are connected to each other

fixed points

[CHAP.

of

motion are given by

Y,

where

= k(Y 2 -2Y
ST I ml.

Y2 =

1 ),

k(Y 1

~2Y 2

Fig. 8-27

A
.39.

Prove that the natural frequencies of the vibration


respectively by

/3T

JL

2tt

Problem 8.38 are given

9T

and

2w \ ml

in

V ml

and describe the modes of vibration.


.40.

Find the normal frequencies and normal modes of vibration for the system of
m x and m 2 connected by springs as indicated in Fig. 8-28.

particles of masses

CHANGING MASS. ROCKETS


8.41.

Prove that the

(a)

by the rocket of Problem

total distance traveled

<t

/m

at\
In

What

is the maximum height which the rocket can


achieve this maximum height?

(b)

8.42.

8.5 in

time

is

given by

- -gt
y 2
2

reach and

how long

will

it

take to

falls in a constant gravitational field.


At the
starts to fall it ejects gas at the constant rate a in the direction of the gravitational
field and at speed v with respect to the rocket. Find its speed after any time t.

Suppose that a rocket which starts from rest

instant

Ans.

it

gt

v In

at J
8.43.

How

far does the rocket of Problem 8.42 travel in time tl

gt 2

Ans.

how Problem

at
)

In

at

8.44.

Describe

8.45.

Discuss the motion of a two-stage rocket,


rocket takes over.

8.42 can be useful in

making a
i.e.

"soft landing" on a planet or satellite?

one in which one part falls

off

and the other

COLLISIONS OF PARTICLES
8.46.

gun

which
with horizontal velocity v into a block of wood of mass
If the bullet becomes embedded in the wood, (a) determine
the subsequent velocity of the system and (b) find the loss in kinetic energy.
fires

a bullet of mass

rests on a horizontal frictionless plane.

Ans.
8.47.

8.48.

mv/(M + m)

(a)

Work Problem

ball

which

(b)

mMv 2 /2(M + m)

8.46 if the block is

is

moving away from the gun with

dropped from a height H onto a


Ans. -Jh/H

floor

velocity V.

rebounds to a height

< H. Determine

the coefficient of restitution.

8.49.

mass

If

the

m m2 (l
1

traveling with speed v on a horizontal plane hits another mass


2 which is at rest.
prove that there is a loss of kinetic energy equal to
of restitution is e

coefficient
e 2 )v 2

/2(m 1

+ m2 ).

CHAP.

8.50.

APPLICATIONS TO VIBRATING SYSTEMS, ROCKETS AND COLLISIONS

8]

billiard

ball

another billiard ball obliquely at an angle of 45

strikes

centers at the time of impact.


the first ball will "bounce off".

8.51.

If the coefficient of restitution

Ans. tan" 1

Let the masses of two colliding particles be


If the coefficient of restitution is
be v t ,v2
.

of the collision

is

(v x

=
,

8.52.

is

8.53.

momentum which

Prove that the


(1

-_,

v2

e )(v 1

- v2

is

- c2

2 (l

m
e,

is

1/2, find

219

with their line of


the angle at which

(3/5)

,m2 and

their respective velocities before impact


in kinetic energy as a result

prove that the loss

).

transferred from the

first particle of

Problem

8.51 to the second

).

ball is dropped from a height h above a horizontal plane on to an inclined plane of angle a which
resting on the horizontal plane. Prove that if the coefficient of restitution is e, then the ball will
next hit the incline at a point which is at a distance 4e(l + e)h sin below the original point
of impact.
is

FOURIER SERIES, ODD AND EVEN FUNCTIONS, FOURIER SINE AND COSINE SERIES
8.54.

Graph each of the following functions and find their corresponding Fourier series using properties
of even and odd functions wherever applicable.

= j_
8

f(x)

Ans

8.55.

= {

f{X)

(o)

W |

(h\
{b)

(1-

-
V
**

Peri d 8

4
.

cos

(")

fW ~

n^

nir)

_
nvx

r
C0S

n*

(a)

(b)

no discontinuities

Expand

Ans.

0,

f(x)

2, 4, ...

...

~T

<

<

[1-6

id)

<

a;

3
2

40

?>

4x

'

< x <

Period 10

_ s<x<0

Period 6

\ 6(cos nv

1)

nirx

COS "3

*P

>

0^x<3

(2x

= |

10

6 coanir

(6)

How
=5

(a)

0,

10, 20,

3, 9, 15,

20

what value

a Fourier series of period

8.

5 ffa;
J^
^cos
52

v,

,,

,,^

(6)

/(0)

in

< x <

cos x,

(d)

nirx\

should f(x) be defined at

f(x)

nV~ Sin -3-|

^+

<?
J E. j_
+ 1.
cos
^cos^F
+
JHcos^f
32
4

Expand

(c)

4<<8

i 6-

(a)

a
Ans.

a Fourier sine series.

in

and

a;

so that the series will converge to fix) for

7T?

8.58.

Period 4

cos n,)

(2-

8.57.

2
4

In each part of Problem 8.54, tell where the discontinuities of f(x) are located and to
the series converges at these discontinuities.

Ans.

8.56.

<
<

x
x

-A^x^O

C-x

IX

{b)

<
<

(a)

-v
2

ttsin2/i#

4n2-1

,,

fW)

(a)
Expand in a Fourier series f(x) - cos a;,
< x < v if the period is w, and
the result of Problem 8.57, explaining the similarities and differences if any.

Ans. Answer

is

the

same as

in

Problem

8.57.

(6)

compare with

APPLICATIONS TO VIBRATING SYSTEMS, ROCKETS AND COLLISIONS

220

fx
8.59.

Expand

f{x)

< n

[8
.

32 ,

Ans.

(a)

ir

<
<

<
<

x
x

0,

4
in a series of

(6) cosines.

(a) sines,

nvx

^ Bm T sin

v%

[CHAP.

^2
16

/L ,
(6)

/ 2 cos

- cos mr

n-ir/2

l\
mrx
) cos

THE VIBRATING STRING


8.60.

Solve the boundary-value problem

(a)

d2

Yx (0,t) =

Y(r,t)

0,

0<x<,,t>0

Y(x,0)

h,

Give a physical interpretation of the problem in

(6)

Aws.

8.61.

d2

1W =

F(a,

t 2 2w
=i

*)

'

sin (w
1

- 4)
2

sin (2n

0,

(x,0)

(a).

- l)t

Solve the boundary-value problem

Y
F(0,

t)

= Y xx -

tt

Y{ir,t)

0,

<

Y(x,0)

0,

<

*,

>

px(ir-x),

(x,0)

and interpret physically.


Ans.

Y(x,

4(2/ta?

t)

flr)

/o

n =i (2w

ir

sin

,^
1)^

2w ~ !)* cos ( 2w

Y = d2 Y
4 -^j
sin 4x, Y (x, 0) =
d2

8.62.

Find a solution of the equation

(a)

Y( v

t)

0,

Y(x, 0)

0.1 sin

cally the

boundary conditions

Ans.

Y(x,

(a)

t)

0.1 sin

+ 0.01

-rp-

and the

in (a)

+ 0.01

x cos 2t

sin

F(2,

Aws.

8.64.

the

Solve

(a)

t)

0,

(a)

boundary-value

4a;

Y(x, 0)

= 0.05x(2 - x), Y

*v
v
F(ar, t) =
,

1-6
-^j

4
2

1
(

2w

(x, 0)

_ i)3 sm

<

<

ir,

3.2 2,
*

Y(x,
=t 2

t)

(2^71)4

(2w

subject

<

where

0,

(2n - l)irx

C0S

conditions
0.

(6)

F(0,

t)

Interpret physi-

to

<2,t>

- l)yg

conditions

the
0.

(6)

Y(0,

t)

0,

Interpret physically.

3(2n-lM
2

<j

>

Solve Problem 8.63 with the boundary conditions for Y(x,0) and
Y(x, 0) = 0, Y t (x, 0) = 0.05sc(2 - x), and give a physical interpretation.

Ans.

cos St

d Y
-^ = 9 -^

problem

the

satisfies

for

solution.

Q2V

8.63.

which

_ !)* ~ i^"' _ x

(x,0)

interchanged,

i.e.

B(2n-l)wt
2

MISCELLANEOUS PROBLEMS
8.65.

spherical raindrop falling in a constant gravitational field grows by absorption of moisture from
surroundings at a rate which is proportional to the instantaneous surface area. Assuming
Ans. ^g
that it starts with radius zero, determine its acceleration.

its

8.66.

8.67.

It fires a
rests on a horizontal plane having coefficient of friction ft.
with muzzle velocity v in a direction making angle a with the horizontal.
Determine how far back the cannon will move due to the recoil.

cannon of mass

projectile of

ball is

the plane.
is

mass

in a direction making angle a with


the coefficient of restitution, prove that the velocity of the ball after the impact
-1 (e tan a) with the horizontal.
1 - (1 e2 ) sin 2 a in a direction making angle tan

thrown with speed v onto a smooth horizontal plane


If

given by v

e is

CHAP.

8.68.

8.69.

APPLICATIONS TO VIBRATING SYSTEMS, ROCKETS AND COLLISIONS

8]

Prove that the

8.70.

8.71.

8.72.

time taken for the particle of Problem 8.67 to come to rest

total theoretical

8.27, page 213, moves from the top


moves a distance {mL cos a)/{M + m).

Prove that while the particle of Problem


incline, the incline

Prove that the loss of kinetic energy of the spheres of Problem


where fi is the reduced mass m 1 m 2 /{m, 1 + m 2 ).
Prove that the acceleration of a double incline of mass
table

is

is

If

(m 1

given by

M + my sin

sin a x cos ay
2

ay

^(^i

[Fig.

8-29]

mass

- 3/t)/(3 -

e *)

on a smooth

cos ay

g sin

by

ay)

m[ +
;

m (A
m2
2

cos a 2

sin a 2 )
.

Pig. 8-30

See Fig. 8-30.

/x).

A gun of mass
The gun fires a
How

(1 ~~

inclined plane of the same mass which is on a horizontal plane with


Prove that the inclined plane moves to the right with acceleration equal

fi.

located on an incline of angle a which in turn is on a smooth horizontal plane.


Find the
horizontally away from the incline with speed v
mass

is

bullet of

Ans. (mv cos a)/M up the incline

recoil speed of the gun.


8.75.

is

2
2)

down an

in slides

coefficient of friction

8.74.

which

cos

the acceleration of the incline in Problem 8.71, prove that the accelerations of the masses

Fig. 8-29

to (1

v2

m2 sin 2 a 2
my(A

bottom of the

to the

cos 0y

8.9 is

is

sin o 2 cos a 2 )g

relative to the incline are given numerically

8.73.

221

far up the plane will the gun of Problem 8.74


(b) has coefficient of friction /*?

move before

it

comes to rest

if

the incline

is

(a) frictionless,

8.76.

is dropped from a height H above the plate AB of


which is supported by a spring of constant k. Find the
speed with which the weight rebounds.

weight

Fig. 8-31

8.77.

thrown with speed v at angle a with a horizontal plane.


rebounds successively from the horizontal plane, determine

ball is

If it

its location
is e

8.78.

after

n bounces. Assume that

and that air resistance

Work Problem
ball is

8.79.

the coefficient of restitution


Fig. 8-31

negligible.

8.77 if the horizontal plane

thrown downward,

(a)

Obtain the equation


for the

is

(b)

is

replaced by an inclined plane of angle

/?

and the

thrown upward.

page 195, for the vibrating string by considering the equations of motion
Problem 8.29, page 215, and letting N - .

(1),

particles of

8.80.

-*
Prove that as
the normal frequencies as given in Problem 8.31, page 216, approach those
for the continuous vibrating string.

8.81.

Prove that for


(a)

(6)

x(ir

x(v

x) =
x)

ir,

t2

f cos

1x

cos
i

l2

6
8 /sin x
l3

sin

3a;

33

cos

4a;

~m

"i"

22
sin

5a;

53

6a;

T^o

32

APPLICATIONS TO VIBRATING SYSTEMS, ROCKETS AND COLLISIONS

222

8.82.

Use Problem
n =i

8.83.

n2

Y =

Prove that

show that

8.81 to

[CHAP.

f(x

+ ct) +

g(x

12

- ct)

n =i (2n

32

a solution of the equation

is

d*Y

dt 2

and discuss the connection of

l) 3

d2

dx 2

with the problem of the vibrating string.

this solution

2
T C /dY\
-r
dx.
V =
x

8.84.

(a)

(b)

8.85.

(a)

(b)

(c)

8.86.

Prove that the total potential energy of a vibrating string

V = ~-

Thus show that

n%

an cos

"

~^i

^n

si n

Prove that the total kinetic energy of the vibrating string

Thus show that

Can the

K.E.

..

kinetic energy be infinite?

n2

a n cos ~i

is

T-

^<r

2
)

is

E =

2T
ttir

4f

2
n=l

n2 ( a n

&!)

Find the potential energy, kinetic energy and total energy for the string of
page 210, (b) Problem 8.28, page 214.

8.88.

If

to the instantaneous transverse velocity

is

problem of the vibrating string, prove that

its

equation of motion

is

(a)

Problem

8.20,

taken into account in the


d2

-ttj-

dY
P

~qT

2
d
c z T-g"

Prove that the frequencies of vibration for the damped string of Problem 8.88 are given by

y/nW&IP - p 2 /4, n =
8.90.

b n sin

8.87.

8.89.

Explain.

Prove that the total energy of a vibrating string

damping proportional

K.E.

is

1, 2, 3,

and
Solve the problem of the damped vibrating string if the string is fixed at the endpoints x =
= I and the string is (a) given an initial shape f(x) and then released, (b) in the equilibrium
(c) given an initial shape f(x) and velocity
position and given an initial velocity distribution g(x),
x

distribution g{x).

8.91.

8.92.

Work
Work

(a)

Problem

8.84(a),

case where the string

8.93.

damped vibrating

the problem of the

is

(6)

Problem

replaced by

string

if

8.85(a),

gravitation

(c)

particles as in

is

taken into account.

Problem 8.86, (a*) Problem 8.88 for the


Problem 8.29, page 215.

In Fig. 8-32 the double pendulum system is free to vibrate in a vertical


Find the normal frequencies and normal modes assuming small

plane.

vibrations.

8.94.

Work Problem

8.93 if there is

a string of length

8.95.

an additional mass

m3

suspended from

m2

by

3.

Generalize the motion of


particles

8.96.

(a)

Problem

8.1,

(6)

Problem

8.34 to

equal

and springs.

In Problem 8.95 investigate the limiting


physical significance of the results.

case

as

N -

Discuss the

Y///////////

CHAP.

8.97.

APPLICATIONS TO VIBRATING SYSTEMS, ROCKETS AND COLLISIONS

8]

223

Solve the boundary-value problem

Y _

d2

F(o,t)

F(U) =

o,

fl2

F
=

r(*,o)

o,

/(),

(*,o)

and give a physical interpretation.

8.98.

Work Problem

8.99.

Work Problem

8.97 if the condition

Q dY
Pin

W+

and interpret physically.

8.101.

is

replaced by

c2

d*Y

(as,

0)

g(x).

by

8.97 if the partial differential equation is replaced

d*Y

8.100.

{x, 0)

+ asmut
.

Jx^

Set up the differential equations and initial conditions for the motion of a rocket in an inverse
square gravitational field. Do you believe these equations can be solved? Explain.

Two

bodies [such as the sun and earth or earth and moon] of masses
and
x
2 move relative
each other under their mutual inverse square attraction according to the universal law of
gravitation. If r x and r 2 are their position vectors relative to a fixed coordinate system, and
r = r t r2
prove that their equations of motion are given by
to

..

ri

This

8.102.

is

called the

0.

ri
,.

new

Thus show that


..

or,

-r

{r 2

t)

^3

G(m x + m 2 )r x

_
~

^3
..

..

on subtracting,

r2

G^ + m

such that
then

2 )r 2

G{m + m2 )r

..
'

i.e.

m2

r3

to

m2

is

exactly the same as

if

the body of

mass

m2

were

Using Problem 8.102, obtain the orbit of mass m t relative to w 2 and compare with the results of
Chapter 5. Are Kepler's first and second laws modified in any way? Explain.

If

is

the period of revolution of


2 prove that

about

about

m2

P2

Compare

and a

is

the semi-major axis of the elliptical path of

a3

4tt 2

G(m l + m 2

this result with Kepler's third law:

does this modified Kepler law have

8.105.

Gm

_
~

Thus show that the motion of m x relative


and its mass increased to m l + m 2

8.104.

r2

'

origin at the center of mass of the two bodies,


if we let r be the position vector of
relative to
x

fixed

8.103.

..

problem of two bodies.

In Problem 8.101 choose a

m r + m2 r2 =

Gm 2 (r - r2
_
~ ~
r3

much

In the case of the earth [or other planet] and sun,


effect? Explain.

Set up equations for describing the motion of 3 bodies under a mutual inverse square law of
attraction.

8.106.

Transform the equations obtained


relative to their center of mass.

8.107.

Work Problems

in

Problem 8.105 so that the positions of the bodies are described


believe these equations can be solved exactly?

Do you

8.105 and 8.106 for

bodies.

Chapter 9

PLANE
of

MO TION

RIGID BODIES

RIGID BODIES

system of particles in which the distance between any two particles does not change
regardless of the forces acting is called a rigid body. Since a rigid body is a special case
of a system of particles, all theorems developed in Chapter 7 are also valid for rigid bodies.

TRANSLATIONS AND ROTATIONS


A displacement of a rigid body is a

change from one position to another. If during


a displacement all points of the body on some line remain fixed, the displacement is called
a rotation about the line. If during a displacement all points of the rigid body move in
lines parallel to each other the displacement is called a translation.

EULER'S THEOREM. INSTANTANEOUS AXIS OF ROTATION


The following theorem,

called Euler's theorem, is

fundamental

in the

motion of rigid

bodies.

to

Theorem 9.1. A rotation of a rigid body about a fixed point of the body
a rotation about a line which passes through the point.

The

is

equivalent

line referred to is called the instantaneous axis of rotation.

Rotations can be considered as finite or infinitesimal. Finite rotations cannot be


represented by vectors since the commutative law fails. However, infinitesimal rotations
can be represented by vectors.

GENERAL MOTION OF A RIGID BODY. CHASLE'S THEOREM


In the general motion of a rigid body, no point of the body
the following theorem, called Chasle's theorem, is fundamental.

Theorem

9.2.

The general motion of a

plus a rotation about a suitable point which

rigid
is

may

be fixed. In such case

body can be considered as a translation

often taken to be the center of mass.

PLANE MOTION OF A RIGID BODY


The motion

1.

body is simplified considerably when all points move parallel


In such case two types of motion, called plane motion, are possible.

of a rigid

to a given fixed plane.

Rotation about a fixed axis. In this case the rigid body rotates about a fixed axis
perpendicular to the fixed plane. The system has only one degree of freedom [see Chapter 7, page 165] and thus only one coordinate is required for describing the motion.

224

CHAP.

2.

PLANE MOTION OF RIGID BODIES

9]

225

General plane motion. In this case the motion can be considered as a translation
parallel to the given fixed plane plus a rotation about a suitable axis perpendicular to

the plane. This axis is often chosen so as to pass through the center of mass. The number of degrees of freedom for such motion is 3: two coordinates being used to describe
the translation and one to describe the rotation.

The axis referred to is the instantaneous axis and the point where the instantaneous
axis intersects the fixed plane is called the instantaneous center of rotation [see page 229].

We

shall consider these two types of plane motion in this chapter. The motion of a
body in three dimensional space is more complicated and will be considered in
Chapter 10.

rigid

MOMENT OF INERTIA
A
bodies

geometric quantity which

moment

called the

is

The moment

of great importance in discussing the motion of rigid

is

of inertia.

of inertia of a particle of

mass

where r

is

The moment

where

AB

is

defined as

= mr2

(-0

of inertia of a system of particles, with


is defined as
N about the line or axis

^m r

=
r%,

about a line or axis

the distance from the mass to the line.

masses mi, ra 2
/

. ,

mr

mr

mNr N

(2)

rN are their respective distances from

AB.

r2 ,

AB

The moment of inertia of a continuous distribution of


mass, such as the solid rigid body
of Fig. 9-1, is given by

where r

is

= J

r2

dm

(S)

the distance of the element of

mass

dm from AB.

Fig. 9-1

RADIUS OF GYRATION
N

Let /

^m

v=i

^m

r 2 be the

moment

of inertia of a system of particles about

be the total mass of the system. Then the quantity

2 r
2m
>

AB, and

such that

(*)

is called

the radius of gyration of the system about

For continuous mass distributions

(4) is

AB.

replaced by

Jr dm
2

(5)

dm

PLANE MOTION OF RIGID BODIES

226

[CHAP.

THEOREMS ON MOMENTS OF INERTIA


Theorem 9.3: Parallel Axis Theorem. Let / be the moment of inertia of a system
1.
about axis AB and let Ic be the moment of inertia of the system about an axis parallel to
AB and passing through the center of mass of the system. Then if b is the distance between
the axes and
is the total mass of the system, we have

Ic

+ Mb 2

2.
Theorem 9.4: Perpendicular Axes Theorem. Consider a mass distribution in the
xy plane of an xyz coordinate system. Let h, I y and h denote the moments of inertia about
the x, y and z axes respectively. Then

h =

IX

+Iy

en

SPECIAL MOMENTS OF INERTIA


The following
practice.

In

all

table

cases

shows the moments of inertia of various rigid bodies which arise


assumed that the body has uniform [i.e. constant] density.

it is

Rigid Body
1.

Moment

Solid Circular Cylinder

of radius a and mass


about axis of cylinder.
2.

Hollow Circular Cylinder


of radius a and mass

about axis of cylinder.

Wall thickness
3.

is

Ma*

Solid Sphere

Hollow Sphere
of radius a and mass
about a diameter.
Sphere thickness is negligible.

5.

\Ma?

negligible.

of radius a and mass


about a diameter.
4.

of Inertia

Rectangular Plate
of sides a and 6 and
mass
about an axis

|Ma2

Ma2

JLM(a2

62)

perpendicular to the plate


through the center of mass.
6.

Thin Rod

of length a and mass


about an axis perpendicular
to the rod through the
center of mass.

^Ma*

COUPLES
A set of

two equal and parallel forces which act in


opposite directions but do not have the same line of action
[see Fig. 9-2] is called a couple. Such a couple has a turning
effect, and the moment or torque of the couple is given
by

-F

F.

The following theorem

is

important.

Fig. 9-2

in

CHAP.

PLANE MOTION OF RIGID BODIES

9]

227

Theorem 9.5. Any system of forces which acts on a rigid body can be equivalently
replaced by a single force which acts at some specified point together with a suitable couple.

MOMENTUM ABOUT A FIXED

KINETIC ENERGY AND ANGULAR

AXIS

Suppose a rigid body is rotating about a fixed axis with


angular velocity
which has the direction of the axis AB
[see Fig. 9-3].
Then the kinetic energy of rotation is
given by
<>

T =
where

is

the

moment

i/o>

(8)

of inertia of the rigid body about

the axis.
Similarly the angular

momentum
O = 7o>

is

given by
(9)

Fig. 9-3

MOTION OF A RIGID BODY ABOUT A FIXED AXIS


Two important methods for treating the motion of a rigid body about a fixed axis are
given by the following theorems.
all

Theorem 9.6: Principle of Angular Momentum. If A is the torque or the


external forces about the axis and O = I<a is the angular momentum, then

A
where a

is

dt

(/)

IS,

It

moment

of

(10)

the angular acceleration.

Theorem 9.7: Principle of Conservation of Energy. If the forces acting on the rigid
body are conservative so that the rigid body has a potential energy V, then

T + V

i/w 2

+ V = E =

constant

()

WORK AND POWER


%

Consider a rigid body


capable of rotating in a
plane about an axis O perpendicular to the plane, as
indicated in Fig. 9-4. If a is the magnitude of the torque
applied to the body under the influence of force F at
point A, the work done in rotating the body through
angle de is

dW = AdO

(12)

and the instantaneous power developed


cp

where

We

is

dW =
dt

is

A.

(13)

the angular speed.

Fig. 9-4

have the following

Theorem 9.8. The total work done in rotating a rigid body from an angle 0i where
the angular speed is Wl to angle
where the angular speed is a is the difference in the
2
kinetic energy of rotation at w and o>
In symbols,
x
2
.

Ade

/<of

/o>f

(U)

PLANE MOTION OF RIGID BODIES

228

IMPULSE. CONSERVATION OF ANGULAR


The time

[CHAP.

MOMENTUM

integral of the torque


2

Jt

Adt

(15)

h
is

called the angular impulse

We

from time U

t2 .

have the following theorems.

Theorem

9.9.

The angular impulse

symbols

is

equal to the change in angular

momentum. In

tf

Adt

Theorem
rigid

to

body

9.10:

is zero,

O2

Oi

(16)

Conservation of Angular Momentum. If the net torque applied to a


then the angular momentum is constant, i.e. is conserved.

THE COMPOUND PENDULUM

Let
[Fig. 9-5] be a rigid body which is free to oscillate
in a vertical plane about a fixed horizontal axis through O
under the influence of gravity.
call such a rigid body a

We

compound pendulum.
Let C be the center of mass and suppose that the angle
between OC and the vertical OA is 0. Then if h is the
moment of inertia of % about the horizontal axis through O,
is the mass of the rigid body and a is the distance OC,
we have for the equation of motion,

f sm0

(17)

Jo

For small

Fig. 9-5

oscillations the period of vibration is

P =
The length

of the equivalent simple

pendulum
I

The following theorem

is

2ir\/Io/Mga

(18)

is

= hiMa

(19)

of interest.

Theorem 9.11. The period of vibration of a compound pendulum is a minimum when


the distance OC = a is equal to the radius of gyration of the body about the horizontal
axis through the center of mass.

GENERAL PLANE MOTION OF A RIGID BODY


The general plane motion of a rigid body can be considered as a translation parallel to
the plane plus a rotation about a suitable axis perpendicular to the plane. Two important
methods for treating general plane motion of a rigid body are given by the following
theorems.
of

Theorem 9.12: Principle of Linear Momentum. If


mass of a rigid body relative to an origin O, then
(Mr)

Mr =

r is the position vector of the center

jfi

where

the body.

is

the total mass, assumed constant, and

is

(20)

the net external force acting on

CHAP.

PLANE MOTION OF RIGID BODIES

9]

229

Principle of Angular Momentum. If Ic is the moment of inertia of the


rigid body about the center of mass, o> is the angular velocity and A c is the torque or
total moment of the external forces about the center of mass, then

Theorem

9.13.

Tt^

>)

7 c"

(21)

Principle of Conservation of Energy. If the external forces are conservative so that the potential energy of the rigid body is V, then

Theorem

9.14.

T + V = %mr2 +

i/ c o

+V = E =

constant

(22)

2
Note that imr2 = \mv 2 is the kinetic energy of translation and i/ c *
energy of rotation of the rigid body about the center of mass.

is

the kinetic

INSTANTANEOUS CENTER.
SPACE AND BODY CENTRODES
Suppose a rigid body % moves parallel to a given
xy plane of Fig. 9-6. Consider
an x'y' plane parallel to the xy plane and rigidly
fixed plane, say the

attached to the body.

As the body moves there will be at any time t


a point of the moving x'y' plane which is instantaneously at rest relative to the fixed xy plane.
This point, which may or may not be in the body,
is called the instantaneous center. The line perpendicular to the plane and passing through the instantaneous center is called the instantaneous axis.

O
Fig. 9-6

As the body moves, the instantaneous center also moves. The locus or path of the
instantaneous center relative to the fixed plane is called the space locus or space centrode.
The locus relative to the moving plane is called the body locus or body centrode. The motion
of the rigid body can be described as a rolling of the body centrode on the space centrode.
The instantaneous center can be thought of as that point about which there is rotation
without translation. In a pure translation of a rigid body the instantaneous center is
at infinity.

STATICS OF A RIGID BODY


The statics or equilibrium of a rigid body
The following theorem is fundamental.

Theorem
is

9.15.

A necessary and

is

the special case where there

sufficient condition for

is

no motion.

a rigid body to be in equilibrium

that

F =
where F

is

0,

A =

the net external force acting on the body and

PRINCIPLE OF VIRTUAL
Since a rigid body

is

work and D'Alembert's

(23)

is

the net external torque.

WORK AND D'ALEMBERT'S PRINCIPLE

but a special case of a system of particles, the principle of virtual


principle [see page 171] apply to rigid bodies as well.

PLANE MOTION OF RIGID BODIES

230

[CHAP.

PRINCIPLE OF MINIMUM POTENTIAL ENERGY. STABILITY


At a position of equilibrium the net external force
conservative and V is the potential energy,

F =

?I =

or in components,

dx

-vV

dV

U
'

so that if the forces are

(24)

dV_

dy

is zero,

'

dz

(25)

In such case V is either a minimum or it is not a minimum. If it is a


minimum the
equilibrium is said to be stable and a slight change of the configuration will restore
the
body to its original position. If it is not a minimum the body is said to be in
unstable
equilibrium and a slight change of the configuration will move the body
away from its
original position. We have the following theorem.

Theorem
equilibrium

9.16.

is

that

its

necessary and sufficient condition for a rigid body to be in stable


potential energy be a minimum.

Solved Problems
RIGID BODIES
9.1.

rigid

body

[Fig. 9-7] is

DEF,

i.e.

form of a triangle

in the

moved

the vertices A,

ABC

in a plane to position

and C are car-

Show that
the motion can be considered as a translation plus a rotation about a suitable point.
ried to D,

and

respectively.

Choose a point G on triangle ABC which corresponds to the point


on triangle DEF. Perform
the translation in the direction GH so that triangle
ABC is carried to A'B'C. Using as center of

rotation perform the rotation of triangle A'B'C


through the angle 9 as indicated so that A'B'C is
carried to DEF. Thus the motion has been accomplished by a translation plus a rotation.

9.2.

Give an example to show that

Fig. 9-7

finite rotations

cannot be represented by vectors.

Let A x represent a rotation of a body [such as the rectangular parallelepiped of Fig. 9-8(a)]
about the x axis while A y represents a rotation about the y axis. We assume that such rotations
take place in a positive or counterclockwise sense according to the right hand rule.

tf

V
r

(6)

Fig. 9-8

CHAP.

231

PLANE MOTION OF RIGID BODIES

9]

indicated position and perform the rotation


In Fig 9-8(a) we start with the parallelepiped in the
indithe rotation about the axis
then
and
9-8(6)
Fig.
indicated
in
axis
as
x
the
A x about
9-8(o).
Fig.
A
on
A
rotation
the
of
+
result
x
y
Thus Fig. 9-8(c) is the
cated in Fig. 9-8(c).
position as in Fig. 9-8(a), but this
In Fig 9-9(a) we start with the parallelepiped in the same
as indicated in Fig. 9-9(6) and then the
time, we first perform the rotation A y about the y axis
Thus Fig. 9-9(c) is the result of the
9-9(c).
Fig.
in
rotation A x about the x axis as indicated

Ay + Ax

rotation

on Fig.

9-9(a).

not the same as that of Fig. 9-9(c),


Since the position of the parallelepiped of Fig. 9-8(c) is
A y + A x Thus the commutative
as
same
the
not
A
is
+
A
x
we conclude that the operation
y
by vectors.
law is not satisfied, so that A x and A y cannot possibly be represented
.

MOMENTS OF INERTIA
9.3.

masses m and ra 2 respectively are connected by a rigid massless


inertia of the
rod of length a and move freely in a plane. Show that the moment of
the center of
through
passing
system about an axis perpendicular to the plane and
2
mass is iacl where the reduced mass

Two

ix

particles of

m\m<2.l{m\

+ W2).

Let r x be the distance of mass mj from the


center of mass C. Then a r x is the distance
Since C is the center
of mass
2 from C.
of mass,

r1

= m 2 (a r

Thus the moment

of inertia about

m r\ + m
x

2 (a

2
x)

= m.

an axis through C

ma
m + m2 / + m
2

9.4.

m^a
mj + W2

from which

t)

Fig. 9-10

and

w^a
+ W^

rt

!!

is

m a \2
\ m + m2/
x

wii%

fllj+W^

Find the moment of inertia of a solid circular cylinder of radius


about the axis of the cylinder.
mass

a2

a,

height h and

Method

1,

using single integration.

Subdivide the cylinder, a cross section of which appears


in Fig. 9-11, into concentric rings one of which is the element shown shaded. The volume of this element is
(Area) (thickness) = (2irrdr)(h) = 2rrrh dr

dm = 2varh dr.
inertia of dm is
r2 dm 2irora h dr

and the element of mass

The moment
where a

is

of

is

the density, and thus the total

moment of

inertia is

id

2wars h dr

= ^wahaA

(1)

ndP-

Fig. 9-11

PLANE MOTION OF RIGID BODIES

232

Then

since the

mass

= |Ma2

find /

Method

is

M
we

[CHAP.

2worh dr

aira 2 h

using double integration.

2,

Using polar coordinates


the

(r, 8), we see from Fig. 9-12 that


of inertia of the element of mass dm distant
the axis is

moment

r from

r2
since hr dr do

dm =

r2 ahrdrdo

ahr* dr do

the volume element and a is the mass per


unit volume (density). Then the total moment of inertia is
is

sa
J-2ir

ahr^drdo

\irohcfi

(1)

0=0 "r=0

The mass

of the cylinder

is

given by

Fig. 9-12

pa

s2tt

ahr dr do

aira2 h

^6=0 ^r=0
which can also be found directly by noting that the volume of the cylinder
equation (1) by (2), we find I/M = %a 2 or I = \Ma2

is

ira2 h.

Dividing

9.5.

Find the radius of gyration, K, of the cylinder of Problem

Since

9-6.

= I/M = \a

2
,

K=

a/y/2

\a^2.

Find the (a) moment of inertia and


with sides a and b about a side.
Method
(a)

1,

9.4.

radius of gyration of a rectangular plate

(b)

using single integration.

The element of mass shaded in Fig. 9-13 is ab dx, and its moment of
(ab dx)x 2 = abx 2 dx.
Thus the total moment of inertia is

inertia about the

y axis

is

ia

J
Since the total mass of the plate

.(b)

K2 = I/M = %a2

or

K = a/y[S =

is

abx 2 dx

M = aba,

%aba*

we have I/M =

-Ja

or

= $Ma2

%ayfz.

y
dx

dm =

a dy dx

Fig. 9-13

Method

2,

Fig. 9-14

using double integration.

Assume the plate has unit thickness. If dm = adydx is an element of mass [see Fig. 9-14],
the moment of inertia of dm about the side which is chosen to be on the y axis is x 2 dm = ax 2 dy dx.
Then the total moment of inertia is

Xa
The

total

mass of the

plate

is

M = aba.

s\h
I

ax 2

dydx

Then, as in Method

j^aba?

1,

we

find

= \Ma2

and

K = $ayfs.

CHAP.

9.7.

PLANE MOTION OF RIGID BODIES

91

231

Find the moment of inertia of a right circular cone of height h and radius a
about

its axis.

Method

1,

using single integration.

The moment of

inertia of the circular cylindrical

PQR

which is represented by
by Problem 9.4,

disc one quarter of

Fig. 9-15

is,

\{irr 2 o dz)(r 2 )

From
Then the

volume vr dz and radius


h

Fig. 9-15,
total

iror*dz

since this disc has

moment

in

or

r.

of inertia about the z axis

dz

is

JL^jjTra4 <rh

Also,

Fig. 9-15

Thus /

Method

2,

^Ma2

ira2 ha

using triple integration.

Subdivide

shown

dz

the

one quarter of which is


elements of mass dm as indi-

cone,

in Fig. 9-16, into

cated in the figure.


In cylindrical coordinates (r, 8, z) the element of
dm of the cylinder is dm or dr de dz where
the density.

mass
a

is

The moment of
r2

As

in

Method

Then the

total

inertia of

dm =
h

1,

dm

about the

ha

moment

z axis is

or3 dr de dz

a
\
of inertia about the z axis

pa
J27r

0=0

The

total

mass of the cone

Fig. 9-16

is

sn,(a
>h(a r)/a,
r)/a

dr de dz

Jr =0 ^z=0

is

Jiiarj/r

or dr de dz
*S0=O

9.8.

^Ma

^n-aPho

^r=0 *^2=0

which can be obtained directly by noting that the volume of the cone

Thus

^iraAoh

is

\tra2 h.

2.

Find the radius of gyration

K2 = I/M = ^a2

and

of the cone of Problem 9.7.

K = ay/^ =

^ay/SO.

THEOREMS ON MOMENTS OF INERTIA


9.9.

Prove the parallel axis theorem [Theorem

9.3,

page 226].

Let OQ be any axis and ACP a parallel axis through the centroid C and distant 6 from OQ.
In Fig. 9-17 below, OQ has been chosen as the z axis so that
is perpendicular to the xy plane
at P.

AP

PLANE MOTION OF RIGID BODIES

234

[CHAP.

If b x is a unit vector in the direction OP,


then the vector OP is given by

where

b is

6bi

constant and

(1)

the distance between

is

axes.

Let

mass m

and

be the position vectors of

x'v

O and C respectively.
C relative to O

relative to

If r

then

the position vector of

is

we have

rv

The

moment

total

OQ

about axis

x'v

(jg)

of inertia of

masses

all

is

The

m^.bi)*

moment

total

Fig. 9-17

(3)

of inertia of all masses

ACP

about axis

is

Then using

(2)

we

2 m^-b^

Ic

find

N
I

2 m^.bi)
K=l

2 m^-b^ +
V=l

/c

The

^ m
r=l

b,

result

22=
V

m^.b^Cf-bj) +

m, <

2 ^(f-b^
1>=1

\
.

bx

and

62

2 m

Ic

+ Mb*

wh-rj

[Problem 7.16, page 178].

v=l

easily extended to continuous

is

fbi) 2

i'b l =

26

2 mX'bi +
V=l

since

(4)

mass systems by using integration

in

summation.

9.10.

Use the

parallel axis

theorem

to find the

moment

of inertia

of a solid circular cylinder about a line on the surface of


the cylinder and parallel to the axis of the cylinder.
Suppose the cross section of the cylinder is represented as in
Then the axis is represented by C, while the line on the
surface of the cylinder is represented by A.
Fig. 9-18.

If

is

the radius of the cylinder, then by Problem 9.4 and the

parallel axis

theorem we have

jA

9.11.

Ic

Ma?-

= Ma2 + Ma* -

f Ma*

Fig. 9-18

Prove the perpendicular axes theorem [Theorem


9.4, page 226].
Let the position vector of the particle with mass
xy plane be

in the

z axis

is

w^|r| 2

Then the

x vi

+ yj

The moment of

[see Fig. 9-19].

inertia of

m about

the

total

about the z axis

is

moment

of inertia of all particles

*
Fig. 9-19

place of

CHAP.

PLANE MOTION OF RIGID BODIES

9]

2hr|r|
v= \

=.

2 m

v=l

where Ix and JB are the

The

9.12.

result

moments of

total

235

v= l

xl

U(^ + a

m,!^

I~

v =i

inertia about the x axis

and y axis respectively.

easily extended to continuous systems.

is

Find the moment of inertia of a rectangular plate with sides


a and b about an axis
perpendicular to the plate and passing through a vertex.
Choose the rectangular plate [see Fig. 9-20] in the
xy plane with sides on the x and y axes. Choose the
z axis perpendicular to the plate at

From Problem

we have for the moments of


and y axes,

9.6

inertia about the x

Ix

a vertex.

$Mb*,

= ^Ma2

Iy

Then by the perpendicular axes theorem the moment


of inertia about the z axis

Iz

Ix

is

Iy

M(&2

+ a2)

= $M(a + b 2
2

Fig. 9-20

COUPLES
9.13.

Prove that a force acting at a point of a rigid


body can be equivalents replaced
by a single force acting at some specified
point together with a suitable couple.
Let the force be F acting at point P
as in
t

Fig. 9-21. If
the effect of
forces t t and

Q
F

any

is

alone
at Q.

In particular

if

specified point, it is seen that

we

same

the

is

choose

ft

if

= F u

has the same magnitude as Fj but


direction,

we

same as the

is

i.e.

if

opposite in
alone is the

see that the effect of F


x
effect of the couple formed

- ~ F [which
force fj = F

fi

we apply two

by F t and

has moment r,XF,] together with the

Fig. 9-21

1#

9.14.

Prove Theorem

9.5

witlTsS cXr ^

By Problem 9.13 we can replace the force F


at P by the force P at
Q plus a couple of moment
r X F. Then the system
of forces P lf F2 ,
, F
N at
points P lt P 2 ,...,P
N can be combined into forces
Fj, F 2
at Q having resultant
, FN
.

"*" "*

page 227: Any system of forces which acts


on a rigid body can
a
3 Single f0Ke WhiCH
at
er

.Py

F = F + F2 +
1

+ Fn

together with couples having moments

'iXFj,

r2

XF2

..., r N

XFN

which may be added to yield a single couple.


Thus
the system of forces can be equivalent^
replaced by

the single force

acting at

together with a couple

Fig. 9-22

PLANE MOTION OF RIGID BODIES

236

MOMENTUM

KINETIC ENERGY AND ANGULAR


9.15.

[CHAP.

body rotates about a fixed axis with


, prove that the kinetic energy
of rotation is T = ^/w 2 where / is the moment of

If a rigid

angular velocity

inertia about the axis.


Choose the axis as AB in Fig. 9-23. A particle P of
m v will rotate about the axis with angular speed .
it will describe a circle PQRSP with linear speed

mass
Then
vv

where r v

ur v

is

its

distance

and the

energy of

total kinetic

^rn v

^rl

v=

w r 2

is

-I (

the

Thus

its

^m v v% = mv w 2 r 2

is

particles

all

=
where

from axis AB.

AB

kinetic energy of rotation about

is

wr2

)<o 2

v= l

" 2

moment

of inertia about

AB.

The result could also be proved by using integration in


place of summation.

9.16.

Prove that the angular

momentum

The angular momentum of

momentum

AB

where

The

v=l

wrf

is

2 m

the

moment

result could also be proved

is

O=

r*m.

Then the

total

/.

angular

is

AB

about axis

is

Problem 9.15

of the rigid body of

particle

of all particles about axis

Fig. 9-23

r2
,

o>

iv r

2
)

/<*

AB.

of inertia about

in place of

by using integration

summation.

MOTION OF A RIGID BODY ABOUT A FIXED AXIS


9.17.

momentum

Prove the principle of angular


axis [Theorem 9.6, page 227].
By Problem

A=

7.12,

page 176, since a rigid body

the torque or moment of


total angular momentum about the axis.
dtl/dt

Since

9.18.

where

O=

/<>

for a rigid body rotating about a fixed

is

by Problem

9.16,

Aa _
-

all

d ir \
(Ia)
di

is a special case of a system of particles,


external forces about the axis and Q is the

'

!T

da

Tt

/i.

Prove the principle of conservation of energy for a rigid body rotating about a
fixed axis [Theorem 9.7, page 227] provided the forces acting are conservative.
The principle of conservation of energy applies to any system of particles in which the forces
acting are conservative. Hence in particular it applies to the special case of a rigid body rotating
about a fixed axis. If T and V are the total kinetic energy and the potential energy, we thus have

T+ V =
Using the result of Problem

9.15, this

constant

can be written

^Iu> 2

E
+ V -

E.

CHAP.

PLANE MOTION OF RIGID BODIES

9]

237

WORK, POWER AND IMPULSE


9.19.

Prove equation

page 227, for the work done in rotating a rigid body


about a

(12),

fixed axis.

Refer to Fig.
unit vector

9-4, page 227.


Let the angular velocity of the body be
the direction of the axis of rotation.
The work done by F

dW =

F-tfr

fJr

F-f dt

where

9.20.

in the last

two steps we use

Prove equation

(18),

From Problem

9.21.

Prove Theorem

9.8,

A=

k and a

Ak,

% dt

'-*

C-

Jh

dt

dt

9 19
-

^d

V4-M

is

Q2 - O

equal to the change in

page 229, on the conservation of angular

momentum

zero.

From Problem

9.22, if

A=

then

Q2 = Qv

(17),

page 228, for

the fact that

The angular impulse

tl

9.10,

Then from Problem

Jf\dt

is

de/dt.

A = Idu/dt

tHat

Prove Theorem 9.9, page 228:


angular momentum.

torque

page 227.

Ade

Prove Theorem

= A w dt = Ade

m dt

=a
=

dW/dt = A de/dt = Aw

ol

9.23.

o>

and the fact that de/dt

deJl dt Zlhte

9.22.

(rXF)-(lf

if

the net

THE COMPOUND PENDULUM


9.24.

Obtain the equation of motion


a compound pendulum.
Method

1.

Suppose that the vertical plane of vibration of the


pendulum is chosen as the xy plane [Fig. 9-24]
where
the z axis through origin O is the
horizontal axis of
suspension.

Let point C have the position vector a relative


to O.
Since the body is rigid, |a| = a is constant
and is the
distance from O to C.

The only external force acting on the body is its


= -Mgj acting vertically downward. Thus

weight Mg
we have

A =
=
where k

is

total external torque

about z axis

X Mg = -a X Mgj = aMg

a unit vector in the positive

sin e

is

the

moment

_
-

(1)

Fig. 9-24

z direction [out of the

Also, the instantaneous angular velocity

so that if I

page 227, for the power developed.

Id /dt

9.19

is

is

= F.(.Xr)<ft

F-vd*

= k where k

-<ok

plane of the paper toward the reader].

is

de
~^k

of inertia about the z axis

= -*k

PLANE MOTION OF RIGID BODIES

238

O =
Substituting from

(1)

and

aMg
Method 2.
The force

angular

sin

(2)

into

fk

Mg = Mgj

_ vv =

is

A=

dil/dt,

or

|r(-7o*k)

V = Mgy +

or

cos o

= a

constant

Differentiating equation

(5)

with respect to
1 6

or, since

Show

P=

is

that

not identically zero,

for

small

- Mga

I e

'S

Sz

is

^I

<>

the height of

Then the

2-

C below
principle

the

of

(5)

t,

+ Mga

+ Mga

the

vibrations

cos

f = *,, =
(4)

cos e

I u 2

such that

is

is

|/o

(5)

92/

By Problem 9.15, the kinetic energy of rotation


conservation of energy gives

0,

Mga

d.

T + V

sin

sin $

as required.

pendulum

Problem

of

9.24

has

period

27T\/Mga/h.

For small vibrations we can make the approximation sin


becomes
,

%-+**, =

Show

that the length

of Problem 9.24

is

of a simple

so that the equation of motion

a)

Then, as in Problem 4.23, page 102, we find that the period

9.26.

^^ sin

IS

cos

-M k

/<><*

This could be seen directly since


1/
x axis taken as the reference level.
since

9.25.

z axis

dz

fly

from which

= a

about

conservative, so that the potential energy

E, _ *E j _
da;

momentum

[CHAP.

is

P=

pendulum equivalent

2jry/I

to the

/Mga.

compound pendulum

I of Ma.

The equation of motion corresponding to a simple pendulum of length


from O is [see Problem 4.23, equation (2), page 102]
6

Comparing

this equation

with

(1)

+ j

sin e

of Problem 9.25,

suspended vertically

we

(-0

see that

/Ma.

GENERAL PLANE MOTION OF A RIGID BODY


9.27.

Prove the principle of linear momentum, Theorem


plane motion of a rigid body.

9.12,

page 228, for the general

This follows at once from the corresponding theorem for systems of particles [Theorem 7-1,
page 167], since rigid bodies are special cases.

9.28.

Prove the principle of angular momentum, Theorem


plane motion of a rigid body.

9.13,

page 229, for general

This follows at once from the corresponding theorem for systems of particles [Theorem
page 168], since rigid bodies are special cases.

7-4,

CHAP.

9.29.

PLANE MOTION OF RIGID BODIES

9]

239

A solid cylinder of radius a and


mass
rolls without slipping down
an inclined plane of angle a. Show
that the acceleration is constant and

equal to f g sin

a.

Suppose that

point
after

initially the cylinder

has
with the plane and that
the cylinder has rotated

in contact

time t
through angle

[see Fig. 9-25].

The forces acting on the cylinder

at
the weight Mg acting vertically downward at the center of mass C;
(ii) the reaction R of the inclined
plane act-

time

are:

(i)

ing perpendicular to the plane; (iii) the


f acting upward along the

frictional force
incline.

Fig. 9-25

Choose the plane

which motion takes place as the xy plane, where the * axis


positive down the incline and the origin is at O.
in

If r is the position of the center of

mass at time

then by the principle of linear

t,

Mr = Mg + R +
But

sin a

-g

cos a

R=

j,

Mr*
The

total external torque

Rj,

(Mg

-/i.

sin a

Hence

(1)

(R

f)\

The

total

momentum about

angular

O =
where

moment

Ic is the

Substituting

Using

yj

can be written

- Mg

cos )j

(2)

CBXf ^

(-aj)X(-/i)

is

= - a/k
is

= -7c *k

7c (-*k)

into

we

in (2),

Mx

U)

A=

we

dCl/dt,

-a/k = -7cVk or

find

Ic e

af.

obtain

= Mg

sin a

My

f,

= R - Mg

cos a

(5)

Nowjf there is no slipping, x = ae or


= x/a. Similarly, since the cylinder remains
incline, y = 0; hence from (5), R = Mg cos a.
Using e = x/a in Ic e = af, we have f = I '/a*. From
Problem 9.4, Ic = JLMa*
c
substituting / = iMx into the first equation of
(5), we obtain
x = \g sin a as required.
9.30.

Prove that
The

Problem 9.29 the

in

coefficient of friction is

From Problem

9.29
slipping will not occur, n

9.31.

(a)

Work Problem

plane
(a)

is

^ and

In equation

(b)

(5)

we have
must

coefficient of friction

\M x = Mg
f/R

Then

at least * tan.

sin a

1 tan

and

B = Mg

cos

a.

Thus

in order that

a.

if the coefficient of friction between the


cylinder and inclined
discuss the motion for different values of /*.

9.29

of Problem 9.29, substitute

in this case the center

a particle sliding

must be

on the

f/R.

/ =
be at least

***

Note that

(3)

of inertia of the cylinder about this axis.

and U)

(3)

xi

momentum,

the horizontal axis through the center of mass


/c o,

taken as

about the horizontal axis through the center of mass

A = OXMg + OXR + CBXf =

is

down an

= pR = ^Mg

f(sin

ix

cos a

and obtain

cos a)

of mass of the cylinder moves in the same manner


as
inclined plane.
However, the cylinder may slip as well as roll.

PLANE MOTION OF RIGID BODIES

240

The acceleration due


The acceleration due
(b)

If

(sin

i.e.

/*

= ^

3/i

tan

to rolling

is

to slipping is

[CHAP.

a2f

a2/iMg cos a

Ic

\M&

ae =

g(sin a

3/*

cos

2ng cos

a.

a).

> 0, i.e. u < ^ tan a, then slipping will occur. If (sin a 3fi cos a) ^ 0,
then rolling but no slipping will occur. These results are consistent with

cos a)
a,

those of Problem 9.30.

9.32.

Prove the principle of conservation of energy [Theorem

9.14,

page 229].

This follows from the corresponding theorem for systems of particles, Theorem 7-7, page 169.
The total kinetic energy T is the sum of the kinetic energy of translation of the center of mass
plus the kinetic energy of rotation about the center of mass, i.e.,

T =

is

-imr2

|/c <o 2

If V is the potential energy, then the principle of conservation of energy states that
a constant,
T + V = -|mf2 + /c <o 2 + V =

if

9.33.

Work Problem
The

9.29

by using the principle of conservation of energy.

is composed of the potential energy due to the external forces [in this
case gravity] and the potential energy due to internal forces [which is a constant and can be
omitted]. Taking the reference level as the base of the plane and assuming that the height of
and h respectively, we have
the center of mass above this plane initially and at any time t to be

potential energy

IMf +

l/c <o 2

or,

Hh =

using

Substituting w
t, we obtain

x2

+ y2 =

\Mx2 +

^7c w2

r2

and

x sin a

\Ma 2 we

x/a and Ic

= gx

2x x

+ Mgh - MgH
x2 since y

= Mgx
x2

find

or

sin a

0,

sin a

\gx
x

sin a.

fg

Differentiating with respect to

sin a

INSTANTANEOUS CENTER. SPACE AND BODY CENTRODES


9.34.

Find the position vector of the instantaneous center for a rigid body moving
parallel to a given fixed plane.

XY

plane of Fig. 9-26 as the fixed


Choose the
plane and the xy plane as the plane attached to
and moving with the rigid body %. Let point P of
the xy plane [which may or may not be in the
rigid body] have position vectors R and r relative
to the
and xy planes respectively. If v and
v^ are the respective velocities of P and A relative
to the
system,

XY

XY

where RA
If

so

is

vA
is

+ Xr =

the position vector of

X (R RA

relative to O.

X(R-RA = -vA

Multiplying both sides of

()

(2)

by

o>

X and using

Fig. 9-26

(7),

5,

{(R
Then

(1)

to be the instantaneous center, then

that

page

VA

A )}

(R

R RA this
(R - RA )w 2 = X vA

since is perpendicular to

A )()

= -XvA

becomes
or

R = RA +

X vA

(3)

CHAP.

9.35.

PLANE MOTION OF RIGID BODIES

9]

cylinder moves along a horizontal plane.


Discuss the case where slipping

centrode.

Find the

may

241

space centrode,

(a)

(o)

body

occur.

The general motion is one where both rolling


and slipping may occur. Suppose the cylinder
is moving to the right with velocity v A [the
velocity of its center of mass] and is rotating

(a)

about

with angular velocity

Since
x va =

o>

= wk

-wv A j

and vA

so that (3) of

becomes

R = RA -

we have
Problem 9.34

v A i,

( v a)j

vA#

X~ = RA

Fig. 9-27

In component form,

Xi

Yj

- XA i +

oj

(v A /)j

x>r

X = XA

Y =

v A /a

Thus the instantaneous center is located vertically above the point of contact of the cylinder
with the ground and at height a v A /u above it.
Then the space centrode is a line parallel to the horizontal and at distance a v /u
A

above it. If there is no slipping, then v A = aa and the space centrode is the
axis while
the instantaneous center is the point of contact of the cylinder with the
axis.

(b)

The body centrode is given by


= ao3 and the body centrode

9.36.

|r|

is

v /o, or a circle of radius v /u.


the circumference of the cylinder.

In case of no slipping,

Solve Problem 9.29 by using the instantaneous center.

By Problem 9.35, if there is no slipping then


the point of contact P of the cylinder with the
plane is the instantaneous center. The motion of
P is parallel to the motion of the center of mass,
so that we can use the result of Problem 7.86(c),
page 191.

The moment of

inertia of the cylinder about

P is, by the parallel axis theorem, ^Ma2 + Ma2 =


2
The torque about the horizontal axis through
2 Ma
P is Mga sin a. Thus
.

-^{lMa2
or

Since

ae,

'e)

= Mga

sine

-r^smfl

6a

the acceleration

is

fg

sin

9.

Fig. 9-28

STATICS OF A RIGID BODY


9.37.

ladder of length I and weight Wi has one end


against a vertical wall which is frictionless and
the other end on the ground assumed horizontal.
The ladder makes an angle a with the ground.
Prove that a man of weight
m will be able to
climb the ladder without having it slip if the
coefficient of friction /* between the ladder and

the ground

is

at least

Wm + Wl

Wm+Wr

COt a.

Let the ladder be represented by AB in Fig. 9-29 and


choose an xy coordinate system as indicated.

Fig. 9-29

PLANE MOTION OF RIGID BODIES

242

[CHAP.

The most dangerous situation in which the ladder would slip occurs when the man is at the
Hence we would require that the ladder be in equilibrium in such case.

top of the ladder.

the reaction R x = R t i of the wall;


(ii) the weight
of the ladder concentrated at C, the center of
of tn ground; (v) the friction force f = /i.

The forces acting on the ladder are:


of the man; (iii) the weight
j

W m = Wm
gravity;

(iv)

R2 = R 2 j

the reaction

(i)

= W j

For equilibrium we require that

F =
where F

A =
and A

0,

(I)

the total external force on the ladder


the total external torque taken about a
suitable axis which we shall take as the horizontal axis through A perpendicular to the xy plane.

We

is

have

Wm + W, + R

F = Rx +
if

JBj

Also,

(0)

X Bi +

(0)

= tyWi

(I

^Wi

if

Wm

(0)

cos a

cos a

Then the minimum

= R

(R 1 -f)i

Wm

coefficient of friction

W, +

R2 =

(AB) X

B2 +

($1 cos a

X (R 2 j) +

j)

a.

and

(2)

XW,+

j)

sin a

+ R2

COS a

+ (-Wm -W + R 2 )j =

-Wm -

(AC)

IR 2 cos

= (WTO +

X (-

Solving simultaneously equations


/

and

(0)

(flji)

A =

If sin

COS a

we

(3),

(AB) X

1 sin a

cos a

j)
I

x (-TFJ)
j) X ( /i)

sin a

f sin a

(3)

find

R,

and

)cot

(I

(2)

Wm + W,

necessary to prevent slipping of the ladder

wm + jW
wm + w

Ra

is

cot a

MISCELLANEOUS PROBLEMS
9.38.

m2 are connected by an inextensible string of negligible mass


which passes over a frictionless pulley of mass M, radius a and radius of gyration K
which can rotate about a horizontal axis through C perpendicular to the pulley.

Two masses mi and

Discuss the motion.


Choose unit vectors
as shown in Fig. 9-30.

and

in the plane of rotation

If we represent the acceleration of mass


then the acceleration of mass ra2 is Aj.

by Aj,

Choose the tensions T x and T 2 in the string as shown


By Newton's second law,

in the figure.

w-iAj

-mgAj = T2
Thus

m g = TJ + m^j
+ m2g = -T2 + m^j

= T +

mA =
x

mxQ

(2)

lt

T t = m^g-A),

or

CO

m A T2 m g
T 2 = m2 {g + A)
2

(3)

U)

The net external torque about the axis through C

A =
The

total

angular

momentum

(-oi)

about

X (-TJ)

O
CI

Since

A=

dil/dt,

we

find

from

(5)

(ai)

mg
Fig. 9-30

is

X (-r^)

a(7\

- T2)k

(5)

is

Ic

and

(6),

a{T x

-T

2)

7c cok

Ic

'o

IcOk

= MK*$

(6)

(7)

CHAP.

PLANE MOTION OF RIGID BODIES

9]

If there is

243

no slipping about the pulley, we also have

A
Using

(8) in (7),

Using

(4) in (9),

a'e

~ T2

Tx

(8)

MK2
(9)

(wn - m2)g
m + m2 + MkVa?

A =

(10)

Thus the masses move with constant acceleration given in magnitude


the result (10) reduces to that of Problem 3.22, page 76.

9.39.

Find the moment of inertia of a

by

(10).

Note that

if

M 0,

sphere about a diameter.

solid

Let O be the center of the sphere and AOB be the


diameter about which the moment of inertia is taken
[Fig. 9-31]. Divide the sphere into discs such as QRSTQ
perpendicular to AOB and having center on AOB at P.

Take the radius of the sphere equal to a, OP = z,


r and the thickness of the disc equal to dz. Then
by Problem 9.4 the moment of inertia of the disc about

SP =

AOB

is

\(trr 2 o dz)r 2

=
From triangle OSP,
the total moment of inertia
r2

=
The mass

a2

\ira(a2

of the sphere

^var^dz
z2 .

(1)

Substituting into

(1),

is

- z2

dz

is

-f_
2=

7ra(a 2

- z2

dz

gwa 3 a

which could also be seen by noting that the volume of the sphere

From

9.40.

(2)

and

cube of edge

we have I/M =

(S)

and mass

is

\a2

ox

- %Ma

is

P=

OC

from axis

2v -^2y/s/Sg.

to the center of

2.

Since the diagonal of a square of side 8 has length


the distance

is

ova3

suspended vertically from one of

that the period for small vibrations


the equivalent simple pendulum?
(a)

($)

mass

The moment of inertia / of a cube about an edge


that of a square plate about a side.
Thus by
/ = %M(s 2 + s 2 ) = Ms 2

V2 + s2 =
is

(b)

its

What

edges,
is

(a)

Show

the length of

8^/2,

^8^/2.

the same as
Problem 9.6,

is

Then the period for small vibrations

P =
(b)

by Problem

9.25,

2wyf2y[slZg~

The length of the equivalent simple pendulum


I

9.41.

2v *sl%Ms 2/[Mg($ 8 V2)]

is,

= %MsV[M(%sV2)] = fv^s

is,

by Problem

9.26,

Fig. 9-32

Prove Theorem

The period of small vibrations of a compound


9.11, page 228:
pendulum is a minimum when the distance OC = a is equal to the radius of gyration
of the body about a horizontal axis through the center of mass.
If I c is the moment of inertia about the center of mass axis and I is the moment of inertia
about the axis of suspension, then by the parallel axis theorem we have

Then the square

where K*

Ic /

4fl-

/p

Mga

Kc

2/Kr

\Ma

with

da^
from which a -

given by

is

mass

the square of the radius of gyration about the center of

is

Setting the derivative of

the theorem

4ir 2

+ Ma 2

ic

of the period for small vibrations

p2

respect to a equal to zero,

we

axis.

find

'

a?

g \

This can be shown to give the

minimum

d 2 (P 2 )/da2

value since

<

0.

Thus

proved.

is

The theorem

9.42.

[CHAP.

PLANE MOTION OF RIGID BODIES

244

is

also true even if the vibrations are not

assumed small.

See Problem 9.147.

rests on top of a fixed rough sphere of radius b.


sphere of radius a and mass
The first sphere is slightly displaced so that it rolls without slipping down the second
sphere. Where will the first sphere leave the second sphere?

Let the xy plane be chosen so as to pass


through the centers of the two spheres, with the
center of the fixed sphere as origin O [see Fig.
Let the position of the center of mass C of
9-33].
the first sphere be measured by angle 0, and suppose that the position vector of this center of
mass C with respect to O is r. Let r l and 9 X be
unit vectors as indicated in Fig. 9-33.

Resolving the weight


ponents in directions r t and

Problem

page

1.43,

W = mgj

into com-

we have [compare

9 lt

24]

(WT^

=
=
=

+ (W *!)#!
(-mgj-rj)^ + (-mgj 9
mg sin o r mg cos e 9

9X

t)

and frictional force f


Using Theorem 9.12,
page 228, together with the result of Problem 1.49,
page 26, we have

The reaction force


are
N = Nr lt f = f9 v

m(r

from which
Since r

[the distance of

We now
center of

b)'e 2

apply Theorem

mass C

is [since

m[(r

N mg

re*)

m(a +

r'o 2 )r + (r'e + 2ro)9


W+ N+
(N mg sin 0)^ + (/ mg

= ma =
=
=

C from

9.13,

W and

A =

1]

m(r'o

sine,

sin

m(a +

e,

cos

2re)

mg

cos e

mg

cos 6

(-ar,)

b)

-^(0 +

(-ar 1 )X(f9 1 )

d2

v^)k

all

-a/k

-(<

vOk

AP

equals arc BP, or

</>

-(0 + vOk

-{ -6

-~a $

forces about the

is

Since there is only rolling and no slipping it follows that arc


= v/2 - e and ^ = (b/a)(ir/2 o), so that
Then

(1)

become

page 229. The total external torque of


pass through C],

Also, the angular acceleration of the first sphere about

O], these equations

N mg

Fig. 9-33

^-* )*k

b$

af.

CHAP.

PLANE MOTION OF RIGID BODIES

9]

Since the
I

is

moment

&ma2 we

A =
Using

of inertia of the first sphere about the horizontal axis of rotation through

have by Theorem

= |wa2(^-i-^ )Vk

-afk

la,

this value of / in the second equation of (1),

we

5g

= "7(^+6)

Multiplying both sides by $ and integrating,

9.13,

245

we

or

= -im(a+b)e

find

C0S6

find after using the fact that

at

or

r/2,

Using

(*) in the first of equations (J),


leaves the second sphere where
= 0,

we

find

i.e.

where

A/"

= lwflr(17 sin - 10). Then


sin-* 10/17.

the first sphere

Supplementary Problems
RIGID BODIES
9.43.

9.44.

Show that the motion of region


of Fig. 9-34 can
be carried into region %' by means of a translation
plus a rotation about a suitable point.

Work Problem

9.1,

page 230, by

translation of the point

9.45.

first

of triangle

applying a

ABC.

If A X ,A ,A Z represent rotations of a rigid body about


V
the x, y and z axes respectively, is it true that the
associative law applies, i.e. is A x + (A + A ) =
z
y
(A x + A y ) + A z t Justify your answer.

Fig. 9-34

MOMENTS OF INERTIA
9.46.

Three particles of masses 3, 5 and 2 are located at the points (-1, 0,


1), (2, -1, 3) and (-2, 2, 1)
Find (a) the moment of inertia and (6) the radius of gyration about the x axis.
Ans. 71
respectively.

9.47.

Find the moment of inertia of the system of particles in Problem 9.46 about
(fe) the z axis.
Ans. (a) 81, (6) 44

9.48.

Find the moment of inertia of a uniform rod of length I about an axis perpendicular to it and
(a) the center of mass,
(b) an end,
(c) a point at distance 1/4 from an end.
Ans. (a) r&MP, (b) j^MP, (c) ^MP

(a)

the y axis,

passing through

9.49.

Find the
Ans.

9.50.

9.51.

(a)

moment

^Ma*,

(b)

of inertia and

(b)

radius of gyration of a square of side a about a diagonal.

^ay/S

Find the moment of inertia of a cube of edge a about an edge.

Ans. Ma2

Find the moment of inertia of a rectangular plate of sides a and


Ans.

9.52.

(a)

Ma 2 b 2 /(a 2 +

about a diagonal.

b 2)

Find the moment of inertia of a uniform parallelogram of sides a and 6 and included angle a
it and passing through its center.
Ans. J*M(a2 + b 2 ) sin2 a

about an axis perpendicular to

PLANE MOTION OF RIGID BODIES

246

[CHAP.

9.53.

Find the moment of inertia of a cube of side a about a diagonal.

9.54.

Find the moment of inertia of a cylinder of radius a and height h


about an axis parallel to the axis of the cylinder and distant b from
its center.
Ans. ^M(a2 + 2b 2 )

9.55.

A solid of constant density is formed from a cylinder of radius a


and height h and a hemisphere of radius a as shown in Fig. 9-35.
Find its moment of inertia about a vertical axis through their
+

Ans. M(2a3

centers.

15a2 h)/{10a

+ 15h)

9.56.

Work Problem 9.55


and height h.

9.57.

Find the moment of inertia of the uniform solid region bounded


by the paraboloid cz = x 2 + y 2 and the plane z = h about the
z axis.
Ans. ^Mch

9.58.

9.59.

if

the cylinder

Use your

definitions in

a vertex and

(6)

Problem 9.58 to
Ans.

a face.

KINETIC ENERGY AND ANGULAR

9.61.

(a)

find the

Ma 2

moment

(a)

Fig. 9-35

a point,

(6)

a plane?

of inertia of a cube of side

Is there

a about

(b) |ilfa2

MOMENTUM

uniform rod of length 2 ft and mass 6 lb rotates with angular speed 10 radians per second
about an axis perpendicular to it and passing through its center. Find the kinetic energy of
rotation.
Ans. 100 lb ft2 /sec 2

Work Problem
Ans. 400

9.62.

replaced by a cone of radius a

is

How might you define the moment of inertia of a solid about


any physical significance to these results? Explain.
(a)

9.60.

9.60

if

the axis of rotation

is

perpendicular to the rod and passes through an end.

lb ft 2 /sec 2

hollow cylindrical disk of radius a and mass


total kinetic energy.
Ans. Mv 2

along a horizontal plane with speed

rolls

v.

Find the
9.63.

9.64.

9.65.

Work Problem

flywheel having radius of gyration 2 meters and


speed of 5 radians/sec about an axis perpendicular to
energy of rotation.
Ans. 1000 joules

Find the angular momentum of


Ans.

9.66.

5 lb ft2 /sec,

(a)

Ans.

9.62 for a solid cylindrical disk of radius a.

(b)

200 kg

Prove the result of (a) Problem


summation.

(a)

&Mv 2

mass 10 kilograms rotates at angular


through its center. Find the kinetic

it

the rod of Problem 9.60

(6)

the flywheel of Problem 9.64.

m2 /sec
9.15,

page 236,

(6)

Problem

9.16,

page 236, by using integration

in place of

9.67.

Derive a "parallel axis theorem" for


the physical significance.

(a) kinetic

energy and

MOTION OF A RIGID BODY. THE COMPOUND PENDULUM.


WORK, POWER AND IMPULSE
9.68.
A constant force of magnitude F is applied tangentially

9.69.

How

long will

from rest?
9.70.

angular

momentum and

explain

a flywheel which can rotate about a


If the flywheel has radius a, radius
and mass M, prove that the angular acceleration is given by F a/MK2

fixed axis perpendicular to

of gyration

(6)

it

and passing through

to

its center.

it

be before the flywheel of Problem 9.68 reaches an angular speed w


2 a /F a

Ans.

if it

starts

MK

Assuming that the flywheel of Problem 9.68 starts from rest, find (a) the total work done,
the total power developed and (c) the total impulse applied in getting the angular speed
Ans ( a) %MK2 <%> ( b ) *>o. (c) MK2 u
up to <o
(6)

CHAP.

9.71.

PLANE MOTION OF RIGID BODIES

9]

Work

KAns.
9.72.

Problem

(a)

0.5 meter,

2 rad/sec 2

(a)

9.68,

(b)

Problem 9.69 and (c) Problem 9.70


and w = 20 radians/sec.

if

M = 20 kilograms
;

(b)

10 sec;

247

F =

10 newtons,

meter,

250 joules, 200 joules/sec, 100 newton sec

(c)

Find the period of small vibrations for a simple pendulum assuming that the string supporting
the bob is replaced by a uniform rod of length I and mass
while the bob has mass m.

Ans. 2v a\

2(M + 3m)Z
3(M + 2m)g

M=

m=

9.73.

Discuss the cases

9.74.

rectangular plate having edges of lengths a and b respectively hangs vertically from the edge
(a) Find the pe riod fo r small oscillations and
(6) the length of the equivalent simple
pendulum.
Ans. (a) 2wy/2b/Zg
(b) |6

(a)

and

(b)

in

Problem

9.72.

of length a.

9.75.

uniform

9.76.

sphere of radius a and mass


is suspended vertically downward from a point
Find the period for small oscillations in a plane and (6) the length of the
equivalent simple pendulum.
Ans. (a) 2ir^la/hg, (b) 7a/5

on

its

solid

surface,

(a)

yo-yo consists of a cylinder of mass 80 gm around which a string of length 60 cm is wound.


end of the string is kept fixed and the yo-yo is allowed to fall vertically starting from rest,
find its speed when it reaches the end of the string.
Ans. 280 cm/sec
If the

9.77.

9.78.

Find the tension in the string of Problem


Ans. 19,600 dynes

9.76.
Y/////////////////////'

hollow cylindrical disk of mass


moving with constant speed v
to an incline of angle a. Prove that if there is no slipping
will rise a distance v\/{g sin a) up the incline.

comes
it

20 kg
9.79.

If the hollow disk of

high will
9.80.

it

rise

Problem 9.78

is

up the incline?

replaced by a solid disk,

Ans. Svl/(4g sin

how

a)

In Fig. 9-36 the pulley, assumed frictionless, has radius 0.2 meter
its radius of gyration is 0.1 meter. What is the acceleration of
the 5 kg mass?
Ans. 2.45 m/sec2

10

kg
|

and

kg

Fig. 9-36

INSTANTANEOUS CENTER. SPACE AND BODY CENTRODES


9.81.

ladder of length I moves so that one end is on a vertical wall and the other on a horizontal
Find (a) the space centrode and (b) the body centrode.

floor.

Ans.

(a)
(b)

9.82.

9.83.

circle

having radius

circle

with the ladder as diameter

and center at point

O where

the floor and wall meet.

AB

A long rod
moves so that it remains in contact with
the top of a post of height h while its foot B moves on a
horizontal line CD [Fig. 9-37]. Assuming the motion
to be in one plane, find the locus of instantaneous centers.
What
in

9.84.

A
A

is

the

(a)

body centrode and

(b)

space centrode

Problem 9.82?

Work Problems 9.82 and 9.83


by a fixed cylinder of radius a.

if

the post

is

replaced
Fig. 9-37

STATICS OF A RIGID BODY


9.85.

uniform ladder of weight


and length I has its top against a smooth wall and its foot on a
having coefficient of friction p. (a) Find the smallest angle a which the ladder can make
with the horizontal and still be in equilibrium. (6) Can equilibrium occur if ft - 0? Explain.
floor

PLANE MOTION OF RIGID BODIES

248

9.86.

9.87.

Work Problem

9.85

and

AB

is

at

the wall has coefficient of friction n v

if

a uniform bar of length


so that AC a and
so that the system will be in equilibrium?
In Fig. 9-38,

at

[CHAP.

and weight

CD

b.

W supported

at C.

Where must a weight

weights
be placed on

It carries

AC

D
Fig. 9-39

Fig. 9-38

9.88.

9.89.

uniform triangular thin plate hangs from a fixed point O by strings OA, OB and OC of
lengths a, b and c respectively. Prove that the tensions T it T% and T 3 in the strings are such that
T x /a = T 2 /b = T 3 /c.

is supported
uniform plank AB of length I and weight
C and D distant a from A and b from B respectively [Fig. 9-39]. Determine the reaction forces at C and D

at points

respectively.

9.90.

In Fig. 9-40, OA and OB are uniform rods having the same


is a right angle.
density and connected at O so that
is in a vertical
The system is supported at O so that
plane. Find the angles a and /? for which equilibrium occurs.
Ans. a = tan -1 (a/b), /? = v/2 tan -1 (a/b)

AOB
AOB

Fig. 9-40

MISCELLANEOUS PROBLEMS
9.91.

9.92.

A circular cylinder has radius a and height h. Prove that the moment of inertia about an axis
perpendicular to the axis of the cylinder and passing through the centroid is -J^M(h2 + 3a2 ).
Prove that the

effect of

a force on a rigid body

is

not changed by shifting the force along

its line

of action.

9.93.

rolls without slipping down an inclined plane of


cylinder of radius a and radius of gyration
angle a and length I, starting from res t at the top of the in cline. Prove that when it reaches the

bottom of the incline


9.94.

is

speed will be ^(2gla2 sin a)l(a2

+ K2

cylinder resting on top of a fixed cylinder is given a slight displacement so that


Determine where it leaves the fixed cylinder.
sin -1 4/7 where e is measured as in Fig. 9-33, page 244.
Ans.

it rolls

without

slipping.

9.95.

Work Problem

9.42 if the sphere is given

9.96.

Work Problem

9.94 if the cylinder

9.97.

A
an

9.98.

9.99.

initial

given an

initial

speed v
speed v

about a diameter rolls without slipping down


sphere of radius a and radius of gyration
2
2
2
).
incline of angle a. Prove that it descends with constant acceleration given by (ga sin a)/(a +

Work Problem

9.97 if the sphere

Ans.

a,

is

an

(a)

f g sin

(&)

(a) solid,

is

(b)

hollow and of negligible thickness.

f g sin a

hollow sphere has inner radius a and outer radius


of inertia about an axis through its center is

6.

Prove that

moment

|M
Discuss the cases

and

b.

'a4

a3 &
i2

+ a2 & 2 + a& 3 +
+ ab + &2

&

4>

if

is

its

mass, then the

CHAP.

9.100.

PLANE MOTION OF RIGID BODIES

9]

249

Wooden

plates, all having the same rectangular


shape are stacked one above the other as indicated
in Fig. 9-41.
(a) If the length of each plate is
2a, prove that equilibrium conditions will prevail
if the (n + l)th plate extends a maximum distance
of a/n beyond the wth plate where n = 1,2,3, ...
.

What

(b)

is

the

maximum

which can be reached


are added?
9.101.

as assumed

more and more plates

9.100 if the plates are stacked on


instead of on a flat surface
in that problem.

cylinder of radius a rolls on the inner surface of a smooth cylinder of radius 2a.
is

2v \/3a/2g

Prove that

ladder of length I and negligible weight rests with one end against a wall having coefficient of
and the other end against a floor having coefficient of friction /i 2 It makes an angle a
with the floor, (a) How far up the ladder can a man climb before the ladder slips? (6) What is
the condition that the ladder not slip at all regardless of where the man is located?

friction

Ans.
9.104.

Fig. 9-41

the period of small oscillations


9.103.

distance

Work Problem

a sphere of radius

9.102.

if

horizontal

(a)

/j.

+ t&n a)/ (finx 2 +

2 l(m

Work Problem

9.103

if

the

1),

(b)

tana >

l//t 2

weight of the ladder

is

not

negligible.
9.105.

AB

[Fig. 9-42] has one end A on an


I
and the other end B on a vertical wall.
The ladder is at rest and makes an angle /? with the incline.
If the wall is smooth and the incline has coefficient of

ladder

of length

incline of angle a

friction

n,

the smallest value of n so that a

find

man

of

weight
m will be able to climb the ladder without having it
slip.
Check your answer by obtaining the result of Problem
9.37, page 241, as a special case.
9.106.

Work Problem

9.107.

9.105 if the wall has coefficient of friction

0i-

Fig. 9-42

uniform rod AB with point A fixed rotates about a vertical axis so that it makes a constant
angle a with the vertical [Fig. 9-43], If th e length of the rod is I, prove that the angular
speed needed to do this is = ^J(3g sec a)/2l
.

'//////////////////'.

Fig. 9-43
9.108.

Fig. 9-45

circular cylinder of mass


and radius a is suspended from the ceiling by a wire as shown in
Fig. 9-44. The cylinder is given an angular twist 6 and is then released. If the torque is assumed
proportional to the angle through which the cylinder is turned and the constant of proportionality

is X,

9.109.

Fig. 9-44

prove that the cylinder will undergo simple harmonic motion with period 2iray/m/2\

Find the period

in

Problem 9.108

if

the cylinder

is

replaced by a sphere of radius

a.

Ans. 2jra\/2m/5X
9.110.

9.111.

Work

(a) Problem 9.108 and


(6) Problem 9.109
angular velocity is present. Discuss physically.

AB

if

damping proportional

to the

instantaneous

A uniform beam
of length I and weight
[Fig. 9-45] is supported by ropes
and BD of
lengths a and 6 respectively making angles a and /? with the ceiling CD to which the ropes
are fixed. If equilibrium conditions prevail, find the tensions in the ropes.

AC

9.112.

9.114.

9.115.

Prove that the acceleration of the mass

ga2 /(a2 +

K2

Problem 9.112

in

is

).

Describe how Problem 9.112 can be used to determine the radius of


gyration of a pulley.

w////////////////.

is attached to a rope which is wound around


In Fig. 9-46 the mass
which can rotate
and radius of gyration
a fixed pulley of mass
freely about O. If the mass is released from rest, find (a) the angular
speed of the pulley after time t and (b) the tension in the rope.

9.113.

[CHAP.

PLANE MOTION OF RIGID BODIES

250

uniform rod

AB

[Fig. 9-47] of length

and weight

having

Fig. 9-46

its

OA and floor OB respectively, slides


starting from rest when its foot B is at a distance d from O. Prove
that the other end A will eave the wall when the foot B is at a
distance from O given by ^yoT^ + 4d2
ends

on

a frictionless wall

9.116.

cylinder of mass 10 lb rotates about a fixed horizontal axis through


rope wound around it carries
center and perpendicular to it.
a mass of 20 lb. Assuming that the mass starts from rest, find its
Ans. 128 ft/sec
speed after 5 seconds.

its

be the length of a rod suspended from one end so that


Ans. 149 cm
on making small vibrations in a plane?

9.117.

What must

9.118.

it

Fig. 9-47
will be a seconds

pendulum

sphere and a hollow sphere of the same radius both start from rest at the top of an
and roll without slipping down the incline. Which one gets to the
Ans. The solid sphere
bottom first? Explain.
solid

inclined plane of angle a

9.119.

and radius of gyration K about a horizontal axis is displaced


compound pendulum of mass
vertical and is then released. Prove that if the center of
it makes an angle
O with the
mass is at distance a from the axis, then the reaction force on the axis is given by

so that

Mg
#2
9.120.

9.121.

Find the least

9.123.

y/([K2

rectangular parallelepiped of sides


Find the period of small oscillations.

incline of angle

9.122.

coefficient of friction

+ 2a 2

a, b,

cos*

and

c is

a2 cos

sin 0) 2

needed to prevent the sliding of a circular hoop down an

suspended vertically about a point %l

and r 2
pulley system consists of two solid disks of radius r x
rotating
of
capable
and
other
each
to
respectively rigidly attached
weight
freely about a fixed horizontal axis through the center O. A
shown
as
disk
smaller
the
around
wound
string
is suspended from a
If the radius of gyration of the pulley system is
in Fig. 9-48.
and its weight is w, find (a) the angular acceleration with which the

weight descends and

(a)

the tension in the string.

(b)

WgrJiWri + wK2 ),

sphere of radius

a.

(b)

WwK2/(Wr\ + wK

2
)

rolls on the inside of a smooth hollow


the period for small oscillations is
that
Prove

solid sphere of radius

given by 2vy/7(a

b)/5g

from a
thin circular solid plate of radius a is suspended vertically
If it
[see Fig. 9-49].
horizontal axis passing through a chord
makes small oscillations about this axis, prove that the frequency
is at distance a/2 from
of such oscillations is greatest when

AB

AB

the center.

a.

Ans. tan a

a.

Find the period of small vibrations of a rod of length


from one end.

Ans.

9.125.

+ (K2

suspended vertically from the side of length

9.124.

2
)

Fig. 9-49

CHAP.

9.126.

PLANE MOTION OF KIGID BODIES

9]

uniform rod of length

end
1
1_

suspended vertically from a string of length 21 which has

Prove that the normal frequencies for small oscillations in a plane are

fixed.

[^
-y

51 is

251

other

its

\\
2
^ *

and describe the normal modes.

and

5l

2ir

9.127.

9.128.

(a)

uniform rod of mass


and length I is suspended from one of its ends. What is the minimum
speed with which the other end should be hit so that it will describe a complete vertical circle?
If the

bob of a simple pendulum

is

a uniform solid sphere of radius a rather than a point

mass, prove that the period for small oscillations


(b)

9.129.

A
to

For what value of

9.131.

is

the period in (a) a

2iry/l/g

+ 2a2/5gl

minimum?

sphere of radius a and mass


rolls along a horizontal plane with constant speed v
It comes
an incline of angle a. Assuming that it rolls without slipping, how far up the incline will it

Ans.

travel?
9.130.

is

10v/(7flr sin a)

Prove that the doughnut shaped solid or torus of Fig. 9-50 has
a moment of inertia about its axis given by |Af(3a 2 + 4b 2 ).

cylinder of mass
and radius a rolls without slipping down
a 45 inclined plane of mass
which is on a horizontal frictionless table.
Prove that while the rolling takes place the incline
will move with an acceleration given by mg/(SM + 2m).

9.132.

Work Problem

9.131 if the incline is of angle

Ans. (mg sin 2a)/(3M


9.133.

Find the

(a)

+ 2m - ra

a.

cos 2a)

tension in the rope and (6) accelerashown in Fig. 9-51 if the radius
of the pulley is 0.5
and its mass

Fig. 9-50
200 kg

tion of the system

of
is

gyration
20 kg.

9.134.

Compare the

9.135.

Prove that

9.136.

1\

.2

result of Problem 9.133 with that


obtained assuming the pulley to have negligible mass.

is

the net external torque about an axis


zero, then it is also zero about any other axis.

solid cylindrical disk of radius

a
100 kg

if

from the center of the

disk.

Fig. 9-51

a has a circular hole of radius b whose center


down an inclined plane of angle a, find

If the disk rolls

is

at distance c

its acceleration.

[See Fig. 9-52/

Fig. 9-52
9.137.

9.138.

Find the moment of inertia of the region bounded by the lemniscate r2


about the x axis.
Ans. Ma2 (3ir - 8)/48

Find the largest angle of an inclined plane down which a


if

9.139.

Fig. 9-53

the coefficient of friction

Work Problem

is

/t.

9.138 for a solid sphere.

a2 cos 2e

solid cylinder will roll

[see

Fig

9-531

without slipping

9.140.

Discuss the motion of a hollow cylinder of inner radius a and outer radius b as
inclined plane of angle

9.141.

[CHAP.

PLANE MOTION OF RIGID BODIES

252

it

rolls

down an

a.

ABC

of side s. The legs


table top of negligible weight has the form of an equilateral triangle
is placed on
heavy weight
of the table are perpendicular to the table top at the vertices.
the table top at a point which is distant a from side BC and 6 from side AC. Find that part
of the weight supported by the legs at A, B and C respectively.

Ans.

2Wb

2Wa

Wll- 2a + 26
sV3

9.142.

Discuss the motion of the disk of Problem 9.136 down the inclined plane
friction is

9.143.

hill

if

the coefficient of

/<.

has a cross section in the form of a cycloid


x

a{e

sin

$),

a(l

cos

9)

A solid sphere of
as indicated in Fig. 9-54.
radius b starting from rest at the top of the hill
is given a slight displacement so that it rolls
without slipping down the hill. Find the speed
of its center when it reaches the bottom of the hill.
Ans. y/l0g(2a

9.144.

- b)/l

Work Problem

Fig. 9-54

3.108,

page

85,

if

the masses and

moments

of inertia of the pulleys are taken

into account.

9.145.

Work Problem

9.146.

if friction

is

taken into account.

uniform rod of length I is placed upright on a table and then allowed to fall. Assuming that its
point of contact with the table does not move, prove that its angula r velocity at the instant when

it

9.147.

page 242,

9.38,

makes an angle

with the vertical

is

given in magnitude by y/3g{i--cose)/2l

Prove Theorem 9.11, page 228, for the case where the vibrations are not necessarily small.
Compare Problem 9.41, page 243.

9.148.

9.149.

9.150.

rigid body moves parallel to a given fixed plane. Prove that there
of the rigid body where the instantaneous acceleration is zero.

AB

solid cylinder of radius a and height h is suspended from axis


as indicated in Fig. 9-55. Find the period of small oscillations about

Prove that a

B
~e*%%$mk

solid sphere will roll

plane of angle a

9.152.

one and only one point

If it is
solid hemisphere of radius a rests with its convex surface on a horizontal table.
displaced slightly, prove that it will undergo oscillations with period equal to that of a simple
pendulum of equivalent length 4a/3.

this axis.

9.151.

is

if

without slipping down an inclined


is at least
f tan a.

the coefficient of friction

coefficient of friction for an inclined plane of angle a


order that a solid cylinder will roll down it without slipping.

Find the least


in

Ans. ^ tan a

Fig. 9-55

Chapter 10

SPACE MOTION
of RIGID BODIES

GENERAL MOTION OF RIGID BODIES IN SPACE


In Chapter 9 we specialized the motion of rigid bodies to one of translation of the
center of -mass plus rotation about an axis through the center of mass and perpendicular
to a fixed plane. In this chapter we treat the general motion of a rigid body in space.
Such general motion is composed of a translation of a fixed point of the body [usually the
center of mass] plus rotation about an axis through the fixed point which is not necessarily
restricted in direction.

DEGREES OF FREEDOM
The number

of degrees of freedom [see page 165] for the general motion of a rigid body
6 coordinates are needed to specify the motion.
usually choose 3 of
these to be the coordinates of a point in the body [usually the center of mass] and the
in space is 6,

We

i.e.

remaining 3 to be angles [for example, the Euler angles, page 257] which describe the
rotation of the rigid body about the point.
If

the

a rigid body is constrained in any way, as for example by keeping one point
of degrees of freedom is of course reduced accordingly.

fixed,

number

PURE ROTATION OF RIGID BODIES


Since the general motion of a rigid body can also be expressed in terms of translation
of a fixed point of the rigid body plus rotation of the rigid body about an axis through the
point, it is natural for us to consider first the case of pure rotation and later to add the
effects of translation. To do this we shall first assume that one point of the rigid body
is

fixed in space.

by using the

The

effects of translation are relatively

result (10),

easy to handle and can be obtained

page 167.

VELOCITY AND ANGULAR VELOCITY OF


A RIGID BODY WITH ONE POINT FIXED
Suppose that point
is fixed.

Then

of the rigid

body

% of Fig.

at a given instant of time the

10-1

body

will be rotating

with angular velocity <> about the instantaneous axis through O. A particle P of the body
having position vector r with respect to O will have
an instantaneous velocity vv given by

See Problem

rv

Xr

CO

10.2.

Fig. 10-1

253

SPACE MOTION OF RIGID BODIES

254

[CHAP. 10

ANGULAR MOMENTUM
The angular momentum
axis through the fixed point

where

is

particles of

given by

is

2 <

fixed about the instantaneous

body with one point

of a rigid

v {t v

Xxv

2 <v{rv X

r v )}

{2)

the mass of the vth particle and where the summation

taken over

is

all

%.

MOMENTS OF

INERTIA. PRODUCTS OF INERTIA

Let us choose a fixed xyz coordinate system having origin

a = a + a y j +

nji,

Xvi

rv

Then equation

(2)

9-y

=
=

7 *z w s

ty^x

Q z ~ hx^x
l xx

y vj

<*j.

o>

^m^ +

z*),

Ixy
lyz
lzx

Iyy

Jm^ + 4

us write

ajs.

[see

Problem

10.3].

+ 7 xy<% + Ixz"z
+ hy% + hz^z
+ hy% + hz^z

^ Wl
-2m

yj

let

z vk

can be written in component form as

Qx

where

O and

I zz

X v yv

"

V* z

Xv

lxz

W/v

Z11

(4)

^m^ +

yl)

(5)

':

(6)

quantities hx, Iyy, Lz are called the moments of inertia about the x, y and z axes respecFor continuous mass
tively. The quantities I xy hz, ... are called products of inertia.
distributions these can be computed by using integration.

The

Note that the products of inertia in (6) have been defined with an associated minus
sign. As a consequence minus signs are avoided in (4).

MOMENT OF INERTIA MATRIX OR TENSOR


The nine

quantities Ixx

xy ,

...,hz can be written in an array often called a matrix or

tensor given by

and each quantity

is called

la

lyx

Iyy

lyz

\lzx

Izy

lz

the principal or

called
Ixy

it

lxy

an element of the matrix or tensor. The diagonal consisting

of the elements hxjyyjzz is

is

lxx

lyx,

Ixz

lzx,

lyz

main diagonal.
Izy

Since
\P)

is seen that the elements have symmetry about the main diagonal.
often referred to as a symmetric matrix or tensor.

For

this reason (7)

KINETIC ENERGY OF ROTATION


The

kinetic energy of rotation

is

given by

T = Wxx-l + hy< +

i<o-0

hz-l

+ 2Ixy<x<y +

^xz^z + 2I

yz<y<z)

{9)

CHAP.

SPACE MOTION OF RIGID BODIES

10]

255

PRINCIPAL AXES OF INERTIA

mutually perpendicular axes having origin O which are fixed in the body
it and which are such that the products of inertia about them are zero,
are called principal axes of inertia or briefly principal axes of the body.
set of 3

and rotating with

An important property of a principal axis [which can also be taken as a definition]


that if a rigid body rotates about it the direction of the angular momentum is the same
as that of the angular velocity. Thus
is

O =
where

From

/ is a scalar.

we

this

find [see

Jo,

(10)

Problem

Vx + ^-^K +
l zx<x

lzy(A y

10.6] that

=
=

- 7 )

Vzz

()

In order that (11) have solutions other than the trivial one
require that
ixx

Ixy

*xz

lyx

Iyy-I

hz

Izx

i-zy

wx

0,

ay

0,

0,

we

(12)

<>

Izz

This leads to a cubic equation in / having 3 real roots h,h,h. These are called the
principal moments of inertia. The directions of the principal axes can be found from (11),
as

shown

An

in

Problem 10.6 by finding the

symmetry

axis of

ratio

>

of a rigid body will always be a principal axis.

ANGULAR MOMENTUM AND KINETIC ENERGY ABOUT


THE PRINCIPAL AXES
If we call
v 2
momenta about the
<a

<a

(o

and Q lt Q 2 o 3 the magnitudes of the angular velocities and angular


3
principal axes respectively, then
,

The

A*

!*

n2

_7

a,

2'

n 3 - h^s

kinetic energy of rotation about the principal axes is given

t =
which can be written

in vector

(13)

by

WA + *A + W)

form as [compare equation

T =

(U)
(9)]

|a>-0

(15)

THE ELLIPSOID OF INERTIA


Let n be a unit vector in the direction of
o>

where
axes.

cos

a,

cos

Then the

a>n

w(C0S a

<>.

Then
COS P

cos y are the direction cosines of or


kinetic energy of rotation is given by

/3,

T = \h

COS y k)

n with respect

(16)

to the x,

y and

(17)

SPACE MOTION OF RIGID BODIES

256

where

+ I zz cos 2 y
+ 27 xy COS a COS /? + 27 COS /3

I xx cos 2 a

yy

cos 2 ft

'

By

COS y

27 xz COS a COS y

(18)

defining a vector
p

where

[CHAP. 10

Px i

+ PJ + P

k,

7xxrx
p2

(18)

2
7 yy"y
p

= n/yT

(19)

becomes

2
7 zz"z
p

op =1
p+ 27yzryrz
P p+ 27 zx"z"x

27xyrxry
P

In the coordinates p x ,p ,p z equation (20) represents an


y
ellipsoid of inertia or the momental ellipsoid.

ellipsoid

which

(20)
is

called

the

If the coordinate axes are rotated to coincide with the principal axes of the ellipsoid,
the equation becomes
21 )
(
*A + h?\ + 1 = 1

where

p v p 2 ,p 3

represent the coordinates of the

new

axes.

EULER'S EQUATIONS OF MOTION


It is convenient to describe the motion of a rigid body relative to a set of coordinate
axes coinciding with the principal axes which are fixed in the body and thus rotate as the
body rotates. If A lf A 2 a 3 and c^, <o 2 w 3 represent the respective components of the external
torque and angular velocity along the principal axes, the equations of motion are given by
,

-^K^ = A
h^2 + (*1 ~ -^sK^l ~ A 2
Vs + (^2 _ ^l) *^ ~ A 3

Vl +

(*s

(22)

These are often called Euler's equations.

FORCE FREE MOTION.


THE INVARIABLE LINE AND PLANE
Suppose that a rigid body is rotating about a
O and that there are no forces acting
on the body [except of course the reaction at the
fixed point

Then the total external torque is


Thus the angular momentum vector Q is
constant and so has a fixed direction in space as
indicated in Fig. 10-2. The line indicating this

fixed point].
zero.

direction

Since

Problem

is

called the invariable line.

the

kinetic

10.34],

energy

is

we have from

wo

constant

constant

[see

(15)
(23)

Fig. 10-2

This means that the projection of on O is constant, so that the terminal point of
describes a plane. This plane is called the invariable plane.

As

<*

the rigid body rotates, an observer fixed relative to the body coordinate axes would
see a rotation or precession of the angular velocity vector a> about the angular momentum
vector CI.

CHAP.

SPACE MOTION OF RIGID BODIES

10]

257

POINSOTS CONSTRUCTION. POLHODE. HERPOLHODE.


SPACE AND BODY CONES
As noted by Poinsot, the above ideas can
be geometrically interpreted as a rolling
without slipping of the ellipsoid of inertia
corresponding to the rigid body on the
invariable plane. The curve described on the
invariable plane by the point of contact with
the ellipsoid is called the herpolhode [see
Fig. 10-3]. The corresponding curve on the
ellipsoid is called the polhode.

To an observer fixed in space it would


appear that the vector a traces out a cone
which is called the space cone. To an
observer fixed on the rigid body it would
appear that <> also traces out a cone which is
called the body cone. The motion can then
be equivalently described as a rolling without
slipping of one cone on the other.
See
Problem 10.19.

Fig. 10-3

SYMMETRIC RIGID BODIES. ROTATION OF THE EARTH


Simplifications occur in the case of a symmetric rigid body.

In such case at least


h, are equal and the ellipsoid of inertia
is an ellipsoid of revolution.
We can then show [see Problem 10.17] that the angular
velocity vector < precesses about the angular momentum vector O with frequency given by

two principal moments of

inertia, say

/
where the constant
axis of symmetry.

is

h and

h-h

2tt

in)

the component of the angular velocity in the direction of the

In the case of the earth, which can be assumed to be an ellipsoid of revolution flattened
300 days. In
practice, however, the period is found to be about 430 days. The difference is explained as
due to the fact that the earth is not perfectly rigid.
slightly at the poles, this leads to a predicted precession period of about

THE EULER ANGLES


In order to describe the rotation of a rigid

body about a point we use 3 angular coordinates


called Euler angles. These coordinates denoted
by 4>, 9, xp are indicated in Fig. 10-4. In this
figure the xyz coordinate system can be rotated
into the x'y'z' system by successive rotations
through the angles < and then 6 and then ^
[see Problem 10.20]. The line OA is sometimes
called the line of nodes.

In practice the x',y',z' axes are chosen as


the principal axes or body axes of the rigid
body while the x, y and z axes or space axes
are fixed in space.

Fig. 10-4

SPACE MOTION OF RIGID BODIES

258

[CHAP.

10

ANGULAR VELOCITY AND KINETIC ENERGY


TERMS OF EULER ANGLES

IN

In terms of the Euler angles the components w w 2' w of the angular velocity along the
l
3
x f y' and z' axes are given by
,

<

sin

$ sin
<

The

kinetic energy of rotation

is

where h, h, h are the principal

sin ^
6

cos 8

cos

xp

6 sin
6 sin

ip

(25)

then given by

T = *(/,? + V5 +
moments of inertia.

W)

(26)

MOTION OF A SPINNING TOP


An interesting example of rigid body motion
occurs when a symmetrical rigid body having one
point on the symmetry axis fixed in space is set
spinning in a gravitational field. One such example
is that of a child's top as shown in Fig. 10-5, where
point O is assumed as the fixed point.
For a discussion of the various kinds of motion
which can occur, see Problems 10.25-10.32 and 10.36.

Fig. 10-5

GYROSCOPES
Suppose a circular disk having its axis mounted in gimbals [see Fig. 10-6] is given a
spin of angular velocity a. If the outer gimbal is turned through an angle, the spin axis
of the disk will tend to point in the same direction as previously [see Fig. 10-7]. This

assumes of course that friction at the gimbal bearings

is

negligible.

In general the direction of the spin axis remains fixed even

which is attached
mechanism, which
direction [or

some

moves freely

when the

outer gimbal,

Because of this property the


is called a gyroscope, finds many applications in cases where maintaining
following some specified course] is important, as for example in navigation
to

and guidance or control of

object,

in space.

ships, airplanes, submarines, missiles, satellites or other

moving

vehicles.

A
the

gyroscope

symmetry

is

another example of a symmetric spinning rigid body with one point on

axis [usually the center of mass] taken as fixed.

Spin

Fig. 10-6

Fig. 10-7

CHAP.

SPACE MOTION OF RIGID BODIES

10]

259

Solved Problems

GENERAL MOTION OF RIGID BODIES IN SPACE


10.1.

Find the number of degrees of freedom for a rigid body which


in space, (b) has one point fixed, (c) has two points fixed.
6

[see

Problem

7.2(a),

page 172]

(6)

[see

Problem

7.2(6),

page 172]

(c)

If

(a)

10.2.

can move freely

(a)

two points are fixed, then the rigid body rotates about the axis joining the two fixed
points. Then the number of degrees of freedom is 1, such as for example the angle of rotation
of the rigid body about this axis.

rigid body undergoes a rotation of angular velocity a about a fixed point O. Prove
that the velocity v of any particle of the body having position vector r relative to O
is

given by v

= xr.

This follows at once from Problem

moving system

dr/dt\

is

M=

dr/dt\ b

page 147, on noting that the velocity relative

6.1,

to the

0.

ANGULAR MOMENTUM. KINETIC ENERGY.


MOMENTS AND PRODUCTS OF INERTIA
10.3.

page 254, for the components of angular momentum in terms


of inertia given by equations (5) and (6), page 254.
angular momentum is given by

Derive the equations


of the

(4),

moments and products

The

total

G
where we have used Problem

Now

by equation
r

2 m
v=l

XvJ

m(r

v {r v

X (w X

r)}

10.2 applied to the ?th particle.

page

(7),

2
v=l

5,

r)

we have

(r

(w x i

r)

uj

r(*

r)

+ y% + z v
+ z vk)(a x x v +

w xk)(*2

{x v i

+ z l) -

WxiVv

y v\

Uyy v

u> z z v )

- <*z x V Z V }i
+
UxXvVv ~ u zVv z v}J
+ {o> z (xl + y v u x x v z v u y y v z v }k

UyXvVv

{a y ($

z\)

Then multiplying by m, summing over


respectively,

we

Vy

and equating the

coefficients of

i, j

and k

to

Sl x , S2

and

Sl z

find as required

]2 (yl +z*)><*x

Ixx u x

"f"

'xj/

2 mXy

j/

v >

<

2 mxy

Ixy u x

*xz u x

lyy^y

OJ x

~^~

nh*v*v

lyz^y

<

2 mxz

v v

>v(xl

+ *v)
f

>vVvZv \

">x

lyz^z

ux

"j

rn^y^,

>

ay

-l

>v(tf

+ Vv)\<*z

*zz u z

For continuous mass distributions of density

*xz u z

v >

a(rX\)dr

a,

we can

obtain the
<x{r

(<

same
r)}

dr

results

by starting with

SPACE MOTION OF RIGID BODIES

260

10.4.

If a rigid

[CHAP.

10

body with one point

momentum

CI,

fixed rotates with angular velocity w and has angular


prove that the kinetic energy is given by T = |<*> CI.

|2m,{(Xr,)-(Xr v )}

2 wir X((.Xr) =

2 w{

[r

(.

r)]}

i tt -0
N

where we have used the abbreviation

10.5.

Prove that the kinetic energy

Problem 10.4 can be written

in

10.4,
l>

W A + y + A + xM + ^

T =
From Problem

in place of

2I xz

21

IZ

2I yz

z)

we have
CI

+ 4i/ w y + 4z")
+ Uyilyx^x + ^/y w y + ^2/Z w z)
+ w z(4x w +
+ 4zz)}

i{a;(4a; w x

-fz2,3,

a:

i(^xx w x

using the fact that /.

I vx

Ixz

Izx

IZ z<*z

+ ^xy u x u y + ^xz a x u z +

2Iyz w y a z )

/..

I.

lyyUy

PRINCIPAL MOMENTS OF INERTIA AND PRINCIPAL AXES


10.6.

Derive equations

(11), page 255,


directions of the principal axes.

Using

CI

together with equations

(3)

and

(-4),

moments

for the principal

of inertia

and the

I*

(1)

page 254, we have


"xy^y
^j/y w r/

^j/ w x:

Ia y

uu
xz u x ~r *uz
yz>y

'z2 z

"*

*yx u x

~ I)<*x + ^"j/ + 4 w z =
=
lyx^x + Uj/j/ ~ I) u y
^j/z w z
(hx

"+"

4zx

The principal moments of


ux

o)

u z in

(2)

equal to zero,

inertia are

lyz^y

(I zz

~ I)<*z

found by setting the determinant of the

coefficients

of

i.e.,

T.V.

*xy
*yy

'xz
*

*yz
I,,

This

(2)

-I

a cubic equation in / leading to three values I lt / 2> I3 which are the principal moments
I = 7 X in (2) we obtain ratios for a x u
y u z which yields the direction of
or the direction of the principal axis corresponding to I v Similarly, by substituting I 2 and I 3
we find the directions of the corresponding principal axes.
is

of inertia.

By putting

CHAP.

10.7.

SPACE MOTION OF RIGID BODIES

10]

Find the

moments

(a)

an element dx dy of the plate


the density is a is ay 2 dx dy. Then the
of inertia of the entire plate about the x axis is

The moment of

inertia of

about the x axis

moment

if

/a

pa
ay 2 dx dy

since the

mass of the plate

moment

Similarly, the
the y axis is
id

is

is

|a4

M aa

Ma

jjj
f

(1)

dx dy

2.

of inertia of the plate about

pa
ax2

J
as

and (b) products of inertia of a uniform square


y and z axes chosen as shown in Fig. 10-8.

of inertia

plate of length a about the x,


(a)

261

also evident

dxdy

i<ra4

Ma 2

(2)

Fig. 10-8

by symmetry.

The moment of

inertia of

dx dy about the

z axis is a{x 2

+ y2

dx dy, and so the moment of

inertia of the entire plate about the z axis is

hz

f"

<r(x 2

+ y2

dx dy

%Ma2 + %Ma2

\Ma2

(3)

This also follows from the perpendicular axes theorem [see page 226].

(6)

The product of inertia


and so the product of

of the element dx dy of the plate about the x


inertia of the entire plate about these axes is

/,.

is

axydxdy,

pa

pa

and y axes

2
-\Ma
4

j<t 4

axy dx dy

(4)

*'a =

x=0

The product of inertia of the element dx dy of the plate about the * and
product of adxdy by the distances to the yz and xy planes, which are x and

axes

is

the

respectively.

Thus we must have

10.8.

Find the

(a)

principal

/,

moments

hu =

and similarly

0,

and

of inertia

(5)

the directions of the principal

(b)

axes for the plate of Problem 10.7.


(a)

By Problem

10.6

and the results

Problem

(l)-(5) of

%Ma2 -

10.7,

(1)

|Ma2
[(%Ma2

I)(Ma2

obtain

-\Ma2
\Ma2 - I

-\Ma2

or

we

-- J

- (-lMa 2 )(-lMa 2 )][%Ma 2 -

I)

I]

which can be written


[P
Setting the

first

- |Ma2J + -M 2 a*][%Ma 2 -

1}

factor equal to zero and using the quadratic formula to solve for

/,

we

find for the three roots of (1),

I^-^Ma2
which are the principal moments of
(6)

/2

= JLMa2,

73

= \Ma2

inertia.

find the direction of the principal axis corresponding to I lt


the equations

To

(Ma2 -I)a x - lMa2u y +


-Ma 2 x + (Ma2 -I)a y +
a>

0<a x

(2)

Wy

<o 2

<* z

(|Ma2-7)a> 2

=
=
=

we

let

Ix

= j^Ma2

in

(3)

SPACE MOTION OF RIGID BODIES

262

The

[CHAP.

10

two equations yield u y w x while the third gives w x = 0. Thus the direction
is the same as the direction of the angular velocity vector

first

of the principal axis

<

wxi

w yj

Then the principal axis corresponding


Similarly, by letting

uy

u x

uz

we

uy

let

while u z

The

we

(3)

+ j.

find

j.
= u x u x j = a x (i j) or
2
I = I3 = %Ma
in (3) we find
a x = 0,
= z k which
is arbitrary.
This gives

is

<>

shows that the third principal axis

+ j,

in

+ j)

u x (i

to /j is in the direction

~ -^Ma2

I2

+ uj =

ux i

so that the direction of the correspond-

ing principal axis


If

u zk

to

the direction k.

is in

directions of the principal axes are indicated by

j and

Note that these are


j have the
j and i
directions of the diagonals of the square plate which are
lines of symmetry.
k in Fig. 10-9.
mutually perpendicular and that
i

The principal moments of inertia can also be determined by recognizing the lines of symmetry.

10.9.

Fig. 10-9

Find the principal moments of inertia at the center of a uniform rectangular plate
of sides a and b.
The principal axes lie along the directions of symmetry and thus must be along the x axis,
y axis and z axis [the last of which is perpendicular to the xy plane] as in Fig. 10-10.

By Problems
I2

= ^-Mb 2

I3

9.6, 9.9

and

= ^M(a 2 +

9.11 the principal

moments

of inertia are found to be

It

^Ma2

b 2 ).

dr

V* 2 +

dz dy dx

Fig. 10-10

10.10.

Fig. 10-11

Find the principal moments of inertia at the center of the

_|_

ellipsoid

^2

,2

,.2

4-

a"

One eighth of the

The moment of inertia of the volume


+ y 2 )a dr, and the total moment of

ellipsoid is indicated in Fig. 10-11.

element dr of mass a dr about the


inertia about the z axis is

axis or "3" axis

yJl- x 2 /a 2

rr
x =o

s*c\ll

(x 2

is

(r2/2

.,2,^2,

(x 2

"y=0

+ y 2 )a dz dy dx

Integration with respect to z gives

pa
8ac

To perform

x=0

s>b\/l

x 2 /a 2

(* 2

Jy=0

this integration let


integral can be written

aX,

(a 2
j

V 2)

Vl -

(aW + y

2 /b 2

bY where X and Y

X 2 + b 2 Y 2 Vl )

{X 2

are

dy dx

new

+ Y 2 dY dX
)

variables.

Then the

CHAP.

SPACE MOTION OF RIGID BODIES

10]

XY

Introducing polar coordinates R, 6 in this

plane, this becomes

s*tt/2

263

(a 2

R2

cos 2

62

Vl - R 2 R dR dQ

sin 2 6)

*^0 = O

'R =

2iraabc{cfi

b 2)

R^VT^WdR

7roabc(a2

b 2)

R=

where we use the substitution

R2 =

Since the volume of the ellipsoid

By symmetry we

10.11.

find

Suppose that the


while
(7 3

From Problem

73

LM(a2

ft

).

c/a.

(a

10.10, if

then

+c

a2

and

2a

-I

(/ 3

)/I

c2

a2
2a2
.

c)(a + c)
a2

c2

-|-

But

c2

if

c differs

Thus, approximately,

- c)(2a)/2a 2 =

(a

10.11 for the case of the earth

Work Problem

and hence

).

Problem 10.10 is an oblate spheroid such that a = b


from a or b. Prove that to a high degree of approximation,

only slightly from a then

10.12.

M = ^Traabc

is

ellipsoid of

c differs slightly

- 7i)//i =

mass

the

= iM(o2 + c 2

J2

),

evaluating the last integral.

in

is ^-n-abc,

= iM(6 2 + c 2

/j

t/ 2

c/a

assumed

to be

an oblate spheroid.

Since the polar diameter or distance between north and south poles is very nearly 7900 miles
"3" axis
while the equatorial diameter is very nearly 7926 miles, then taking the polar axis as the
we have 2c = 7900, 2a = 7926 or c = 3950, a = 3963.

Thus by Problem

(Z 3

10.11,

- h)/I =

~ 3950/3963 =

.00328.

ELLIPSOID OF INERTIA
10.13.

Suppose that the moments and products of inertia of a rigid body % with respect
to an xyz coordinate system intersecting at origin O are I xx hv, Izz> I*y> Ixz, hz respectively. Prove that the moment of inertia of % about an axis making angles a, p, y
with the x, y and z axes respectively is given by
,

Ixx

+
A

n
if

m has

about the axis

rXn

cos a

21 xy COS a COS P

cos

position vector

OA

is

Tv

r, its

cos Y

yv

zv

cos a

cos p

cos y

(z v

Izz

COS 2 y

21 xz COS a COS y

21 yz COS p COS y

given

is

moment
v

(y v cos y

n| 2

m T)1 where D -

of inertia

|r

n|.

But

z v cos /3)i

cos a

+
\

)3

xv

and

I yy COS 2

unit vector in the direction of the axis

by

Then

COS 2 a

x v cos

(x v cos

(y v cos y

(vl

+ 4)

/3

y)j

z v cos

cos 2 a

Fig. 10-12

y v cos a)k

(3)

(x

(z v

+ 4)

2x v y v cos a cos

(3

cos a
cos 2

x v cos
2

(x v

y)

+ yl)

2xz cos a cos y

(* cos

fi

y v cos

cos 2 y

2y v z v cos

/?

cos y

a) 2

SPACE MOTION OF RIGID BODIES

264

Thus the

moment

total

J2

of inertia of all masses ra

+ 4)

v(vl

Ixx cos 2 a

+
10.14.

2/j.j,

2^-2 m y
v

lyy cos 2

cos a cos

Find an equation for the


Problem 10.7.

We

have from Problem


Ixx

= \Ma\

+ /^

/?

/?

cos a cos

vzv >

cos

10

is

2 m^ajj! + 4) \

- 2 myX v V v

-j

+
=

co S 2 a

[CHAP.

/?

cos2

/?

2 m^

VA

-j

2
.

+ yl)

cos 2 y

cos cos y

cos y

cos 2 y

2/X2 cos a cos y

ellipsoid of inertia

2Iyz cos

/?

cos y

corresponding to the square plate of

10.7,

m = Ma2

Then the equation of the

Izz

*Ma*, Ixy

= -i-Ma2

by equation

ellipsoid of inertia is

?\

P%

2 Pl

h xPy

0,

/,

page 256,

(20),

\MaVx + iMaVy + \Mtfpl - \Ma? 9xPy =


or

Ixz

= 3/Ma*

EULER'S EQUATIONS OF MOTION


10.15.

Find a relationship between the time rate of change of angular momentum of a rigid
body relative to axes fixed in space and in the body respectively.
body axes are chosen as principal axes having directions of the unit vectors
and e 3 respectively, then the angular momentum becomes

If the rigid

e lt e 2

= ^U)^ +

CI

Now

by Problem

6.1,

page 147,

and

if

/2 2e 2

refer

^3"3C3

to

space

(fixed)

and body (moving)

axes

respectively, then

at

dt

Is

\b

hu^! +

10.16.

{ Jl"l

72 2 e 2

+
+

the principle of angular

~ ^2) w d 3} e + {-^2"2 +
+ (12 ~ ^l) w 2 w l}e 3
2'

(22),

is

+ I2 ^2^2 + h w aPs)

(^ 1 ~~ ^3)"l3}e2

page 256.

momentum, we have

A
where

(/l"iei

{h^3

Derive Euler's equations of motion

By

+ "2^2 + "3^3) X

W 11

(^3

+ h^3 e 3

the total external torque.

Tt\.

Writing

A = A^ +

A 2e 2 + A 3 e 3

(2)

where A 1? A 2 A 3 are the components of the external torque along the principal axes and making
use of (1) and Problem 10.15, we find
,

/jwj

72 w 2
73w 3

+
+
+

/2 )u 2 w 3 = A
(h ~ h)^3 u A 2
(/ 2 /i)wi<o 2 = A 3

(I 3

(3)

CHAP.

SPACE MOTION OF RIGID BODIES

101

265

FORCE FREE MOTION OF A RIGID BODY. ROTATION OF THE EARTH


10.17. A rigid body which is symmetric about an axis has one point fixed on this

axis.

Discuss the rotational morion of the body, assuming that there are no forces acting
other than the reaction force at the fixed point.
Choose the axis of symmetry coincident with one of the principal axes, say the one having
Then I t = 7 2 and Euler's equations become

direction e 3

+
+

A"i
/i<o 2

=
=

(^3 "~ 7i)o> 2 o> 3

I 3 )w 3 ui

(Ii

/3 3

From

(3),

o>

constant

-A

(5)

with respect to

Aw,

h
~^~

we

(U),

73-/^2

o> 2

U)

(5)

find

A 2w 2

(6)

k2 <2

(7)

73-^
h

where
Solving

we

(7),

we choose

Then from

find

=
w2

we have

(5)

=
=

o>

the time scale so that

Thus the angular

From

)Aw 2

h~h

If

/j \

and using

(2)

become after dividing by I lt

(2)

/ 7S

0) 2

Differentiating

and

so that (1)

(1)

wjej

velocity

cos xt

+ C

when

0,

then

= C

sin Kt

(9)

ut

= C

cos Kt

(10)

is

+ 10363
+ C sin Kt

e2

+ Ae s

(11)
is

constant in magnitude equal to w = || = VC2 +


and precesses around the "3" axis with frequency

A2

\h

w 2e2

cos Kt e x

follows

it

sin Kt

w2

that the angular velocity

this

(8)

2,7

-7/

^A

(12)

2tt7!

as indicated in Fig. 10-13.

Note that the vector <> describes a cone about the


However, this motion is relative to the
body principal axes which are in turn rotating in
space with angular velocity u.
"3" axis.

10.18. Calculate the

about

Fig. 10-13

precession frequency of Problem 10.17 in the case of the earth rotating

its axis.

Since the earth rotates about its axis once in a day, we have
the precessional frequency is from Problems 10.12 and 10.17,

~(\
cir

7
l

The period of precession


and is explained as due

is

=
77-(l--M
Ci/
&tt
\

thus

P=

1//

305 days.

<o 3

IT- (.00328)(2*-)

=A =
=

2w radians/day.

Then

.00328 radians/day

Zir

The actual observed period

to the non-rigidity of the earth.

is

about 430 days

SPACE MOTION OF RIGID BODIES

266

[CHAP.

10

THE INVARIABLE LINE AND PLANE.


POLHODE, HERPOLHODE, SPACE AND BODY CONES
10.19.

Describe the rotation of the earth about

From Problem

10.17 the angular velocity

o)

w1e1

CI

71 1e 1

Let a be the angle between


3

w 2e 2

+
o>

w 3e3

7 1 w 2 e2

-0

~^~

w 3e 3

axis in terms of the space and body cones.

its

and angular momentum

o>

C(cos Kt ej

= hC

^3 w 3 e 3

= Ae 3

and

/?

be the angle between 3 and


3

a>

|* 3

and

From

(1)

and

we

(2)

cos

72

cos a

7 3A2

(1)

A2

/?

Ay/C2 + A 2

cos

A2

/3

V^ + A2

see that

7X C

sin/8

VC 2 +

V/fC2 + 7fA 2
Thus

tan

IS

tan

Now

for the earth or any oblate spheroid

that

A2

Q
=

ft

<*)

tan a
tan /3

<

7 3Ae 3

Then

m.

cos/3

|<a|

V/fC2 +

Similarly, let

e 2)

Then

CI.

73

and

are given respectively by

CI

+ Ae 3

+ sm Kt

cos Kt e i

A V/f C2 + ZfA 2

| 3 ||0|cosa

sin Kt e 2 )

(5)

h
at the poles]

[flattened

we have

7X

<

73

It

follows

(3.

The situation can be described geometrically by Fig. 10-14. The cone with axis in the direction
fixed in space and is called the space cone. The cone with axis <* 3 = u 3 e 3 is considered as
fixed in the earth and is called the body cone. The body cone rolls on the space cone so that the
CI

is

element in

common

is

the angular velocity vector


ei

<l)

e2

e3

I t C sin Kt

A
I3A

<!>

I^C cos

Kt

<>.

Now
=

A7 C
X

sin Kt e 1

+ A7 C
X

cos Kt e 2

Thus
CI

(o> 3

(I^C cos Kt ej

It
lie

follows
in

( A7 t C

from Problem

1.21(6),

Kt e 2 + 7 3 Ae 3
+ AI C cos/ct e 2

7jC sin

sin Kt ex

page

16,

that

CI,

o> 3

and

o>

one plane.

An observer fixed in space would see the vector <o


tracing out the space cone [see Fig. 10-14]. An observer
fixed in the body would see the vector tracing out the body
Cone

cone.

In the case of the earth the space cone is inside the


For the case of a
to the fact that l x < 7 3
prolate spheroid the reverse situation is true, i.e. 7 t > 7 3 and
the space cone is outside the body cone [see Problem 10.121].

body cone due

Fig. 10-14

CHAP.

SPACE MOTION OF RIGID BODIES

10]

267

THE EULER ANGLES


10.20.

three separate figures how the xyz coordinate system of Fig. 10-4,
x'y'z' coordinate system by successive rotations through

Show by using

page 257, is rotated into the


the Euler angles
6 and ^.
<j>,

z or

Figures 10-15, 10-16 and 10-17.


Fig.
10-15 indicates a rotation through angle
of the x and y
axes into an X and Y axis respectively while keeping

Refer

to

<f>

the

same

z or

axis.

In Fig. 10-16 a rotation about the


axis through
angle e is indicated so that the Y and Z axes of
Fig. 10-15 are carried into the Y' and Z' axes of
Fig. 10-16 respectively.

In Fig. 10-17 a rotation about the Z' or z' axis


is indicated so that the X' and Y' axes
are carried into the x' and y' axes respectively.

through angle ^

In the figures
axes; X, Y,

x, y, z

axes by i,j,k;

I,

we have

J,

axes;

K;

I', J',

indicated unit vectors on the


X' , Y' Z' axes and x', y', z'
,

K' and

i',

j',k'

respectively.

Z' or

Fig. 10-15

z'

XorX'

Fig. 10-16

10.21.

Fig. 10-17

Find the relationships between the unit vectors (a) i,j, k and I, J, K of Fig. 10-15,
(b) I, J, K and F, J', K' of Fig. 10-16,
(c) I', J', K' and i', j', k' of Fig. 10-17.
(a)

From

(i-I)I

=
=

(k-I)I

(6)

From

=
=
K =
(c)

Fig. 10-15,

(j

1)1

+
+
+

(I-I')I'

(J'I')r

F
J'

K'

=
=
=

(j-J)J

(k-J)J

+ (i-K)K
+ (j-K)K
+ (k-K)K

cos

sin

sin

cos

<f>

<f>

J
J

Fig. 10-16,

From

(i-J)J

(K'I')I'

+
+
+

(I-J')J'

(J-J')J'

(K'J')J'

+
+
+

(I'K')K'

I'

(J-K')K'

cos e J'

sin e K'

(KK')K'

sin e J'

cos e K'

(I'-k')k'

cos

\p

sin

(J'-k')k'

sin

i'

cos ^

(K'-k')k'

k'

Fig. 10-17,
(I'-i')i'

(J'-i')i'

(K'i')i'

+
+
+

(I'-j')J'

(J'j')j'

(K'j')j'

+
+
+

\p j

j'

10.22.

Express the unit vectors


From Problem

cos

cos

cos

sin

cos

sin

Derive equations

(25),
o>

cos

JK'

co^k

sin

Wfl I'

(0 sin

u x \'

w a 'j'

co

cos

3'

sin

k =

3,

K'

cos

i'

<p

cos

J'

sin

cos

j'

K'

cos0j',

k'

sin

K'

sin

sin

k'

sin

k'

<p

I'

cos 0)j'

sin

cos

cos

3'

cos

sin

cos

cos

3'

cos

sin * k'

sin

k'

K'

j'

cos

sin 0)i'

sin

cos 0)3'

cos

cos

sin

i'

cos
cos

sin

3'

cos

k'

i'

(0 sin

0k
cos

sin

sin

(9

i'

cos

c^K'

sin

Si'

3'

3'

0K'
cos

k'

0k'

cos 0)i'

cos

sin 0)3'

(0 cos

0)k'

2 'k',

'

<V
<o

sin

(9

+
=

sin

<p

page 258.

o>

sin

sin

cos

cos

i'

sin

cos

K sin

sin

sin

sin

cos

( sin

J'

(9

sin 0)i'

sin

cos

cos

since

j'

i'

cos

sin

i'

<f>

sin

sin

cos

cos

cos

(sin

Then

sin

cos

sin

s in

k\

K',

sin

( cos

e 3'

cos

cos

(cos

+
j

i', j',

J'

i'

sin

3,

<p

sin0j',

sin

cos

sin

cos

cos0i'

V,

I'

Then

j,k in terms of

i,

10

10.21,
i

10.23.

[CHAP.

SPACE MOTION OF RIGID BODIES

268

'

wt

sin

sin

cos

"2

sm *

cos

~~

sin

<o 3

cos

Write the kinetic energy of rotation of a rigid body with respect to the principal
axes in terms of the Euler angles, (b) What does the result in (a) become if h = hi

10.24. (a)

(a)

Using Problem

10.23, the required kinetic

(/ lW ?

^/j(0 sin

+
(6)

If

/j

72

12

energy

is

seen to be

4 + 1 A)
sin

-1-/2(0

+
sin

cos 0) 2
cos

sin 0) 2

i/ 3 (0 cos

0)

the result can be written

i/ x (02 sin 2

02)

1/3(0 cos

0)

MOTION OF SPINNING TOPS AND GYROSCOPES


10.25. Set

up equations for the motion of a spinning top having

fixed point

[see Fig. 10-18].

CHAP.

SPACE MOTION OF RIGID BODIES

10]

269

Let xyz represent an inertial or fixed set


of axes having origin O. Let x'y'z' represent
principal axes of the top having the same
origin.
Choose the orientation of the x'y'
plane so that Oz, Oz' and Oy' are coplanar.
Then the x' axis is in the xy plane. The
line
in the x'y' plane making an angle ^
with the x' axis is assumed to be attached
to the top.

ON

The angular velocity corresponding to


the rotation of the x'y'z' axes with respect to
the xyz axes is

wie^

w 2e2

~^~

w3e3

(1)

In obtaining the angular momentum we must


use the fact that in addition to the com-

ponent w 3 due to rotation of the

x'y'z'

there

the component

also

is

since the top

is

spinning about the

Then the angular momentum

system
= ^e 3

se 3

Fig. 10-18

O
Now

if

we

/xwje!

6.1,

<ZO

dt
(1)

and

+ h^z + s

in (3),

(2)

~~fo

total torque about

we

(3)

~ h)^2^3 + h u 2 s )^i
+ ih^2 + (^1 ^3) w u 3 ~

ih^i

(h

(k

sin

e 1 )e 1

e2

(k

dCl

with

dt

e 2 )e 2

wl

^12

(k

x (-mgk)

(le s )

e 3 )e 3

mgl(e 3 X

cos

I2

we

mgl

k)

find

from

+
+

A) W 2W 3 +
^3)wi"3
(^1

(^3

^3"2S

(ir/2

fl)e 2 +

(5)

cos 6 e 3

(4)

sin e e x

and

(6)

(6),

(7)

= m 9l sm *

=
+ s) =

^3<0lS

73 (<u 3

Express equations

-^l)"l"2} e 3

If

-^l

10.26.

(h

U)

e3

cos

It

X (mg)

(fc g )

A =
A =

+ *) +

{-^("3

^3 w l s ) e 2

is

is

Then using

we have

find

A =

the torque

(*)

\b

Since

e3

dt

The

page 147,
dCl

Using

7 2 <o 2 e 2

subscripts / and b denote the fixed system and body system respectively,

let

by Problem

axis.

z'

is

(7)

of Problem 10.25 in terms of the Euler angles 6

and

of

<

Fig. 10-18.

The components

We

o>

v 2 w 3 can be obtained from Problem 10.23 by formally letting


,

0.

find
Ul

Then equations

(7)

0,

u2

of Problem 10.25 become

sm

9,

a5 3

COS 6

(1)

SPACE MOTION OF RIGID BODIES

270

[CHAP.

+ (^3 h)$ 2 sm e cos e + h$ s sm 6 = m ^


+ $e cose) + (7j I 3 )e^>cose I 3 es =0
73
cos o $e sin e + s)

''

hC<P sin<9

10

sin e
V

(2)

velocity of precession,

The quantities <, and s are often known as the magnitudes of the angular
and of spin respectively.

of nutation

10.27.

Prove that the equations


(a)

he -

(b)

h{'4 sin

where

Z"

cose)

2j>6

+ 3 A</>sin0
- hA8 =

cos

sin

mgl

sin e

a constant.

is

From

2
7i<J)

Problem 10.26 can be written as

of

(2)

the third equation in

w3

Then substitution

of

(I)

= A

or

= A

and second of equations

into the first

Using the results

of Problem 10.25,

(7)

7 1 w 2 w3

73 2

/ 1 6j 2

7 1 <o 1 3

/3W1A

Problem

we

10.26,

(1)

yields

(7)

A m 9l

7i"i

W
W

sin e

=
and

find that equations (2)

(3)

reduce to the required

equations.

10.28. (a)
(b)

Find the condition for steady precession of a

Show
Since

top.

that two precessional frequencies are possible.


is

constant so that

(Irf

=
2

we have from Problem

0,

cos e

Irf 2 cos

or

from which

73

<t>

7 3 A<
I3

10.27(a),

mgl) sin

A$ + mgl =

A V7fA 2 - Amglh

cos e

g^-^

>

Thus there are two frequencies provided that


/fA 2
If

A 2 = 4mgll
If A is very

73

> Amglli

only one frequency

cos

is

possible.

large, e.g. if the spin of the top

very great, then there are two frequencies,

is

one large and one small, given by

mgl/I 3A

I 3A/(I lC ose),

10.29.

(2)

cos e

Prove that

(a)

%h(e 2

(b)

2
h4> sin d

<J

sin 2

8)

+ hA

|/ 3 A

cos e

Multiply equations

(7)

of Problem
/ifttji!

*>22)

result.

10.25 by Wl

7 3("3

constant

constant

and give a physical interpretation of each


(a)

+ mgl cos e

co 2

and

+ *)(S +)

+s

<* 3

respectively,

m^

and add

to

obtain

Sin ' *

which can be written as

f {#M + 4) +
t

Integrating and using

<*

+s = A

ll^e*

<

a*

+s

2
>

>

f (~ m9lcose)

as well as the results

2 sin 2 0)

|7 3A

+ mgl

Wl

cos e

and

= E

<o

sine,

we

find

CO

CHAP.

SPACE MOTION OF RIGID BODIES

10]

where

The

constant.

is

result

271

equivalent to the principle of conservation of energy,

is

since the kinetic energy is

T =
while the potential energy

T+V

and

(6)

is

AJ^fl 2

^ 2 sin2

V = mgl

is

e)

%I3A*

(2)

cos e

(8)

the total energy.

Multiplying the result of Problem 10.27(6) by sin

I^sirfo +

e,

11 x <t>6 sin e cos e

I3Ae sine

which can be written

Irf sin 2

Integrating,

I3

I 3A costf)

2
jt(/i0 sin

cos 6

=
= K

constant

(-4)

this result physically, we note that the vertical component of the angular
Irf sin 2 e + I3A cos e [see Problem 10.123], and this must be constant since the
torque due to the weight of the top has zero component in the vertical direction.

To interpret

momentum

10.30.

Let u

cos

is

Prove

6.

that:

u2 = {a-/3u)(l-u2 )- {y-Su) 2 =

(a)

where

= 2(E - ihA 2 )/h, p =

(a)

From Problem

<p

sin 2 e)

(2),

Substituting this into

so that

cos *

(K

I3

sin

= E

(1)

= K

cos

{2)

cos e
<*>

/ lSin2,

cos 0)2

+ *A + mgU se = E

and

8 h

27.(1-^)
2

K I 3A

2/ lS in2 g

y.r^ +
Thus

I3

cos 6

(I),

i' 1 '

= hA/h;

constant

+ mgl

I3A*

Irf sin2 6

Letting

7m

10.29,

i/ 1 (ff 2

From

= K/I u

du

(6)

2mgl/h, y

f(u)

(^p)

2rog

sin2 9

u2

+ "*"" =

'^-"

2)

this

becomes

-*7^

&^a v _ 1 / A,
1

which can be written as

w2 =
where

(2tf

- I^A

(a
2

Note that with this notation

- fiu)(l - u*) -

)/^,
{3)

2mgl/I 1

(y

- 8u) 2 =

/()

= K/I

lt

(4)

= /aA/^

(5)

can be written
.

_
~

8u"

T^

(<?)

[CHAP.

SPACE MOTION OF RIGID BODIES

272

(b)

From

the result of

(a)

we

u >

have, since

10

0,

<&*

or

Integrating,

The integral can be evaluated

10.31. (a)

Prove that

(a)

elliptic

(7)

functions which are periodic.

which

at those values of m for

= (-/8M)(l-M 2 )-(y-8M) 2 -

Prove that the equation in (a) has three real roots u u u 2 ,u 3 but that in general
not all the angles corresponding to these are real.
From Problem

Since

-sin

e 0,

it

u2

10.30(a),

/( M )

Equation

/?

>

- m)(1 ~ 2 -

(a

where

- fiu)(l - m 2 )

(y

(y

- 8u) 2

or

S)2

/()

(1)

0.

Thus

at the
(2)

can be written as

(1)

f(u)

Since

f(u)

follows that

roots of the equation

(b)

terms of

in

f(u)
(b)

0,

it

/3tt

(S 2

+ a)u 2 +

(2yS

- /3)u +

follows that

= -(y-S) 2

/(I)

/(-l)

Thus there is a change of sign from


to + as u goes from 1 to , and
consequently there must be a root, say
u 3 between 1 and as indicated in
,

Fig. 10-19.

Now we know that in order for


the motion of the top to take place we
must have

^
It

f(u)

= u2 s

o.

we must have

ir/2,

since

Also,

tt

1.

thus follows that there must be two

roots

m x and u 2 between

and

1,

as

indicated in the figure.


It

follows that in general there are

e x and e 2 such
that cos 0i
cos 2 u 2- ^ n special
cases it could happen that u x
U<l = Ug = 1.

two corresponding angles

10.32.

Fig. 10-19

Give a physical interpretation of the results found in Problem 10.31.

and u 2 corresponding to 6 X and 2 respectively, shows


The fact that there are two roots
that the motion of the top is such that its axis always makes an angle e with the vertical which
This motion, which is a bobbing up and down of the axis between the
lies between e y and e 2
nutation and takes place at the same time as the precessional motion
limits <?! and
is
called
2
of the axis of the top about the vertical and the spinning of the top about its axis. Because
the motion can be expressed in terms of elliptic functions [see Problem 10.104], we can show
.

that

it

is

periodic.

In general the tip of the axis of the top will describe one of various types of curves such as
indicated in Figs. 10-20, 10-21 and 10-22. The type of curve will depend on the root of the
equation [see equation (6) of Problem 10.30]
<t>

_
=

Su
u2

(1)

CHAP.

SPACE MOTION OF RIGID BODIES

10]

If this root

given by y/8

the curve of Fig. 10-20 occurs. If it is the same as


between u x and u 2 the curve of Fig. 10-22 occurs. Other
the same as u x or is less than u x [see Problems 10.124].

greater than u 2

is

u 2 the curve of

Fig. 10-21 obtains.

cases can arise

if

y/8

the root

273

is

If it is

> u2

y/8

u2

Aside from the general motion which is made up of nutation


various special cases which can arise. One of these is the case of
nutation [see Problem 10.28]. In this case u x = u 2 so that e x ~ e 2
case is the "sleeping top" which occurs where u 2 = u 3 = 1 and the axis
[see

Problem

y/8

< u2

Fig. 10-22

Fig. 10-21

Fig. 10-20

remains vertical

<

u-l

and precession, there are


steady precession with no
or e = constant. Another
of the spinning top always

10.36].

MISCELLANEOUS PROBLEMS
T

the total kinetic energy of rotation of a rigid body with one point fixed,
prove that dT/dt - <o A where all quantities refer to the body principal axes.

10.33. If

is

Multiplying both sides of the Euler equations

of

(3)

Problem 10.16 by w 1 ,w 2

<o 3

respectively

and adding, we obtain


^l w l w l

But

iiCOjWj

0)^ +

and

Thus

^
+

0) 2

~^~

^2 (0 2

^3 W 3 W 3

"l-^l

~^~

c0 2-^2

~^~

w 3-^3

(7)

2 \ dT
_l^/r2j_r2.r

2
3< 3
+lt'3 W 3 W 3 o di lWl
~dt
2<

=
=

"3-^3

("l^l
6>

2 e 2

<">3

e 3)

* (

(2)

'

A lel

+ A 2e2 + A 3e3

(3)

dT/dt

becomes

(1)

~f~

i 2 O) 2 0) 2

A2

w2

at

Prove that if there are no forces acting on a rigid body with one point fixed,
then the total kinetic energy of rotation is constant, (b) Thus prove that O =

10.34. (a)

IT

constant.

(a)

Since there are no forces,

(6)

Since

O=

/jo^ej

0.

+ I2 u 2e 2 + 730)363
*

10.35.

A=
O =

Then by Problem
and

/ x o)f

7 2 w|

o^e^

o> 2

/ 3 o>3

The

momentum

dT/dt

or

T =

constant.

+ 0)363,

in

constant

terms of the kinetic energy and

of the rigid body.

kinetic energy is

T =
so that

e2

= 2T

Find the precession frequency of Problem 10.17


angular

10.33,

i(/,o)

72 o)|

I8 |)

/jC2

4(/ lW?

73A2

h4 + 73W

= 2T

|)

= h2 +

J sA2 )
(1)

SPACE MOTION OF RIGID BODIES

274

The angular momentum

O =
=
=

so that

|Q|

(i)

and

/2 w 2 e 2

7 1 C(cos Kt e l

= V/fC2 + /|A2

we

sin id e 2 )

^l w l e l

C2 +

^3 w 3 e 3

^3-Ae 3

10.17, equation

1%A 2

fi2

S22

(2)

_ 277,
h(h-h)

fi 2

A2 -

hda-It)'

the precession frequency

(12),

\(n*-2Ti 1 )(i3

-i

(3)

is

l )\

04)

2^/i

10.36.

^l"2e 2

find

2773

(72

Then from Problem

^3 w 3 e 3

or

simultaneously,

(2)

10

is

Il03 1 e 1

Solving

[CHAP.

Find the condition for a "sleeping top".


and
For a "sleeping top" we must have e =
and no nutation can take place. Then from Problem

hA

K,

from Problem 10.30 we have a


and y = 8, and so
Also,

/(h)

It follows

(a

- j8u)(l - h2) -

that

/(h)

(y

I3

2mgl/I l

- 8h)2 =

(1

will "sleep" if this root is greater

must remain

since the axis

0,

vertical

10.29,

A 2 - 2(E-mgl)
=

2mgl/I v y

- u)(l - it*) -

has a double root at


v2

Then the top

1,

2
y (l

l3All x

- u)2 =

(1

- h)2 [(1 + u) -

while the third root

J|A

2mgll 1

I 3 A/I V

is

Thus a

($

y2]

given by

than or at most equal to

1,

so that

A2 ^ Amglljll
Of course, even though this condition may apply at the beginning, energy will in practice
2
be diminished due to friction at the support so that after some time we will have A < Amglljl^.
In such case precession combined with nutation will be introduced. Further loss of energy will
ultimately cause the top to fall down.

10.37.

Find the torque needed to rotate a rectangular plate of sides a and


about a diagonal with constant angular velocity .
By Problem

10.9 the principal

plate at the center


lx

We

= ^M<fi

have
a>

moments

b [see Fig. 10-23]

of inertia of the

are given by
I2

= ^Mb*,

(wi)i

4-

Iz

= -^M(a2 + b*)

(1)

(*j)j
(2)

Va2 + 6 2

Va2 + 6 2

Thus
Substituting

w2

'

Va2 +
(1)

and

Fig. 10-23

62

osb
'

V2 + 6 2

into Euler's equations


/jWi

I2 a 2
/3

<J)

(7 3

72) W 2 W 3 = A

+ (/ 73)<3 W = A 2
+ (J 2 Ji)"i"2 = A 3
x

w3

CHAP.

SPACE MOTION OF RIGID BODIES

10]

we

find

Aj

0,

A2 =

A3 =

0,

M(b 2 az)abu 2

U(

the rectangular plate

if

re(l uired

torque about

^
O

is

- cfi)ab<J-

M(&2

12(a2

a square,

is

Thus the

b*)

A ~
Note that

,,

275

i.e.

(4)

62)

if

b,

then

A=

0.

Supplementary Problems
GENERAL MOTION OF RIGID BODIES IN SPACE
10.38.

Find the number of degrees of freedom of (a) a sphere free to roll on a plane,
free to rotate about a fixed point, (c) an airplane moving in space.
Ans.

10.39.

(a)

(b)

3,

an

ellipsoid

(c)

3,

(b)

In Fig. 10-24 a displacement of a tetrahedron in


space is indicated. Show directly that the displacement can be accomplished by a translation
plus a rotation about a suitable axis, thus
illustrating Chasle's theorem [page 224] for
space.

10.40.

Give an illustration similar to that of Problem


10.39 involving a rigid body whose surfaces are
not plane surfaces.

10.41.

Derive the result of Problem 10.2, page 259,


without using Problem 6.1, page 147.

Fig. 10-24

ANGULAR MOMENTUM. KINETIC ENERGY. MOMENTS AND PRODUCTS OF INERTIA


10.42.
A rigid body consists of 3 particles of masses 2,1,4 located at (1,-1,1), (2,0,2), (1,1,0)
Find the angular momentum of the body
velocity = 3i - 2j + 4k.
Arts. -6j + 42k
tively.

10.43.

10.45.

(a)

What

is

Find the

I xx

(a)

(a)
(&)

AB =

16,

16;

(6) I xy

of inertia and

and

AD

6, I yz

2,

= iM62, Iyy = l M a?, Igz = M(a? +


hy = ~lMab, lyz = 0, Ixz =

AB

and

AD

the products of inertia

1 = -6
Ans. 180

(6) products of inertia of a uniform rectangular plate


taken about axes AB, AD and the line perpendicular to

Ixx

calling axes through

62)

the x and y axes respectively.

(a) moments of inertia and (6) products of inertia of a cube of side a taken about x,y, z
axes coinciding with three intersecting edges of the cube.

Find the
Ans.

10.47.

moments

of sides
the plate at B.

Ans.

12, I yy

(b)

the kinetic energy of rotation for the system of Problem 10.42?

ABCD

10.46.

respecrotated about the origin with angular

Determine the (a) moments of inertia about the x, y and z axes and
for the rigid body of Problem 10.42.
Ans.

10.44.

if it is

(a)

Ixx

Iyw

Izz

%Ma2,

(b)

Ixy

Iyz

Ixz

- -\Ma*

Find the (a) angular momentum and (6) kinetic energy of rotation of the cube of Problem 10.46
about the point of intersection O of the three edges if the cube has an angular velocity
Ans. (a) ^MaHlOi + 43j - 45k), (6) 185MaV12
a, = 2i + 5j - 3k about
O.

SPACE MOTION OF RIGID BODIES

276

10.48.

Find the
x2 + y2 + z2
Ans.

moments of

(a)

(a)

PRINCIPAL MOMENTS OF INERTIA.


10.49.

10.50.

and

inertia

= a2 in the first octant,


Ixx = Im = Izz = Ma 2
(b)

products of inertia

(b)

i.e.

in the region

Ixy

Iyz

*g0,

2/

[CHAP.

the

of
0, z

uniform

10

sphere

solid

0.

= -2MaV5ir

Ixz

PRINCIPAL AXES.

ELLIPSOID OF INERTIA

Prove that the principal moments of inertia for a system consisting of two particles of masses
m l and m2 connected by a massless rigid rod of length I are I t = J2 m l m 2 ^/{m l + m 2 ), Iz = 0.

Find the

principal

(a)

moments

of inertia and

(b)

directions of the principal axes for the system

of Problem 10.42.

Ans.

Ix

(a)
(b)

10.51.

= 13 - t/T3 I 3 = 13 + \/73
+
V73)i-J + k, j(i_-/73)i_j + k>
(/

18, I 2

+ k,

Determine

the
(a)
principal moments of inertia
directions of the principal axes for right triangle
of Fig. 10-25 about point C.
(b)

10.52.

and

ABC

Find the principal moments of inertia at the center of a


b and acute angle a.

parallelogram of sides a and


10.53.

Find the

principal

(a)

moments

of inertia and

tions of the principal axes for the cube of

Ans.

Zj

(a)

72

(6)

Problem

direc-

10.46.

= Ma2

flilfa2 , I 3

Axis associated with Is is in the direction of the diagonal from the origin. Axes
associated with J x and I2 have any mutually perpendicular directions in a plane perpen-

(6)

dicular to this diagonal.

10.54.

Find the principal moments of inertia of a uniform cylinder of radius a and height
Ans. J a

I2

= jLM(3a 2 + h2

h.

= \Ma2

I3

),

10.55.

Obtain the principal moments of inertia and directions of principal axes for a rectangle of
sides a and b by using Problem 10.45 and equations (11), page 255. Compare with Problem 10.9,
page 262.

10.56.

Find the lengths of the axes of the

Problem
10.57.

4^3/Ma

Ans.

10.55.

2
,

4V3/llMa

Prove that the

10.59.

If I V I 2 ,I 3 are the principal


I,

10.62.

2v

ellipsoid

Ay/Z/Mb 2

of

inertia

4^S/M(a +
2

b2

corresponding to

ellipsoid of inertia corresponding to the cube of

6/Ma

Under what conditions do any

I2

Suppose that a rigid body

at point O, then

is

is

Problem

h,

inertia,

and determine

10.46.

its

radius.

prove that

^ h + h.

h ^ h+h

or all of the equality signs hold in Problem 10.59?

Prove that if a rigid body is a solid of revolution about a line L, then


corresponding to any part of L.

to

of

moments of

rectangle

the

ellipsoid of inertia for a regular tetrahedron is a sphere

10.58.

10.61.

Find the lengths of the axes of the


Ans. 4\/3/llMa

10.60.

symmetrical about a plane P. Prove that if


a principal axis corresponding to point O.

is

is

a principal axis

a line perpendicular

EULER'S EQUATIONS OF MOTION


10.63.

rigid

body having one point O fixed and no external torque about O, has two equal principal
Prove that it must rotate with constant angular velocity.

axes of inertia.

CHAP.

10.64.

SPACE MOTION OF RIGID BODIES

10]

277

Write Euler's equations for the case of plane motion of a rigid body and discuss their physical
significance.

10.65.

Solve the problem of a

10.66.

Describe

compound pendulum by using Euler's equations.

Euler's equations can be used to discuss the motion of a solid cylinder rolling

how

down

an inclined plane.
10.67.

in case the axes are not principal axes.

Write Euler's equations

FORCE FREE MOTION. INVARIABLE LINE AND PLANE.


POLHODE, HERPOLHODE, SPACE AND BODY CONES
10.68.

If

two principal moments of inertia corresponding

rotates are equal, prove that (a) Poinsot's ellipsoid


is a circle and
(c) the herpolhode is a circle.

which a rigid body


an ellipsoid of revolution, (6) the polhode

to the fixed point about


is

10.69.

Discuss the (a) invariable line and plane, (6) polhode and herpolhode and (c) space and body
cones for the case of a rigid body which moves parallel to a given plane, i.e. plane motion of a
rigid body.

10.70.

(a)

How

(6)

What

would you define the instantaneous axis of rotation for space motion of a rigid body?
is the relationship between the instantaneous axis of rotation and the space and body

cones?
10.71.

Prove that relative to its center of mass the axis about which the earth spins in a
rotate about an axis inclined at 23.5 with respect to it in 25,780 years.

day

will

THE EULER ANGLES


10.72.

Using the notation of Problem 10.20, page 267, find:


terms of I,J,K; (c) i',j',k' in terms of I',J',K'.
J = sin $ i + cos
I = cos
i + sin
Ans.
(a)
j,
<f>

10.73.

(6)

I'

(c)

i'

=
=

Prove the results

<f>

=
F+

J'

I,

cos ^

ux

ay
ug
10.74.

If

Ix

axes

=
is

I2

I3,

cos e J

sin ^ J',

=
=
=

9 cos

<f>

sin

<f>

yp

sin e K,

J'

cos

K'

sin f

^ sin

e sin

^ sin

I'

(a) I,

<f>

cos $

J',

terms of

in

K =

j,

sin

J,K

(6)

I\J',K' in

cos e
k'

i,j,k;

K'

<j>

cos

prove that the kinetic energy of rotation of a rigid body referred to principal

T = \I X (4? +

o2

j<

+ 2w

cos

e).

MOTION OF SPINNING TOPS AND GYROSCOPES


10.75.

10.76.

top having radius of gyration about its axis equal to 6 cm is spun about its axis. The spinning
point is fixed and the center of gravity is on the axis at a distance 3 cm from this fixed point.
If it is observed that the top precesses about the vertical at 20 revolutions per minute, find the
Ans. 3.10 rev/sec or 19.5 rad/sec
angular speed of the top about its axis.

uniform solid right circular cone of radius a and height h is spun so that its vertex is fixed
and its axis is inclined at a constant angle a with the vertical. If the axis precesses about the

vertical with period P, determine the angular speed of the cone about its axis.

10.77.

Work Problem
same

10.78.

10.76 if the cone

is

surmounted by a uniform

solid

hemisphere of radius a and the

density.

Explain physically why the spin axis of the gyroscope of Figures 10-6 and 10-7, page 258,
should maintain its direction.

SPACE MOTION OF RIGID BODIES

278

10.79.

[CHAP.

10

Explain how a gyroscope can be used to enable a ship, airplane, submarine or missile to follow

some

specified course of motion.

MISCELLANEOUS PROBLEMS
10.80.

uniform

solid

cube of side a and mass

has

its

of a coordinate system with vertex at the origin O.

angular velocity
10.81.

the angular

momentum.

Find the moment of inertia of a uniform


the base,

(a)

10.82.

w, find

the vertex.

(b)

Arts,

(a)

edges lying on the positive x, y and z axes


about the z axis with constant

If it rotates
Arts,

J^Ma 2 <o(3i + 3j 8k)

cone of radius

solid

^Ma2

(b)

height h and mass

about

Find the principal moments of inertia at the center of a uniform elliptical plate having semi-major
Ans. I t = %Mb 2 I 2 = \Ma2 I 3 = \M{a 2 + b 2 )
axis a and semi-minor axis b.
,

10.83.

a,

-^M{2h2 + a2 )

top has the form of a solid circular disk of radius o and mass
and length I attached to its center
with a thin rod of mass
Find the angular velocity with which the top
[see Fig. 10-26].
should be spun so as to "sleep". Assume that the base point O

is fixed.

10.84.

Work Problem

10.85.

Work Problem

10.86.

10.83 for a cone of radius a, height h and

10.83 for a cone of radius a, height h and mass


surmounted by a hemisphere of radius a and mass m.

coin

of

radius a

angular velocity
if

10.87.

mass M.

to

>

o>

is

set

M
Fig. 10-26

spinning about a vertical axis with


Prove that the motion is stable

[see Fig. 10-27].

Agla.

Suppose that the coin of Problem 10.86 is spun with angular speed
s about a diameter which is inclined at an angle a with the vertical
and which is fixed at point O. Assuming there is no nutation,
find the angular speed with which the coin precesses about the
vertical.

10.88.

10.89.

Discuss how gyroscopes can be used to control the motions of a ship


on a stormy sea.

Fig. 10-27

is fixed at point
of a uniform solid cone of radius a, height h and mass
horizontal plane. Prove that if the cone rolls on the plane with angular velocity <o about

The vertex

.....

perpendicular to the plane through O, then the kinetic energy of rotation

10.90.

Explain how the principal axes of a rigid body can be found


is known.

principal axes

10.91.

uniform

its altitude.

has the radius of its base equal to twice


Prove that the ellipsoid of inertia corresponding

solid cone

to its vertex is

10.92.

a sphere.

Explain how a gyroscope can be used as a compass, often


called a gyrocompass.

10.93.

dumbbell consists of two equal masses

attached to a
of length I and negligible mass [see Fig. 10-28].
with constant
The system rotates about a vertical axis
angular velocity * such that the rod makes a constant
angle 8 with the vertical. Prove that the angular momentum
Q of the system describes a cone of angle irl2 e about u

rod

ABC

DCE

and has magnitude \Ml2 u

sin e

if

is

of a

an axis

3Mh2 (a? + 6fc 2)co 2


40(a2 + h2

'

the direction of one of the

CHAP.

10.94.

SPACE MOTION OF RIGID BODIES

10]

Prove that the magnitude of the torque needed to keep the system of Problem 10.93

(a)
is

279

\Ml

10.95.

Work

10.96.

2 v> 2 sin2<?

(a)

(&)

What

is

Problem 10.93 and

in

motion

the direction of the torque?

Problem 10.94

(6)

if

the rod

ACB

has mass m.

thin solid uniform circular plate of radius a has its center


attached to the top of a thin fixed vertical rod OA [see
Fig. 10-29]. It is spun with constant angular speed w about
an axis which is inclined at angle a with the normal OB to
the plate,
(a) Prove that the angular velocity vector *
precesses about the normal OB with period 2tt/(u cos a).
(b) Prove that the axis OB describes a space cone with period
2tt/(<o

\/1

+3

cos 2 a)

10.97.

In Problem 10.96 find the angle through which the plate


turns during the time it takes OB to describe the space cone.

10.98.

Find the principal moments of inertia of a uniform solid cone


taken about the (a) vertex,
of radius a, height h and mass

(6)

center of mass.

Ans.

(a)
(b)

10.99.

= h = ^M(a2 + Ah2
h=h= i>M(h* + 4a2

Ix

73

= ^Ma2
= J^Ma2

oscillates

I3

),

),

compound pendulum of mass

with respect to the principal axes of

Fig. 10-29

about a horizontal axis which makes angles

a, /?, y
of inertia are 7 1 ,72 3
to the axis of rotation is I, prove that for

If the principal

inertia.

from the center of mass


the period is 2v^Mgl/I where I = Ml2

respectively and the distance

small oscillations
10.100.

+ 1^

moments

cos2 a

+ I2

cos 2

(3

+ /3

cos2 y.

Find the period of small oscillations of a uniform solid


cone which rotates about a horizontal axis attached to
the vertex of the cone.

10.101.

An

elliptical plate [see Fig. 10-30] having semi-major


and semi-minor axes of lengths a and 6 respectively is
rotated with constant angular speed about an axis
making a constant angle a with the major axis. Find

the torque required to produce this motion.


10.102.

Work Problem
by an

10.103.

if

the elliptical plate

is

replaced
Fig. 10-30

Given Euler's equations of motion for a rigid body having zero external torque about a fixed
point O,

i.e.,

I^X

10.104.

10.101

ellipsoid.

(/ 3

jT

2 )u 2 <o 3

prove that

^i w i

and

/?i

Prove
dy/dx

from

V(l

Problem

722

0,

Cl

~"

-^3)

W 3W 1

Z 3 3

0,

~^~

^2 W 2

~t"

^3 U 3

constant

2T

/|u|

73<o|

constant

(/ 2

^l)l"2 =

H2

that a lt 2 and 3 satisfy a differential equation of the form


and thus show that the angular velocity can be expressed in terms

10.103

# 2)(1 ~ k 2 x 2

),

of elliptic functions.
10.105.

Find the moment of inertia of a uniform solid cone of radius a, height h and mass
line which lies in its surface.
Ans. ^Ma2 (a 2 + 6h2 )/(a2 + h2 )

10.106.

The moments and products of

inertia of a rigid

^=

4/3, Iyz
8/3, Ixy
4/3, I
10/3, Izz
(6) the directions of the principal axes.

Iyy

Ans.

(a)

Ix

(6)

ei

= 3, J2 = 2, 73 = 4
= i - 2j - 2k, e2 = -2i + j - 2k,

e3

0.

about a

body about the x, y and z axes are Ixx = 3,


Find (a) the principal moments of inertia and

-2i

- 2j + k

10.107.

[CHAP.

SPACE MOTION OF RIGID BODIES

280

10

cone having semi-vertical angle o rolls with constant angular speed w on a horizontal plane with
vertex fixed at a point O. Prove that the axis of the cone rotates about the vertical axis
through O with angular speed w tan a.
its

10.108.

uniform
horizontal plane rotates about a vertical axis with constant angular velocity <.
a is placed on this plane. Prove that the center describes a circle with
angular velocity given in magnitude by |-w.

solid sphere of radius

10.109.

Work Problem 10.108 if the


Ans. aK2 /(K2 + a 2 where K
)

10.110.

sphere
is

not necessarily of constant density.

is

the radius of gyration about a diameter

Show how to find the relative maximum and minimum


* = Ax2 + By2 + Cz 2 + Dxy + Eyz + Fxz - 1.

distances from the origin to the ellipsoid

Maximize or minimize the function * = x 2 + y 2 + z z subject to the condition * = 1.


method of Lagrange multipliers, i.e. consider the function G = * + X* where
the (constant) Lagrange multiplier and set dG/dx, dG/dy, dG/dz equal to zero.]

[Hint.

To do
X
10.111.

is

this use the

Explain the relationship of Problem 10.110

moments
10.112. (a)

(b)

10.113.

to

the method of page

Find the relative maximum and


9x 2 + 10y 2 + Sz 2 + Axy - 4scz = 3.

minimum

Discuss the connection of the results of

(a)

from

distances

the

origin

(2,

1, 3)

in

the direction 3i

to

the

line

through the

2j + 4k.

10.114.

Prove that the motion of the "sleeping top" of Problem 10.36

10.115.

Find the moment of inertia of the lemniscate

10.116

r2

a 2 cos 2e

is

stable if

A2 ^

Amglljl^

about the z axis

Ans. jfMa2

plane rigid body (lamina) has an xy and x'y' coordinate system with common origin
that the angle between the x and x' axes is a [see Fig. 10-31]. Prove that

(a)

Ix x
>

>

Ixx cos 2 a
I rT sin 2 a

Use Problem 10.116

ellipsoid

with those of Problem 10.106.

Find the moment of inertia of the system of particles of Problem 10.42 about a
point

10.117.

255 for obtaining principal

of inertia and directions of principal axes.

+
+

2Ixy sin a cos a


21rv sin a cos a

such

Iyy sin 2 a
2
/ sin a

to prove that

and give a physical interpretation.


10.118.

10.119.

Referring to Problem 10.116, find an expression for Ix

in

terms of Jxx ,Ixy ,Iyy and

Use the results of Problems 10.116 and 10.118 to prove that for a plane region having moments
system,
and products of inertia defined by Ixx Ixy> Iyy corresponding to a particular xy coordinate
angle a given by
the principal axes are obtained by a rotation of these axes through an
,

tan 2a
10.120.

a.

Ixy /(Iyy

Ixx

).

Prove that the lengths of the principal axes

in

Problem 10.116 are given by

CHAP.

10.121.

10.122.

SPACE MOTION OF RIGID BODIES

10]

Discuss Problem 10.19, page 266,

if

lx

>

/3

281

Find the moment of inertia of a uniform semicircular wire of mass


Ans. 2M(ir 2)a2/v
center.

10.123.

Prove that the expression on the left side of equation


component of the angular momentum.

10.124.

Discuss Problem 10.32

10.125.

if

the root of equation

(1)

(a)

is

(4)

in

and radius a about

Problem 10.29

equal to u lf

(b) less

its

the vertical

is

than u v

body consists of 3 particles of masses m u ra2 and ra3 The distance between m l and
2
m s and m l are l 12 hz an d Z3 i respectively. Prove that the moment of inertia of the
system about an axis perpendicular to the plane of the particles through their center of mass is
given by
,2
.2
,2

m2

rigid

and

ra 3 ;

m m + m^m^
mi + m2 + m
+

Wi2*i2

3 l2 3

10.126.

Derive a "parallel axis theorem" for products of inertia and illustrate by means of an example.

10.127.

Prove that the principal moments of inertia of a triangle of sides a,b,c and mass
center of mass are given by
It

10.128.

I2

= ^(2 +

coin of radius 1.5

cm

fe2

rolls

C2

2 Va4 +

64

+ C4 -

tt

2&2

fc2 c 2

_ C2 a2

/3

ff

( 2

&2

about the

C2)

without slipping on a horizontal table such that the plane of the


coin moves at a speed of 3 m/sec,
Ans. 2.5

makes an angle of 60 with the table. If the center of the


prove that the coin moves in a circular path and find its radius.
coin

),

Chapter 11

LAGRANGE'S

EQUATIONS

GENERAL METHODS OF MECHANICS


Up to now we have dealt primarily with the formulation of problems in mechanics by
Newton's laws of motion. It is possible to give treatments of mechanics from rather
general viewpoints, in particular those due to Lagrange and Hamilton.
Although such treatments reduce to Newton's laws, they are characterized not only
by the relative ease with which many problems can be formulated and solved but by their
relationship in both theory and application to such advanced fields as quantum mechanics,
statistical mechanics, celestial mechanics and electrodynamics.

GENERALIZED COORDINATES

particles moves subject to possible constraints,


Suppose that a particle or a system of
as for example a particle moving along a circular wire or a rigid body moving along an
inclined plane. Then there will be a minimum number of independent coordinates needed
to specify the motion.

These coordinates denoted by


q\,

<?2,

.,q n

(1)

are called generalized coordinates and can be distances, angles or quantities relating to
them. The number n of generalized coordinates is the number of degrees of freedom
[see

page

165].

Many

sets

of generalized coordinates

may

be possible in a given problem, but a

strategic choice can simplify the analysis considerably.

NOTATION
In the following the subscript a will range from 1 to n, the number of degrees of
freedom, while the subscript v will range from 1 to N, the number of particles in the
system.

TRANSFORMATION EQUATIONS
Let

x vi

+ Vvj + z v k

be the position vector of the vth particle with respect to an


1

xyz coordinate system. The relationships of the generalized coordinates


coordinates are given by the transformation equations
Xv
ijv

zv

where

denotes the time.

X v (qi,

#2,

Qn, t)

Vv{qi, q%,

qn, t) [

z v (qi, q 2 ,

.,

In vector form,
rv

The functions

=
=
=

in (2) or (3) are

(2)

(1)

to the position

(2)

qn, t)

can be written

r v {qi, q%,

qn

t)

(#)

supposed to be continuous and to have continuous derivatives.

282

CHAP.

283

LAGRANGE'S EQUATIONS

Ill

CLASSIFICATION OF MECHANICAL SYSTEMS


rheonomic,
Mechanical systems can be classified according as they are scleronomic or
below.
defined
as
non-conservative
or
conservative
and
holonomic or non-holonomic,

SCLERONOMIC AND RHEONOMIC SYSTEMS


explicitly in the
In many mechanical systems of importance the time t does not enter
others, as for
equations (2) or (3). Such systems are sometimes called scleronomic. In
example those involving moving constraints, the time t does enter explicitly. Such systems
are called rheonomic.

HOLONOMIC AND NON-HOLONOMIC SYSTEMS


let t
Let q it q 2 ...,q n denote the generalized coordinates describing a system and
having
equations
denote the time. If all the constraints of the system can be expressed as
or their equivalent, then the system is said to be holonomic;
the form <j>{qi, q 2
q n t) =
otherwise the system is said to be non-holonomic. Compare page 170.
,

CONSERVATIVE AND NON-CONSERVATIVE SYSTEMS


on a system of particles are derivable from a potential function
non-conpotential energy] V, then the system is called conservative, otherwise it is

If all forces acting


[or

servative.

KINETIC ENERGY. GENERALIZED VELOCITIES


The

total kinetic

energy of the system

is

= I 5>r2
6 v=l

(*)

kinetic energy can be written as a quadratic form in the generalized velocities q a


quadratic form
If the system is scleronomic [i.e. independent of time * explicitly], then the
are
also present.
in
terms
linear
a
rheonomic,
is
If
it
q
only terms of the form a^haq&.

The

has

GENERALIZED FORCES
If

is

the total

work done on a system

of particles

particle, then

dW =

*i$adq a
N

where

2= Fv

called

Problem

the

generalized

associated

force

(5)

=l

<x

is

by forces F acting on the vth

dr v
*

T^~
dq a

with

the

generalized

coordinate

qa

See

11.6.

LAGRANGE'S EQUATIONS
The generalized force can be
Problem 11.10]

related to the kinetic energy

(9T) -iT
dt\dq a J

dq a

*8

by the equations

[see

(7)

LAGRANGE'S EQUATIONS

284

[CHAP.

11

If the system is conservative so that the forces are derivable from a potential or potential
energy V, we can write (7) as

d(BL\ _

BL

dt\dqa J

dq a

L = T-V

where
is

called the

(9)

Lagrangian function of the system, or simply the Lagrangian.

The equations
systems which
If

or (8) are called Lagrange's equations and are valid for holonomic
be scleronomic or rheonomic.

(7)

may

some of the forces

system are conservative so as to be derivable from a potential


etc., are non-conservative, we can write Lagrange's

in a

V while other forces such as friction,


equations as

*(*) - *k =

dt\dq a J
where

L-T-V

and

$>L

*;

(10)

dq a

are the generalized forces associated with the non-conservative

forces in the system.

GENERALIZED MOMENTA
We define

at
dq a

momentum

to be the generalized

pa

call

the

momentum

associated with the generalized coordinate q a


conjugate to q a or the conjugate momentum.

We

often

system is conservative with potential energy depending only on the generalized


coordinates, then (11) can be written in terms of the Lagrangian L = T - V as
If the

BL
Pa

(12)

Bq a

LAGRANGE'S EQUATIONS FOR NON-HOLONOMIC SYSTEMS


Suppose that there are

equations of constraint having the form

VAadqa + Adt =

^Badq a + Bdt =

0,

or equivalents

^A

+ A =

qa

^B

0,

We

must

m<n

of course have

...

0,

(13)

where n

is

the

qa

+ B =

number

0,

(U)

...

of coordinates q a

may not be integrable so as to obtain a relationship


the constraints are non-holonomic or nonintegrable
involving the q a 's. If they are not
integrable; otherwise they are holonomic or integrable.
The equations

(13) or

(U) may or

In either case Lagrange's equations can be replaced by


d_

(dT\ _ aT

dt\dq a J
where the
If

L=

parameters

Ai,

A 2 ... are called


,

the forces are conservative,


_ x
as
,
,

T-V

dq a

(15)

XiAa

XzBa

...

Lagrange multipliers

[see

(i 5)

Problem

11.18].

can be written in terms of the Lagrangian

r
9L\ _dL_

dt\dq a J

dq a

XiAa

X2Ba

...

(16)

CHAP.

LAGRANGE'S EQUATIONS

Ill

285

It should be emphasized that the above results are applicable to holonomic (as well as
non-holonomic) systems since a constraint condition of the form

...,Qn,t)

<l>{Ql,Q2,

(17)

can by differentiation be written as

^p-d Qa
which has the

form

&dt =

(18)

(13).

LAGRANGE'S EQUATIONS WITH IMPULSIVE FORCES


Suppose that the forces F acting on a system are such that
dt

Then we

call

C F

lim

where

represents a time interval.

Sv

(19)

F impulsive forces and

are called

impulses.
If we let the subscripts 1 and 2 denote respectively quantities before and after application of the impulsive forces, Lagrange's equations become [see Problem 11.23]

- (2L)

'H.)
where
If

fa

we

call

fa

is

2*

*Jv

fa

(20)

(21)

dq a

the generalized impulse, (20) can be written

Generalized impulse

which

\dq a /i

^dq a /2

a generalization of

Theorem

change in generalized

2.6,

momentum

(22)

page 36.

Solved Problems

GENERALIZED COORDINATES AND TRANSFORMATION EQUATIONS


11.1.

Give a set of generalized coordinates needed to completely specify the motion of


each of the following: (a) a particle constrained to move on an ellipse, (b) a circular
cylinder rolling down an inclined plane, (c) the two masses in a double pendulum
[Fig. 11-3] constrained to move in a plane.
(a)

moving on the
Let the ellipse be chosen in the xy plane of Fig. 11-1. The particle of mass
ellipse has coordinates (x,y). However, since we have the transformation equations x = a cos 6,
y = 6 sin e, we can specify the motion completely by use of the generalized coordinate 0.

Fig. 11-1

Fig. 11-2

LAGRANGE'S EQUATIONS

286

(b)

11.2.

11

The position of the cylinder [Fig. 11-2 above] on the inclined plane can be completely specified
by giving the distance x traveled by the center of mass and the angle e of rotation turned
through by the cylinder about its axis.
is

(c)

[CHAP.

If there is no slipping, x is related to e so that only one generalized coordinate [either x or


needed. If there is slipping, two generalized coordinates x and e are needed.

Two

coordinates &i and 6 2 completely specify the positions of masses


above] and can be considered as the required generalized coordinates.

Write the transformation equations for the system in Problem

and

w2

e]

[see Fig. 11-3

11.1(c).

Choose an xy coordinate system as shown in Fig. 11-3. Let {x x ,y^) and {x^y^t be the rectangular
coordinates of
and ra 2 respectively. Then from Fig. 11-3 we see that
1

=
=

xx
x2

l-i

cos $i

cos e i

cos

2/i

V2

l\

sin $i

h sm

*l

+ h

sin

which are the required transformation equations.

11.3.

Prove that

We

have

^.
dq a

=
|t
dq<x
=

r v (q u q 2

..., q n >

Then

t).

Thus

We

11.4.

dqa

()

dqa

can look upon this result as a "cancellation of the dots".

Prove that

We

()
dt\dqj

have from

(1)

= !r3 #

of Problem 11.3,

d2 rv

drv

oq a

A (Oil)

N W

dt\dqj

dq a

d 2 r

d<li

.
'

dtfl^

dqa dt
dr

"

*
"

d(

v\ ln
d (
dqn \d qoi J dt

- JL { ?IL) -! 4~ d qi \dqa J dt + '"


d*r v

fl2r v

d <la d <ln

d 2r v

a~ dq<*
z~
dq
n

d ( ?>*v
-dt\dqa

d2r

+ IT^T
dtdq a

Since r is assumed to have continuous second order partial derivatives, the order of differThus from (2) and (3) the required result follows.
entiation does not matter.

The

result can be interpreted as

an interchange of order of the operators,

A(jL\ =

i.e.,

{A\

CLASSIFICATION OF MECHANICAL SYSTEMS


11.5.

Classify each of the following according as they are (i) scleronomic or rheonomic,
(iii) conservative or non-conservative.
(ii) holonomic or non-holonomic and
(a)

sphere rolling

down from the

top of a fixed sphere.

CHAP.

LAGRANGE'S EQUATIONS

Ill

cylinder rolling without slipping

(c)

A
A

(d)

particle

(b)

287

a rough inclined plane of angle

down

the inner surface, with coefficient of friction


paraboloid of revolution having its axis vertical and vertex downward.
particle sliding

down

^,

of a

which rotates with constant

frictionless wire

moving on a very long

a.

angular speed about a horizontal axis.


(a)

(b)

scleronomic

[equations do not involve time

non-holonomic

[since rolling sphere leaves the fixed sphere at

conservative

[gravitational force acting

is

explicitly]

some

point]

derivable from a potential]

scleronomic
[equation of constraint

holonomic

that of a line or plane]

is

conservative
(c)

scleronomic

holonomic
non-conservative
(d)

[since force

due to friction

[constraint involves time

rheonomic
holonomic

[equation of constraint

not derivable from a potential]

is

explicitly]

that of a line which involves

is

t explicitly]

conservative

WORK, KINETIC ENERGY AND GENERALIZED FORCES


11.6.

Derive equations

(5)

and

(6),

page 283, for the work done on a system of

Suppose that a system undergoes increments dq lt dq 2


rth particle undergoes a displacement

particles.

dq n of the generalized coordinates.

Then the

"

Thus the

total

work done

dW =

dr v

dq a
2t9

dr v

a=l

is

Fdrv

v=l

F,

[a=l

v=l

*a

where

We

11.7.

call

* a the generalized

Prove that

We

have

<i>

= dW/dq a

W?
oqa

11.8.

a =X

* a dq a

dr

2t

v=i

t
oq
a

force associated with the generalized coordinate qa

dW = ^j^-dq a

Also,

by Problem

dW =

11.6,

5 (.-)*. =
Thus

since the dq a are independent, all coefficients of dq a

2*<*g a

Then

must be

zero, so that

*a

Let F be the net external force acting on the vth particle of a system.

f^

By Newton's

dr~|

7u \Z, mvrv
dt
\v

dq a )

_
^
>m

second law applied to the yth particle,

mr

dr
=
t
5<
v

"V
2/
v

= dW/dq a

Prove that

dTv

F " ^r
oQct
i?

we have
(1)

LAGRANGE'S EQUATIONS

288

drv

=
mv rv >-
dq

Then

Now

by Problem

3r\
dq a J

d /.
11.4,

-rr

dt\
(

r
v

dr

dr
- v

..

"

..

Hence from

we

(2)

have, since ra

d (
-r:

dt\

Summing

'

(a)

d (

dt\

is

rav r

"

dT

^
v

dr\
dq a J

to v \
dq a /

m v rv

over
9rv

The

...

dq a

kinetic energy is

Bq a

dq a

dT

r2%rj
dT

v?

^
2,m

'

Thus

K-

9rv

rv

'

dqa

page

11.3,

d *v

"

Prove that

dtv

m K

have by the "cancellation of the dots" [Problem

dT

drv

? ^'^

dq a

3ga

dr
- v
dq a

_F

= -2
-

we have
9rv

dQa

dqa

We

&v

3r
-

all particles,

(b)

dqa

m
^'^q-J-^ ^-Iq-a

dic v

<*

be the kinetic energy of a system of particles.

Bqa

constant,

both sides with respect to

d f
d-t\^

Let

9tv

dqa

d 'v

dqa
_,,

d / r

dt\dq

^+

11

(2)

rv

dqa

_ v

[CHAP.

dq a

tt

11.9.

286],

3r"

dqa

dqa

LAGRANGE'S EQUATIONS
11.10.

^ (|)

Prove that

From Problem

11.9(a)

and

3r

(2)

and

*l

1,

(3) in (1),

we

The quantity

where

*=|F

^
drv

the generalized

<?

it

ar"
.

U)

Pa

momentum

{S)

find

d(8T_)_f_ =

nate qa

.,n

a *"

'3Ta

dt\dqa J

is called

11.9(6),

Then substituting

*,

11.8,

d f

From Problems

J^ =

or conjugate

dqa

W
(5)

J^~

momentum

associated with the generalized coordi

CHAP.

11.11.

LAGRANGE'S EQUATIONS

11]

289

Suppose that the forces acting on a system of particles are derivable from a potential
is
function V, i.e. suppose that the system is conservative. Prove that if
the Lagrangian function, then

L-T-V

dt\Bqa )
If the forces are derivable

dq a

from a potential V, then


*

Since the potential, or potential energy

is

_ dW =

dV

3qa

Bqa

a function of only the

L = 4-<r-v) =

(b)
(a)

[and possibly the time

t],

3qa

3V_

d /3L\
dt\9qj

11.10,

dt

11.12. (a)

q's

11.7],

^?-

dqa

Bqa

Then from Problem

Problem

[see

/BL\ _

_ _

BT_

dqa

9qa

\9qa )

dL

Set up the Lagrangian for a simple pendulum and


obtain an equation describing its motion.
Choose as generalized coordinate the angle e made by string
OB of the pendulum and the vertical OA [see Fig. 11-4].
If I is the length of OB, then the kinetic energy is

T = mv 2 =
where

the

mass of the

(1)

bob.

[taking as reference
mass
a horizontal plane through the lowest point A] is given

The
level

is

= mP 2

%m{le)2

potential energy of

by

V = mg(OA - OC) =
=
Thus the Lagrangian

Lagrange's equation

From

e)

cos

e)

()

V = ^ml2 e 2 dL

dL
(3),

mgl

Be

(4),

ml
is

cos

dt\Bi)

2 e

11.13.

mgl(l

is

Substituting these in

which

is

L
(b)

mg(l

we

mgl(l

cos e)

(4)

Be

3L

sin 9 ,

= ml2 e

(5)

Be

find

+ mgl

sin e

-jrsin*

the required equation of motion [compare Problem 4.23, page 102].

mass Af2 hangs at one end of a string which passes over a fixed frictionless nonAt the other end of this string there is a
rotating pulley [see Fig. 11-5 below].
non-rotating pulley of mass Mi over which there is a string carrying masses mi and m%.
(b) Find the acceleration of mass Af.
(a) Set up the Lagrangian of the system,
Let

pulley.

X2 be the distances of masses x and 2 respectively below the center of the fixed
t and
Let x x and x 2 be the distances of masses
2 respectively below the center of the
x and

movable pulley

Mv

Since the strings are fixed in length,

X + X2 =
t

constant

a,

xx

x2

constant

LAGRANGE'S EQUATIONS

290

Then by

differentiating with respect to time

or

X2 = X

or

x2

and

xx

x2

= X2 = -X

11

t,

X + X2 =
x

[CHAP.

= x x

Thus we have

M
Velocity of M
Velocity of

Velocity of

-j-

Velocity of

m2 =

(X

Then the

(X x

X +

xx

+ x2 = X +

x2

+x

x)

energy of the system

total kinetic

= Xx

Flg 11_5
*

is

= M X2 + \M2\ + m

(X x

The total potential energy of the system measured


of the fixed pulley as reference is

Then the Lagrangian

)*

(1)

- m 2 g{X + x 2
+ x - m2 g(X + b - x
)

1)

(2)

x)

+ \M2 k\ + %m (X + x )2 + \m2 {X - xtf


+ M gX + M2 g{a-X + m g(X + x + m 2g(X + b - x

X.\

x)

Lagrange's equations corresponding to

From

(3)

jg-

Mg

dL
r-

MX

mg - m

dL

= mi(X + x

1)

(3)

x)

and x x are

_d

Xj

dL

+ Lm 2 (k -x

is

= %M

dX x

)2

from a horizontal plane through the center

x)

T - V

+x

M gX - m g(X + x
M fif(a-Z]) - m g(X

V = -M gX = -M gX 1

xx

fdL_\ _dL_

dt\dx x J

3*1

U)

dx x

we have
x

#
Mg

MX

+ m

+ m

2fl

{m x

+x

(X x

x)

+ m 2 (X x
x

x)

(M x +

m 2 {X x

(4)

x)

m 2 )X +

(m x

(m x

Then the downward

+ m + m 2 )X +
x

(m x

+ m2 )x

become

(M x + M2 + m x + m 2 )X x + (m x m 2 ) x x

Solving simultaneously,

dXi

Thus equations

-m )g

x)

(M x - M2 + m x + m 2 )g

mg

we

m2)X +

(m x

(M x M2 + m x + m 2 )g

+ m^) x x =

(m x

m 2)g

find

X,

(M x M 2 )(m x + m2 ) + 4m m 2
(M x + M2 )(m x + m2 ) + 4m 1 m2

xx

(M x + M 2 )(m x + m 2 ) + 4m x m 2

acceleration of

2M2 (m m2
x

mass

X2 X

2 is

constant and equal to

(M2 M x )(m x + m 2 ) 4m x m 2
(M x + Af2)(m 1 + m2) + 4m 1ra2

(m x

m 2 )x

CHAP.

11.14.

291

LAGRANGE'S EQUATIONS

Ill

Use Lagrange's equations


of Problem 8.1, page 197.

to set

differential equation of the vibrating

up the

The

Refer to Figs. 8-7 and 8-8 of page 197.

masses

kinetic energy of the system is

T = \mx\ + \mx\

CO

Fig. 8-8 are numerically equal to x lf x 2


Since the stretches of springs AP,PQ and QB of
is
system
the
of
energy
potential
x 2 respectively, the

V =
Thus the Lagrangian

%kx 21

-x

%k(x 2

2
1)

+ &x 22

and

(*)

is

L = t - V = \mx\ + \mx\ -

\kx\

\k[x 2

- xj* - &x\

(3)

Lagrange's equations are

SL

d /BL\
dt{axj

dL

Then

since

-kx,

'

- *i) =

k(x 2

/dL\

dL

dt

\dxJ

dx 9

"(a?2

- 2x i)>

dx 2
equations

(4)

-k{x 2

m'x\

become

x)

k(x 2

agreeing with those obtained in Problem

kx 2

(!

2x 2),

mx 2 -

-2xJ,
8.1,

(-4)

dL

- mx

T*~
dx l

dx-,

dL

11.15.

dx i

dL

mx 2

dX 2

k(x 1

-2x 2

(5)

page 197.

pendulum
Use Lagrange's equations to find the differential equation for a compound
axis.
horizontal
fixed
about
a
plane
vertical
which oscillates in a
Let the plane of oscillation be represented by the xy plane
where O is its intersection with the axis of rotation and C is the center of mass.
of Fig. 11-6,

Suppose that the mass of the pendulum is


of inertia about the axis of rotation is I =
of gyration], and distance OC = h.

M,

MK

moment

its

[K

radius

OC

makes with^the
If e is the instantaneous angle which
2
vertical axis through O, then the kinetic energy is T = %I 6 =
plane
2 2
horizontal
a
relative
to
energy
The potential

MK

through

is

Since dL/do
equation is

V = Mgh cos 6. Then the Lagrangian


L = T -V = \MK2 e 2 + Mgh cos e
- -Mgh

sin e

and

dLIdd

dL

dL

dt[do

de

d_ /

MK

2 e,

is

Lagrange's

i.e.,

MK

2 6

+ Mgh

Compare Problem

11.16.

particle of

function,

page

1.147,
(a)

(b)

The

sin
9.24,

or

V +

sin

Fig. 11-6

page 237.

moves in a conservative force field. Find (a) the Lagrangian


of motion in cylindrical coordinates ( P 4>, z) [see Problem
equations
the

mass

32].

total kinetic

energy T

the Lagrangian function

- %m[p 2 + p 2 $ 2 + z 2

The

potential energy

is

L =

T-V

lm[p2

+ p 2^2 + |2] -V( p ,0,s)

V =

V(p,<p,z).

Then

LAGRANGE'S EQUATIONS

292

11

Lagrange's equations are

(b)

d /8L\

dL

d /8L\

dL

dL

fdL\

11.17.

[CHAP.

Work Problem

d.

&

dV\

\,dV

dV

xy plane and

in the

dV

..

moves

11.16 if the particle

d..,av

..

if

the potential

depends only on the distance from the origin.


In this case
11.16

depends only on P and z

Then Lagrange's equations

0.

in

part

(6)

of Problem

become

rnG-p'^)
These are the equations for motion

A( P 20) =

-^,

a central force

in

field

obtained in Problem

5.3,

page 122.

LAGRANGE'S EQUATIONS FOR NON-HOLONOMIC SYSTEMS


11.18.

Derive Lagrange's equations (15), page 284, for non-holonomic constraints.


Assume that there are m constraint conditions of the form

2a A a dqa +
where

As

m<

n,

number

the

Problem

in

11.10,

Adt =

2a B a dqa

0,

of coordinates q a

+ B dt =

...

0,

(1)

page 288, we have

dT

8T

..

drv

If Sr are virtual displacements which satisfy the instantaneous constraints [obtained by considering that time t is a constant], then

Now

the virtual

work done

SW
Now

since the virtual

**

(4-)

work can be written

subtraction of

(3)

dr

=
- S?a
2 2a m r oq

8rv

in

SW
we have by

is

2v

^ dr
2^-5g
a

Sr

and

2 Ya 8qa
a

terms of the generalized forces

2*
a
a

<t>

a as

S 9a

(5)

(6)

(5),

2(^a-*a)Sg a
a

Since the 8qa are not all independent, we cannot conclude that
Lagrange's equations as obtained in Problem 11.10.

Prom

(1),

since

t is

constant for instantaneous con train ts,

J,A a 8qa
a

Multiplying these by the

(4)

0,

lB
a

Lagrange multipliers

2 (M a + X2B a +

a Sqa

X lt X 2 , ...

8qa

Ya = $ a

we have
0,

the

which would lead

to

equations

...

(7)

and adding, we have

(8)

CHAP.

293

LAGRANGE'S EQUATIONS

11]

Subtraction of

and

(6)

(8) yields

2 (Y a -

X 2B a

8q a

(9)

of the quantities 8q a [say 5^,


Now because of equations (7) we can solve for
.,
Thus in (9) we can consider 8 ffl>
., 8q n ].
of the remaining 8q a [say 8g m + 1
and Sg m + i,
., 8q n as independent.

.,8q m ] in terms
m as dependent

S<?

equal to zero,
Let us arbitrarily set the coefficients of the dependent variables

*a

X l^a

X2

Ba

1,2,

o,

...,m

Equations

and

(2), (10)

11.19.

* a+Xl A a +

These equations together with

Derive equations

From Problem

ar-

()

.,

thus lead to

(11)

d/3T\_|T =
as required.

m+1,

0,

(10)

and since these are

will be left in the sum (9) only the independent quantities 8q a


bitrary it follows that their coefficients will be zero. Thus

Then there

i.e.,

lead to n

(i)

X a Fa

+m

l,2,...,n

equations in n

+m

unknowns.

page 284, for conservative non-holonomic systems.

(16),
11.18,

/dT

dT

<f>

d <la

dt \dq a
J

Then if the forces are derivable from a


qa Thus (1) can be written

potential,

X xA a

<t>

X 2B

= -dV/dq a

(1)

where

does not depend on

11.20.

dt\dq a J

L = T V.

where

d <la

moves under the influence


particle of mass
of gravity on the inner surface of the paraboloid
x 2 + y 2 = az which is assumed
of revolution
frictionless [see Fig. 11-7]. Obtain the equations
of motion.

By Problem
ordinates

is

11.16, the

L =
Since x 2

+ y2 =

m('p 2

2
,

2p 8p

we

call

in cylindrical co-

Qi

the equations

+ P2

+ I 2 - mgz

q% = <t>, <lz z and compare (2) with


of Problem 11.18, we see that

p,

(7)

since only one constraint is given.

<tt\dp)
(1),

az =
(2)

Using

is p

a 8z

A =

1,e "

{D

the constraint condition

so that

If

Lagrangian

given by

these become

3p

A2 =

0,

A 3 = -a

Lagrange's equations

/1L\

2X lP

2p,

Fig. 11-7

-*k
<ft^4

[see

xA
d^

o,

(3)

Problem

11.19]

can thus be written

1,2,

dt\

XjO.

dz

dz

LAGRANGE'S EQUATIONS

294

[CHAP.

11

m('p-p$ 2 ) = 2X lP

= mg

m'z

We

also

have the constraint condition

2pp

The four equations &),

11.21. (a)

(5), (6)

and

(4)

\ta

(6)

az

(7)

enable us to find the four unknowns

(7)

p, $, z,

\v

Prove that the particle of Problem 11.20 will describe a horizontal circle in the
plane z = h provided that it is given an angular velocity whose magnitude is
(o

y/2g/a

(b)

Prove that if the particle is displaced slightly from this circular path
undergo oscillations about the path with frequency given by (lh)^2g/a

(c)

Discuss the stability of the particle in the circular path.

(a)

The radius of the


p

az

circle obtained as the intersection of the

is

Po

Letting z

in equation (6) of

Problem

(1)

2{mg/a)p Q

or

and
w2

(*) in

2g/a

equation

(1)

find

and

we

find

Assuming that the

{2)

\j

and

(7) in

equation

(4)

constant

circle,

p
is

small compared with p

(p

But

= A

(5)

we

find

1-1

A =

aha so that
(6)

h,

we

by

find

letting

= mg/a

a2 h2 a 2 /p 3

Now if the path departs slightly from the


are led to make the transformation
where u

m(- Po

ahu/p 2

Then

-2p/a

+ u) 6

(8)

+u

(8)

find

then p will depart slightly from p


po

(7)

we

of Problem 11.20,
P

in (8),

find

/,=>/?

Since the vibration takes place very nearly in the plane


in equation (6) of Problem 11.20 that

(6)

= w, we

u2 )

(3)

particle starts with angular speed w,

Using

h with the paraboloid

= V2g/a

2,^

of Problem 11.20,

(5)

we

of Problem 11.20 and calling

(4)

particle in this circular path are given respectively by

P1 =
equation

from which

The period and frequency of the

From

plane

= mg/a

<o

(b)

Va>h

11.20,
Xi

Then using

will

it

Thus we
(9)

becomes

to a high degree of approximation,

(p

+ tt) 3

p3(l+w/p

_ i/ 1+ iiV 3 =
)3

p3\

po

l/i_ 3w
p3\

by the binomial theorem, where we have neglected terms involving


values of p and a given by (1) and (5) respectively, (10) becomes

p
2

u u

Using the

CHAP.

295

LAGRANGE'S EQUATIONS

11]

u
whose solution

is

it

et

Po

cos V8r/o

(Sg/a)u

4-

+ u = Vah +

VSg/a

sin

e2

ej

cos

It follows that if the particle is displaced slightly


it

will

undergo

oscillations

t>

e2

sin y/Sg/a

from the circular path of radius

Po

Vah,

\~L

(7)

"\2g

oscillation in the circular

It is interesting that the period of

the period of oscillation about the circular path given by

11.22.

about the path with frequency

or period

(c)

Thus

t.

VSgJa

5)

Since the particle tends to return to the circular path


the motion is one of stability.

path given by

(4)

is

twice

(7).

when

displaced slightly

it is

Discuss the physical significance of the Lagrange multipliers

Xi,

A2

in

from

it,

Problem

11.18.

Problem 11.10,
In case there are no constraints the equations of motion are by

/dT\ _

9T_

dt

d Qa

\dq a )

In case there are constraints the equations are by Problem 11.18,

dt \dq
aJ
It follows

that the terms

\x

Aa +

d<*

\ 2B a

correspond to the generalized forces associated with

constraints.

acting on the
Physically, the Lagrange multipliers are associated with the constraint forces
account
system. Thus when we determine the Lagrange multipliers we are essentially taking into
explicitly.
forces
these
finding
actually
without
forces
constraint
the
of
the effect

LAGRANGE'S EQUATIONS WITH IMPULSIVE FORCES


11.23.

Derive the equations

(20),

page 285.

For the case where forces are

finite

we have by Problem

dt{ dq
*

where

dqa

2 F t

Jo
so that

7-

(1)

dt

with respect to

\*Qa)

Jo

Integrating both sides of

11.10,

from

to

r,

a<*

\dqa /t=T

Taking the limit as

r -* 0,

we have

-lim f*Ldt
auSL-(SL>-rs*
lim

g)

*ia

=
-

(Aim C *,*)>
2
n*s;'-*)'Z

(5)

LAGRANGE'S EQUATIONS

296

11.24.

a
tfa

dt

since -

JA^

f) -(f)

and

is finite,

[CHAP.

9o

lim

t-0

11

T.

Fv dt =

tf.

A square ABCD formed by four rods of length

and mass
hinged at their ends, rests on a
horizontal frictionless table. An impulse of
magnitude 3 is applied to the vertex A in the
direction AD. Find the equations of motion.
21

After the square


eral be a

rhombus

struck, its shape will in gen-

is

[Fig. 11-8].

Suppose that at any time t the angles made by


AD (or BC) and AB (or CD) with the x axis
are e t and 6 2 respectively, while the coordinates of
sides

the center
are (x,
eralized coordinates.

Thus x,y,0 x ,6 2 are the gen-

y).

From Fig. 11-8 we see that the position vectors


of the centers E,F,G,H of the rods are given respectively by

rG

=
=
=

rH

rE
rF

The

(x
(x
(x
{x

velocities of

(y
(y +
(y +

+
+ cos 6 2 )\ +
+ cos 0i)i +
cos e 2 )i +
I

cos 0{)i

(y

E, F,

and

=
vF =
vg =
vH =
vE

at

sin e t )j

sin e 2 )j

sin e^j

sin 6 2)j

any time are given by

=
=

rE
rF

*g
rH

Fig. 11-8

{x

(x

(x
(x

sin

sin e 2 $ 2 )i

sin $! #i)i

sin

(y

+
+
+
+

eji

2 #2)*

(y

(y
(if

+
+

cos $i

cos o 2 e 2 )j

cos e x b i)j

co s *2 *2)j

x )j

The kinetic energy of a rod such as AZ? is the same as the kinetic energy of a particle of
mass m located at its center of mass E plus the kinetic energy of rotation about an axis through
E perpendicular to the xy plane. Since the angular velocity has magnitude e 2 aR d the moment of
inertia of a rod of length 21 about its center of mass is IAB = $ml2 the total energy of rod AB is
,

Similarly, the total kinetic energies of rods

TBC =
Thus the

%mrj.

total kinetic

is

CD

AD

and

are

TAD = %miH2 + %IAD o\

TCD = %mr% + $ICDl,

lIBC el,

energy

BC,

[using the fact that I

IAB

IBC

Icd

T TAB + T BC + TCD + TAD


= |m(r| + rF + r% + rH2 + I(e\ + f
= im(4^2 + 4y 2 + 2Z2 *2 + 2J2*2 + mZ2(^ +

= m 2
*

.)

= 2m(x 2 + y 2 + %ml2 {e\ +


)

2
2)

Let us assume that initially the rhombus is a square at rest with


coordinate axes and its center located at the origin. Then we have
x
If

we

use the notation

0,

0,

H and

<?!

)2

v/2, e 2

0,

0,

to denote quantities before

have

0,

^=

(4i)i

"

\ d V/i

0, e 2

sides parallel to the

and after the impulse

'2Z\
Bx/x

its

(4my)i

is

applied,

we

CHAP.

LAGRANGE'S EQUATIONS

11]

dT

(Amx) 2

3*/ 2

Am'x

ST

V%/ 2

(Amy) 2

= 4m

raZ2 2

/dT

\de 2 /2

\l/2

(5).-.

'

or

4m* = fx

ftf

or

Amy - fy

or

JmPff!

= y
6i

Te*

or

fmPffg

(i?),"(i
{'df2 ) 2

~\dfj

fx f y f $i f h we
,

We

are the impulsive forces.

04)

2j-

(5)

thus have

Fig. 11-8

we

dr

A
Sa-+

T.

3rB

drc

drD

A, -

dr R

3r c

3rD

3rA

3rB

9rc

drD

A,B,C,D

(x

cos 9 1

cos

2 )i

(2/

rB

(x

cos

cos

2 )i

(2/

rc

(x

cos

cos e 2 )i

td

(x

cos e x

cos e 2 )i

dr n

3rA

rA

tf

is initially in

(T)

given by

sin

sin

sin e 1

sin

(y

sin 8 X

sin $ 2 )j

(y

+ sm$i +

sin

2 )j

fl

2 )j

2 )J

the direction of the positive y axis,

we have

= Si

do)

(6)-(9) yield

fx

Then equations

drr

A-^ + A-^+

ca

Thus equations

drB

find the position vectors of

Since the impulsive force at

7^

0,

y 9l = - J*

J,

cos e x ,

Te2

= SI

cos 6 2

(11)

become

(1)-(U)

4m* =

11.25.

fe2

drA

Now from

(S)

note that

T*
where

(2)

where for simplicity we have now removed the subscript


find

\mV-h

(fe) 2 "(fe)i

To

297

Amy = J,

0,

fm/2 ^

= SI

cos

|mZ2

tf^

ff

= 1

cos

(-Z2)

Prove that the kinetic energy developed immediately after application of the impulsive
forces in Problem 11.24 is T = S2 /2m.
From

equations (12) of Problem 11.24,

'

= s

4m>

we have

%s
* = ~8mT C0S ^'

'*

%s

= 8m7 cos '*

LAGRANGE'S EQUATIONS

298

[CHAP.

Substituting these values in the kinetic energy obtained in Problem 11.24,


3

<72

we

11

find

(12

But immediately after application of the impulsive


Thus (1) becomes T = J 2 /2m.

forces,

ex

and

tt/2

62

approximately.

MISCELLANEOUS PROBLEMS
11.26. In Fig. 11-9, AB is a straight frictionless wire
fixed at point A on a vertical axis OA such that
AB rotates about OA with constant angular
A bead of mass m is constrained to
velocity
<>.

Set up the Lagrangian.


Write Lagrange's equations, (c) Determine

move on
(b)

the wire,

(a)

the motion at any time.


(a)

Let r be the distance of the bead from point A of


The rectangular coordinates
the wire at time *.
of the bead are then given by
x

y
z

=
=

r sin a cos at
r sin a sin at

r cos a

the wire
t
it is assumed that at
the xz plane and that the distance from O to A

where

The

is in

Fig. 11-9

is h.

kinetic energy of the bead is

T = m(x 2 + y 2 +

m{(r

z2 )

sin a cos at

+
= ^m(r2 +

co

ar sin a sin

(r sin a sin at

2r2

at) 2

ar sin a cos

at) 2

( r cos

a) 2 }

sin 2 a)

The potential energy, taking the xy plane as reference level, is V = mgz


Then the Lagrangian is
L = T V \m(r2 + a 2 r2 sin2 a) - mg(h r cos a)
(b)

We

have

flL

= m 2 r

sin 2 a

+ mg

L_

cos

a,

is

dL
(

mr

(c)

The general

sin 2 a

+ w#

(w 2 sin 2 a)r

c e (a sin a)t -f

(1)

is

r cos

a).

= mr

or

fif

(co

CO

cos a

hand

C2 e-

cos a)

solution of equation (1) with the right

Since the right hand side of


general solution of (1) is

dr

(ma 2 r

i.e.,

mgr(/i

dr

dr

and Lagrange's equation

side replaced

C2 e-Ca, sin

is

sin a)t

a constant, a particular solution

^CasinaOt

by zero

aK

is

-^-^-.
z
z

__cos a
fif

to

sin z a

functions as
This result can also be written in terms of hyperbolic
g cos a
2 sin 2 a
r = c 3 cosh ( sin a)i + 4 sinh ( sin a)t
to

<o

Inus tne

sin a

(2)

(3)

CHAP.

11.27.

LAGRANGE'S EQUATIONS

11]

299

Suppose that in Problem 11.26 the bead starts from rest at A. How long
to reach the end B of the wire assuming that the length of the wire is It
Since the bead starts from rest at
tion (2) of

Problem

+
,

c 1x

Co

we have

0,

0,

cx

c2

at

it

take

Then from equa-

0.

11.26,
Ci1

Thus

will

= -^2t-tt2
sm

2w

and

g cos a
o
o
w 2 sin 2 a

c2

and

of Problem 11.26 becomes

(2)
v '

_JL2!L_ / e (<a
2 2 sin2 a

& C0S a

sin a)t J- e -(u sin a)t\

/*\

a
4COS
r-^ {cosh (w sm a) 1}
,

flr

which can also be obtained from equation

Z 2

= 14-

" 2si

11.28.

....

When

sin2

Z,

yields

(2)

9 cos a

is

= -4-coshVl-f
w sm a
=

of Problem 11.26.

(3)

cosh ( sin a)*


so that the required time

Sir a

w-4

flr

cos 2 a /

dn^ ln {( 1+ ^^UA/fl
V
g COS a /
2

^^)
gr

-1

cos 2 a

double pendulum [see Problem 11.1(c) and Fig. 11-3, page 285] vibrates in a
(a) Write the Lagrangian of the system,
(b) Obtain equations for
the motion.
vertical plane,

(a)

The transformation equations given

yield

The

xx

lx

cos 9 X

x2

cos

= ~h 9

-l x e x

sin

sin

in

4- Z2

Problem

cos 6 2

^
tf

Z2

page 286,

11.2,

2 sin e 2

yx

V2

lx

sin $ x
sin *i 4- l2 sin e 2

y2

= Z^

lx e

cos

cos e x

4-

2e 2

cos e 2

kinetic energy of the system is

T =

$m

+ y 2 + ^m&l + fy

x {x\

x)

= %m x l\e\

4-

\m2 \$o\ 4-

Z^a

2Z 1 Z2 * 1 * 2 cos

The potential energy of the system [taking as reference


below the point of suspension of Fig. 11-3] is

V =

m g[l
x

Then the Lagrangian

4- Z2

lx

cos e x ]

+ m 2g[l

4- Z
2

(Z x

(^

level

- *2

)]

a plane at distance

cos 0!

Z2

cos

lt

2 )]

is

L = T - V
= \>xA<>\ +

l2

+ |*| + 2Z 1 2 2 cos (e - e 2
-l x cos $ - m2g[l + 2 - (l

%>2{l j\

-m g[l +
(b)

The Lagrange equations associated with

x]

ex

and

are

(i)

))
x

cos e x

4- Z
2

cos 6 2 )]

LAGRANGE'S EQUATIONS

300

From

we

(1)

[CHAP.

11

find

dL/de 2

m2 2 e x e 2 sm e ~ e z) ~ m idh sin e ~ m 20h


= ml\e + m 2 l\e + m 2 2 e 2 c s o 2
= m 2 2 e e 2 sin (*i 2) ~ m*zQh sin 2

dL/de 2

= m 2 l\e 2 + m 2

dL/de x

l l
x

dL/de x

Thus equations

(<?i

+ m 2 l\ e\ + m 2l x

e\

and

sin e x

cos

29 x

e2

{e x

become

{2)

x x

-m 2

2 ?2 * 2

l x l2
l

'e

i cos

l2

e2 m
)

(<?!

sin

xe2

~ #2) ~

cos (*i

sin (e x

l l 6 x62
x 2

+ m2
= m2

(<?!

rn 2 l x l 2 e 2 (e x

m 2 gl

gl x sin e x

e 2 - m2
2 e\(e - e 2
9 2 m 2 gl 2 sin 2
lxl

e2

sin (e x

2)

sin e x

sin {e x

- o2

<9

which reduce respectively to

+ m 2)l\ e\ +

(m x

m2

and

11.29.

*0

m2

2 e2

cos

(<?!

- e 2 + m2

'e\

cos

(0 x

-6 2 - m 2

m = m2
y

lx

21 e\
I 'e x

=
+

in equations

sin (e x

- e2

-{m x + m 2 )gl

x l 2 o\

sm(o x

-e 2

-m

- e2 +
+ 2 -

e 2 cos (e x

cos (e x

- e2

and

(3)

I 'e

m =m
2

sin e x

sine 2

(3)

(4)

and h = h = I

Problem 11.28 and simplifying, they

of

(4)

2 gl2

lei sin {e x

- e2

= -2g

le\ sin {e x

- e2

-g

sin e x

{1)

sin e 2

(2)

are assumed
Obtain the equations of Problem 11.29 for the case where the oscillations
to be small.
Using the approximations sin e = 6, cos
tions (1) and (2) of Problem 11.29 become

11.31.

2 e\

Write the equations of Problem 11.28 for the case


Letting
can be written

11.30.

m2

Find the

(a)

Let

ex

l'e 2

l'e\

l'o 2

= go 2

normal frequencies and


of the double pendulum.

=A

cos*t,

*2

= A2

and neglecting terms involving

2l'e x

(b)

(a)

oscillations

A e^, A 2 e^]

[or

cos<ot

-lo> 2

In order for A x and


equal to zero, i.e.,

A +
x

A 2 to be different from
-

2{g

2 co 4

- 4tow 2 + 2g 2 =

we

Solving,

0.

,.&

4lg

A 2 = 0|
= Oj

CO

zero,

we must have

the determinant of the coefficients

-Zco 2

Zo) )

to 2

find

y/l6l2 g 2

- M29 2

(2V^)9

2l2

{2

A =

(g-l<* 2 )A

-hfl
or

1<*

+ y/2)g
1

W2
'

to the small

the equations of Problem 11.30.

in

2(g-lo> 2 )A x

the equa-

-2ge x

normal modes corresponding

Then they can be written

6 2 e,

_
-

y[2)g
I

CHAP.

LAGRANGE'S EQUATIONS

Ill

301

Thus the normal frequencies are given by

(b)

Substituting

Wl

a2

a*

l(2

+ V2)g
and

= 2^\

2^

(2

_
~

ft

+ -y/2)g/l

Part

in equations (1) of

A 2 = -V2A

i J (2-V2)g
&\

_
~

tt2

tor

yields

(a)

U)

This corresponds to the normal mode in which the bobs are moving in opposite directions.

u2

Substituting

w2

(2

\/2 )gll

in equations

of Part (a)

(1)

yields

A 2 = V2A!

(5)

This corresponds to the normal mode in which the bobs are moving in the same directions.

Set up the Lagrangian for the motion of a symmetrical top [see Problem 10.25,
page 268] and (b) obtain the equations of motion.

11.32. (a)

(a)

The

T = \{h\ + 12 4 + h4) =
The

Problem

kinetic energy in terms of the Euler angles [see

potential energy

%h& sin2 e

V = mgl

is

(6)

T - V

= -J/^ 2

3L/d0

= i^ 2

dL/dd

3L/30

= J^
=

dL/d$

3L/d^

dL/dj,

sin2 e

sin 8 cos

Irf sin 2 8

7 3 (0 cos 8

2)

cos e

is

+ fo

(i)

(2)

and the height of the center of

|/3 (0 cos

/ 3 (< cos 8

Is ($ cos

y^

g2)

page 268]

cos 8

as seen from Fig. 10-18, page 269, since distance OC


Thus
mass C above the xy plane is therefore I cos 8.

10.24,

J)2

^){<t> sin 8)

mflr ; C os

mflfZ

(5)

sin *

cos

4>)

^)

Then Lagrange's equations are

/9L\_9L I\0

or

h4>

cos

sin

A/^.\_^

[7 t

sin2 6

Use the results of Problem 11.32


(a) Problem 10.29(&), page 270, and
(a)

From

equations

(5)

and

of

(6)

Itf

sin 2 8

(6)

(2) in (1),

Using

(2) in

we

find

equation

(4)

ij o

Problem

obtain

to

Problem

(b)

I3 ($ cos 8

I-Jj>

of

^)]

sin

$) cos

0]

mflrZ

-Ok =

sin

(4)

(5)

(6)

agreement with the equation of


10.27(a), page 270.

Problem 11.32 we obtain on integrating,

<

Using

<L/i>LL\

\p)$

J 3 (0 cos 8

j^[/3 (^cos0

11.33.

/3 (^ cos

cos *

^) cos 8

sin 2

11.32,

we

2
Ji< sin 8 cos

constant

= A

73

cos

(1)
(2)

find

/ 3A< sin 8

mgrZ sin 8

LAGRANGE'S EQUATIONS

302

11.34.

[CHAP.

11

Derive Euler's equations of motion for a rigid body by use of Lagrange's equations.
kinetic energy in terms of the Euler angles

The

T = (J lW ? + I 2 4 + 7 8 i)
- -J/^ sin sin + 6
- I Q> sin
dT/dxp
<?

Then

ar/af

Then by Problem

/1 w 1 w2

i 3 ($ cos

11.10,

\J-)

sin ^

cos

,/<

/2 (< sin

(9

cos

I 2 (< si n * cos

J2 (w 2 )( a^)

/>)

-?3

e sin

6 sin

^)( < sin

(^1

-^2)

ized force corresponding to a rotation ^ about an axis


A 3 of the torque about this axis [see Problem 11.102].

The remaining equations

/T

/<)

./>)

sin

>/<

cos ^)

is

- it2

J2 2

(h

- ^3)"3l = A 2

w 2w 3

(*)

,\

(I 3

The quantity *^ represents the generaland physically represents the component

to

*</;

7 lWl

w l w2

^i) w i"2

page 256.

Euler's third equation of (22),

%h(4> cos

9T

(h ~

^3"3

page 268]

2
6 sin ^)

page 288, Lagrange's equation corresponding

or
is

10.24,

w3

d_/dT\

This

e cos

sin e cos^p

</<)(0

\p

Problem

is [see

= A
.

2)

can be obtained from symmetry considerations by permutation of the indices. They are not directly
and < but can indirectly be deduced
obtained by using the Lagrange equations corresponding to
Problem
11.79].
[see
from them

11.35.

A bead slides without friction on a f rictionless


wire in the shape of a cycloid [Fig. 11-10]
with equations
x

where
function,
(a)

a(0

sin

9),

a(l

cos

9)

the Lagrangian
motion.
the equation of

(b)

Find

2tt.

Kinetic energy

= T =

Potential energy

(a)

+ y2

-|m(x 2

Fig. 11-10

= lma 2 {[(l -

cos e)d] 2

ma 2 (l

e)e 2

cos

= V - mgy

mga{l

[-sin

cos

e]

2
}

0)

Then
Lagrangian
...

d fdL\

= L = T

dL

-V
d

= ma2 (l [2ma2 (l

cos 0)0 2

cos 0)0]

mga(l

[ma 2

cos

sin 6 e 2

0)

+ mga

sin

e]

dt

[(1

which can be written

11.36. (a)

Show

written

(1

cos 9)e]

cos 6) e

- \

+ %

sin 6

sin 6 e 2

sin 6

g- sin

that the equation of motion obtained in part

(b)

of

a2u
dt 2

SL W

4a

where w = cos

(5/2)

Problem 11.35 can be

CHAP.

LAGRANGE'S EQUATIONS

11]

and thus

If

(a)

show that the bead

(b)

with period 2 7r\/ 4a/g

oscillates

cos (0/2), then

~r = -
Thus

303

-v-g-

+ j-u =

the

is

^=

sin (0/2)0,

-A

sin (0/2)0*

-A cos (0/2)02

same as

- i4~~v,-/=
cos (0/2)02 + &- cos (0/2)
4a

sin (0/2) V
\
2 --'-'-

which can be written as

Since
it

follows that equation

The

(b)

sin (0/2)

'

cot (0/2)

(i)

sin
1

cos

the same as that obtained in Problem 11.35(&).

(1) is

solution of the equation

is

|L

2 sin (0/2) cos (0/2)


2 sin2 (0/2)

= ^os (<?/2) =

cot (0/2)

cot (0/2)0*2

cos (0/2)

cos V4a/flr

Cj

c 2 sin y/ba/g t

from which we

see that cos (0/2) returns to its original value after a time 2iry/a/g which
is the required period.
Note that this period is the same as that of a simple pendulum with
length I 4a.

An

11.37.

application of this

is

the cycloidal pendulum.

See Problem 4.86, page 112.

Obtain equations for the rolling sphere of Problem 9.42, page 244 by use of Lagrange's
equations.
Refer to Fig. 9-33 in which
and ^ represent generalized coordinates. Since the sphere of
CP = a rolls without slipping on the sphere of radius OP = b, we have
<t>

radius

6 d<p/dt

which shows that

if

<p

when ^

</>

and $ [and therefore

The

and d> or

d<p

8<t>

kinetic energy of the rolling sphere

using the fact that I = %ma?through its center of mass.

The

is

af

then

0,

b<j>

Thus

b$

or

d\f//dt

a\p

and

(i)

3^] are not independent.

is

= im(a +

6)2^2

6)2^2

+ ^(|ma2 )( ; + ^2

m(a +

the

moment

_ij w 2

of inertia of the sphere about a horizontal axis

potential energy of the rolling sphere [taking the horizontal plane through

as reference

level] is

V = mg(a +
Thus the Lagrangian

We

T - V

cos

= {m(o + 6) 202 + ma2

<f>

(4>

+ J)2 - mg(a + b)

use Lagrange's equations (16), page 284, for non-holonomic systems.


b

so that if

we

b)

is

we

call

gx

and

q2

8<f>

a S^

f and compare with equation

find

A =
t

Thus equations

(16),

b,

A 2 = a

page 284, become


dL /dL\

_d

/BL\

dL

dL

_
_

cos

From

(2)

<f>

(1)

we have
(#\

(7)

of Problem 11.18, page 292 '


(4)

LAGRANGE'S EQUATIONS

304

Substitution of

(2)

and

intd (5)

(6)

m(a+b) $ + fma2 (0 + ) - mg(a +

fraa 2 (0

[from

(b/a)</>

m(a +
.

(1)]

b) 2 5?

into (7)

and

fma 2 (l +
(10)

and using

this in (9) it

we have

'vfr)

<f>

\xb

(7)

(8)

find

wfir(o

6/a)0

b) sin

-X x a

we

(8),

+ 6/a) -

raa2 (l

Now from

6) sin

<p

\xb

(9)

-XjO

(10)

= %m(a + b)$

Xi

becomes after simplifying and solving for

*<p,

5flf

~ 7(^+6) 8m *

This is the same equation as that of (2) in Problem 9.42, page 244, with
required angle at which the sphere falls off, see Problem 11.104.

<f>

w/2

- e. To

Solve the equations of motion obtained in Problem 11.24, page 296, and
pret physically.

11.38. (a)

(a)

From

at

0.

0.

Similarly,

since

From

at

In cot

or on integrating,

we

Thus
have

since
$i

From

n-/2

at

<

- ~g^J +

c2

In cot

tan

i.e.,

0,

e%

ten/E-M

all

time

we must

so that c4

(7

1.

-8^8i

similarly,

- z^jdt
3Jt
-^f

e2 \

(i"^)

we have

='

c3

W~ ZSmml

Then
or

,2

|-

2 tan-i( e

-3 ^/8w

and (2) show that the center moves along the y axis with constant speed J/4m.
rotate until
and BC are always parallel to the y axis while rods AB and CD slowly
axis.
on
the
be
rods
will
all
four
that
y
so
[*-"] the rhombus collapses,

Equations

The rods
finally

eW*ml

This means that for

0.

the fourth of equations (12) in Problem 11.24

or on integrating,

Now when

c2

ci

n72.

sec 9 2 de 2

(6)

on separating the variables,

find

we have

0,

(12)

we have

= is'

tan(|-^) =

i.e.

(0

from the second of equations

the third of equations (12)

(b) inter-

constant

find the

we have

the first of equations (12) in Problem 11.24


<c

since

11

yields

Substituting

[CHAP.

(1)

AD

CHAP.

LAGRANGE'S EQUATIONS

11]

305

Supplementary Problems
GENERALIZED COORDINATES AND TRANSFORMATION EQUATIONS
11.39.

Give a set of generalized coordinates needed to completely specify the motion of each of the following: (a) a bead constrained to move on a circular wire; (6) a particle constrained to move on a
sphere; (c) a compound pendulum [see page 228]; (d) an Atwood's machine [see Problem 3.22,
page 76]; (e) a circular disk rolling on a horizontal plane; (/) a cone rolling on a horizontal plane.

11.40.

Write transformation equations for the motion of a

triple

pendulum

in

terms

'of

a suitable set of

generalized coordinates.
11.41.

particle

is

x2

moves on the upper surface of a frictionless paraboloid of revolution whose equation


cz.
Write transformation equations for the motion of the particle in terms of a suit-

+ y2 =

able set of generalized coordinates.


11.42.

Write transformation equations for the motion of a particle constrained to move on a sphere.

CLASSIFICATION OF MECHANICAL SYSTEMS


11.43.

Classify each of the following according as they are (i) scleronomic or rheonomic,
or non-holonomic, and (iii) conservative or non-conservative:

(ii)

holonomic

(a)

a horizontal cylinder of radius a rolling inside a perfectly rough hollow horizontal cylinder of

(6)

a cylinder rolling [and possibly sliding]

(c)

a sphere rolling
plane;

(d)

a particle constrained to move along a line under the influence of a force which is inversely
proportional to the square of its distance from a fixed point and a damping force proportional
to the square of the instantaneous speed.

radius b

Ans.

>

a;

down an inclined plane of angle a;


down another sphere which is rolling with uniform speed along a

horizontal

scleronomic, holonomic, conservative


scleronomic, non-holonomic, conservative

(a)
(b)

rheonomic, non-holonomic, conservative


scleronomic, holonomic, non-conservative

(c)

(d)

WORK, KINETIC ENERGY AND GENERALIZED FORCES


11.44.

Prove that
the time

if

the transformation equations are given by


rv = r v (q v q 2
then the kinetic energy can be written as
n
n

.,

qn ),

i.e.

do not involve

explicitly,

T
where aa/3 are functions of the qa

2 2

aa a la 9b

11.45.

Discuss Problem 11.44 in case the transformation equations depend explicitly on the time

11.46.

If F(Xx, Xy, Xz)

of order n.
in

X n F(x, y, z)

Determine which

is

each case:

(a)
(6)
(c)

(d)

x 2 + 2 + z 2 + xy + yz +
3x-2y + 4z
xyz + 2xy + 2xz + 2yz
(x + y + z)/x
2/

Ans.

(a)

xz

homogeneous of order

geneous of order zero,


order 1.
11.47.

(if

If F{x,y,z) is

(e)

(e)

x s tan -1 (y/x)

(/)

4 sin

(g)

(x

homogeneous of order n

[see

a
+
dx
This

is

called Euler's

xy

+ y + z)/(x 2 + y 2 + z 2

2,
(c) non-homogeneous,
(6) homogeneous of order 1,
(d) homohomogeneous or order 3, (/) non-homogeneous, (g) homogeneous of

dF

Problem

dF

y-z
"

by

11.46],

prove that

dF

= nF
2tdz

theorem on homogeneous functions.

Differentiate both sides of the identity


then place X = 1.]
[Hint.

11.48.

t.

a parameter, then F is said to be a homogeneous function


any) of the following functions are homogeneous, giving the order

where

Generalize the result of Problem 11.47.

F(\x,\y,\z)

X nF(x,y,z) with respect to

X and

LAGRANGE'S EQUATIONS

306

11.49.

[CHAP.

Prove that if the transformation equations do not depend explicitly on time


energy, then

t,

and T

11

the kinetic

is

SJ

Can- you prove this directly without the use of Euler's theorem on homogeneous functions [Problem
11.47]?

LAGRANGE'S EQUATIONS
11.50.

Set up the Lagrangian for a one dimensional harmonic oscillator

(a)

Ans.

equations.
11.51.

(a)

and
11.52.

11.53.

(a)

L = mx 2 ^kx 2

(b)

and

(6)

write Lagrange's

mx + KX =

Set up the Lagrangian for a particle of mass


(b) write Lagrange's equations.

falling freely in a uniform gravitational field

Work Problem

11.51 in case the gravitational force field varies inversely as the square of the distance from a fixed point O assuming that the particle moves in a straight line through 0.

Use Lagrange's equations

down a

to describe the

motion of a particle of mass

to describe the

motion of a projectile launched with speed v

frictionless in-

clined plane of angle a.


11.54.

Use Lagrange's equations

at angle

a with the horizontal.


11.55.

Use Lagrange's equations

to

solve the

problem of the

(a)

two-dimensional

and

(6)

three-

dimensional harmonic oscillator.


11.56.

mass m is connected to a fixed point P on a horizontal plane by a string of length


The plane rotates with constant angular speed w about a vertical axis through a point O of the
plane, where OP = a.
(a) Set up the Lagrangian of the system.
(6) Write the equations of motion

particle of

I.

of the particle.
11.57.

(as, y, z) defining the position of a particle of mass tn moving in a force


are given in terms of spherical coordinates (r, 6, <f>) by the transformation

The rectangular coordinates


field

having potential

equations

Use Lagrange's equations


Ans.

m[ r
~

r$ 2

-tz (r 2 0)

m- .. (r2 e
,

r sin

r 2 e 2 sin

</>)

it

<f>

cos

e,

r sin

<

sin e ,

<f>

cos

r sin e

ay

r Be
d

1
.

<t>

move

Work Problem

11.56

if

11.59.

Work Problem

4.23,

page 102, by use of Lagrange's equations.

the particle does not necessarily

LAGRANGE'S EQUATIONS FOR NON-HOLONOMIC SYSTEMS


(a) Work Problem 11.20, page 293, if the paraboloid is
11.60.
(b) What modification must be made to Problem 11.21, page

11.62.

r cos

11.58.

11.61.

up the equations of motion.

to set

re 2 cos 2 0]

sin2

in

a straight line through O.

replaced by the cone


294, in this case?

Use the method of Lagrange's equations for non-holonomic systems to


particle of mass m sliding down a frictionless inclined plane of angle a.

Work Problem

3.74,

x2

+ y2 =

c2z 2

solve the problem of a

page 82 by using the method of Lagrange's equations for non-holonomic

systems.

LAGRANGE'S EQUATIONS WITH IMPULSIVE FORCES


11.63.

An impulse of
is at rest on a horizontal frictionless table.
uniform rod of length I and mass
(a) the
Prove that
magnitude $ is applied to one end A of the rod and perpendicular to it.
(c) the rod
and
is J/M
mass
of
center
the
of
velocity
(b)
the
velocity given to end A is 4J/M,
rotates about the center of mass with angular velocity of magnitude 6J/MI.

CHAP.

11.64.

LAGRANGE'S EQUATIONS

Ill

In Fig. 11-11, AB and BC represent two uniform rods


smoothly hinged
having the same length I and mass
at B and at rest on a horizontal frictionless plane.
An impulse is applied at C normal to BC in the direction indicated in Fig. 11-11 so that the initial
Find (a) the initial velvelocity of point C is v
ocities of points A and B and (6) the magnitudes of
the initial angular velocities of AB and BC about
their centers of mass.

307

Ans.
11.65.

/7,

Prove that the

2v

/7;

(6)

3v

/ll,

Fig. 11-11

9v /7l

energy developed by the system of Problem 11.64 after the impulse

total kinetic

\Mv%

is

11.66.

(a)

square of side a and mass M, formed from 4 uniform rods which are smoothly hinged at their
An impulse is applied at a vertex in a direction
edges, rests on a horizontal frictionless plane.
Prove
of the diagonal through the vertex so that the vertex is given a velocity of magnitude v
.

move about

that the rods


11.67.

If

(a)

their centers of

mass with angular speed 3v

/4a.

the magnitude of the impulse applied to the vertex in Problem 11.66, prove that the
by the rods is given by 5<5 2 /4ilf. (b) What is this kinetic energy in terms of
Explain.
Does the direction of the impulse make any difference?
is

kinetic energy developed


i>

11.68.

(c)

In Problem 11.24, page 296, suppose that the impulse is applied at the center of one of the rods in
Prove that the kinetic energy developed is <jj2 /8m.
is perpendicular to the rod.

a direction which

MISCELLANEOUS PROBLEMS
11.69.

11.70.

moves on the inside of a smooth hollow hemisphere of radius a having its


particle of mass
With what horizontal speed must it be projected so that it will
vertex on a horizontal plane.
remain in a horizontal circle at height h above the vertex?
is constrained to move inside a
particle of mass
thin hollow frictionless tube [see Fig. 11-12] which is
rotating with constant angular velocity a in a hor-

xy plane about a fixed vertical axis through


Using Lagrange's equations, describe the motion.

izontal

O.

11.71.

Work Problem

11.72.

particle of

11.70 if the

xy plane

in

mass

moves

is vertical.

a central force

field

the distance from


Using spherical coordinates, (a)
the force center.
set up the Lagrangian and (6) determine the equations of motion.
Can you deduce from these equations that the motion takes place in a plane [compare
Problem 5.1, page 121]?

having potential V(r) where r

11.73.

is

Fig. 11-12

moves on a frictionless horizontal wire of radius a, acted upon by a resisting force which
proportional to the instantaneous speed.
If the particle is given an initial speed v
find the
position of the particle at any time t.
particle

is

Ans.

(?nv Ac)(l

e~ Kt/ma) where

radius such that


11.74.

Work Problem
no,

11.75.

11.76.

Work Problem

at

e is
0,

11.73 if the resisting force

is

proportional to the square of the instantaneous speed.

spherical pendulum is fixed at point


equations for its motion.
9.29,

the angle which a radius drawn to


makes with a fixed
k is the constant of proportionality.

and

but

is

otherwise free to move in any direction.

page 239, by use of Lagrange's equations.

Write

LAGRANGE'S EQUATIONS

308

11.77.

11.78.

11.79.

11.80.

Work Problem 11.20


az = bx 2 + cy 2 where

the paraboloid of revolution


are positive constants.

if

is

[CHAP.

replaced

by the

elliptic

11

paraboloid

a, b, c

Prove that the generalized force corresponding to the angle of rotation about an axis physically
represents the component of the torque about this axis.
(a) Obtain Lagrange's equations corresponding to B and <p in Problem 11.34, page 302, and show
(6) Show how to obtain
that these are not the same as equations (2) and (3) of that problem.
equations (2) and (3) of Problem 11.34 from the Lagrange equations of (a).

m m

K K

Two circular disks, of radius of gyrations V 2 and masses u 2


respectively, are suspended vertically on a wire of negligible mass [see
They are set into motion by twisting one or both of the
Fig. 11-13].
Let

disks in their planes and then releasing.

made with some

m K\el)
2

is

v = iMf + T^-*!) 2
where

tx

be the angles

is

^(m.Kl'e 2

Prove that the potential energy

(b)

e2

specified direction.

Prove that the kinetic energy

(a)

and

V//////////////////A

and

t 2 are torsion constants,

i.e.

the torques required to

rotate the disks through one radian.

11.81.

Fig. 11-13

Set up Lagrange's equations for the motion.

(c)

Solve the vibrating system of Problem 11.80, finding

(a)

the normal frequencies

and

(6)

the normal

modes of vibration.

11.82.

Generalize the results of Problem 11.80 and 11.81 to 3 or more disks.

11.83.

(a)

Prove that

if

>

m2

and

lx

l2

in the double

frequencies for small oscillations are given

(m t

by

pendulum of Problem
where

+ m 2 )(h + h) V(h + w 2
2l 1 l 2

[!(!

- y 2 + m 2 (*i +

11.84.

Examine the

11.85.

Use Lagrange's equations

11.86.

special case

h -

2,

m,

in

normal

Problem

l2 )

Discuss the normal modes corresponding to the frequencies in

(b)

11.28, then the

w/2tt

(a).

11.83.

surface
to describe the motion of a sphere of radius a rolling on the inner
of a smooth hollow hemisphere of radius b > a.

particle on the inside surface of a frictionless paraboloid of revolution


Find the value of v
above the vertex is given a horizontal velocity v
.

oscillate

between the planes z

11.87.

Find the period of the

11.88.

oscillation in

and

Problem

H2

Ans. v Q

az

x2

+ y2

at a height

in order that the particle

= y2gH2

11.86.

angle a in a
sphere of radius a is given an initial velocity v up a frictionless inclined plane of
which is not along the line of greatest slope. Prove that its center describes a parabola.

direction

11.89.

wire of radius a
bead of mass m is constrained to move on a frictionless horizontal circular
through a point
which is rotating at constant angular speed a about a fixed vertical axis passing
pendulum.
simple
like
a
oscillates
the
bead
wire
the
to
Prove that relative
on the wire.

CHAP.

11.90.

LAGRANGE'S EQUATIONS

Ill

If a particle of

mass

and charge

B, the force acting on

field

it is

moves with velocity v

in

* and a vector

e(E

11.92.

11.93.

Suppose that the potential

B = VXA

dA/dt,

|m;2

e(A

v)

is

e*

page 278, by use of Lagrange's equations.

10.86,

has its ends constrained to move on the circumference of a


uniform rod of length I and mass
smooth vertical circular wire of radius a > 1/2 which rotates about a vertical diameter with conObtain equations for the motion of the rod.
stant angular speed u.

depends on q v as well as q v

T + V
is

Prove that the quantity

dV
dq v

a constant.

11.94.

Use Lagrange's equations


[see for example page 121.]

11.95.

Find the acceleration of the 5


Ans. 71^/622
Fig. 11-14.

11.96.

to set

up and

gm mass

two body problem as discussed

solve the

in the pulley

circular cylinder of radius a having radius of gyration K


with respect to its center, moves down an inclined plane of
angle a.
If the coefficient of friction is n, use Lagrange's
equations to prove that the cylinder will roll without slipping
ii

<

K2
a2

+ K2

tan

r~\
gm

Discuss the cases where n does not

a.

15

10

Use Lagrange's equations

to solve

Problem

8.27,

gm

page 213.

2gm

11.99.

Describe the motion of the rods of Problem 11.64 at any time


t after the impulse has been applied.
In Fig. 11-15, AB represents a frictionless horizontal plane
having a small opening at O.
A string of length I which
passes through O has at its ends a particle P of mass m and
a particle Q of equal mass which hangs freely. The particle
P is given an initial velocity of magnitude v at right angles
to string OP when the length OP = a.
Let r be the instantaneous distance OP while e is the angle between OP and
some fixed line through O.
(a)

gm
Fig. 11-14

A_

O IF

Q
Fig. 11-15

Set up the Lagrangian of the system.

(6)

Write a

(c)

Find the speed of

Ans.

(a)
(6)
(c)

11.100.

Chapter 5

system of

satisfy this inequality.

11.98.

in

if

11.97.

E and magnetic

the fields can be expressed by the relations

Prove that the Lagrangian defining the motion of such a particle

Work Problem

electric field

v X B)

potential

E = -V* -

11.91.

an

given by

F =
In terms of a scalar potential

309

differential equation for the

motion of

in

terms of

r.

at any position.

L = %m[2r2 + r2 e 2 + mg(l - r)
r = a i%/r2 g
r = y/2av? + 2g(a - r) - 2a2 i%/r

Work Problem

2/

11.99 if the masses of particles

and

are ra x and

m2

respectively.

3gm

LAGRANGE'S EQUATIONS

310

11.101.

11.102.

[CHAP.

11

Prove that if v = yfag the particle P of Problem 11.99 remains in stable equilibrium in the circle
r = a and that if it is slightly displaced from this e quilibrium position it oscillates about this
position with simple harmonic motion of period 2irv2ct/3flr.
Prove that the quantity fy in Problem 11.34, page 302, physically represents the component A 3
of the torque.

11.103.

Describe the motion of the system of


the impulse has been applied.

11.104.

Show how

11.105. (a)
(6)

11.106.

to find the angle at

(a)

Problem 11.63 and

which the sphere of Problem

(6)

Problem 11.66 at any time

11.37,

page 303,

after

falls off.

Set up the Lagrangian for the triple pendulum of Fig. 11-16.

Find the equations of motion.

Obtain the normal frequencies and normal modes for the triple pendulum of Problem 11.105 assuming
small oscillations.

11.107.

Work Problems

11.105 and 11.106 for the case where the masses and lengths are unequal.

m
Fig. 11-16

11.108.

Fig. 11-17

is placed on the spring and


If a mass
vertical spring [Fig. 11-17] has constant k and mass M.
with simple harmonic
set into motion, use Lagrange's equations to prove that the system will move

motion of period

2v^(M +

3w)/3<c.

Chapter 12

HAMILTON IAN
THEORY

HAMILTONIAN METHODS
In Chapter 11 we investigated a formulation of mechanics due to Lagrange. In this
we investigate a formulation due to Hamilton known collectively as Hamiltonian
methods or Hamiltonian theory. Although such theory can be used to solve specific problems in mechanics, it develops that it is more useful in supplying fundamental postulates
in such fields as quantum mechanics, statistical mechanics and celestial mechanics.

chapter

THE HAMILTONIAN
Just as the Lagrangian function, or briefly the Lagrangian, is fundamental to Chapter
Hamiltonian function, or briefly the Hamiltonian, is fundamental to this chapter.

11, so the

The Hamiltonian, symbolized by H,

is

defined in terms of the Lagrangian

as

2 Vaq

- L

(1)

a=l

and must be expressed as a function of the generalized coordinates q a and generalized


To accomplish this the generalized velocities q a must be eliminated from (1)
by using Lagrange's equations [see Problem 12.3, for example]. In such case the function

momenta p a

H can be written
H(pi,

or briefly H(p a q a
,

t),

and

is

.,p n

..

also called the

qi, ...,

qn

t)

(2)

Hamiltonian of the system.

HAMILTON'S EQUATIONS
In terms of the Hamiltonian, the equations of motion of the system can be written in
the symmetrical form

dH

qa

(3)

dp*
dH

These are called Hamilton's canonical equations, or briefly Hamilton's equations. The
equations serve to indicate that the p a and q a play similar roles in a general formulation
of mechanical principles.

THE HAMILTONIAN FOR CONSERVATIVE SYSTEMS


If a

(kinetic

can be interpreted as the total energy

= T+ V

(4)

system is conservative, the Hamiltonian


and potential) of the system, i.e.,

H
Often this provides an easy

way

for setting up the Hamiltonian of a system.

311

HAMILTONIAN THEORY

312

IGNORABLE OR CYCLIC COORDINATES


A coordinate q a which does not appear explicitly

*
pa

is

Lagrangian

is called

12

an ignorable

In such case

or cyclic coordinate.

so that

in the

[CHAP.

<

5>

a constant, often called a constant of the motion.

In such case

we

also

have dH/dq a

0.

PHASE SPACE
The Hamiltonian formulation provides an obvious symmetry between the p a and q a
which we call momentum and position coordinates respectively. It is often useful to imagine
a space of In dimensions in which a representative point is indicated by the In coordinates
(Pu

Such a space

is called

.,Pn,

<7i,

q n)

(6)

a 2n dimensional phase space or a pq phase space.

the state of a mechanical system at time t, i.e. we know all position


this corresponds to a particular point in phase space.
Conversely, a point in phase space specifies the state of the mechanical system. While the
mechanical system moves in the physical 3 dimensional space, the representative point

Whenever we know

and momentum coordinates, then

describes

some path

LIOUVILLE'S

in the

phase space in accordance with equations

(3).

THEOREM

Let us consider a very large collection of conservative mechanical systems having the
In such case the Hamiltonian is the total energy and is constant, i.e.,

same Hamiltonian.

H(pi, ...,Pn,qi,...,Qn)

which can be represented by a surface

in

constant

= E

{7)

phase space.

Let us suppose that the total energies of all


Then the
these systems lie between Ei and E 2
paths of all these systems in phase space will lie
E2
= Ei and
between the two surfaces
as indicated schematically in Fig. 12-1.
.

Since the systems have different initial condithey will move along different paths in the
phase space. Let us imagine that the initial points
are contained in region %i of Fig. 12-1 and that
For
after time t these points occupy region
2
example, the representative point corresponding to
one particular system moves from point A to
point B. From the choice of %i and
2 it is clear
that the number of representative points in them
are the same. What is not so obvious is the following theorem called Liouville's theorem.
tions,

Theorem
same, or
is

if

constant.

12.1: Liouville's

we

define the

Theorem.

number

Fig. 12-1

The 2n dimensional volumes

of points per unit

of

%i and ^2 are the

volume as the density then the density

CHAP.

HAMILTONIAN THEORY

12]

as particles of an incompressible fluid which

We can think of the points of %i


to

%2 in time

313

move from %i

t.

THE CALCULUS OF VARIATIONS


A problem which often arises in mathematics is that of finding a curve
the points where x

a and x =

Y(x) joining

such that the integral


>b

f F(x,y,y')dx
*J

(8)

where y' - dy/dx, is a maximum or minimum,


The curve itself is often called an extremal.
necessary condition for (8) to have an extremum

an extremum or extreme value.


[see Problem 12.6] that a

also called

can be shown

It
is

d_/dF\_ dF =
dx\dy'J

(g)
v

by

'

which is often called Euler's equation. This and similar problems are considered
branch of mathematics called the calculus of variations.

in a

HAMILTON'S PRINCIPLE
The obvious

similarity of

to Lagrange's equations leads one to consider the

(9)

problem

of determining the extremals of


2

Jt L{Qu

Qn, Ql,

.,

t)dt

On,

(10)

J-t

Ldt

where L = T

We

is

the Lagrangian of a system.

can show that a necessary condition for an extremal

dt\dq a J
which are precisely Lagrange's equations.
variational principle

t2

known

is

dq a

The

result led

Hamilton to formulate a general

as

Hamilton's Principle.
in such a way that

A conservative mechanical system moves from time U to time


C*Ldt

(12)

sometimes called the action integral, has an extreme value.


Because the extreme value of

(12) is often a

minimum, the

principle

is

sometimes referred

to as Hamilton's principle of least action.

The

fact that the integral (12) is

where

S is

the variation symbol.

an extremum

Vhdt =

is

often symbolized by stating that

(13)

HAMILTONIAN THEORY

314

[CHAP.

12

CANONICAL OR CONTACT TRANSFORMATIONS


The ease in solution of many problems in mechanics often hinges on the particular
generalized coordinates used.
Consequently it is desirable to examine transformations
from one set of position and momentum coordinates to another. For example if we call
q a and p a the old position and momentum coordinates while Q a and P a are the new position
and momentum coordinates, the transformation

Pa = Pa

(Pi,

.,Vn,

tf 1,

is

t),

Qa

Qa(Vu

<?a, t),

Qa

Qa(Pa, Qa,

<7n,

Pn, 01,

qn

t)

(14)

denoted briefly by

Pa = Pa

We

(Pa,

t)

(15)

restrict ourselves to transformations called canonical or contact transformations for

which there

exists a function Jl called the

P
we

In such case

Qa

--Wa>

Q a and P a

often refer to

Hamiltonian

in the

new

coordinates such that

{W)

~^Pa

as canonical coordinates.

The Lagrangians in the old and new coordinates are L(p a ,q a ,t) and oC(Pa ,Qa,t) respectively.
They are related to the Hamiltonians H(p a ,q a ,t) and Jl(Pa ,Q a ,t) by the
equations

2 Paka

where the summations extend from

~
=

L,

Ji

2 PaQa

~ *

(17)

1 to n.

CONDITION THAT A TRANSFORMATION BE CANONICAL


The following theorem

Theorem

12.2.

is

of interest.

The transformation
Qa = Qa(Pa,q a ,t)

Pa = Pa(Pa,qa,t),
is

canonical

is

an exact

^p

if

^PadQa

dq a

(18)
(I 9 )

differential.

GENERATING FUNCTIONS
By

Hamilton's principle the canonical transformation (14) or (15) must satisfy the con*

ditions that

f L dt and f
8

These

'

-C

C* Ldt

will be satisfied if there is

dt are both extrema,

and

sf'^f

simultaneously have

(20)

L -

JZ

()

a generating function.

position cothat Q is a function, which we shall denote by d", of the old


t, i.e.,
time
the
well
as
P
as
coordinates
a
momentum
and the new

By assuming
ordinates q a

We call Q

we must

a function Q such that

=
4?
dt
See Problem 12.11.

i.e.

CHAP.

HAMILTONIAN THEORY

12]

Q =

we can prove

t)

(22)

that [see Problem 12.13]

where

P.

Similar results hold

Problem

effaa Pa,

315

-M

the generating function

if

q. =
is

(w)

a function of other coordinates [see

12.12].

THE HAMILTON-JACOBI EQUATION


we can

= 0, then we see from (24) that


be ignorable coordinates]. Thus by means of
the transformation we are able to find p a and q a and thereby determine the motion of the
system. The procedure hinges on finding the right generating function. From the third
equation of (23) we see by putting Jl =
that this generating function must satisfy the
partial differential equation
If

find a canonical transformation leading to Jl

Pa and Q a will be constants

[i.e.,

Pa

and Q a

|^-

will

+ H(p a

qa

f + *(f.
This

is called

t)

-.)

(25)

<>

the Hamilton-Jacobi equation.

SOLUTION OF THE HAMILTON-JACOBI EQUATION


To accomplish our aims we need

to find a suitable solution of the Hamilton-Jacobi


since this equation contains a total of n + 1 independent variables, i.e.
t, one such solution called the complete solution,
, q n and
will involve n + 1 conOmitting an arbitrary additive constant and denoting the remaining n constants by
p n [none of which is additive] this solution can be written

Now

equation.

Qu

qz,

stants.

Pu (32,

of

When

this solution is obtained

offai,

tf2,

we can

...

qn,

pu

fa, ..., ,

then determine the old

(27)

*)

momentum

coordinates by

def

Also, if

we

identify the

new momentum

O
where

ya

1,

Using these
the system.

n are

we can

coordinates

Pa

M_

with the constants p a then


,

doJ*

dfia

constants.

then find q a as functions of p y and


a
a

CASE WHERE HAMILTONIAN

IS

t,

which gives the motion of

INDEPENDENT OF TIME

In obtaining the complete solution of the Hamilton-Jacobi equation,


assume a solution of the form

it is

often useful to

HAMILTONIAN THEORY

316

of

Si(qi)

-S 2 (g 2 )

[CHAP. 12

Sn{q n )

F(t)

(30)

where each function on the right depends on only one variable [see Problems 12.15 and
12.16]. This method, often called the method of separation of variables, is especially useful
when the Hamiltonian does not depend explicitly on time. We then find that F(t) = Et,
and if the time independent part of of is denoted by

5 =

Si(qi)

S2M +

+ Sn {q n

(31)

the Hamilton-Jacobi equation (26) reduces to

*()
where

is

(S2)

a constant representing the total energy of the system.

(32) can also be obtained directly by assuming a generating function


independent of time. In such case equations (23) and (24) are replaced by

The equation
which

is

Pa

= |S

Qa =

dq a

H = E

(33)

OJTa

Pa = ~tq~>

where

JH

j-,

Q<*

Tp~

(**)

PHASE INTEGRALS. ACTION AND ANGLE VARIABLES


Hamiltonian methods are useful in the investigation of mechanical systems which are
periodic. In such case the projections of the motion of the representative point in phase
space on any p a q a plane will be closed curves C. The line integral

Ja
is

Jc a
<f>

Padq a

35 )

called a phase integral or action variable.

We

can show

[see

Problems 12.17 and 12.18] that

S =
where

S(qi,

p.

...,qn,Ju ...,Jn)

(*6)

Jg,

customary to denote the new coordinates


pIacedby
as
It is

Thus Hamilton's equations become

[see

Q a by w a

in this case

equations (33) and

where / and

(m
(34)]

dJa

depends only on the constants J a

tion in (39),
}
V

so that equations (37) are re-

bs

dW a
where Jl =

|J

Then from the second equa-

nn
(*)
s

Wa = fat +
ca

are constants.

We

call

wa

angle variables. The frequencies / are given by

dJa

See Problems 12.19 and 12.20.

Ca

CHAP.

HAMILTONIAN THEORY

12]

317

Solved Problems

THE HAMILTONIAN AND HAMILTON'S EQUATIONS


12.1.

where the summation extends from


Vqa - L,
coordinates
of
the
function
q a and momenta p a , prove
a = 1 to n, is expressed as a
Hamilton's equations,
vrr
d jj
^
^

H = 2

Hamiltonian

If the

= 'Wee'

V"

regardless of whether
(a)

does not contain

Taking the

does not or

(a)

2Pa
=

2?#a

<*<7

and p a

dL/dq a

dH

H is

does contain the variable time

(b)

H - 2 Va Qa L

differential of

Then using the fact that pa

since

W*

^^dqa

dL/dq a

dp a

*2,q a

and

(3)

we

- 2p a dq a

contain

we have

(1), (2)

_
-

2?>a<4*

dH

2qa dpa -

(5)

and

(6),

is
(a)

(b)

Hamiltonian

BH

Pa

>

dq a

of part (a) are replaced

(3)

2g<*P

dqa

*2>pa

2f^<*<7

2 f|-^ +

by the equations

- 2^-<*<L - ~^dt

dt

ff

equation

(2)

Thus

is

U)

(5)

,*

_
~

dH_
dp a

Pa

'

_ _d#
~
dq

8H _ _3L
'

tt

H is independent of

explicitly,

dt

dt

prove that

it is

(a)

a constant and

equal to the total energy of the system.

From

By

dp

of Problem 12.1

-jfe

(6)

dH_
dp a

we have

If the

and

dH

12.2.

(3)

explicitly.

In this case equations

Then comparing

{2)

have, as required,
.

H does

(1)

2jr-dp
a + 1^-dqa
dp
dq

q <*

(b)

- 1jr-dia

this reduces to

(2)

explicitly.

we have

>

expressed as a function of p a and q a

dH
Comparing

explicitly.

dH =

But

*"

we have

2 ka Pa ~ 2 Pa Qa

a constant, say E.

Euler's theorem on homogeneous functions [see Problem 11.47, page 305],

dq a

HAMILTONIAN THEORY

318
where T

is

Then

the kinetic energy.

since

2pa

we have

does not depend on q a ],

pa

<Za

dL/dq a

dT/dq a

12

[assuming the potential

Thus as required,

2T.

H = 2,PaQa~L = 2T-(T-V) =
12.3.

[CHAP.

= E

T + V

xy plane under the influence of a central force depending only


distance from the origin, (a) Set up the Hamiltonian for the system. (6) Write

A particle moves in the

on its
Hamilton's equations of motion.
(a)

Assume that

the particle

the central force

Lagrangian

is

V(r).

is

located by its polar coordinates (r, e) and that the potential due to
2 2
2
Since the kinetic energy of the particle is T = -m(r + r * ), the

is

We

have

so that

Then the Hamiltonian

Note that

(6)

p a q

mr,

S +"

2m

2mr2

dL/de

(1)

mr2 6

(2)

= p e /wr2

(3)

P rr

PeO

{$m(r*

P!.
4* m2 +

Pe

+ r&) -

.^LV
mW

r2

V(r)>

V(r)'

(4)

'

energy expressed in terms of coordinates and momenta.

Hamilton's equations are

qa
;

Thus

dH/dp a

= dH/dp r =

= -dH/dr = ppmr* -

Note that the equations

pg

p r/m,

- L =

Pr

this is the total

pr

given by

is

H =

dL/dr

Pr

2
= im(r.,,-,,
+ rW) - V(r)

T - V

(5)

pa

= -dH/dq a

p rlm,

V(r),

pe

dH/dpe

p Q/mr*

(5)

= -dH/de =

are equivalent to the corresponding equations

(6)
(3).

PHASE SPACE AND LIOUVILLE'S THEOREM


12.4.

freedom.
Prove Liouville's theorem for the case of one degree of
can think of the mechanical system as
in terms of the motion of repdescribed
being
resentative points through an element of volume in phase space. In the case of a mechanical
system with one degree of freedom, we have a
two dimensional (p, q) phase space and the vol-

We

ume element

reduces to an area element

B(q, p

+ dp)

Let p = p(p, q, t) be the density of representative points, i.e. the number of representative points per unit area as obtained by
an appropriate limiting procedure. Since the
speed with which representative points enter
is q, the number of representative
through
per unit time is
points which enter through

..

Mq,v)

AB

AB

pq dp
of representative points

+ dq, P + dp)

dpdq

[Fig. 12-2].

The number

C(q

Fig. 12-2

(1)

which leave through


Pq

j-

(pq)

dq

CD
dp

is

D(q

+ dq,

p)

CHAP.

HAMILTONIAN THEORY

12]

Thus the number which remain

element

in the

minus

is (i)

319

or

(2),

^(pq)dpdq
Similarly the
respectively

points which enter through

number of representative

Thus the number which remain

in the

element

AD

BC

and leave through

are

-.

<pp +

and

pp dq

The increase

(3)

(pp)

dp

>

dq

is

(pp)dpdq

(-4)

and

in representative points is thus [adding (3)

(4)]

{*#+^V**
Since this

is

we must have

equal to -^dpdq,

dp
dt

dp

Hamilton's equations

dq

dt

Thus

But

since

dp/dp

this

we suppose that

dp
dp
dp

P-^-

-r-p

dp

(5)
v

so that

dm

dq
dq

dpdq'

op

= dH/dp

dm

this in (5), it

dq dp

dp/dt
in

follows

it

becomes

can be written

which shows that the density

12.5.

dp
-f-q
dq *

d(pfr )

the Hamiltonian has continuous second order derivatives,

Using

dq/dq.

dq

dH/dq, q

dp
dp

that

Kpq)

+ PF"+
dq

tt

Now by

phase space

(7)

constant and thus proves Liouville's theorem.

is

Prove Liouville's theorem in the general case.


In the general case the element of volume in phase space

dV =
manner

In exactly the same

dq y

dq n dp t

is

dp n

as in Problem 12.4 the increase of representative points in

to be

(-+
<(3(pi)
,

{ dq t

and since

this is equal to

d(pqn

a(pp t )

)
1-

dqn

d(pP n)\ _.

1-

3p!

>

dpn

dV

-dV, we must have


dt

dp
-

-f-

d( pqi )

\.

. .

_|_

dq x

dt

d(pq n

d(

)
-J-

dqn

dp

dt

a <Pg)

a =l

dqa

_^pPl )
dp t

_|_

d(ppn

dpn
d

-j.

^)
dpa

=i

This can be written as


dp

(dp

dp

fdqa ,dpa \

dV

is

found

HAMILTONIAN THEORY

320

Now

by Hamilton's equations

Hence dpa /dp a

= -dqa /dqa

= dH/dq a

pa

dpa

^K

~~

and

d2

qa

'

at

dPa

(- +
\^a

a=l

d2

d <I a d Pa

s^O
"Pa

()

<V<tt

i.e.,

so that

_
~

dqa

becomes

(1)

&+
or p

dH/dp a

~ d P<x d<la

[CHAP. 12

(#)

constant.

Note that we have used the fact that


dp

4j

/ dp dqa

dp

if

p(q u

dp a \

q n Pi,

dp

>

>

Pn>

then

*)

,dp_- \

fdp_.

dp

CALCULUS OF VARIATIONS AND HAMILTON'S PRINCIPLE


F(a,

mum

or

minimum]

dL (dF_\_dF
dy
dx \ by' )

is

Then

where

is

independent of

via)

The value of

an extremum [maxi

given by

is

CO

= Y +

V (x)

a neighboring curve through

x, is

to be

a^x^b

Y(x),

Y(x)

dx

Suppose that the curve which makes I an extremum

y, y')

<*)

eV

a and x

if

we

v(b)

7 for this neighboring curve is

Y + ev

F(x,

Y'

e V ')

U)

dx

jr

that -^
an extremum for e = 0. A necessary condition that this be so is
e=o
we find
differentiation under the integral sign, assuming this is valid,

This

choose

is

0.

de

J a \dy

=o

dy'

which can be written on integrating by parts as

where we have used

(3).

Since

dF
~dy

which

is

v is

'{-=(5)}*
arbitrary,

_d_/dF\ _
~
dx\dy'J

n
u

to the Euler's or

{L\-.W

nr

The

Lagrange's equations

dy

dx\dy'J

result is easily extended to the integral

i/i, If J, 2/2 1/2.

and leads

we must have

called tf wtor** or Lagrange's equation.

Xb F{X,

Vn n)

d*

But by

CHAP.

HAMILTONIAN THEORY

12]

By using a Taylor
/( e )

The

coefficient of

j(o)

we

find

that

(4)

higher order terms in

and

in (5) is often called the variation of the integral

f.
F(x, y,

dx

y')

is

F(x, y,

an extremum

is

3
e2 , e ,

etc.

(5)

denoted by

dx

y')

thus indicated by

is

12.7.

from

+ T)')dx +

f j- v

1,2, ...,n

dya

series expansion

dx\by'a J

321

F(x, y, y') dx

Discuss the relationship of Hamilton's principle with Problem 12.6.


By identifying the function F{x,y,y') with the Lagrangian L(t,q,q) where x,y and
placed by t, q, q respectively, we see that a necessary condition for the action integral

r
to be

an extremum [maximum or minimum]

is

y'

are re-

Ldt

(1)

given by

l(f)-f

<2)

seen that (2) describes the motion of a particle, it follows that such motion
requiring that (1) be an extremum, which is Hamilton's principle.
by
achieved
be
can also

Since

we have already

For systems involving n degrees of freedom we consider the integral

L =

L(t, q v q lt q 2 q 2
,

(1)

where

q n , qn )

which lead to the Lagrange equations

dt\dqa J

12.8.

dqa

1,2,

.,

from rest at one point on a


wire in a vertical plane to another
point under the influence of gravity. Find the
total time taken.
particle slides

f rictionless

Let the shape of the wire be indicated by curve


and suppose that the starting and finishing points are taken to be the origin and the point

in Fig. 12-3

A{xQ,yo) respectively.

Let P(x, y) denote any position of the particle


which we assume has mass m. From the principle

A(

of conservation of energy, if we choose the horizontal


line through A as reference level, we have

Potential energy at

O +

kinetic energy at

mgy

or

where ds/dt

is

mg(y

If

we measure

is positive,

The

Fig. 12-3

potential energy

y) +

at P +

kinetic energy at

t.

Then

= Vzgy

the arc length s from the origin, then s increases as the particle moves.

so that

ds/dt

y/2gy

total time taken to go

or

from y

dt

ds/y/2gy.
to

is

%m(ds/dt) 2

the instantaneous speed of the particle at time


ds/dt

Vo)

(-0

Thus ds/dt

HAMILTONIAN THEORY

322

dt

^0

But

(cte) 2

(do;) 2

+ (dy) 2

or

= ^1 +

cte

y>

^=0

Vi +

rv

If the particle of

show

sible time,

the wire

A
is

is

y'

dx

(2)

vv

Problem 12.8 is to travel from point O to point A in the least posthat the differential equation of the curve C denning the shape of

+ y' 2 + lyy" =

0.

necessary condition for the time r given by equation

that

(d_F_\_8F =
dx\dy'J

F =

where

Now

is

j=-

V2gJ y=0
12.9.

'2gy

Thus the required time

dx.

[CHAP. 12

BFlby'

(l

+ y'

)-

(1

y'y-^ 2

1/2

1 '2
)

K1)

V V2

(2)
2

1(1 + y' )^ y

dF/dy

minimum

dy

+ y'

of Problem 12.8 to be a

(2)

-s/2

Substituting these in (1), performing the indicated differentiation with respect to x and simplifying,
obtain the required differential equation.

we

The problem of finding the shape of the wire

often called the brachistochrone problem.

is

Solve the differential equation in Problem 12.9 and thus


curve is a cycloid.

12.10. (a)

(a)

Since x

missing in the differential equation,

is

Then the

u*

dw dy_
dy dx

=u
du
dy

show that the required

so that

du
dy

becomes

differential equation

du
dx

y'

let

(b)

2yu

^=0
dy

-^
+^ =
+

or

uz

Integration yields
In (1

where

6 is a constant.

+ u2 +
)

Letting y

must be

b sin 2 e,

26

26 sin

sin 2 6 de

Thus the parametric equations

Since the curve

J
\

+ u2 )y =

this can be written

f J b sin2 9
J V 6 cos2 e

= * =

(1

Separating the variables and integrating, we find

positive.

or

In 6

Thus

since the slope

In

6(2*

ecosede +

(1

|6(1

cos 2e) de

6(2*

sin 2e)

of the required curve are

sin 2d)

c,

must pass through the point x

2(9,

sin

<f>),

b sin 2 e

= 0,

= 0, we

have

= |6

cos 2e)
c

0.

Then

letting

CO

the required parametric equations are

a(<p

a(l

cos

<p)

(2)

CHAP.

(b)

323

HAMILTONIAN THEORY

121

cycloid [see Fig. 12-4]. The constant a must


(2) are parametric equations of a
A. The cycloid is the path taken by a
point
through
passes
curve
the
that
so
determined
be
Problem 12.89].
[see
line
given
a
fixed point on a circle as it rolls along

The equations

7<

/ \

CANONICAL TRANSFORMATIONS AND GENERATING FUNCTIONS


12.11.

Prove that a transformation


dq/dt = L-Ji.
The

integrals

tions are zero,

Ldt and

canonical

is

must simultaneously be extrema

dt

there exists a function

if

that

q such

so that their varia-

i.e.,

Ldt -

and

SJ

-C

dt

t.

Thus by subtraction,
This can be accomplished

if

-C

-r-dt

12.12.

) dt

there exists a function

L-

The function Q

(L

Q such that
d/ctt

S{( 2 )

(*i)}

called a generating function.

is

Suppose that the generating function is a function


ordinates q a and Q a respectively as well as the time
Va

Pa =

||-,

By Problem

-*[1,
dQ a
'

Qa

t),

(1)

and

(2),

we have
Pa

The equations

dQa

'

dPa

Si

H
(Si~H)dt

(1)

then

2^dQ
a
2*Ia+
3Q
39o

dT
Comparing

H - V2Pa Q a -

2pa dqa - 2Pa dQ a +

dT
T{q a

2, Pa q a

Spg ~ 2PQ a + Si -

T=

and new position coQ a t). Prove that

T(q a

P =

where

tf

T=

dt

L-j =

dt

if

^+

of the old

t, i.e.

12.11,

dT =

But

+ dt
dt

as required

dT
dqa

P
*

3t

'

*c#

= - aQ

_ djj
Qa ~ 8P
-

'

follow from the fact that Si is the Hamiltonian in the coordinates


tions hold as in Problem 12.1.

Pa Q a
,

so that Hamilton's equa-

HAMILTONIAN THEORY

324

12.13.

Let

efbea generating
Va

~W'

Qa

From Problem

function dependent only on q a

lt

^T + ^PaQa)

Or

since gf is a function of qa

Pa

The

(2)

and

(4),

W~a

(1)

(2)

^Pa Qa

T +

(3)

Pa =

fl

Prove that the transformation

Q a = 75-

-777-,

the Hamiltonian.

is

P=

i(p

tan -1 (q/p)

+ q 2 ), Q =

canonical.

is

1.

and P, Q be respectively H(p, q) and J#(P, Q)


Since p, q are canonical coordinates,

Let the Hamiltonians


that H(p, q) Jfl(P, Q).

dH

= ~rdq

^ 9J(dP

dq

dP dq

djdQ

_
~ V

dP
dq

'

__

dH

(*)

*>
=

dp

Solving simultaneously,

we

dp

__

dP

"

+ <

?MdP

3JtdQ

dP dp

dQ dp

'

-q

__

p2

dp

dQ _
q

'

()

p
p2

dq

Also, differentiating the transformation equations with respect to

we have
dQ

/^x

- 7
dp

dQ dq'

the given transformation equations

dp

3jF/

i^H4,

dH

5P

so

in the coordinates p, q

From

Qa =

2^^+ SJ^p. + J^

results

Method

-Wa'

t,

follow as in Problem 12.12, since J#

12.14.

Pa =

2Pa^a + 2 Q <*P + (Jl~H)dt

gf

Comparing

where

Prove that

^,Pa dQ a + (J(-H)dt

where

But

t.

= 2pdga + 2QdP a + (J(-H)dt

def

i.e.,

Pa

12

we have

^pa dqa -

+H

J(

eK'

12.12, equation (1),

dT

[CHAP.

and Q respectively, we

(>&-<&)A +

find

*>

find
dq.

P
p2

'

dP

~ p2 +
9.

'

*L - - aq
dQ

'

M. = Vv
dQ

U)

CHAP.

HAMILTONIAN THEORY

12]

Then equations

and

(1)

become

(2)

sh _

dq

Thus from equations

djj
dP
and

(1), (5)

p2

Solving these simultaneously

we

Method

dQ

'

p
*

ajl
dQ

dj(

p2

q2

dQ

find

_ 3_M
* ~ dP
.

'

(7)

are canonical and that the transformation

is

therefore canonical.

2.

By Theorem

page 314, the transformation

12.2,

1pa dq a is

Bjj

p2

dP

p2

9Jl

A
+r **
pQ =
~

JL

2*P

sji
8P

and

dp

_ej(
F _
"
dQ

BJj

en
'

qQ

2*
q

dQ

we have

(6)

djj

P +
2

which show that

325

an exact

differential.

In this case

(8)

pdq - PdQ

is

canonical

J,P a dQ a

(8)

becomes

= pdq -

(p 2

+ q2

Pdq-qdp
)

=
an exact

%(P dq

Thus the transformation

differential.

if

q dp)

d{pq)

canonical.

is

THE HAMILTON-JACOBI EQUATION


Write the Hamiltonian for the one dimensional harmonic oscillator of mass m.
(b) Write the corresponding Hamilton-Jacobi equation, (c) Use the Hamilton-Jacobi
method to obtain the motion of the oscillator.

12.15. (a)

(a)

Method

1.

Let q be the position coordinate of the harmonic oscillator, so that q is its velocity. Since
the kinetic energy is T = ^mq2 and the potential energy is V = ^*cg2 the Lagrangian is
,

L = T - V = \mq - \
2

The momentum

is

so that

Then the Hamiltonian

(1)

= mq

(2)

= p/m

(3)

is

*2,pa q a

- L =

= p2/m +
Method

dL/dq

K q2

pq

- (%mq 2 - % K q 2

K q2

U)

2.

By Problem

12.2, since the

Hamiltonian

is

the

same as the

total

energy for conservative

systems,

= mq2 +

\nq 2

\m{plm) 2 +

<7

= \p 2lm +

ieq 2

HAMILTONIAN THEORY

326

(b)

(c)

Using p

tion (26),

page 315]

Assume a

and the Hamiltonian of part

dof/dq

2^W7

Setting each side equal to the constant

whose

SM + S

/dS x \ 2

becomes

(5)

the Hamilton-Jacobi equation

is

[see

equa-

form

solution to (5) of the

ef

Then

(a),

[CHAP. 12

/?,

we

(6)

2 (t)

dS 2
~~dt

(7)

find

solutions, omitting constants of integration, are

Si

so that (6)

becomes

Let us identify

/?

f V2m( 8 - l/cg 2 )

with the new

V2m(/3

momentum

s 2 = -pt

- ^/cg 2

dq

coordinate P.

(8)

pt

(9)

Then we have for the new

position

coordinate,
B<

_
~

__ \

dp

dp

y/2m(p

- ^/cg 2

dq

r
V2m C
J
2
But since the new coordinate

is

a constant

ViJm

/"

VW*

Then solving for


which

is

g,

the required solution.

0*

-v/rt

y,

dq

s
or on integrating,

y/p - %Kq*

dq

2
y/p - &Q

sin" 1 (qy/ic/2p

y/2plic sin

yJTJm

(t

+ y)

(-*

The constants p and y can be found from the

initial conditions.

the
quantity p is physically equal to the total energy E of
316.
page
on
(SI)
equation
=
illustrates
E
The result (9) with p

It is of interest to note that the

system

12.16.

[see

Problem

12.92(a)].

Use Hamilton-Jacobi methods


square central force
The Hamiltonian
Then

since

pr

problem for a particle in an inverse

field.

H =

is

d^/dr, p e

It
Let

to solve Kepler's

p?

~^

~
J

~r

d^/de, the Hamilton-Jacobi equation

+ 2m

*
\V Br J

J =

is

r2 V Be J J

S 1 (r) + S 2 (e) + S 3 (t)

CHAP.

HAMILTONIAN THEORY

12]

Then

f/dSA
^ ^r

becomes

(2)

+
,

Setting both sides equal to the constant

dS

A\

^{^fj

we

/? 3 ,

(4) yields,

(5)

by 2mr2 and write

it

r2

-/? 3 *

form

in the

2m ^ + -r-{-dir

dS

S
Thus

Qr

and

and

/? 3 ,

we

and

<? fl

/? 3

gT

V2m/? 3

\/2m/?3

Qe

Wz

=
W~2

2mtf/r

Pr

with the new momenta

Qr

0/r*

2raK/r

and

(7)

af~ J*

Pe

dr

(8)

/3 2 e

(3 3 t

/r2 dr

% /r2 dr

/?

On performing

(9)

we have

respectively,

^ 2m/?3 + 2mX/r ~

are constants, say y 1 and y 2

/3^/r 2

0*/r2 dr

S ^ 2m^3 + 2mK/r -

Yi

72

the differentiations with respect to

find

Pidr
2 \/2m^3

7"'

V2m/? 3

The integral

(0)

Then

on taking the positive square root.

/? 2

follows that each

\ dr

= V2m/3 3 + 2mK/r -

-j-

ui

S2 = P 2

or

p2

Identifying

r, it

Thus
dS 2 /de

/3 2

(4)

since one side depends only on e while the other side depends only on

side is a constant.

since

-/? 3

apart from a constant of integration,

-dv)
Then

"^

S3 =
Multiply both sides of

dS s

_
~

^(f )} - f

^{(^)
Integration of

K
" ~

find

dS z ldt
2

327

in (10)

+ 2mX/r -

+ 2mK/r -

7i

(JO)

72

(**)

jSf/r

2
/3

/r2

can be evaluated by using the substitution r

II u,

and after integrating we

find as the equation of the orbit,

- Vl + ZPzpymK* cos

(*

+ B-/2 - y

()
x)

The constant /? 3 can be identified with the energy E [see Problem 12.92(b)], thus illustrating equation
it is a parabola; and if
If E = /3 3 < 0, the orbit is an ellipse; if E =
{31), page 316.
3 = 0,

/3 3

>

0,

it is

The equation

a hyperbola. This agrees with the results of Chapter


(11)

when

5.

integrated yields the position as a function of time.

HAMILTONIAN THEORY

328

[CHAP.

12

PHASE INTEGRALS AND ANGLE VARIABLES


12.17.

Let gT be a complete solution of the Hamilton-Jacobi equation containing the n constants


(3i,

We

Ja =

Let

/?.

have

where the constant p 1

p a dq a
1

E,

(q l ,

lt

Using

.,

pn

(4)

we can

(3 n )

pxt

(1)

3<2f

dSa

dqa

dqa

~dq

{S )

remaining are the con-

solve for

pu

>

Pn)

..,p n in terms of J u

l,...,n

(4)

.,Jn and express

..

(1) in

terms of the Ja

Suppose that the new position and momentum coordinates are taken to be
Prove that if Ji is the new Hamiltonian,

wa

and

respectively.

Deduce from

where f a and

(a)

Wa =

ca

By Hamilton's

fa =

are constants and

dJl/dJa

ca

Q a ,P,

Qa = dJ{/dP a

-dJ(/dQa,

momentum

fat

equations for the canonical coordinates

Ja
^[

wa

and

constant

Then since the new position and


these equations become

Since

dJl/dJa

that

Pa

(b)

only.

Thus we have the n equations

Ja

(a)

fi lt

/? a

Now

Ja = -dM/dWa,
(b)

in this integration q a is integrated out, so that the only quantities

Ja = Ja (fiv

Ja

+ S n (q n

Padq

Ja

stants Pi,

12.18. (a)

n)

.,

the total energy.

Thus
But

Prove that the Ja are functions of the

coordinates are taken as

Wa =

-dJl/dWa,

(1)

Q a w a and P a = Ja

dJl/dJa

E, the new Hamiltonian depends only on the Ja and not on the

>

(2)

wa

Thus from

(2)

we have

Ja

where f a

djK/dJa

From

(3)

we

Ja

The quantities J a are

wa =

0,

find,

constant

= fa

as required,

wa

constant,

fa t

ca

Padq a

Ja

(5)

where the integration is performed over a complete cycle of the coordinate q a


phase integrals. The quantities w a are called angle variables.

Let

Aw a

denote the change in


Prove that

lar coordinate q r

(-4)

called action variables while the corresponding integrals

12.19. (a)

(3)

wa

are called

corresponding to a complete cycle in the particu-

Aw a

if

if

a ^r

CHAP.

HAMILTONIAN THEORY

12]

Give a physical interpretation to the result in

(b)

329

(a).

Aw a

(a)

M-a

'-

dS

dJr

yw

dqr

5Ja

fl

if

if

#r

where we have used the fact that w a = aS/d7a [see Problems 12.17 and
sumed that the order of differentiation and integration is immaterial.

as-

(a) it follows that w


a changes by one when qa goes through a complete cycle but that
there is no change when any other q goes through a complete cycle. It follows that
qa is a
periodic function of w a of period one. Physically this means that the f
a in equation (4) of
Problem 12.18 are frequencies.

Determine the frequency of the harmonic

comple te cycle of the

from q

-y/2(3/ K

to

pdq

c oordin ate q [see

+V2/3//C

equation

and back

J -V2/3/K

oscillator of

to

Problem

12.15.

12.15] consists in the

-y/2ph. Then the action variable

- \ K q*)

y/2m(/3

(10),

Problem

dq

y/2m(p

- % K q*)

motion

is

dq

27T/?VW/C

Thus

12.21.

and have

From

(6)

12.20.

12.18]

'

= * =

>

= .*

Determine the frequency of the Kepler problem

' =

[see

Problem

i{i
12.16].

^ A

complete cycle of the coordinate r consists in the motion from r = r


min to rmax and back to
r ~ rmin> where r min and r max are the minimum and maximum values of r
given by the zeros
of the quadratic equation [see equation (10), Problem 12.16]
2m/? 3

We

then have from equations

J6

Jr

Ve de

(2)

and

(3)

we have on

^-ds

p 3 = E,

(4)

^de

rds<

J-j-dr =

f* fi%d,

u Pt

r rmax
Jrmin V2m/? 3 + 2mK/r

(f)

&/r* dr

2tt/3 2

elimination of p 2

Je
Since

(1)

of Problem 12.16,

= y-^rdr =

Pr dr

= 2wmK/y/-2m/3 3 From

(7)

r d<of

r
J>

and

(6)

+ 2mK/r - ppr2 =

+ Jr =

2TrimKly] 2m/3 3

yields

2^2

mg2

-othat

^-"OVfT^

jr

m
-__
2^2

/f2

Then the frequencies are


J6

dJ9

(./,

Since these two frequencies are the same,


is degenerate.

system

fr

)3'

i.e.

there

dJr
is

(Je

+7r)3

only one frequency,

we say that

the

HAMILTONIAN THEORY

330

MISCELLANEOUS PROBLEMS
12.22. A particle of mass m moves in
and

(b)

The

(a)

a force

of potential V. Write

field

Hamilton's equations in spherical coordinates

(r, 9,

the Hamiltonian

(a)

</>).

kinetic energy in spherical coordinates is

T = \m(r2 +
Then the Lagrangian

r2 e 2

r 2 sin 2 9

r2 e 2

r2 s i n 2

(1)

is

-V

L = T

We

[CHAP. 12

= \m(r2 +

tf

2)

y( r

^^

have

pr

= mf

dL/dr

and

The Hamiltonian

= mr2 8,

dL/do

Pr

pe

mr
Pe

mr

-,

p^,

mr2

3L/30

sin 2

(5)

P<*>

(4)
v
'

sin 2

given by

is

H =

^tP a q a

p rr

n
Pr_

p ee

^m(r2

2
P<&

Pe

2m

pqQ

2mr2

r 2 sin2 e

V(r,

e,

0)

v &>'>*)
+ v
,

2mr 2

r2 e 2

sin 2

where we have used the results of equations

5)

<?

(4).

We can also obtain (5) directly by using the fact that for conservative systems the
= T + V.
Hamiltonian is the total energy, i.e.

Hamilton's equations are

(b)

dH_

qa

Pr

dp r

dp a

_Po_
Pe

dp e

mr2

mass

particle of

is

V (K cos 0)/r
By Problem

m
2
.

pr

AqJ"

pe

1
M-l.
+ 2m
3*

12.24. (a)

sin 2

p*
_ Z
= -EL3 +
dr
mr
mr3 sin 2 e

dV
do

-^R - __
=
~~
30

30

in a force field whose potential in spherical coordinates


Write the Hamilton- Jacobi equation describing its motion.

12.22 the Hamiltonian

-r

P4

mr2

dp$

moves

1
N n
~ 2m

Writing

(a),

dH _
_ dH_

.
'

_ _dH_ _ P% cos
~
~ mr2 sin 3 e
do
'

Then from part

dH_ _
_ dH

P*

12.23.

dqa

_^
dr

= t

pa

3<of

-r

p^

is

( a*
r
V

9(of

Kc0Sff

r2 sin 2 0/

r2

(1)'
K

r2

the required Hamilton- Jacobi equation is

iYMY. r^^Vi
U
/
V 3* /
3r-

P*
I

1
r-

2 sin 2

/^Y)
\ 30

gcosg

Find a complete solution of the Hamilton- Jacobi equation of Problem 12.23 and
how the motion of the particle can be determined.

(b)

indicate

(a)

Letting

= S

(r)

+ S 2 (&) + 3(0) ~ Et

in

equation

(2)

of Problem 12.23,

it

can be written

CHAP.

HAMILTONIAN THEORY

12]

fds t y

2m\dr
Multiplying equation

/ds2
i
2mr2 \ do J

(1)

331

/ds3

Kcos9

2rar2 sin2 e\d<f>)

r2

by 2mr2 and rearranging terms,

/dS2 \ 2

/rfSiV

/dSo\2

Since the left side depends only on r while the right side depends on e and
each side must be a constant which we shall call /?j. Thus

2mEr2 =
\d7) ~

/dS2 \ 2

and

sin 2 *

by

(5)

/dS 3 y
\~d&)

/dSo\2

-a^^j

-(^j

Multiplying equation

(1)

<f>,

it

follows that

Pl

(2)

+fcOr. =

fc

and rearranging terms,

= 2mKain20cos9 -

/^sin 2

/dSa \*

sin 2 *

(4)

Since the left side depends only on <p while the right side depends only on e each side must be a
constant which we can call /3 2 However, since
.

^
we can

write

ordinate.

/3 2

Then

(4)

p^. This
becomes

2mK
By

solving equations

- J y/2mE + pjr2

<

a consequence of the fact that

is

sin 2 e cos 6

(2), (6)

= 1? = ~dj

and

(5),

px

we

obtain

S2 =

dr,

sin 2 e

yJImK

sin 2 e (

- p2

cos

a cyclic or ignorable co-

is

p|

esc 2 *

(6)

/?i ete,

S3 =

p^

where we have chosen the positive square roots and omitted arbitrary additive constants.
complete solution
of

(6)

of motion are found

30J

2m K'
J

t,

cos *

2
p$ esc e

Yl

'

~dE

l2

dp^

'

r, 0,0

y (**L!*L
\dp*dq
a

Prove that
dF/dpr,

(d)

\F,G\

(a) [F,G] = -[G,F],


[F,p r ] = -dF/dq r

/3 1

de

+ p^ - #<

Y3

as functions of time using initial condi-

the functions F and G depend on the position coordinates q a


the Poisson bracket of F and G is denned as

\f G]

(a)

by writing

and then solving these to obtain the coordinates


tions to evaluate the arbitrary constants.

12.25. If

(6)

[Fi

- dF dG

momenta p a and time

dq a dpaj

+ F2 ,G] =

[F U G]

[F*,G],

(c)

[F,q r]

The

is

= J V2mE~+JJr^dr +
J

The required equations

5>

^(**L*G-*LdG\
a

\3p 3ga

9ga ap^/

_^(dG_dL_dGdF_\ _ _ {rp]
a

\dpa 3ga

dq a dpa J

This shows that the Poisson bracket does not obey the commutative law of algebra.

HAMILTONIAN THEORY

332

]F+F
+F
[F 1

(h)
(b)

y
i|P>(*

_
-

G]
2 ,G]

+ * 2) 8G

dpa

[^i,G]

+ FJ dG \
dp

dga

dga

d
_ M\
V ( ll*<L

a{F l

[CHAP. 12

- /^2 ^G _

8G_\

Mj, _3G

[F 2 ,G]

This shows that the Poisson bracket obeys the distributive law of algebra.

{C)

[,<lrl

^ f dF
%\d Pa

since dq r/dqa = 1 for a = r and


bitrary, the required result follows.

(d)

for

,Pr]

dF dq r

dqa

dq a dpa J

a = r,

a\dp a

while

for all a, while dp r/dp a


dp r/dqa =
bitrary, the required result follows.

qF_
d Pr

dq r/dp a

for

for all

a.

Since r

is

ar-

Since r

is

ar-

dq r

dq a dp a J

dqa

since

12.26. If

dq r

and

for

a >

r.

H is the Hamiltonian, prove that if / is any function depending on position, momenta

and time, then

-,,

or

dt

But by Hamilton's equations,

at

qa

+
Pa
dpa
\dqa q

dH

T~r
dp

Pa

>

Then

(2)

dH
_
~TZ~
dq

(*\
K

'

can be written

L
dt

U.
dt

2
H

f-^-

^L--&- ) = &-+
dt
dqj

[H,

dp a

\dq a dp a

f]

Supplementary Problems
THE HAMILTONIAN AND HAMILTON'S EQUATIONS
12.27.

Ans.

(a)

Write the Hamiltonian and

(6)

Ham-

(b)

12.28.

moves in a force field of potential V.


mass
equations in rectangular coordinates (x,y,z).
= (p| + p\ + p|)/2m + V(x, y, z)
(a)

particle of

ilton's

= pjm,

Use Hamilton's equations


plane of angle

p y /m,

p z /m, p x

to obtain the

-dV/dx, p y

-dV/dy, p z

motion of a particle of mass

= -dV/dz

down a

frictionless inclined

a.

Hamilton's equations.
the problem of small oscillations of a simple pendulum by using

12.29.

Work

12.30.

Use Hamilton's equations


with the horizontal.

to obtain the

motion of a projectile launched with speed v

at angle a

CHAP.

HAMILTONIAN THEORY

12]

333

12.31.

Using Hamilton's equations, work the problem of the harmonic


(b) two dimensions, (c) three dimensions.

12.32.

Work Problem

3.27,

oscillator in

(a)

one dimension,

page 78 by uging Hamilton's equation.

PHASE SPACE AND LIOUVILLE'S THEOREM


12.33.

Explain

why

the path of a phase point in phase space which represents the motion of a system of

particles can never cross itself.

12.34.

Carry out the

details in the proof of Liouville's

theorem for the case of two degrees of freedom.

CALCULUS OF VARIATIONS AND HAMILTON'S PRINCIPLE


12.35.

Use the methods of the calculus of variations


plane which has the shortest length.

12.36.

Prove that
is

if

the function

an extremum

if

in the integral

F y'Fy
>

12.37.

Use the

12.38.

It is desired to revolve the

where

Ja

c is

result of Problem 12.36 to solve

(a)

to find that

F(x,

y, y')

curve connecting two fixed points in a

dx

is

independent of

x,

then the integral

a constant.

Problem

12.9,

page 322,

(6)

Problem

12.35.

curve of Fig. 12-5 having endpoints fixed at P(x v y x ) and Q(x 2 y 2 ) about
the x axis so that the area / of the surface of
revolution is a minimum.

Q(x2 y2 )
,

/**

+ y' % dx.

Obtain the differential equation of the curve.


Prove that the required curve is a catenary.

(b)
(c)

Ans.
12.39.

yy/l

(6)

yy"

= 1 + (y')2

Two

identical circular wires in contact are placed


a soap solution and then separated so as to
form a soap film.
Explain why the shape of the
soap film surface is related to the result of Problem 12.38.
in

12.40.

Use Hamilton's principle

12.41.

Work

12.42.

Use Hamilton's
angle

to find the

the problem of a projectile

Fig. 12-5

motion of a simple pendulum.

by using Hamilton's

principle.

principle to find the motion of a solid cylinder rolling

down an

inclined plane of

a.

CANONICAL TRANSFORMATIONS AND GENERATING FUNCTIONS


12.43.
Prove that the transformation Q = p, P = q is canonical.
Q =

P =

12.44.

Prove that the transformation

12.45.

(a)

Prove that the Hamiltonian for a harmonic oscillator can be written in the form

(b)

Prove that the transformation

q tan p,

In sin

is

canonical.

H = 4p /ra +

= \P/y/^sinQ, p - -\j mPyf^ cos Q is canonical.


(a) in terms of P and Q and show that Q is cyclic.

(c)

Express the Hamiltonian of part

(d)

Obtain the solution of the harmonic oscillator by using the above results.

HAMILTONIAN THEORY

334

12.46.

Prove that the generating function giving


is

[CHAP.

rise to the canonical

transformation in Problem 12.45(6)

^yficq cotQ.

12.47.

Prove that the result of two or more successive canonical transformations

12.48.

Let

12.49.

Let

L>

be a generating function dependent only on

Prove that the generating function

Problem 12.12 by
12.51.

Q a ,p a ,t. Prove

be a generating function dependent only on the old and


t.
Prove that

and the time

12.50.

12

V = T -

of Problem 12.48

is

also canonical.

that

new momenta p a and P a

respectively

w
T

of

related to the generating function

related to the generating function

is

2p<7a-

Prove that the generating function "V of Problem 12.49


Problem 12.12 by "V = T +
a qa
a Qa -

^P

^p

is

THE HAMILTON-JACOBI EQUATION


12.52.

Use the Hamilton-Jacobi method


gravitational

to

determine the motion of a particle falling vertically in a uniform

field.

down a frictionless
Set up the Hamilton-Jacobi equation for the motion of a particle sliding
thus determine the
and
(a)
in
equation
Hamilton-Jacobi
inclined plane of angle a. (b) Solve the
motion of the particle.

12.53.

(a)

12.54.

Work

the horizontal by using


the problem of a projectile launched with speed v Q at angle a with

Hamilton-Jacobi methods.
12.55.

Use Hamilton-Jacobi methods


lator in

12.56.

2 dimensions,

(a)

Use Hamilton-Jacobi methods

motion and find the frequencies of a harmonic

to describe the

(b) 3

oscil-

dimensions.

to arrive at the

generating function of Problem 12.46.

PHASE INTEGRALS AND ANGLE VARIABLES


12.57.

Use the method


of length

12.58.

I,

of a simple
of phase integrals and angle variables to find the frequency

assuming that

Find the frequencies of

An$.

oscillations are small.

(a)

pendulum

^^j

a 2 dimensional harmonic oscillator,

(b)

a 3 dimensional harmonic

oscillator.

12.59.

pendulum by using phase integrals.


Obtain the frequency of small oscillations of a compound

12.60.

Two

connected by equal
equal masses
springs to fixed walls at A and B are free
AB
plane ad
frictionless piane
to slide in a line on a irictioniess

V
A
y,

12.61.

Discuss Problem 12.57

if oscillations

1
'W ^
p|
ft

| nS^^-C\-^W^-C\^A
\^# vvv \l#
^

[see Fig. 12-6]. Using phase integrals determine the frequencies


freauencies of the normal modes.

are not

t<Ig-

assumed small.

MISCELLANEOUS PROBLEMS
12 62

particle of

coordinates.

Ans.

mass
Give

cylindrical
moves in a force field having potential V( P ,*,z) where P ,*,z are
particle.
the
for
equations
Hamilton's
(b)
(a) the Hamiltonian and

+ p%h 2 + P2z )/2m + V( P


= Pp/m, I = Po/mr*. z = Vz /m,

(a)

H =

(b)

(p 2p

<p,

z)

pp

= p%lm P * -

dV/d P

P<t>

-dV/d<f>,

pz

= ~dV/dz

CHAP.

12.63.

12.64.

HAMILTONIAN THEORY

121

particle of mass
which moves in a plane relative to a fixed set of axes has a Hamiltonian
given by the total energy. Find the Hamiltonian relative to a set of axes which rotates
at constant
angular velocity * relative to the fixed axes.

Set up the Hamiltonian for a double pendulum. Use Hamilton-Jacobi methods to


determine the
normal frequencies for the case of small vibrations.

12.65.

F{t,x,x'x)dt

X*2
BF

d /BF

Bx

Can you generalize


12.66.

12.67.

Work Problem

particle of

* 2 tan 2

a.

(6)

Use the
z

(a)

(a)

Arts.

12.68.

335

>

dt

\dx J

dV\dx)

an extremum

is

that

this result?

3.22,

page

76,

by Hamiltonian methods

m moves on the inside of a frictionless vertical cone having equation x 2 + y^ =


Write the Hamiltonian and (b) Hamilton's equations using cylindrical
coordinates.

mass

-^- +
2m

ir

+ mgp

2mP 2s

P " Sin2a

P<ft

rap3

cot a

mg

cot a

Problem 12.67 to prove that there will


find the frequency in this orbit.

results of
0,

to be

and

be a stable orbit in any horizontal plane

12.69.

Prove that the product of a position coordinate and its canonically


conjugate momentum must
have the dimension of action or energy multiplied by time, i.e. AfL 2 T-i.

12.70.

Perform the integration of equation


Kepler problem in Chapter 5.

12.71.

Verify the integration result

12.72.

Prove that Euler's equation

(3)

(9),

(10) of

of Problem 12.21.

page 313, can be written as

.n^E.,+

V dy'2
'

12.73.

Problem 12.16 and compare with the solution of the

&F

y dy'dy

+
'

dW

dF

dy' dx

by

A man

can travel by boat with speed v 1 and can walk


with speed v2 Referring to Fig. 12-7, prove that in
order to travel from point A on one side of a river
bank to a point B on the other side in the least time
he must land his boat at point P where angles 9 and
t
2 are such that
.

sm

0j

sin e 2

Vj

v2

Discuss the relationship of this result to the refraction of light in the theory of optics.

12.74.

Prove that

a particle moves under no external


a free particle, then the principle of
least action becomes one of least time. Discuss
the
relationship of this result to Problem 12.73.
forces,

12.75.

i.e.

if

it is

Fig. 12-7

Derive the condition for reflection of light in optical theory


by using the principle of least time.

HAMILTONIAN THEORY

336

12.76.

[CHAP. 12

shape of a curve lying in a plane and having fixed endpoints such that its
of inertia about an axis perpendicular to the plane and passing through a fixed origin is a

It is desired to find the

moment

minimum.
(a) Using polar coordinates

(r, e),

show that the problem


r2 V 1

is

equivalent to minimizing the integral

r2 (de/dr) 2 dr

where the fixed endpoints of the wire are (r v e^, (r2 2).
Write Euler's equation, thus obtaining the differential equation of the curve.
Solve the differential equation obtained in (b) and thus find the equation of the curve.
,

(b)
(c)

(c) r3 = c t sec (30


the fixed points.

Ans.

where

c2)

12.77.

Use the Hamilton-Jacobi method

12.78.

Use Hamilton-Jacobi methods

12.79.

If [F,G] is the Poisson bracket [see


(a)

[F.F^G]

<>

f [F,G)

= F

dF

_dt

(c)

12.80.

12.81.

A [F,G]

dt

Prove that

(a)

where

t afl

Evaluate [H,

+ F2

,G

F,

fl

if

tf

where

is

(6)

[p a ,

fi ]

(c)

0,

[p a , q p ]

called the

Kronecker

the Hamiltonian and

Illustrate Liouville's

12.84.

(a)

Is the

(6)

Discuss the uniqueness of the generalized

t is

the time.

Are

and

canonically conjugate

[F3,[F lf F 2]]

=
oscillator.

Explain.

momenta and Hamiltonian

of a system.

particles [see Problem 8.29, page 215]


Set up the Hamiltonian for a string consisting of
Use Hamilton-Jacobi methods to find the normal modes and frequencies.

Prove that the Poisson bracket

12.87.

Prove that Liouville's theorem

12.88.

(a)

Let
a

Qa =
1, 2,

where A

and A ajLt

12.89.

S a(3

delta.

Lagrangian of a dynamical system unique?

12.86.

(6)

theorem by using the one dimensional harmonic

12.83.

(6)

prove that

fi

Prove JacobVs identity for Poisson brackets


{F lt [F2 ,F 8 ]-] + \F 2 [Fa ,F^

(a)

page 294.

dG

12.85.

12.26],

11.21,

dt

is

t]

page 293, and

11.20,

[F lt G\

w dG

0,

up the equations of motion of a spherical pendulum.

Problems 12.25 and

are determined so that the curve passes through

c2

Problems

,G

[q a , q fi ]

and

Explain.

variables?
12.82.

dF_

dt

to set

to solve

[F2 ,G]

ct

is

is

<*>anQ,

is

is

invariant under a canonical transformation.


equivalent to the result

p* =

Prove that the


n.
the determinant.

*W?V

where

transformation

is

dp/dt

<W

a 12

a21

a 22

a 2n

a n\

a n2

[p,H],

and

canonical

11

6 <*m
if

are

iven constants and

and only

if

b ail

A a(l/A

*ln

the cof actor of the element a aix in this determinant.

Prove that the conditions in

(a)

are equivalent to the condition

Prove that the path taken by a fixed point on a

circle as it rolls

2 Pa Qa

2 Pa 9a-

along a given line

is

a cycloid.

CHAP.

12.90.

HAMILTONIAN THEORY

121

337

Express as an integral the total potential energy of a uniform chain whose ends are suspended
from two fixed points. (6) Using the fact that for equilibrium the total potential energy is a minimum, use the calculus of variations to show that the equation of the curve in which the chain
hangs is a catenary as in Problem 7.32, page 186. [Hint. Find the minimum of the integral subject
(a)

to the constraint condition that the total length of the chain is

12.91.

Use the methods

a given constant.]

of the calculus of variations to find the closed plane curve

which encloses the

largest area.

12.92.

12.93.

Prove that the constants


the total energy.

(a)

/?

in

Problem 12.15 and

(b)

fo in Problem 12.16 can be identified with

If the theory of relativity is taken into account in the motion of a particle of


field of potential

V, the Hamiltonian

H
where

in a force

= vVc 2 + m2 c4 + V

Obtain the equations of motion for this particle.

speed of light.

c is the

mass

given by

is

problem of a particle moving

12.94.

Use Hamiltonian methods

to solve the

12.95.

Use spherical coordinates

to solve Kepler's problem.

12.96.

Suppose that

m of the n coordinates q

lt

q2

in

an inverse cube force

q n are cyclic [say the first

ra, i.e.

q x q2 ,
,

field.

q m]'

Let

TO

%
Prove that for a

m + 1,

a=l

ca h a

.,n

-77

~ L

where

tj-

dt \ dqa J

t
dq

ca

dL/dq a

By using it a problem involving n


is called Routh's function or the Routhian.
The function
degrees of freedom.
degrees of freedom is reduced to one involving n

12.97.

SL

Using the properties

rlT

8y

rlT

^-7 8y',

(8y)'

8y'

of the variational symbol 8 [see Problem 12.6] and assuming that the operator 8 can be brought
under the integral sign, show how Lagrange's equations can be derived from Hamilton's principle.

12.98.

P = Pip,
are given by

Let

q),

Q=

Q(p,

H - H(p, q)

q).

and

Suppose that the Hamiltonian expressed


J[ = J((P, Q) respectively. Prove that
q

Q =

then

8H/dp,

dJl/dP,

in

terms of

p, q

and P,

if

= -dH/dq

P = -dJ{/dQ

provided that the Jacobian determinant [or briefly Jacobian]


d(P, Q)

Hp,q)

_
~

dP/dp

dP/dq

dQ/dp

dQ/dq

Discuss the connection of the results with Hamiltonian theory.

12.99.

(a)
(b)
(c)

12.100.

Set up the Hamiltonian for a solid cylinder rolling down an inclined plane of angle a.
Write Hamilton's equations and deduce the motion of the cylinder from them.
Use Hamilton-Jacobi methods to obtain the motion of the cylinder and compare with part

Work Problem

7.22,

page 180, by Hamilton-Jacobi methods.

(6).

HAMILTONIAN THEORY

338

12.101.

Write (a) the Hamiltonian and (6) Hamilton's equations for a particle of charge
moving in an electromagnetic field [see Problem 11.90, page 309].
Ans.

(a)

H - ^-(p- eA) +

(6)

(p-eA),

[CHAP.

12

and mass

e<*>

= -eV* + eV(Av)

Obtain the Hamilton-Jacobi equation for the motion of the particle in Problem 12.101. (6)
the result to write equations for the motion of a charged particle in an electromagnetic field.

12.102. (a)

Write the Hamiltonian for a symmetrical top and thus obtain the equations of motion.
pare the results obtained in (a) with those of Chapter 10.

12.103. (a)

12.104.

Prove Theorem

12.105.

An atom

12.2,

(6)

Use

Com-

page 314.

consists of an electron of charge

e moving

charge Ze such that

in a central force field

F about a

nucleus of

F =

r3
r is the position vector of the electron relative to the nucleus and Z is the atomic number.
In Bohr's quantum theory of the atom the phase integrals are integer multiples of Planck's con~

stant h, i.e.,
= n^h.
d) p e de n 2 h
<j) p r dr

where

Using these equations, prove that there

will be only a discrete set of energies given

where n

= n + n2 =
x

1, 2, 3, 4,

is called

mZ2 e4

2?r2

the orbital

quantum number.

by

Appendix

Units

and Dimensions

UNITS
Standardized lengths, times and masses in terms of which other lengths, times and
masses are measured are called units. For example, a distance can be measured in terms of
a standard foot or meter. A time interval can be measured in terms of seconds; hours or
days. A mass can be measured in terms of pounds or grams. Many different types of units
are possible. However, there are four main types in use at the present time.

The

1.

The CGS

2.

The

3.
4.

first

or

centimeter-gram-second system.

or

meter-kilogram-second system.

The FPS

or

foot-pound-second system.

The FSS

or

foot-slug-second system, also called the


English gravitational or engineering system.

MKS

two are sometimes called metric systems, while the last two are sometimes
There is an increasing tendency to use metric systems.

called

English systems.

The following
with the equation

indicates four consistent sets of units in these systems

F=

CGS

System:

MKS

System:

FPS
FSS

which can be used

ma:

System:
System:

F (dynes)
F (newtons)
F (poundals)
F (pounds weight)

m (grams)
= m (kilograms)
= m (pounds)
= m (slugs)
=

x a (cm/sec 2 )
x a (m/sec2
x a
x a

(ft/sec

(ft/sec

In the third through sixth columns of the table on page 340, units of various quantities in
these systems are given.
In the table on page 341, conversion factors

among

units of the various systems are

given.

DIMENSIONS
The dimensions of all mechanical quantities may be expressed in terms of the fundamental dimensions of length L, mass M, and time T. In the second column of the table on
page 340, the dimensions of various physical quantities are listed.
339

UNITS AND DIMENSIONS

340

UNITS

[APPENDIX A

AND DIMENSIONS
MKS

FPS System

FSS System

ft

ft

gm

kg

lb

slug

sec

sec

sec

sec

Velocity

LT-i

cm/sec

m/sec

ft/sec

ft/sec

Acceleration

LT-*

cm/ sec 2

m/sec 2

ft/sec2

ft/ sec 2

MLT-*

gm cm/sec 2
= dyne

kg m/sec 2
= newton

lb ft/sec 2

Force

gm cm/sec
= dyne sec

lb ft/sec

MLT-i

kg m/sec

Momentum, Impulse

ml* r-2

gm cm2 / sec2
= dyne cm
= erg

Dimension

CGS System

Length

cm

Mass

Time

Physical Quantity

Energy,

Work

ML 2 r-a

Power

gm cm 2 /sec 3
= dyne cm/sec
= erg/sec

System

nt sec

m2 /sec2
nt m

kg

=
=
kg

joule

m 2 /sec 3

=
=

joule/sec

watt

slug ft/sec2

poundal

slug ft/ sec

pdl sec

ft pdl

ft

lbwt

slug f t2 /sec 3
= ft lbwt/sec

lb f 2 /sec 3

lbwt sec

slug ft2 /sec 2

lb f 2 /sec 2

lbwt

ft pdl/sec

Volume

L3

cm 3

m3

ft3

ft 3

Density

ML-3

gm/cm 3

kg/m 3

lb/ft 3

slug/ft3

radian (rad)

rad

rad

rad

T -i

rad/sec

rad/sec

rad/sec

rad/sec

ji-2

rad/sec 2

rad/sec 2

rad/sec 2

rad/sec 2

kg

slug ft2 /sec 2

Torque

gm cm2 /sec 2
= dyne cm

lb ft2 /sec 2

ML 2 T-2

Angular momentum

ML

gm cm 2 /sec

kg

Angle

Angular

velocity

Angular acceleration

Moment

of inertia

Pressure

T~i

Ml?

gm cm 2

ML-i T-2

gm/(cm sec 2 )
= dyne/cm2

m2/sec 2
= nt m

ft pdl

ft lbwt

m 2 /sec

lb f Wsec

slug ft2 /sec

m2

lb ft 2

slug ft2

pdl/ft2

lbwt/ft2

kg

kg/(m sec 2 )
= nt/m 2

"

APPENDIX

UNITS AND DIMENSIONS

A]

341

CONVERSION FACTORS
Length

kilometer (km)

1000 meters

1 inch (in.)

2.540

cm

meter (m)

100 centimeters

1 foot (ft)

30.48

cm

m
m
10" 6 m
10-9 m
10- 10 m

1 centimeter (cm)
1

Area

Volume

millimeter (mm)

micron

millimicron (m/t)

angstrom (A)

square meter (m2 )

square foot

1 liter

(/t)

(I)

(ft2 )

meter (m

1 cubic

Mass

U.S. gallon (gal)

kilogram (kg)

1 slug

Speed

Density

mi/hr

gm/cm 3 =

0.01602

centimeter

0.3937

meter

39.37 in.

kilometer

0.6214 mile

square mile (mi2 )

acre

pound

kilogram weight (kgwt)

(nt)

weight (lbwt)

U.S. short ton

m =

1 joule

nt

lbwt

watt

horsepower (hp)

kilowatt (kw)

nt/m 2

1 joule/sec

m3

1.602

6895 nt/m 2

=
=

57.296;

453.6

1.201 U.S. gallon

gm =

277.4 in3

0.03108 slug

ft/sec

0.2248 lbwt

kgwt =

2240 lbwt;

10

Btu

3413 Btu

joule

=
=

0.2389 cal/sec
33,000 ft lbwt/min

cm mercury =
nt/m

cm mercury =

745.7 watts

0.9483 Btu/sec

5.171

860.0 kcal

X lO" 6 atmosphere =

9.481

X 10~ 3 Btu

3.968

9.869

0.2389 cal

2205 lbwt

0.293 watt hr

737.6 ft lbwt/sec

10 5

metric ton

1.285 X 10" 3 Btu

10 6 joules
19

32.17 poundals

9.807 nt

3.087 ft lbwt

X 10~

1.013

76

0.4536

0.7376 ft lbwt

550 ft lbwt/sec

=
=

radian (rad)

10 7 ergs/sec

10 dynes/cm 2

43,560 ft2

0.03532 ft3

28.32

1 lb

kgwt

long ton

3.60

hp

atmosphere (atm)

0.3239 cal

1.341

0.1020

1055 joules

640 acres

= 1.940 slug/ft3
= 0.5154 gm/cm 3

2.205 lbwt

kilowatt hour (kw hr)

1 lbwt/in 2

4.186 joules

778 ft lbwt

10 7 ergs

Btu

=
=

in.

km/hr = 0.4470 m/sec

4.448 nt

2000 lbwt;

1.356 joules

61.02 in3

1 British gallon

I;

1 slug/ft3

10 5 dynes

0.02832

62.43 lb/ft3

gm/cm

in.

0.06852 slug;

1.609

kg/m3 =

10 3

1 electron volt (ev)

Angle

km

10- 3

kg

14.59

newton

3.785

1.609

ft3

35.32

2.2046 lb

1.467 ft/sec

1 calorie (cal)

Pressure

in 3

231

1.057 quart (qt)

1000

1 ft

Power

cm 2

929

7.481 U.S. gal

32.174 lb

Energy

10.76 ft2

mile (mi)

km/hr = 0.2778 m/sec = 0.6214 mi/hr = 0.9113

1 lb/ft3

Force

lmil

=
=

1 cubic foot (ft3 )

10" 3

cm 3 =

1000
3

10-2

27.68

2.089
in.

X 10~ 2 lbwt/ft2

water

= 1.013 X 10 6 dynes/cm2
406.8 in. water

0.017453 rad

14.70 lbwt/in 2

Appendix

Astronomical Data
THE SUN
X 10 30

X 10 30 kg

or

2.0

or

6.96

89.2 lb/ft3

or

1.42

gm/cm3

896 ft/sec 2

or

273 m/sec2

385 mi/sec

or

620 km/sec

25.38 days

or

2.187

X 10 6

or

6.673

X 10 ~ 8 cm 3 /gm-sec 2

or

3.84

X 10 5 km
X

Mass

4.4

Radius

4.32

Mean

density-

Mean

surface gravitational

lb

X 10 5 mi

10 5

km

acceleration

Escape velocity
at surface

Period of rotation
about axis

Universal gravitational
constant G

X 10~ 9

1.068

ft 3 /lb-sec 2

MOON

THE

Mean

distance from earth

239

sec

10 3

mi

Period of rotation
about earth

27.3 days

or

2.36

Equatorial radius

1080 mi

or

1738

Mass

1.63

or

7.38

208 lb/ft3

or

3.34

gm/cm3

5.30 ft/sec 2

or

1.62 m/sec 2

Escape velocity

1.48 mi/sec

or

2.38

km/sec

Period of rotation
about axis

27.3 days

or

2.36

X 10 6

Orbital speed

.64 mi/sec

or

1.02

km/sec

Orbital eccentricity

.055

Mean

density

Mean

surface gravitational

X 10 23

lb

10 6 sec

km
10 22

kg

acceleration

342

sec

APPENDIX

ASTRONOMICAL DATA

Bl

bfl

>>
c

-a

X X

o
co
t-

o
eo

10

a
a

44

co

4)

44

00

OS

*
00
*
CO

co

2 iH

oo
OS

CO

t#

x x

50
CO

co

<o

Co"

bo

T3

09

CO
eo

SI

CO
co

i-l

^B

us

00
CO
co

S
4*i

us
CO

11

X X

OS

h-

if

tH

^.

a)

a
44
tP
Tjt

us

4*

IM
CO

00

<v

os

t*

Cv,

i-(

CO

00
(M

"<

02

43

bo

O 2
iH

(4

t"J

00

a
44

X X
2!
00
trH

os

44

T3

*h

<u

1-1

si

X x
\a
o

a
a

o
iH
rH

343

x x

x x

too

to
CO

?o

Is
B

o
t-

jo

eo
co

a)
teo

00
CO

T^
?o

0)

<v

(1

i-l

CO

10

43
iH
eo
<*
CO

US

bo

^a
o
tH

a
a

eo

co

,3.

i-<

X X
31
00

t-

co

00

X X

43

x
1-1

5;
tH

bo

iH
rH

14

.58

>

x
tco
CO

44

TH
CO

Tj<

CO

Tf
f)

55

c3

il
OS
eo

JO
to

X x
3

3
3

00

o o

73

10
00

x X
co
00

O s
P
s S w

1 !
tH

CO

<u

a
U3
00

eo

*o

oa

>>

FH

00
CO

to
00

eo

44
CO

CO

eo

<o

'-J

OS

3 &

T3

s
44
00
os
CO

o
o

CO

-IJ

c3

4J

00
iH
CO

09
01

a
to
03

rj

O
00

CO

i
S2

o>

05

bo

CO

5
fl>

bo

to

x X
*#

tH

- a
O S3

CO

o
rH

n
el

CO

^S

00
OS
CO

CO
rji

a
44
iH

W
CO

jo

a
44
OS
t<^K

*h

ops

a 3

w s
M
bo

*c is

u o

o-g^
a a

h
<B

O O

-S

a
g
o

Appendix

Solutions of Special
Differential Equations

DIFFERENTIAL EQUATIONS
An equation which has derivatives-

or differentials of an unknown function is called a


The order of the differential equation is the order of the highest
differential equation.
derivative or differential which is present. A solution of a differential equation is any
relationship between the variables which reduces the differential equation to an identity.
Example

1.

The equation j*-=2y


y

is

ce 2x

where

c is

is

a differential equation of

any constant,

the identity

2ce 2x

Example 2.
is a
The equation x 2 dx + y3 dy =
where c is any constant, since taking the
d(x s /S

Example

or order

1.

solution of this equation

2ce 2x

differential equation of first order.


differential of the solution we have

y*/4)

2y

we have

x 2 dx

or

y z dy

solution is

x*/3

+ y*l =

3.

^| - 3^ +

The equation

CLOG

first order,

since on substituting this into the given differential equation

= dex +

c 2 e 2x

&dx2

S *!L

dx

2x

4x

is

a differential equation of second order.

+ c 2 e 2x + 2x + 3)

solution

is

0/QCr

2y

since

( Cl

ex

+ 4c 2 e 2x )

3( Cl e x

+ 2c2 e 2x + 2) +

2(c t e x

4*

as independent variable and y as dependent


symbols could just as well have been used.
other
variable. However, it is clear that any
Example 3 could be
of
equation
Thus, for instance, the differential

we have used x

In the above examples

H - 3^ +
dt 2

with independent variable

and dependent variable * and solution x

The above equations are often


from

2x

dt

cie*

c 2 e 2t

2t

called ordinary differential equations to distinguish

partial differential equations such as

a2V

-^ =

r)

c2 -^" involving

S.

them

two or more independent

variables.

ARBITRARY CONSTANTS. GENERAL AND PARTICULAR SOLUTIONS


values and are called
In the above examples the constants c, d, c 2 can take on any
will have a solution
arbitrary constants. In practice an nth order differential equation
is called the general
involving exactly n independent arbitrary constants. Such a solution
giving the constants special
solution. All special cases of the general solution obtained by
Example 3 above if we let
values are then called particular solutions. For instance in
2*
+ c 2 e + 2x + 3, we obtain the particular
= -3 in the general solution y =
=
ci

6,

solution

w*

c2

y =

5e

- 3e 2x + 2x + 3.
344

APPENDIX

SOLUTIONS OF SPECIAL DIFFERENTIAL EQUATIONS

C]

345

Particular solutions are often found from certain conditions imposed on the problem
called boundary or initial conditions. In Example 3 for instance, if we wish
to satisfy the conditions y = 5 when x =
and if = dy/dx = 1 when x = 0, we obtain
ex = 5, c 2 = 3.

and sometimes

in which we are required to solve a differential equation subject to given


often called a boundary-value problem.

problem

conditions

is

SOLUTIONS TO SOME SPECIAL FIRST ORDER EQUATIONS


The following list shows some important methods for finding general solutions of
order differential equations.
1.

first

Separation of Variables
If

first

order equation can be written as

F(x)dx + G(y)dy

(1)

then the variables are said to be separable and the general solution obtained by direct
integration is

J
2.

+ C

F(x) dx

G(y) dy

(2)

Linear Equations

A first order equation

called linear if

is

has the form

it

g + P(x)y
Multiplying both sides by e J

Then

integrating, the general solution

y e)

or

y
C

The factor

Q(x)

(S)

this can be written

-^{yJ

3.

pdx

Pdx
}

Qe

Spdx

is

e~f Pdx

Qe^

Pdx

dx +

Pdx
C Qef dx + ceS Pdx

(4)

p dx
is

often called an integrating factor.

Exact Equation

The equation

Mdx + Ndy =
where

and

(5)

are functions of x and y is called an exact differential equation if


can be expressed as an exact differential dU of a function U(x,y). In such
case the solution is given by U(x, y) = c.

Mdx + Ndy

necessary and sufficient condition that

M =
By

(5)

be exact

is

dN
dx

'

In some cases an equation is not exact but can be made exact by first multiplying
through by a suitably chosen function called an integrating factor as in the case of the
linear equation.

[APPENDIX C

SOLUTIONS OF SPECIAL DIFFERENTIAL EQUATIONS

346

Homogeneous Equation

4.

an equation has the form

If

f(V-)
\x/

Pdx

(7)

homogeneous equation and can be solved by the transformation y


(7) becomes
dv
dx =
dv = r,/
F{v)
or
v + x
1
~x~
Tx

called a

it is

Using

this,

Then the general

which the variables have been separated.

fdx = C
J
J x

dv
F(v)

solution is

where v = /x

vx.

s
{)

fW ^

in

{9)

Occasionally other transformations, which may or may not be evident from the form
of a given differential equation, serve to obtain the general solution.

SOLUTIONS OF HIGHER ORDER EQUATIONS


The following

list

shows certain equations of order higher than one which can often

be solved.

-^n

1.

dx

F(x).
v
'

In this case the equation can be integrated n times to obtain

= f

SlL
dx 2

we

(x !^L)
dx J
\

F(x) dx n

in this case y

find that the equation

ci

missing and

is

dv

'

we make

en x n

~l

the substitution dy/dx

F(x

v)

'

order equation. If this can be solved we replace v by dy/dx, obtaining another


order equation which then needs to be solved.

first

first

if

czx

becomes

Tx
a

c2x

^y = p(va ^}L\
'dxJ
dx
2

Here x

is

missing and

we make

if

the substitution

dy/dx

v,

noting also that

TO

d2y

dx 2

_
~

cLv_

dx

the given equation can be written as a

which then needs

first

dv&n
dydx

_
~

dv
dy'

order equation

to be solved.

LINEAR DIFFERENTIAL EQUATIONS OF ORDER HIGHER THAN ONE


We shall consider solutions of linear second order differential equations. The

results

can easily be extended to linear higher order equations.

linear second order equation has the

^+P

(x)

form

fx +

Q( X )y

IHx)

(10)

APPENDIX
If

yc

is

SOLUTIONS OF SPECIAL DIFFERENTIAL EQUATIONS

C]

347

the general solution of the equation

[obtained

by replacing the right hand

side of (10)

of (10), then the general solution of (10)

The equation
called the

zero]

and

if

yp

is

any particular solution

+ yp

yc

(12)

complementary equation and

(11) is often called the

complementary

by

is

its

general solution

is

solution.

LINEAR EQUATIONS WITH CONSTANT COEFFICIENTS


solution is easily obtained when P(x) and Q(x) are constants
In such case equation (11) can be written

The complementary

respectively.

+
If

we assume

as solution y

e ax

4f

where

the equation
a2

+ *"

+ Aa + B

and

a is constant in (11),

we

find that a

must

satisfy

This equation has two roots, and the following cases arise.
1.

Roots are real and

distinct,

say a x

a2

In this case solutions are e


and e *. It also follows that c x ea & and c2 e a * x are solua
tions and that the sum c x e & + c 2 e a * x is the general solution.
aiX

2.

Roots are real and equal, say 1


In this case
c x ea ^x

3.

c2 xe aiX

we

012

find that solutions are e aiX

and xe aiX and the general solution

is

Roots are complex.


If

and

B are real, these complex roots are conjugate, i.e.

case solutions are e

The general

ia+hi)x

ax

bix

solution can be written

ax

ax

(cos
(c

bx

cos bx

sin bx)

and

+ bi and
e

(a ~ bi)x

(cos

bx

bi.

In such

i sin bx).

c2 sin bx).

PARTICULAR SOLUTIONS
To

find the general solution of

we must

find a particular solution of this equation and add it to the general solution of (13)
already obtained above. Two important methods serve to accomplish this.
1.

Method

Undetermined Coefficients.
This method can only be used for special functions R(x) such as polynomials and the
exponential or trigonometric functions having the form e px cos px, sin px where p
See Problems C.17
is a constant, together with sums and products of such functions.
and C.18.
of

2.

Method

of Variation of Parameters.

In this method we first write the complementary solution in terms of the constants
We then replace c x and c 2 by functions f t (x) and f2 (x) so chosen as to satisfy
c x and c 2
.

the given equation.

The method

is

illustrated in

Problem C.19.

[APPENDIX C

SOLUTIONS OF SPECIAL DIFFERENTIAL EQUATIONS

348

Solved Problems

DIFFERENTIAL EQUATIONS. ARBITRARY CONSTANTS.


GENERAL AND PARTICULAR SOLUTIONS
Cl.

(a)

Prove that y

x-1

ce~ x

the general solution of the differential equation

is

x+v

%~
(b)

Find the particular solution such that y

(a)

If

ce~ x

dy/dx

when x -

= ee~ x + 1 and so
- (-ce' x + 1) - x +

dy/dx

then

1,

(ce~ x

+ a; 1 is a solution; and since it has a


(namely one) equal to the order of the differential equation, it

Thus

(6)

C.2.

(a)

2/

Since y
y = 4e~ x

Prove that x

we have from

0,

= c^ +

c 2 e~ st

sin

dt 2

number

(a)

From

^+

Then

c 2 e~ 3t

=
2

dt2

+ x-1,

ce~ x

sin

Cl e*

= c-1

or

4.

Thus

x-1,

dx/dt

4 sin*

= -3

at

0.

we have

= 2cos* -

3*

at

Find the particular solution such that


Cje*

of arbitrary constants
the general solution.

is

the general solution of

is

(b)

1)

the required particular solution.

^ + 2^ a

+x-

ce- x

= 3 when
+ x 1 is

0.

3e 2 e- 8t

3a;

(c ie t

dt

cost,

+
-

2 cos

9c 2 e-3t

3(cje*

"^Jf

4 sin

sin

9c 2 e- 3t

+ 2( Cl e* + sin t)

t)

c 2 e-3t

c i et

3c 2 e"

sin

cos

t)

the
sin t is a solution; and since it has two arbitrary constants while
c 2 e~ 3t
c^*
Thus a;
differential equation is of order two, it is the general solution.

(6)

From

part

(a),

letting

=
2
-3

Solving,

we

find

cx

in the expressions for x

=
=

1/2, c 2

ct
Cl

and dx/dt, we have


fci

c2

3c 2

3/2.

or

i
L ci

+
_

c2

Then the required particular

^e*

3t

f e~

sin

= 2
= _4
.

3c 2

solution

is

SEPARATION OF VARIABLES
C.3.

+ xy 2 dx + (y + 22/) dy =
= 1.
such that y = 2 when

(a)

Find the general solution of

(b)

Find the particular solution

(a)

Write the equation as


separate the variables,

x(l

we

+ y2

(x

dx

y(l

+ x2

dy

Dividing by

0.

(1

+ x 2 )(l + y 2

>

to

find

xdx
1

Then we have on

0.

a;

a;2

ydy
1

^)

y2

integrating,

xdx

x2

1 In

(1

a;

f ydy

J
)

^y*
+y

In (1

2
?/

Cl

*i

APPENDIX

SOLUTIONS OF SPECIAL DIFFERENTIAL EQUATIONS

C]

This can be written ^ In {(1

+ x 2 )(l + y 2 )} +

(1

which
(6)

is

Since y

solution

is

^ = RH

Solve

+ yl) =

when x =

we have on

1,

R=

if

at

substitution in

+ x 2 )(l + y 2 =

10

when

Separating the variables, we have

(j

g)

1,

R=

we

find

dR

-^ -

t2

y2

10;

thus the required particular

x 2y2

Integrating both sides,

dt.

t3

4
3

+C

Thus

-4/3.
t3

x2

c2

1.

(2),

or

~R =
Substituting

C2

the required general solution.

(1

C.4.

or

cx

X 2 ){1

349

or

it

t3

LINEAR EQUATION
C.5.

dv

This
factor

+ 2xy =

-5

Solve

if

2/

when x =

a linear equation of the form

is
I

eJ

is

x3

2.x dix

x2

0.

page 345, with

(S),

P=

2x,

Q =

Multiplying the given equation by this factor,

e x2

^- + 2xye x2 =

+ x. An

integrating

find

+ x)e x2

(x 3

ax

Xs

we

which can be written

Integrating,

or,

making the

substitution

x2

Thus

Check:

If

when x =

= \x 2 +

Solve

we

0,

2e~ x2

^-+

C.6.

e*

find

~=SU
+ lifU =
dt

Writing the equation

2.

then dy/dx

2xy

in the

(x 3

(# 3

x)ex2

+ x)e x2 dx +

in the integral,

Since y

e x2 )

t~{v
ax

-1x20*2

= x 2 +

ce~x2

Thus

= \x 2 + 2e~ x2

- 4xe~ x2

4xe~ x2

when

Thus

2x(\x2

2e~* 2 )

x3

0.

form

dU

dt~* U =

SOLUTIONS OF SPECIAL DIFFERENTIAL EQUATIONS

350

~ 3dt

we

see that

it is

linear with integrating factor

e-*

e~ st

[APPENDIX C

Multiplying

by e" 8 ',

(1)

can be

it

written as

4i(Ue-M)

e -*t

at

Integrating,

Since

Ue~ 3t = ^e~ 3t +

we have

U=

when

we

0,

Thus

= _ e -3t +

Ue-zt

The equation can

Another method.
we have

obtain c

(e 3t

-l)

(2)

by the method of separation of variables. Thus

also be solved
,r.

dU

Integrating,

U=

u =

or

|.

3U +

Since

when

0,

we

find

(sin

In (3t7

dt

1)

so that

In

(BU +

1)

Thus

t.

*7

(e 3t -l).

EXACT EQUATIONS
C.7.

(Sx 2

Solve

y cos x) dx

Method

cos x

M = Sx2 + y cos x, N = sin g -

with
dx + N dy = 0, we have
dN/dx and so the equation is exact.

Two

4s/

3
.

Then

methods of solution are available.

1.

Since the equation

grouping terms, we

d(x s )

i.e.,

must be an exact differential of a function U.


can be written

exact, the left side

is

find that the equation

3a 2 dx

d(y sin *)

x dx

(y cos

sin

x dy)

d(-y)

Xs

y sin x

y sin x

y4

By

Ay3 dy

d( 3

or

Integration then gives the required solution,

Method

0.

Comparing
3Af/3i/

- 4y 3 dy =

y4)

c.

2.

The given equation can be written as


(3a; 2

y cos

x)

dx

(sin

4j/

dy

dU = -^dx +

dy

Then we must have

^
ATT

(1)
v
'

Integrating

(1)

3a;2

dx

with respect to

x,

this into (2),

sin

where F'(y)

dF/dy.

a;

(2)

a;

keeping y constant,

U =
Then substituting

we

Xs

y sin x

F'(y)

sin x

4?/

F'(y)

or

y sin x

Xs

y sin x

y*

can be written

dU =
xz

4j/

F(y)

= -4^

Integrating, omitting the constant of integration,

differential equation

so the solution is

sin

find

U =

and

oy

we have

that

Then the given

STJ

cos

2/

y4

d(x*

c.

y sin x

y4)

we have

F(y)

= -y*

so

APPENDIX

SOLUTIONS OF SPECIAL DIFFERENTIAL EQUATIONS

C]

351

HOMOGENEOUS EQUATIONS
C.8.

p- =

Solve

dx

Let y

+ %-.

/x

Then the equation can be written

vx.

dv
%-7-

= e~

Separating the variables,


lnsc + e~ y,x c.

dx
v dv.

ev

dv
%~j~

or

= e~ v + c. Thus

Ins

Integrating,

Ax

the general solution

is

SOLUTIONS OF HIGHER ORDER EQUATIONS


C.9.

Solve

cos

U=

where

dU/dt

2,

3 at

0.

Integrating once,

dU/dt

Then

dU/dt

since

at

we

0,

et

find

U =

Now

U=

since

at

we

0,

c2

find

3.

a#" + 2y' = x 2 where

Since y

dy/da;

*g+2*

(i)

cos

*2

a5

2v

x*/4

+c

Cll. Solve yy"


Since x

+
is

{y'f

a^/12

c x lx + c2

where

missing, let y'

+*

(f)

(2/x) dx

2 ln *

=
=

ln * 2

*2

Multiplying

= x

(x 2 v)

dy/dx, y"

_
~

dy/dx

x*/4

cjx*

y'

dy/dx
V

or

v
Integrating again, y

3t

or

A
dx

integration,

solution is

d?y/dx 2

is

Then by

c2

Then the equation can be written

v.

St

The required

dy/dx, y"

cx

sin*

a linear equation in i> with integrating factor


by 05 2 it can be written as

This
(2)

y'

is missi'ig, let

y'

cos

U = \& -

CIO. Solve

sin*

Thus
t

^t2

3.

dU/dt
Integrating again,

v.

dv
dx

d2 y/dx 2

Then

dv^djt
dy dx

__

dv^

dy

and the given equation can be written

yv

so that

From

(1)

(1),

that vy

^+

y'=0
c3 and

or

2/

^.

v2

(
/

From

or

or

(2),

+ -^

% + v)

dv

y-^ +

(2)

0,

i.e.

lnv

+ lny =

c2

or

ln (vy)

c2

so

SOLUTIONS OF SPECIAL DIFFERENTIAL EQUATIONS

352

y 2 /2

Integrating,

dy/dx
c 3x

Thus solutions are y = c t and y 2 Ax + B. Since the


required general solution can be written y 2 = Ax + B.

c3

dx

= Ax + B

y2

or

c4

y dy

or

c 3/y

[APPENDIX C

first is

a special case of the second, the

LINEAR EQUATIONS WITH CONSTANT COEFFICIENTS


C.12.

Solve

4^-4^-52/
dx
dx

Letting y

e ax

0.

we

in the equation,

obtain

- 4a h)e ax =
and a = 5,-1. Then

(a 2

Thus (a 5)(a + 1) =
y = c e*x + c 2 e~ x

Solve

solutions are

0+10^ +

252/

Letting

C.15.

= c^" 2 +
*

Letting

we

~ 2t

2/

g+

e ax ,

+ 4x

e at ,

c 2 te~ 2t

Solve
e -x e 2ix

4^
dt

+
^
at

e 5x

and e~ x and the general solution

we

+ 5)(a + 5) =

Then the general

0,

solution

or
is

(i

+ c2 t).

5v

find

a2

4a

or

-2, -2.

Solve
and

e -i6U!

c x e~^ x

-5, -5.

Since

c2 xe~ 5x .

solution

is

2a

5 =
or a = -1 2t. Then solutions are
(-1 - 2i)a; = e -a: e- 2te = e-z(cos2ar i sin2a;).

e (_1

2i)x

The general

d 2y/dx 2

Letting y

0.

Then the general

and e
t sin2a;)
e -x( cos 2a;
e~ x (c x cos 2x
c 2 sin2x).
solution is y

C.16.

is

0.

a2

find

(a

i.e.

0,

Solve

4a

0.

Letting y = e ax , we find a 2 + 10a + 25 =


the root is repeated, solutions are e~ 5x and xe~ 5x

C.14.

C.13.

a2

or

cos

e ax ,
<*x

we

find

2
<*

0.

sin ax.

or a = ia. Then solutions are e to* = cos wo;


w2 =
The general solution is thus y = c t cos ax + c 2 sin v>x.

a2

sin

METHOD OF UNDETERMINED COEFFICIENTS


C.17.

Solve

0-4||-5i/

By Problem

x2

+ 2e*

C.12 the complementary solution,

i.e.

the general solution, of

&--*
yc

is

c t e&

c 2 e~ x

(1)

Since the right side of the given equation contains a polynomial of the second degree
and an exponential (2e 3x), we are led to the trial particular solution

yp

where A, B, C, D are constants


Substituting

(2)

= Ax 2 + Bx + C +

De* x

(2A

we

find

- 4B - BO + (-8A - 5B)x - SAs 2 - 8De 3 * =

Since this must be an identity,

we must have

x2)

(*)

to be determined.

for y in the given equation and simplifying,

(i.e.

a2

2e^ x

APPENDIX

SOLUTIONS OF SPECIAL DIFFERENTIAL EQUATIONS

C]

2A-4B-5C =
Solving,

we

B=,C = -{%, Z> =

A = -,

find

-8A - 55 =

0,

2/p

-5A =

0,

25

125

-8D =

1,

-j. Then from

353

(2),

solution of the given equation is

Thus the required general

which can be checked by direct substitution.

C.18.

0+lO^|

Solve

The complementary

252/

20 cos 2x.
Problem C.13]

solution [by

yc

we

Since the right side has the term cos 2x,

Cl

c 2 xe- 5x

CO

are led to the trial solution

= A

yp

is

e- 5x

+ B

cos 2x

sin 2x

(2)

Substitution into the given equation yields, after simplifying,

(21A

+ 205)

cos 2x

A = 84i> B =

84i

so that the

P articular solution
y'Pv

Vc

VP

n
2*

420

cos
I_t
841

cie

2x

sin

20 cos 2x

215 - 20/1 =

20,

0.

Solving,

we

find

is

and the general solution of the given equation


V

- 20A)

(215

we have 21A + 205 =

coefficients of like terms,

Equating

400

+
i

-57T

sin 2a

is

~ Sx

+ We~s* +

o
cos 2x

420 ___

fff

+
,

400
841

sin 2x

METHOD OF VARIATION OF PARAMETERS


C.19.

d 2y/dx 2

Solve

+y =

The complementary

tana:.

yc

We

Problem C.16 with u

solution is as in

cos x

c_

c 2 sin

1:

CO

now assume that the solution to the given equation has the form

where

/_

and /2 are suitable functions of

with respect to

From

a;.

cos x

fi

(*)

/2 sin x

we

(2)

have, using primes to denote differentiation

x,

dy/dx

-fi sin *

f2 cos x

/_ cos

/2 sin

(5)

Before finding d2 y/dx 2 let us observe that since there are two functions / t and /2 to be determined
and only one relation to be satisfied [namely that the given differential equation must be satisfied]
we are free to impose one relation between f x and /2 We choose the relation
.

/i cos

so as to simplify (8)

Another differentiation then leads


d2 y/dx 2
(2)

and

(0)

we

(5)

we

find

-fi cos *

/{
and

/1

sin x

f2

cos a

(5)

/ 2 sin

f[ sin

/ 2 cos

as

(0)

see that the given differential equation can be written

Thus
(.4)

(4)

to

dty/dx*

From

/2 sin x

which then becomes


dy/dx

From

fi

sin x

-fi sin x

= - sin 2 x/cos x,

+ f2 cos x =

/2 cos x

/2

sin x.

tan x

Thus

tan x

(7)

(5)

SOLUTIONS OF SPECIAL DIFFERENTIAL EQUATIONS

354

1 - cos2 x
sin 2 x
C
= f
I
ax
ax
I
cos x
J
J cos*
In (sec x + tan
+ sin + c
,

f.
J1

a;)

f2

Substituting in

sin x

(2)

we

(sec

cos x) ax

cos * +

dx

a?

[APPENDIX C

c2

find the required general solution,

= d

cos x

c 2 sin

a;

cos

In (sec x

a;

tan x)

Supplementary Problems
DIFFERENTIAL EQUATIONS. ARBITRARY CONSTANTS.
GENERAL AND PARTICULAR SOLUTIONS
C.20.

C.21.

Check whether each

differential equation

has the indicated general solution.

(a)

- 2^ +
^f
at'at

(*>)

+ V = &;
*4r
at

(a)

Show that

(6)

Determine the particular solution such that z

Ans.

(6)

t?-StU =

e _t (Ci sin t

e -t (3 sin

2 cos

(c t

c 2 t)e*

+t+

c 2 cost)

is

a general solution of

= 2

Solve

dz
dt

ty/T^z*

Vi -

and dz/dt

An*. #

C.25.

Solve

(a;

C.26.

Solve

^
dx

C.27.

Solve

(ye x

C.28.

Solve

(x

l)(y

C.29.

that the differential equation {Ay - a; 2 ) dx + xdy


pends on only one variable and thus solve the equation.

if

(2a;

- hy) dy =

y(l)

e~v) dx +

(xe~y

(xy

+ ex

+ y)dy =

Arts,

5.

if

Ans. \nx

^axz

+ xy) dx +

dy

(x/y)

0.

1/

6a; 2

when

Ans.

(x

Show

Solve

J^

^=

a;

2.

Ans. y

a;

0.

4e- a:2

at

= -^x + c

x*

Ans.

2.

xe~ y -

Ans. ye x

0.

SOLUTIONS OF HIGHER ORDER EQUATIONS


Solve d?U/dtz = t + e~* if U = 3, dU/dt =
C.30.
C.31.

-x

x^--2y =

at

*2

Solve

= -+

^^^_^r-

C.24.

+ 2y) dx +

t)

SOLUTIONS TO SPECIAL FIRST ORDER EQUATIONS


Solve dy/dx = -2xy if y = 4 when x = 0.
C.22.

C.23.

0.

c2

1)

a;

+ 4y - 5y2 -

ce* + y

has an integrating factor which deAns. xty ^x 6 = c

Ans.

U=

t?

e~*

+ 3t + 2

APPENDIX

SOLUTIONS OF SPECIAL DIFFERENTIAL EQUATIONS

C]

355

^ff+ 2 (^)

C.32.

Solve

C.33.

Solve

Ans. US

'

Cl t

c2

^-<*-^ +<*-*>*

H*)"]' -()'

LINEAR HIGHER ORDER DIFFERENTIAL EQUATIONS


C.34.

Solve

|^-2^-82/ =

C.35.

Solve

^?
dt

C.36.

Solve

+ 4^J+
^|
at
acz

C.37.

Solve

4^| +

C.38.

Solve0-4|-2/

C.39.

Solve

20y'

C.40.

Solve

3^ +

2y

C.41.

Solve

^r~
at

2U =

C.42.

4y"

^
at2

U =

Ans.

25y

2/(0)

-*.

6t

10, y'(0)

Ans.

0.

St

sin 2t

e 2x ( Cl

e^ x +

Cl e~*

(6)

y"

As.

(6)

(c)

C.46.

x sin

y
C.47.

C.48.

(6)

a;

cos

a;

4j/'

4y

e" 2*.

cos

sin

dx/dt + y =
c 2 sin t + ^e* +
+ c 2 cos + e*

(e t

dy/dt

Aw*, y

e*,

\e~* x

2/

2/

(6)

1W ~

if

2/(0)

Ans.

2, y'(0)

-1.

2 cos

Is the

sin t

(iv)

62/

+ \xH~^

= 36a;,
(6)
=
+ C2 e2x + c 3 e3x - 6* - 11
= d cos * + c 2 sin x + x{c 3 cos x +

6y"

(a)

18e"*

t.

to solve linear equations of order higher

Ans.

c 2 e~^

t,

Show how
y"'

c 2 a;)e~ 2a;

plicable?

(a)

= ct
= d
2/"

sulfa)

In sec x

4 cos
Explain.

Solve

Problem C.41 by variation of parameters.

Solve simultaneously:

Ans. x

e~^ x

10(cos05

(c) y" + 4y = esc 2*


+ 6y = 50 sin 4a;
- 3y = xe~ x
(d) 1/" + 82/' + 25y = 25x + 33 +
= c e 2x + c 2 e Sx + 2 cos 4* sin 4a?
= Cl e x + c 2 e~ Sx \xe~ x
= c cos 2x + c 2 sin 2a; - %x cos 2x \ sin 2a; In (esc 2a:)
= e -4x( Cl cos 3* + c 2 sin 3a;) + x + 1 + e _x

y"

Solve

c2

(^ y
C.45.

c 2 sin*)

2y'

(a)

+ 2t)e- 2

by use of the method of variation of parameters.

c 2 cos

Problem C.40 and

5y'

(l

cos 2t

Solve each of the following equations by any method.

U =

5.

C.44.

y"

Ans.

Ans. y

25.

Solve

(a)

= 0.

C.43.

(a)

+ c 2 x)e 5x/2

(c t

10 cos 2*

at

Ans. y

+|

+ c^- 2*

e~ 2t (ci cos

Ans. y

0.

c 2 e*

Ans.

0.

c x e* x

U = l,dU/dt =

if

if

sec

c x sin

5z

25y

c x e- 2t

y"

Solve

Ans. y

W
4^+
at

Arts,

0.

2/

coefficients ap-

than two by finding the general solutions of

W+

c 4 sin

method of undetermined

2t sin

a;)

y
a;

x*.

Appendix

D
Index of Special Symbols

and Notations
The following list shows special symbols and notations used in this book together with
number of the page on which they first appear. All bold faced letters denote vectors.
Cases where a symbol has more than one meaning will be clear from the context.
the

Symbols
a

length of semi-major axis of ellipse or hyperbola, 38, 118, 119

an

Fourier cosine

acceleration, 7

coefficients,

acceleration of particle

a P ip

A
A
A
cA
cA max
6

bn

B
B

vector potential of electromagnetic

length of semi-minor axis of ellipse or hyperbola, 38, 118, 119

Fourier sine

magnetic

coefficients,

unit binormal, 7

time derivative operator in fixed and moving systems, 144


unit vectors, 72

total energy, 36

electric intensity, 84

F 12
F av
FD
Fv
F(a) , F(c)

fa
fv

196

intensity, 83

e lt e 2 e 3

fa
F

309

amplitude of steady state oscillation, 90

curve, 6

f vX

field,

amplitude of steady state oscillation, 90

maximum

areal velocity, 122

Pv

relative to particle

area, 122

speed of light, 54

D F ,D M

P2

196

force due to friction, 65


internal force on particle

due to particle

X,

173

frequency, 89
frequencies, 316
force, 33

force of particle 1 on particle

2,

33

average force, 60

damping

force, 87

impulsive forces, 285


actual and constraint forces acting on particle

v,

170

generalized impulse, 285


force (external and internal) acting on particle

356

of a system of particles, 168

APPENDIX

INDEX OF SPECIAL SYMBOLS AND NOTATIONS

D]

acceleration due to gravity, 62

gravitational constant, 120

generating function, 314

Planck's constant, 338

Hamiltonian, 311

Hamiltonian under a canonical transformation, 314

Jl

unit vector in direction of positive x axis, 3

I xx , I yy , I zz

moment of inertia, 225


moment of inertia about axis through center
moments of inertia about x, y, z axes, 254

Ixy Iyz Ixz

products of inertia, 254

/
Ic

357

principal

I U I2 ,I3
J) v

moments

of mass, 226

of inertia, 255

impulses, 285

angular impulse, 228


unit vector in direction of positive y axis, 3

Ja
k

K
I

m
m

M
n

phase integral or action variable, 316


unit vector in direction of positive z axis, 3

radius of gyration, 225


length, 90

Lagrangian, 284

Lagrangian under a canonical transformation, 314


mass, 33
rest mass, 54

mass of a system of particles, 166


number of degrees of freedom, 282
orbital quantum number, 338

total

N
N

normal component of reaction force, 65

unit normal, 7

pa
p

P
Pa
<?

number

of particles in a system, 166

generalized or conjugate momenta, 284

momentum, 33
period, 89

new

generalized

momenta under a canonical transformation, 314

power, 34
electrical charge, 84

qa

generalized coordinates, 282

Qa

new

generalized coordinates under a canonical transformation, 314

spherical coordinate, 32

position vector or radius vector, 4

position vector of center of mass, 166

rx

unit vector in radial direction, 25

r{,

position vector of particle

R
R

range, 75

R max

maximum

R
R

%
%

range, 75

resisting force, 64

resultant of forces, 47
rigid body, 228

Routh's function or Routhian, 337


arc length, 7

spin angular velocity, 269

relative to center of mass, 169

radius of curvature, 8

gj"

generating function, 316


generating function depending on old position coordinates and

new momenta, 314

INDEX OF SPECIAL SYMBOLS AND NOTATIONS

358

T
T
T

tension, 74

unit tangent vector, 7

generating function depending on old and new position coordinates, 323

generating function depending on

v raax) v min

Vp

time, 6
kinetic energy, 35

T
v lim

maximum and minimum

coordinates and old momenta, 334

orbital speeds, 143

velocity, 7

velocity of center of mass, 167

/P i
2

velocity of particle

P2

\'

velocity of particle

new

limiting speed, 70

v i2> v i2

relative to particle

lt

relative to center of mass, 169

relative velocities of particles along

common normal

before and after impact, 194

potential or potential energy, 35

"U

generating function depending on old and new momenta, 334

wa

angle variables, 316

W
W

weight, 62

yc

complementary

yp

particular solution, 347

V
z

[APPENDIX D

work, 34

solution, 347

transverse displacement of vibrating string, 195


cylindrical coordinate, 32

atomic number, 338

Greek Symbols
index of summation, 282

angular acceleration, 29

/?

angle

ratio of speed of particle to speed of light,

angle

made by

logarithmic decrement, 89
variation symbol, 313

Kronecker

delta, 336

coefficient of restitution, 195

eccentricity, 118

cylindrical coordinate, 32

Euler angle, 257

polar coordinate, 25
spherical coordinate, 32

unit vector perpendicular to radial direction, 25

curvature, 8

spring constant, 86

colatitude, 152

A v A 2 A3
,

A
Ac

vie, 54

determinant, 336

9t

i.e.

vector with positive z direction, 24

^2> ^3>

positive y direction, 24

^v

made by vector with


damping constant, 88

f3

S ap

vector with positive x direction, 24

angle

/?

made by

Lagrange multipliers,

280, 284

components of torque along principal axes, 256


torque or moment, 36
torque or

moment about

center of mass, 229

APPENDIX

INDEX OF SPECIAL SYMBOLS AND NOTATIONS

D]

H
fi

359

coefficient of friction, 65

reduced mass, 182

index of summation, 166

cylindrical coordinate, 32

density in phase space, 318

density, 114

torsion, 31

radius of torsion, 31

time, 81

volume, 166

Euler angle, 257

<f>

phase angle or epoch, 88

<p

spherical coordinate, 32
scalar potential, 309

*
4>

lt <o 2 ,

generalized force, 283

angular speed, 8

components of angular velocity along principal axes, 256

angular velocity, 144

ax

fi,,,

components of angular momentum along

az

components of angular momentum

a lt 2 n 3

x, y, z axes,

254

along principal axes, 255

angular momentum, 37

Notations
|A|

AB
A B
AXB

A
AX

C)

scalar triple product, 5

C)

vector triple product, 5

scalar function of u, 6
scalar function of x, y,

du

A(m) du

and B, 5

vector function of x,y,z, 8

0(w)

A(tt)

to B, 11

and B, 4

vector function of u, 6

<p(x, y, z)

cross or vector product of

A,

time derivatives of A,

z,

i.e.

dA/dt, d2 A/dt 2 8

indefinite integral of A(m), 6

definite integral of A(w), 6

integral along curve C, 9

<s>

V
= grad
V A = div A
V X A = curl A
V<

dot or scalar product of

(B

A(x,y,z)

magnitude of distance from

(B

A(w)

magnitude of A, 4

<f>

f(r)

[F, G]

integral around a closed path, 9


del operator, 8

gradient of

<p,

divergence of A, 8
curl of A, 9

magnitude of central
Poisson bracket of

force, 116

and G, 331

INDEX
Arc length,

Absolute motion, 34, 60


Acceleration,

along a space curve,


angular, 8, 145, 148
apparent, 149
centrifugal, 145

8,

20

of triangle, 31

Areas, law of, 116, 123


Associative law, 3, 10
for rotations, 245
Astronomical data, 342, 343

centripetal, 8, 20, 21, 150


Coriolis, 145, 150

due to gravity, 62

Astronomy, definitions
Asymptotes, 119

in cylindrical coordinates, 32
in

moving coordinate systems,

in, 119,

120

Attraction, 120, 121, 129-133, 136

145, 146, 148, 149


in polar coordinates,

Areal velocity, 122, 123


Area, of parallelogram, 15

17-20

1, 7,

Atwood's machine,
Axioms, 1

26

76,

305

in spherical coordinates, 32

instantaneous, 7

Binomial theorem, 106


Binormal, 7, 8
Bob, of simple pendulum, 90
Body axes, 257

linear, 145

normal,

7, 8, 19,

20

relative, 7, 18, 19

tangential,

7, 8, 19,

Body centrode or locus, 229, 240, 241


Body cone, 257, 266
Bohr's quantum theory, 338
Boundary value problems, 195, 345
Bound vectors, 9, 10

20

true, 149

uniform, 62, 65, 66


Action and reaction, 33
Action integral, 313
Action variables, 316, 328
Actual force, 170
Air resistance, 63, 69-72
Amplitude, 86, 87
modulation, 102
of damped oscillatory motion, 88
of steady-state oscillation, 90
Angle variables, 316, 328, 329
Angular acceleration, 8, 145, 148
Angular impulse, 170, 228, 237
Angular momentum, 37, 45-47
about principal axes, 255
conservation of, 37, 45-47, 168, 228, 237
of a rigid body, 227, 236, 254, 259, 260
of a system of particles, 168, 169, 176, 179
of the earth about its axis, 150
principle of, 227, 229, 236, 238

Brachistochrone problem, 322


Calculus of variations, 313, 320-333
connection with Hamilton's principle, 321
Canonical coordinates, 314
Canonical equations, Hamilton's, 311
Canonical transformations, 314, 323-325
condition for, 314
Celestial mechanics, 311
Center, of ellipse, 118
of force, 116
of gravity, 167
of hyperbola, 119
Center of mass, 166, 172-175, 183-186
motion of, 167
motion of system relative to, 169, 178, 179
Central field, 116 (see also Central force)
equations of motion for particle in,
116, 122, 123

relationship to torque, 37, 46,


168, 169, 176

Angular speed, 8
Angular velocity,

(see also

Angular

potential energy of particle in,

velocity)

144, 148

117, 123-125
Central force, 116, 121, 122, 168, 318
(see also Central field)
determination of, from orbit, 118, 125-127
Centrifugal acceleration, 145
force, 146
Centripetal acceleration, 8, 20, 21, 150
force, 146, 150
Centrode, space and body, 229, 240, 241

terms of Euler angles, 258


of rigid body, 253
Anharmonic oscillator, 115
Aphelion, 120
Apogee, 120
Approximations, method of successive, 154, 159
Apsides, 143
Arbitrary constants, 344, 348
in

361

INDEX

362

Curvature, 8, 20
radius of, 8, 20
Cycle, 87

Centroid {see Center of mass)


Cgs system, 33, 62, 339, 340
Chain, hanging, 186, 187
sliding, 83
Characteristic determinant, 198
frequencies, 198
Chasle's theorem, 224, 275
Circular motion, 8, 20, 21, 95, 96

Cyclic coordinates, 312


Cycloid, 84, 85, 106, 107, 302, 322, 323

Cycloidal pendulum, 112, 303


Cyclones, 163
Cylinder, vibrations of, 104, 105
Cylindrical coordinates, 32
acceleration in, 32
gradient in, 61
Lagrange's equations in, 291, 292
velocity and acceleration in, 32

Clock, 2
Coefficient of friction, 65

Colatitude, 152
Collinear vectors, 23
Collisions of particles, 194, 195, 200-202
Comet, 121
Commutative law, for dot and cross
products, 3, 5, 10, 13, 14

D'Alembert's principle, 171, 182, 229

for Poisson brackets, 331


for rotations, 230, 231

Complementary equation and

solution, 347

Components, of a vector, 4
Compound pendulum, 228, 237, 238, 279, 291
Compression time, 194
Conical pendulum, 157
Conjugate momentum, 284, 288
Conservation, of angular momentum,
37, 45-47, 168, 228, 237

37, 167,

notation for time, 8


of vectors, 6, 16, 17

Constraint force, 170


Constraints, 170, 180
holonomic and non-holonomic, 170, 180,
283, 284, 286, 287
reaction due to, 64, 65
Contact transformations (see Canonical
transformations)
Continuous functions, piecewise, 197
Continuous systems of particles, 165, 195
of,

341

of gradient,

9,

29

Dimensions, 2, 339, 340


Direct collision, 194

Discontinuities, 197, 204, 207

Discrete system of particles, 165


Displacement, 2, 224
true, 170
virtual, 170
Dissipative forces, 64
Distance, 2
between two points, 11
Distributive law, 3
for Poisson brackets, 332

88, 96, 98, 99

failure of commutative law for,

22

partial, 195, 344


Differential, exact, 51, 52

Directrix, 118
Dirichlet conditions, 197, 206, 207

Cross products, 5, 13-15


determinant expression for, 14, 15
distributive law, 5, 14
8, 9, 21,

Difference of vectors, 3
and derivative operators,
Differential equations, 344-355

Differential

cosines, 24

moving, 144-164
non-inertial, 144
Coplanar vectors, condition for, 16
Coriolis acceleration, 145, 150
force, 146
Cosines, direction, 24
law of, 27
Couples, 226, 227, 235

Curl,

Determinant, characteristic, 198


Jacobian, 337
secular, 198, 215
Dextral system, 4
Diagonal, main or principal, 254
Difference equation, 216

Direction, 2

inertial, 34, 39

damped motion,

moving coordinate systems,

144, 145, 147, 148

condition for, 35, 50, 51


Constant of the motion, 312
Constrained motion, 64, 65, 72, 73

Critically

Definitions, 1

Derivatives, in

173

fields, 35, 43-45, 283,

Conversion factors, table


Coordinate systems, 3, 4

Definite integrals, 6

Density, 165
in phase space, 312

169, 227, 229, 236, 240

momentum,

Deceleration, 29
Decrement, logarithmic, 89, 97, 98

Del, 8

of energy, 36, 43-45, 124,

of

96-99

Deformable bodies, 165


Degrees of freedom, 165, 172, 224, 225, 253, 282
of a rigid body, 172, 253, 259

Conic section, 118, 127, 128

Conservative force
286, 287

Damped harmonic oscillator, 87, 88,


Damped oscillatory motion, 88, 98
Damping coefficient, 88
Damping forces, 64, 87

5, 13,

14

Divergence, 8, 21, 22
of curl, 9, 29
Dot products, 4, 5, 12, 13

8, 144,

148

INDEX
Dot products (cont.)
commutative law for, 5
distributive law for, 5, 12
Double pendulum, 285, 286, 299-301
Drumhead, vibrating, 195
Dumbbell, 278
Dynamics, 1
Dyne, 33
Earth, flat, 63
motion of particle relative to, 145
rotation of, 150, 257, 265
Eccentricity, 118
Einstein's laws of relativity, 34, 61
Elastic bodies, 165
Elastic collisions, perfectly, 195, 201
Elastic constant, 86
Elasticity, 194
modulus of, 86
Elastic string, vibrations of
(see Vibrating string)
Electrical charge, 83, 84

Electromagnetic field, 84, 309


Hamiltonian for particle in, 338
Lagrangian for particle in, 309
Ellipse, 38, 104, 118, 119, 121, 127, 128

Ellipsoid of inertia, 255, 256, 263, 264


Elliptic functions, 106, 272, 279
integrals, 106, 108

Energy, conservation
227, 229, 236, 240

of, 36, 43-45, 124, 169,

kinetic, (see Kinetic energy)

of simple harmonic oscillator, 87, 99


potential (see Potential energy)
total, 36
English systems, 63, 339
Epoch, 87, 88, 93
Equality of vectors, 2
Equilibrant, 47
Equilibrium, 37, 171
in a uniform gravitational field, 65, 66, 74, 75
of a particle, 37, 38, 47, 48
of a rigid body, 229, 241, 242
of a system of particles, 170, 180, 181
position, 86
stable, 38, 48, 49, 60, 141, 171, 230
Escape speed or velocity, 134
Euclidean geometry, 1, 2
Euler angles, 253, 257, 267, 268, 301
angular velocity in terms of, 258
Euler's equations of motion, 256, 264
from Lagrange's equations, 302
Euler's or Lagrange's equations, 313, 320
Euler's theorem, 224
on homogeneous functions, 305, 306, 317
Even extension of a function, 208
Even functions, 196
Event, 2
E volute, 112
Exact differential, 51, 52
Exact differential equation, 345, 350
Extremal, 313
Extremum or extreme value, 313

363

Field, scalar or vector, 8


Flat earth, 63 (see also Earth)

Focus, 118
Force, 33
axiomatic definition of, 33, 49
centrifugal and centripetal, 145, 146, 150
constraint, 170
damping, 64, 87
generalized, 283
units of, 33, 339, 340

Forced vibrations, 89, 99-102


resonance and, 90, 100, 101
Force fields, conservative (see Conservative
force fields)

non-conservative, 37, 47
uniform, 62, 65, 66

Foucault pendulum, 146, 154-156


Fourier coefficients, 196, 206
Fourier series, 195-197, 203-208
convergence of, 197
half range, 197, 207, 208

Fourier coefficients for, 196, 206


solution of vibrating string by
(see

Vibrating string)

Fps system, 33, 62, 339, 340


Frames of reference, Newtonian,

33, 34
Freely falling bodies, 63, 67
Free vectors, 9, 10
Frenet-Serret formulas, 31
Frequencies, characteristic, 198
Frequency, fundamental, 211
natural, 89, 98
obtained by Hamiltonian methods, 316, 329
of precession, 257, 265, 270, 273, 274
of resonance, 90
of simple harmonic motion, 86, 87
Friction, 65
coefficient of, 65
motion involving, 73
Fss system, 63, 339, 340
Function, scalar and vector, 8

Fundamental frequency, 211


Generalized coordinates, 282, 285, 286
forces, 283
impulse, 285
momenta, 284, 288
velocities, 283
General solution of differential equation,
344, 348
Generating functions, 314, 315, 323-325
Geometry, Euclidean, 1, 2
Gimbal, of a gyroscope, 258

Gradient,

8, 21,

curl of, 9, 29
in cylindrical

22

and spherical coordinates, 61

Gram, 33
Gravitation, universal law of (see Universal
law of gravitation)
Gravitational constant, universal, 120

Gravitational potential, 120, 121, 133, 143

Gravitational system of units, 63, 339

INDEX

364

Gravity, 62
center of, 167
vibrating string under, 214, 215

Gyration, radius

of,

Invariant, 34
Isolation, of a system, 64
Iteration,

method

of, 154,

159

225

Jacobian determinant, 337

Gyrocompass, 278
Gyroscopes, 258, 268-273

Half range Fourier sine and cosine series


197, 207, 208
Hamiltonian, 311, 317, 318, 330
for conservative systems, 311
for particle in electromagnetic field, 338
Hamilton-Jacobi equation, 315, 325-327, 330, 331
for Kepler's problem, 326, 327
for one dimensional harmonic oscillator,
325, 326
solution of, 315, 316

oscillator,

damped,

87, 88, 96-99

simple, 86-90, 92-102


two and three dimensional, 91, 103, 104

Herpolhode, 257, 266


Holonomic, 170, 180, 283, 284, 286, 287
Homogeneous equation, 346, 351
Homogeneous functions, 305
Euler's theorem on, 305, 306, 317

Hooke's law, 86
Hyperbola, 104, 119, 121, 127, 128
Hyperbolic functions, 54
Ignorable coordinates, 312, 315
Impact, 194
Impulse, 36, 45-47, 169, 170, 180
angular, 170, 228, 237
generalized, 285
relation to momentum, 36
Impulsive forces, 285, 295-298
Inclined plane, 64, 65, 72
motion of particle down, 72, 73
motion of sphere down, 239, 240
projectile motion on, 75, 76, 81

Incompressible

fluid,

Kilogram, 33
weight, 63
Kinematics, 1
Kinetic energy, 34, 35, 41-43
about principal axes, 255

terms of Euler angles, 258, 268


terms of generalized velocities, 283, 287, 288
of a rigid body, 227, 236, 259, 260
of a system of particles, 168, 169, 179, 182
of rotation, 229
of translation, 229
relationship to work, 35, 41, 169
in

in

Hamilton's equations, 311, 317, 318


Hamilton's principle, 313, 320-323

Harmonic

Kepler's laws, 120, 128, 129, 223


deduction of from Newton's universal
law of gravitation, 125, 126, 129

313

Indefinite integrals of vectors, 6


Independence of path, 9

condition for, 50, 51


Inelastic collisions, perfectly, 195, 201
Inertial frames of reference, 33, 34, 39
classical principle of relativity for, 39
Inertial system, 34, 39
Initial point, of a vector, 2
Instability, 38

Instantaneous, acceleration, 7
axis of rotation, 224, 225, 229
center of rotation, 225, 229, 240, 241
power, 34 (see also Power)
velocity, 7
Integral equations, 154
Integrals of vectors, 6, 16, 17

Line integrals)
Integrating factor, 345, 351
Internal forces, 177, 178
line (see

Invariable line and plane, 256, 257, 266

relativistic, 55

Kronecker

delta, 336

Lagrange multipliers,

280, 284, 292, 295

Lagrange's equations, 282-310, 320

and calculus of variations

(see Calculus

of variations)

for conservative systems, 284, 288-292


for non-conservative systems, 284
for non-holonomic systems, 284, 285,

292-295, 303
with impulsive forces, 285, 295-298
Lagrangian function, 284, 311
for particle in electromagnetic field, 309

Latitude, 152

Laws,

Lemniscate, 138
Length, 2
Light, speed of, 34, 54
Limiting speed or velocity, 70, 72
Line, 1
of action of a vector, 10
Linear equations, 345-347, 349, 350, 352
Linear harmonic oscillator, 86 (see also

Harmonic

oscillator)

Linear impulse (see Impulse)


Linear momentum (see Momentum)
Line integrals, 9, 22, 23
evaluation of, 22, 23
independence of path of, 9, 23
Liouville's theorem, 312
proof of, 318-320
Lissajous curves or figures, 91
Logarithmic decrement, 89, 97, 98
Lorentz force, 84

Magnetic

field,

83

Main diagonal,

of

moment

matrix, 254

Major

axis, of ellipse,

of hyperbola, 139

118

of inertia

INDEX
Mass, 2, 33
axiomatic definition of, 49
center of (see Center of mass)
changing, 194

Normal

(cont.)

to a surface, 24

Null vector, 3
Nutation, 270, 272

of the earth, 129

reduced, 182, 231


rest, 54, 61
units of, 33
Mathematical models, 1
Matrix, moment of inertia, 254
Matter, 1, 2

Mechanics,

Oblique collisions, 194


Odd extension, of a function, 207
Odd functions, 196
Operators, derivative, 144
Optics, 335
Orbit, 116, 120
determination of from central force,
117, 118, 125-127
weightlessness in, 135, 136
Order of a differential equation, 344

relativistic, 34

Membrane, vibrating, 195


Meteorite, 121
Metric system, 339

Minor

Oscillations, forced, 89 (see also

axis, of ellipse, 119

Forced vibrations)
anharmonic, 115
harmonic (see Harmonic oscillator)

of hyperbola, 139

Oscillator,

Mks system, 33, 62, 339, 340


Mode of vibration, normal, 194

Overdamped motion,

Models, mathematical, 1
Modulation, amplitude, 102
Modulus of elasticity, 86
Moment, of couple, 226
of force, 36
of momentum, 37 (see also Angular
Momental ellipsoid, 256

Moments

momentum)

of inertia, 225, 231-233, 254, 259,

260, 263, 264

matrix, 254
principal (see Principal

moments

of inertia)

226
theorems on, 225, 233-235
Momentum, 33, 167
angular (see Angular momentum)
conjugate, 284, 288
special,

conservation

of, 37, 167,

173

generalized, 284, 288

moment

of,

37 (see also Angular momentum)

of a system of particles, 167, 169, 172, 173


principle of, 238

Momentum
Moon,

88, 98, 99

Overtones, 211

coordinates, 312

119, 342

Pappus, theorems of, 193


Parabola, 63, 104, 119, 121, 127, 128
as curve of motion of projectile, 68
Paraboloid of revolution, 107, 108
Parachutist, motion of, 69, 70
Parallel axis theorem, 226, 233, 234
Parallelepiped, volume of, 5, 15, 16

Parallelogram, area of,


Parallelogram law, 2

5,

Partial differential equation, 195, 344


of vibrating string (see Vibrating string)
Particles, 2

equilibrium of, 37, 38


systems of, 165-193
vibrations of, 194, 197-199
Particular solutions, 344, 345, 347, 348
Path, independence of, 9, 50, 51

Pendulum, bob, 90
compound, 228, 237, 238, 279, 291
conical, 157
cycloidal, 112, 303

Multiply-periodic vibrations, 194

double, 285, 286, 299-301

Musical note, 211

Foucault, 146, 154-156


seconds, 110

Natural frequency and period, 89, 98


Newton, 33
Newtonian frames of reference, 34
Newton's collision rule, 194, 202
laws of motion, 33-41
universal law of gravitation (see
Universal law of gravitation)
Nodes, line of, 257
Non-holonomic, 170, 180, 283, 284, 286, 287
Non-inertial systems, 144, 145

Normal

frequencies, 194, 198

for a double pendulum, 300, 301, 308


for a vibrating string, 210, 211
for a vibrating system of particles, 215, 216
Normal modes of vibration, 194, 198, 199
for a vibrating string, 210, 211
Normal, principal, 7, 8, 20

15

simple (see Simple pendulum)


Perigee, 120
Perihelion, 120
Period, 53
natural, 89
of damped motion, 89
of harmonic oscillator, 87
of motion in a magnetic field, 83
of simple harmonic motion, 86, 87

of simple pendulum, 91, 105, 106


orbital, 135, 136
sidereal, 120

Perpendicular axes theorem, 226, 234, 235


Phase, angle, 87, 88, 93
integrals, 316, 328, 329

out

of,

93

space, 312, 318-320

INDEX

366
Piano string, vibrations of, 195
(see also Vibrating string)
Piecewise continuous functions, 197
Planck's constant, 338
Planets, 119, 343
Poinsot's construction, 257
Point, 1, 2
Poisson bracket, 331, 332
Polar coordinates, 25, 26

54
velocity and acceleration
Polhode, 257, 266

gradient

Reaction

(cont.)

due to constraints, 64, 65


Rectangular coordinate systems,
right handed, 4
Reduced mass, 182, 231
Reference level, 64
Relative acceleration,

7, 18,

3,

19

velocity, 7

Relativistic mechanics, 34

Relativity, classical principle of, 34, 39

in,

in,

26

Position, 2

coordinates, 312
vector, 4
Potential, 35 (see also Potential energy)

relation to stability, 38
scalar, 35, 309

vector, 309

Potential energy, 35, 36, 43-45


(see also Potential)
in a central force field, 117, 123-125
in a uniform force field, 64, 69
of a system of particles, 169, 176-178
principle of minimum, 230
relation of to work, 35, 44

Pound, 33
weight, 63
Poundal, 33
34, 41-43, 227, 237
relation to work, 42

Power,

Precession, 156, 256, 270, 272


frequency of, 257, 265, 270, 273, 274

Principal axes of inertia, 255, 260-263


Principal diagonal, 254
Principal moments of inertia, 255, 260-263
method of Lagrange multipliers for, 280
Principal normal, unit, 7, 8, 20
Products of inertia, 254, 259, 260
Products of vectors, by a scalar, 3
cross (see Cross products)

Einstein's laws of, 34, 61


theory of 54, 55, 61, 143, 337
Representative point, 312
Resistance, air, 63, 69-72
Resisting forces, 64
Resisting medium, motion in, 64, 69-72
Resonance, 90, 100, 101
Restitution, coefficient of, 195
Restitution time, 194
Rest mass, 54, 61
Restoring force, 86
Resultant of vectors, 2
Rheonomic, 283, 286, 287
Right handed system, 4
Rigid bodies, 165, 170, 224, 230, 231, 236
equilibrium of, 229, 241, 242
force free motion of, 256, 257, 265
general motion of, 224, 253, 259
motion of, about a fixed axis, 236
plane motion of, 224-252
symmetric, 257
Rockets, 173, 194, 199, 200
motion of, 199, 200
Rotating coordinate systems, 144, 147, 148
Rotation, 224, 253
associative and commutative laws for, 230,
231, 245
finite, 230, 231
of the earth, 150, 257, 265
pure, 253
Routh's function or Routhian, 337

dot (see Dot products)


Satellites, 119

Projectiles, 62, 63

maximum

height

68
motion of, 68, 69, 71, 72
on an inclined plane, 75, 76, 81
range of (see Range of projectile)
time of flight, 68
of,

Scalars, 2

Pulley, 76, 289, 290

Quantum mechanics, 311


Quantum number, orbital, 338
Quantum theory, 338
Radius, of curvature,
of gyration, 225
of torsion, 31
Radius vector, 4

8,

20

Range of projectile, 68
maximum, 69
on inclined plane, 75, 76
on rotating earth, 164
Reaction, 33

Scalar function, 8
Scalar potential, 35
for electromagnetic field, 309
Scalar product (see Dot product)
Scalar triple product, 5
Scleronomic, 283, 286, 287
Seconds pendulum, 110
Secular determinant, 198, 215

Semi-major and minor axes, 118, 119, 129


Separation of variables, 210, 316, 345, 347, 348
Sidereal period, 120
Simple closed curve, 9
Simple harmonic motion, 86
(see also Simple harmonic oscillator)
amplitude, period and frequency of, 86, 87
Simple harmonic oscillator, 86-90, 92-102
amplitude, period and frequency of, 86, 87
damped, 87, 88
energy of, 87, 99

INDEX
Simple harmonic oscillator

Top

(cont.)

forced vibrations of, 89, 99-102


resonance and, 90, 100, 101

Sliding vector, 9, 10
Slug, 63
Solar system, 119
Solution of differential equation, 344
Space, 1, 2
Space axes, 257

Space centrode or locus, 229, 240, 241


Space cone, 257, 266
Special relativity (see Relativity)
Speed, 7 (see also Velocity)

scalar, 5

angular, 8
escape, 134
of light, 34

vector, 5

Two and

orbital, 135, 136


Sphere, particle sliding down, 76, 77, 82
sphere rolling down, 244, 303, 304
Spherical coordinates, 32
gradient in, 61

Lagrange's equations

in,

and acceleration

Undetermined

Uniform
force

306
32

Units,

37, 38,

acceleration, 62, 65, 66

field, 62, 65,

66

rectangular,

47
field, 65, 66, 74,

75

Statistical mechanics, 311


Steady-state solution, 89
Stiffness factor, 86

of vectors, 2

obtained graphically and analytically, 12, 48


Sun, 119, 342

Superposition principle, 199


Surface, normal to, 24

Symmetric matrix or tensor, 254


Systems of particles, 165-193

62, 65,

66

339-341

2,

3,

Universal law of gravitation, 120, 128, 129


deduction from Kepler's laws, 128, 129
Unstable equilibrium, 171, 230

of a particle, 37, 38, 47, 48


of a rigid body, 229, 241, 242
of a system of particles, 170, 180, 181

Sum

of, 347,

Unit vectors, 3

171, 230

1,

method

Uniformly accelerated motion,

in,

uniform gravitational

88, 98

coefficients,

352, 353

Stable point, 38
Star, 119
in a

three body problems, 121, 223

Underdamped motion,

Spin, 270, 272


Spring constant, 86
Spring, vibrations of, 86, 93-95
Stability of equilibrium, 38, 48, 49, 60, 141,

Statics,

(cont.)

steady precession of, 270


Torque, 36, 45-47, 168, 176
of a couple, 226
relation to angular momentum, 37, 46, 168,
169, 176
Torsion, 31
constant of, 308
radius of, 31
Transformation equations, 282, 285, 286
canonical, 314, 323-325
Transient solution, 89
Translation, 224, 253
Transverse vibration of a string
(see Vibrating string)
Triple products, 5, 15, 16

Lagrange's equations for, 306


Simple pendulum, 86, 87, 90, 91, 102, 103
length of equivalent, 228
Sines, law of, 27

velocity

367

Variation of an integral, 321


Variation of parameters, method of, 347, 353, 354
Variation symbol, 313, 337
Variations, calculus of
(see Calculus of variations)
Vector algebra, laws of, 3, 10-12
Vector field, 8
Vector function, 8
Vector potential, 309
Vector product (see Cross product)
Vector triple product, 5
Vectors, 1, 2
algebra of, 2, 3
bound, 9, 10

components

of,

definition of, 2

Tangent vector, unit, 7, 8, 19


Tautochrone problem, 113

free, 9, 10

magnitude
Velocity,

1, 6, 7,

angular

Terminal point, of a vector, 2


Theorems, 1
Time. 1, 2
principle of least, 335
Top, 258, 268-273, 274
Lagrange's equations for motion
motion of, 258, 268-273
sleeping, 274

of, 11, 13

sliding, 9, 10

Tension, 74, 76
Tensor, moment of inertia, 254

(see

17-19

Angular

velocity)

apparent, 148
areal, 122, 123
escape, 134
generalized, 283
in cylindrical coordinates, 32
of, 301,

302

in

moving cordinate systems,

in polar coordinates,

26

in spherical coordinates, 32

145, 148, 149

368

Velocity (cont.)
instantaneous, 7
limiting, 70, 72

INDEX
Virtual work, principle

of, 170,

Weight, 62

relative, 7

apparent, 162
Weightlessness, 135, 136

true, 148

Work,

of a rigid body, 253, 259

Vertices, of ellipse, 118


of hyperbola, 119
Vibrating string, 195, 202, 203, 209-212
considered as a system of particles, 215-217
under gravity, 214, 215
Vibrating systems of particles, 194, 197-199
Vibrations, of a cylinder, 104, 105
forced (see Forced vibrations)
Violin string, vibrations of, 195
(see also Vibrating string)
Virtual displacements, 170
Virtual work, Lagrange's equations and, 292

229

34, 41-43, 168, 169, 176-178,

237

generalized forces and, 283, 287, 288


in rotation of a rigid body, 227
relationship of to kinetic energy, 168, 169,
176, 177
relationship of to potential energy, 44

virtual (see Virtual work)

x direction, 4

y direction, 4
z direction, 4

Zero vector, 2

You might also like